So what is the Second Darn Law?

With nearly 500 comments on the thread on the Second Law of Thermodynamics there is obviously a need for people to discuss the basic greenhouse theory. Here’s a new thread on that theme.

So what is the Second Darn Law?

From NASA:

But there are variations

As with all these Laws of science there is no exact wording, because There Is No God Who Issues Science Decrees*.  What we have are human efforts to best explain the world around us. Note that the two well known versions of the Second Law both contain the phrase “whose sole result”, meaning that heat transfer can certainly move from a colder to a warmer body if there is some other compensating movement where more heat is transferred from a hotter body to a colder one. Voila… whatever heat transfer goes from greenhouse gases to the Earth is more than countered by the heat moving from the Sun to Earth and on to space. Greenhouse gases can heat the Earth as long as the entropy of the whole system increases.

Clausius statement:

No process is possible whose sole result is the transfer of heat from a body of lower temperature to a body of higher temperature.

Kelvin Statement

No process is possible in which the sole result is the absorption of heat from a reservoir and its complete conversion into work.

Behind the scenes I’m still getting many emails from people wondering about this topic. I’m sorry I can’t reply to them all. Joseph Postma kindly replied to my last post on this topic in comment #97, and in reply to him I say this (his comments are in blue):

“The 2nd Law of Thermodynamics applies to net flows of heat, not to each individual photon, and it does not prevent some heat flowing from a cooler body to a warm one.”

–JoNova

JP: The laws of heat transfer DO forbid a cold body from raising the temperature of a warmer body.  This is a much more physically unambiguous clarification.  “Flows of heat” is ambiguous and confusing.  It is obviously correct to say that a cold body does not raise the temperature of a warmer body.  “Flows of heat” is ambiguous and not important, in terms of what has the ability to raise who’s temperature.

The “ability to raise temperature” is not proscribed by the 2nd law, merely by your interpretation of it. The second law does not mention “abilities” of anything.

“Net heat flows” is not ambiguous. It is the Net heat flow. Are more photons going in one direction? Yes, but the ones coming back the other way affect the NET heat flow.

JP: It is obviously correct to say that a cold body does not raise the temperature of a warmer body.

No it isn’t. It’s obviously correct to say that the heat stored in a cold object can’t raise the temp of a warmer object, but the heat is not coming from a cold object. The heat is coming from the sun. The cold body just stops it leaving the  warmer object at the rate it was previously.

Why do you keep ignoring the sun? There is a continuous input of energy into the Earth?

JP: But there IS a restriction on which body can raise the temperature of its neighbours.  Only the hotter body can raise the temperature of a colder neighbour.

This is preposterous. God doesn’t make rules like “photons can travel in any direction except the way it came”. Atoms radiate photons randomly and some return in the same direction.

“Technically, strictly, greenhouse gases don’t “warm” the planet (as in, they don’t supply additional heat energy), but they slow the cooling, which for all pragmatic purposes leaves the planet warmer that it would have been without them.” — Jo Nova

JP: This statement is fraught with contradiction.  Is it technical or is it real?  I mean this is very important!  If GHG’s, as you correctly point out cannot raise the planets temperature because they are not actually sources of additional energy, then how do we arrange some logic in which they do exactly what we just said they cannot do?

The technical part is the language use, the reality part is the temperature.

The difference between whether we ascribe the raised temperature to the GHG gas or to the SUN is “technical” the reality of the warming is real. Think of the blanket analogy. Technically a blanket on your body does not heat you (if you want to be a pedant) your own body heats you. In reality, you put on a blanket and your temperature goes up.

What if we put it this way: Greenhouse gases don’t  warm^ the planet but they do create a situation where the Sun can warm the world to a temperature above where it previously was without the gases?

That doesn’t break the second law, and it describes reality with a technical accuracy.

^(I don’t like this phrasing at all: the word “warm” here specifically means something other than it’s usual use — here it implies no additional heat energy is supplied that is new to the Earth-atmosphere system. Normally “to warm” means… well you know… . Life doesn’t have to be this complicated.)

In the end this boils down to the energy flows in a whole system, and people who think that the addition of greenhouse gases to our atmosphere won’t warm the planet (due to a conflict with the Second Law) are forgetting that the system includes the sun, and the extra energy flowing in continuously is what drives the system. CO2 may have little effect on Planet Earth, but that’s for other reasons. You can’t just make a comment about “the Earth-Atmosphere” part of the system and ignore the plasma ball that’s 1.4 million kilometers across and 13 million degrees in the middle.

————————————————————

UPDATE #1 From  Michael Hammer

Michael Hammer has suggested this as a line of reasoning that may help people discuss this.  If you don’t agree with the end conclusion (that greenhouse gases can’t warm earth because they are not hotter than Earth), point out exactly which step in the sequence is the one you think is wrong and explain it so we can understand why.

  1. Do you agree that if you stand surrounded by cold objects (say a ring of huge ice blocks) you feel cold?
  2. Do you agree that the colder the object you are surrounded by the colder you feel?  eg: if they were blocks of frozen CO2 (dry ice) instead of water ice would you feel colder?
  3. OK now if you have been standing surrounded by extremely cold objects and then move so that you are now surrounded by merely cool objects does the move make you feel less cold than you were before?
  4. In the absence of green house gases the earth is surrounded by an extremely cold object – outer space at 4K (-269C).  Green house gases make the atmosphere opaque at some wavelengths. With these in place the earths surface is in effect surrounded by a merely cool atmosphere instead of the truly frigid outer space.  Because the surface is now surrounded by a less cold object than it was before it is less cold.
  5. Since warm and cold are opposites, less cold is the equivalent of warmer.  Surrounding earth’s surface with an opaque cool atmosphere make it warmer than it would be if exposed directly to the ultra frigid outer space.

————————————————————-

*Which doesn’t mean there is No God at all. I’m staying right out of that one.

————————————————————-

UPDATE#2  Joseph Postma  in  Comment #47

[My comments in reply inline in green. –JN]

Thanks for posting my comments, Jo.

Firstly, I would like to re-iterate that in my paper, “Understanding the Thermodynamic Atmosphere Effect”, I in fact refer to the Laws of Heat Transfer (i.e. thermodynamics) in total, not just the 2nd Law. You may have an earlier pre-edit copy which had the reference to the 2nd Law specifically.

Secondly, the 2nd Law isn’t even the most important one when it comes to understanding how the atmosphere actually works. Conservation of energy is actually much more important, in terms of the physics. Your diagram from NASA shows just how obscure the 2nd Law can be.

Lastly, the brief mention of the Laws of heat transfer, around page 6-7 of “Understanding…”, is NOT central in any way to the paper! There are MUCH more important issues discussed within.

Now to your post:

“heat transfer can certainly move from a colder to a warmer body if there is some other compensating movement where more heat is transferred from a hotter body to a colder one.”

This was never disputed…it is unclear why there is so much focus on this issue. What is undisputable is that more heat transfers from the warm body to the cold body and so the warm body has to cool down. That is, a cold body does not raise the temperature of a warmer body.

JN: Exactly – the greenhouse gases don’t warm the sun and the sun gets cooler (albeit slowly).

There is too much ambiguity and invitation to equivocation when using terms like “heat transfer” when what is more important is how the temperature is actually controlled. “Heat” does not equal “higher temperature”. 1 Kelvin has “heat”.

“Voila… whatever heat transfer goes from greenhouse gases to the Earth is more than countered by the heat moving from the Sun to Earth and on to space.”

And so by extension the GHG’s cannot increase the temperature of the ground. Because the net heat transfer is from the ground (warmest), outward (coolest).

“Greenhouse gases can heat the Earth as long as the entropy of the whole system increases.”

Not without an additional source of energy, could they do this*. GHG’s do not represent an additional source of energy. Re-emitted, or “back-IR” radiation from GHG’s, do not represent additional energy – this is rather the RESULT of the original Solar energy insolated into the system. The Laws of Thermodynamics, in total, tell you that this energy can’t be replicated for further heating. Never mind the 2nd Law all by itself. The IR and back-IR, etc, is a RESULT of what has already happened…they’re not the cause.

JN: *Never once have I said that GHGs are a source of new energy. Have I not explained at length and repeatedly that the heat comes from the sun, and the GHG’s slow the heat loss from Earth? Remember if the Earth didn’t radiate the heat away it’s final equilibrium temperature would be much higher… what would 4 billion years of input do with no heat loss?

“Why do you keep ignoring the sun? There is a continuous input of energy into the Earth?”

This is unfortunate. If my paper would actually be read it can be learned that I spend a GREAT deal of time discussing the Sun and its continuous input into the Earth system. When understood properly, as I qualitatively and partially quantitatively explained, you discover there is no need to postulate a radiative greenhouse theory in the first place because 1) there’s more than enough Solar insolation to explain the ground temperature, and 2) back-IR heat amplification is not supported experimentally – not that we should have ever expected it to be so, if one utilizes all 3 laws of thermo.

JN: This is a whole new point. If you have other reasons to think CO2’s effect is minimal, lets discuss those but the only way to get past the focus on the Second Law is to admit that as a point of reasoning, there is nothing about the second law which precludes GHGs from theoretically causing an increase in the Earths temperature. Then we can move on and discuss the other reasons. And if I can be so bold — people will find your other reasons more compelling if there is no erroneous reference to the Laws of Thermodynamics being broken at the start.

“This is preposterous. God doesn’t make rules like “photons can travel in any direction except the way it came”. Atoms radiate photons randomly and some return in the same direction.”

Obviously I did not say that. [No, but that’s the implication of a Law that “stops” this happening.–JN] I said there is a restriction on cold bodies raising the temperature of a warmer body. [Only in a closed system where there is not a third body involved like The Sun –JN] If you surrounded yourself by an entire “death star” of ice – you were placed right in the center of it – ALL of that IR energy from the ice-star, which would be tremendously more energy than you radiate at ~30C (or whatever skin temperature humans are), would not raise your temperature. It could be emitting one-billion times more thermal-IR than you do, and you could be right at the center of the sphere, but it would still not warm you. You would still cool. Even if the “death-star” you were in the center of were 29C, you would still cool.

JN:  People keep warmer in Igloos, not because the heat comes from the ice, but because it comes from burning fat internally, and the ice cuts the heat loss. Why continually ignore the real heat inputs? No one is suggesting that ice or GHG’s raise the temp of a warmer body above what it would have been without another energy source…. I’m stuck in ground-hog day: the sun … the sun … the sun…. ?

“What if we put it this way: Greenhouse gases don’t warm^ the planet but they do create a situation where the Sun can warm the world to a temperature above where it previously was without the gases?”

The Sun can only warm the planet to whatever its insolation temperature is. If a planet surface is warmer it is because of additional physical contributions, such as adiabatic compression and convection, etc. Any IR energy as a result of these processes are just that – the RESULT. They cannot self-amplify the temperature or energies of the system as they are the consequence of what has already happened, not the cause of what has already happened.
Also, the discussion of the actual solar insolation and resulting expected temperature can be read about in “Understanding…”. The input insolation upon the Earth is NOT -18C…it is actually +30C on average with a maximum of around +121C, depending on local effects and extinction. It is only the “effective”, or mathematically and theoretically averaged energy OUTPUT of the entire system, as compared to a Black Body, which equates to -18C. This does not mean this is the actual kinetic temperature you should find, nor does it say where you should actually find it even if it was.

“… it describes reality with a technical accuracy.”

Well, I disagree :) It isn’t accurate, ENOUGH.

JN: I’m talking about a principle – not an exact number. The principle is right or wrong. Your calculations are a different thing, and accuracy matters there, but if the principles are wrong, there’s no point doing a calculation.

In regard to Hammer’s post, I will just say this:

The effect of an atmosphere on a planet is to modulate the surface temperature from the extremes it would otherwise experience, as is the case on the moon compared to Earth. ANY atmosphere, independent of GHG’s, has a thermal capacity and ability to retain heat, and so it will ALWAYS be the case that the presence of an atmosphere keeps the “bottom-of-atmosphere” average temperature higher than the effective mathematical equivalent (as just discussed). If adiabatic effects are added to this then the “bottom-of-atmosphere” temperature can rise even further – even to well above the original Solar insolation, as is the case on Venus.

So consider then what the strongest GHG, H2O vapour, does in contribution to this effect: it is the best modulator of temperature that we have in our atmosphere. Under the Sun, in the tropics with lots of H20 vapour, the temperature will raise to LESS-than whatever the Solar insolation says it should. For some reason all the H2O doesn’t amplify the temperature. Very telling.

Then at night, comparing a tropic region to a desert, the tropic region cools by only a fraction of what a desert does. They likely have roughly the same “daily-average” temperature.

So the strongest GHG actually kept it cooler under the Sun than it should have been, and cooler than a desert, even with all the extra back-IR “heating”, of which a desert experiences very little.

There’s nothing wrong with being kept warm at night…but there is a significant concern if Solar insolation heating is modulated downwards by GHG’s.

The only extra heating more CO2 could provide is that contribution to increasing the atmospheric depth and density, such as to increase the physical heating effects.

I think we agree that ideally, CO2 concentration should be above at least 1000ppmv, in order to sustain the biosphere and improve its productivity. I am not sure this can be attained via fossil fuel burning…but I DO think we should try! Even if it DID warm the planet…well that would be great. The climate has always changed and it is only the intellectually inept, and the physically lazy, who now try to use other-people’s MONEY to avoid doing the work and economic improvements to adapt to it.

But as it is, the GH postulate is borne out of an incorrect application of the Stefan-Boltzmann equation, and worse, an illogical interpretation of its result. Under such a scenario, a logician EXPECTS non-physical postulates and theories to be borne out of an illogical premise. Applying the Laws of Thermodynamics, and trying to figure out how the 2nd Law applies, is secondary to this more fundamental physical mistake. Though, again, we should expect to find an inability to agree on the correct application under such a scenario.

We (the Slayers of the Sky-Dragon) will be publishing a new paper in the coming weeks which quantitatively will put this issue to rest…if logic is allowed back into the paradigm. We will also outline (and carry out) the proper physics experiment which anyone can perform, to actually prove or disprove the thesis on either side – such is the way science actually works.


JN: Joseph, you can’t quantitatively put to a rest an argument of reason which is not based on logic. The reasoning underpins everything. Maybe CO2 doesn’t heat the Earth (for other reasons unrelated to this post), but if that’s the case — it’s not anything to do with “Laws of Thermodynamics”. Can you at least acknowledge that in the Sun-Earth-Atmosphere system it is possible that energy flows which are always going from the hot sun to cold space, could be changed by a colder entity (GHGs) which would allow a warmer entity (Earth) to get even warmer, given that a third source (the Sun) is continually adding energy?

8 out of 10 based on 5 ratings

675 comments to So what is the Second Darn Law?

  • #
    overseasinsider

    Do you mean I finally get to be first commenter???

    Yet another good post Jo!!! Great description of 2nd Law!!

    Keep up the good work!!

    20

  • #

    […] So what is the Second Darn Law? […]

    20

  • #
    Phillip Bratby

    I was brought up on the work of Feynman, whose definition of the second law is:

    A process whose only net result is to take heat from a reservoir and convert it to work is impossible.
    No heat engine taking Q1 from T1 and delivering heat Q2 at T2 can do more work than a reversible engine for which
    W = Q1-Q2 = Q1(T1-T2)/T1

    20

  • #
    Steeptown

    Obviously changing the composition of the atmosphere will affect the heat transfer processes.
    The fundamental problem is the crazy statement that the “greenhouse effect” will raise the surface temperature by 30deg due to “back-radiation”. Postma has shown the temperature and lapse rate is just due to radaitive equilibrium and the gravitational compression of the atmosphere.

    40

  • #
    Craig Thomas

    I’m curious – what’s with the americanism? Why is it a “darn” law?

    Still, you’re doing everybody a favour by trying to educate the “It violates the 2nd law of thermodynamics!!!11!1!” cranks.
    Is it the last we ever hear that recurring lunatic assertion, would you say?

    [Err? I didn’t realize it was an Americanism. I thought it was a … word?]

    20

  • #
    pat

    sorry for being O/T but this is why i don’t trust any political party and will vote for none of them. some might like to think the coalition will not touch carbon dioxide, but they are sadly mistaken:

    20 May: Australian: James Massola: Malcolm Turnbull costs Tony Abbott’s carbon plan at $18bn a year
    Mr Turnbull also clarified his comments on the extent to which direct action was a market-based scheme, saying there would be some competition between providers of carbon offsets…
    http://www.theaustralian.com.au/national-affairs/malcolm-turnbull-will-never-lead-again-liberal-mps/story-fn59niix-1226059726470

    20

  • #
    Alistair

    Frankly, the author has got into an Oozlum Bird situation. Unless we stop him, he’s going to vanish up his own contradiction.

    1. The author appears to be confused between the meanings of heat, temperature and heat transfer. Also, in common with most people, he appears to believe the atmosphere is in equilibrium: it’s continuously changing as you get interchange between sensible and latent heat, e.g the cycle of stratus/cumulus in the tropics.

    2. To try and push the idea of back radiation is futile; it doesn’t exist because it’s an artefact of Arthur Milne’s choice of an infinite atmosphere when in 1922 he solved the partial differential equation for IR absorption. [See Miskolczi’s 2006 paper for the proof, but brush up very well on your physics first.]

    3. measured radiation is Prevost’s interchange in a temperature gradient.

    10

  • #
    bananabender

    Clasius and Kelvin predate Bohr Theory relating to the structure of atoms. They both assumed that atoms were solids and that energy was transferred via an invisible aether. Thermodynamics is based on classical Newtonian mechanics and has the same limitations when applied to very large or very small objects – e.g. planets or individual atoms.

    Believe it or not science has progressed in the last 150 years.

    Simply discussing the totally unscientific Greenhouse Effect simply gives credence to the absurd concept of AGW. The temperature of the Earth is entirely due to EMR output of the Sun – any other factor is irrelevant. The Sun heats the surface and heat is transferred to the atmosphere via conduction, convection and phases changes in water. In turn the pressure and density determine the atmospheric temperature. End of story.

    Unconfined gases can’t be heated so the GE is totally impossible. Any heating of the atmosphere simply results in expansion with no net increase in heat. We know this to be fact because the atmosphere temporarirly expands measurably as the Sun passes overhead. This is called the Diurnal Bulge.

    30

  • #
    Ted Middleton

    Just how difficult is this, if a body has energy; that is it is above 0°K it will radiate photons.

    If a body at 273°K is surrounded by objects at 200°k it will stay warmer longer that if it was surrounded by objects at 100°K. Simply the result of the net transfer of photons.

    If it is surrounded by objects at 373°K it will increase in temperature, for the same reason.

    Forget the semantics and accept the science.

    10

  • #
    Alistair

    bananabender: does the diurnal expansion raise the tropopause?

    10

  • #
    Mark

    Methinks you’ve earned a couple of grammatical demerit points Jo.
    * Apostrophe used in a plural noun.
    * Use of word “darn”.
    Penalty: Detention for an indeterminate period, fully suspended in view of previously demonstrated good behaviour and service to the community.

    [I plead guilty. I found one stow-away apostrophe, but not the second.– Jo]

    10

  • #
    RJ

    “JP: It is obviously correct to say that a cold body does not raise the temperature of a warmer body”.

    No it isn’t. It’s obviously correct to say that the heat stored in a cold object can’t raise the temp of a warmer object, but the heat is not coming from a cold object. The heat is coming from the sun. The cold body just stops it leaving the warmer object at the rate it was previously.

    Why do you keep ignoring the sun? There is a continuous input of energy into the Earth?

    Sorry but this article is poor. Take this section above

    The response makes no sense. The Sun is a warmer object so I’m unsure what the author is trying to say.

    10

  • #
    Bryan

    The NASA diagram is correct and similar to others in Physics textbooks.

    From; University Physics Young and Freedman

    Energy transfer that takes place sole because of a temperature difference is called heat flow or heat flow transfer and energy transferred in this way is called heat page 470

    Heat always flows from a hot body to a cooler body never the reverse. pg 559

    There a difference between colloquial “heat” and the term heat used in thermodynamic textbooks.
    Thermodynamic heat transfer, in the case above, IS the radiative flux difference between the two surfaces and is always from the hotter surface to the colder.

    Heat has the property of being changed into useful work in the given situation.
    A steam engine can extract heat from the high temperature reservoir do work like moving pistons and dump the unused heat in the colder reservoir.
    Now try to see if the reverse can happen.
    Can heat be taken from the colder reservoir make the pistons move and then dump unused heat at the higher reservoir?
    Obviously not!

    This shows that the “heat” moving from the colder reservoir is not HEAT as defined in thermodynamic textbooks.

    20

  • #
    Bulldust

    Sorry to go off topic again, but I have had a gutfull of Canberra. JtI (Jack the Insider) is a Canberra government type who spews the CAGW rubbish at The Australian. His latest diversion is to focus on the Libs agenda here:

    http://blogs.theaustralian.news.com.au/jacktheinsider/index.php/theaustralian/comments/turnbull_blows_the_whistle_on_direct_action/

    He seems oblivious to the fact that the Libs are not serious (under Abbott) about climate change policy and that the “direct action” plan is just insurance to keep a few green-leaning Lib voters on side. He is seriously naive if he thinks otherwise, leave alone the fact that the Libs are not in power right now … a simple concept that seems to elude him. I gave him a blast andf a challenge as follows (fully expecting it to be “moderated”):

    So Jack… when are you and the ETS getting married? No doubt Joolya and Bob Brown can be best man and maid of honour, but they will have to figure out who plays what role. Funny how you managed to focus on the insignificant piece of news for the week… like the hole we blasted in your surplus? Do you really think WA will simply lay back and let the Feds molest us through the CGC? Seen the Labor polls lately? In any State or Federally? Seriously mate… you’ve lost the plot.

    This has been cross-posted as evidence of your censorship, which is expected. The first ploy of CAGW alarmists. You are the noise around the real debate… do you really think the Libs are serious about action on “climate change” policy? If so you need a dose of reality salts. A big one.

    The Labor 5% GHG cut achieves nothing, is a revenue raiser, and a fatuous genuflection to the IPCC. If you had the nads you would debate me honestly on this subject. You have been called out mate. Weighed, measured and very much wanting…

    He is a coward and has pulled out a BSc at CSIRO to try and impress me in the past… let’s see how the coward plays this one. I fully expect my comment will not make it up on The Australian. Best case scenario, he will take me on and I rip his feeble treatises to shreds, but I doubt he has the nads. Cowardly Canberra types never do…

    10

  • #
    RJ

    “JP: But there IS a restriction on which body can raise the temperature of its neighbours. Only the hotter body can raise the temperature of a colder neighbour.”

    This is preposterous. God doesn’t make rules like “photons can travel in any direction except the way it came”. Atoms radiate photons randomly and some return in the same direction.

    Surely the key question is not whether protons can travel in any direction. It is whether they have a warming impact when they arrive.

    10

  • #
    Lawrie

    Pat @ 6,

    I understood the costs came about as a result of the “fines” we would have to pay as a result of not meeting our Kyoto obligations plus the costs of the direct action. The latter are put at 3 billion which seems excessive for planting trees and burying biochar. The beauty of Abbotts scheme is that the moment there is a “consensus” that AGW is really and truly crap the scheme can be scrapped.

    On the greenhouse I have a feeling it is being made more difficult to understand than necessary. A proper greenhouse or indeed a sunroom utilise the glass to prevent the warm air inside interacting with the cold air outside. Open a door and the greenhouse starts cooling. Open two doors and the temp drops nearly to the outside temp. At night the air inside soon stabilises to the outside temp.The atmosphere is totally different being exposed to convection day and night.

    20

  • #
    Climate realist number9

    This criticism of Joseph Postma’s replies does not clarify or change the argument- it is semantic only.
    This glib comment is wrong – try the experiment yourself. You will not be made hotter by the blanket.
    In reality, you put on a blanket and your temperature goes up.

    10

  • #

    Simply discussing the totally unscientific Greenhouse Effect simply gives credence to the absurd concept of AGW.

    Making basic errors of reasoning is far worse.
    It gives alarmists good reason to mock, laugh and distract us from the science that matters.

    10

  • #
    RJ

    “Technically, strictly, greenhouse gases don’t “warm” the planet (as in, they don’t supply additional heat energy), but they slow the cooling, which for all pragmatic purposes leaves the planet warmer that it would have been without them.” – Jo Nova

    JP: This statement is fraught with contradiction. Is it technical or is it real? I mean this is very important! If GHG’s, as you correctly point out cannot raise the planets temperature because they are not actually sources of additional energy, then how do we arrange some logic in which they do exactly what we just said they cannot do?

    Regarding Jo’s comment and JP’s response. I could not see why he had an issue with Jo here. Surely GHGs do slow the cooling rate. So they then increase the average temperature

    Maybe if Jo had put -leaves the planet warmer on average– but I would have liked some further explanation from JP on this response. But a jersey leaves me warmer (due to reduced cooling) than without one so can not really see a problem with Jo’s comment.

    I would say though for a jersey – In reality, you put on a jersey and your temperature decreases less might be more accurate than your temperature goes up.

    10

  • #
    Matt b

    “It gives alarmists good reason to mock, laugh and distract us from the science that matters.”

    I’d go further and suggest it exposes many commentators as lacking enough basic science to have any credibility whatsoever when it comes to AGW.

    But don’t worry – it is not an exclusive club there are plenty of loons on my side of the fence. Note I somewhat optimistically don’t consider myself a loon but I know I’ve walked in to whatever you want to throw my way guys:)

    10

  • #
    memoryvault

    Bananabender @ 8

    Way to go BB.

    Now, if we can just get some of these folks to realise that IR light, photons as emitted by a particular substance, and sensible heat (as measured in Joules, for instance) are NOT necessarily the same thing, and certainly not at the same time, we might just get somewhere.

    Unfortunately, given the number of folks still struggling with the difference between heat and temperature, I’m not holding my breath.

    I fear you are going to spend the the next couple of days trying to explain why unconfined gases cannot be heated to people who never heard the expression “at STP” during their high school years.

    10

  • #
    Bulldust

    O/T but I think it is worth reading anyway… the tax stoush is getting a tad bitchy and Colin Barnett is showing backbone with his riposte to Canberra’s carpetbaggers about the royalty rate increase:

    Mr Barnett made light of the stoush today.

    “It’s raining; I’m happy; you’re happy; Wayne Swan’s unhappy; bad luck,” he told a Chamber of Commerce and Industry breakfast in Perth.

    Source: http://www.theaustralian.com.au/national-affairs/budgets/colin-barnett-blows-2bn-hole-in-wayne-swans-budget-target/story-fn8gf1nz-1226059586044

    Here’s a guy who stands for something… there is no question he represents the State, Joolya take note. I don’t know about you lot, but I like the cut of his jib … for the moment.

    10

  • #
    memoryvault

    For those struggling with the definition of the Second Law, here are all four in simplified form:

    FIRST LAW: You can’t win.
    SECOND LAW: You can’t break even.
    THIRD LAW: You can’t can’t ever quit the game.
    ZEROTH LAW: The more things change the more they stay the same.

    10

  • #
    David

    MattyB

    I’d go further and suggest it exposes many commentators as lacking enough basic science to have any credibility whatsoever when it comes to AGW.

    Self description suits you.

    10

  • #
    Popeye

    Bulldust @ 14

    Just checked mate – it’s not up there yet and from the text I doubt VERY much that it will make it.

    But you’ve at least got it on record on this blog and 10 to 1 on jti would be trolling all the blogs so hopefully he’ll see it here – who knows? (I also do the same just to let their censors know that even if they don’t allow it WILL end up somewhere else!

    BTW – agree with your synopsis that the Libs policy is only there at the moment for appeasment to some right wing AGW believers. As soon as he’s PM he has promised to repeal anything to do with CO2 taxes that these fools introduce.

    Cheers,

    10

  • #
    Bryan

    Some people might think it pedantic to distinguish between how heat is used in a colloquial way and its thermodynamic meaning.

    So I will give another example.

    If you heard two people talking on a bus and one said;
    “The whale must be one of the biggest fish in the sea”

    You would understand that the person was incorrect but you would also understand what they were meaning to say(colloquial use).
    However if the same expression was used in a biology textbook you would think that the textbook was not to be recommended.

    This thread is about the SECOND LAW OF………THERMODYNAMICS.
    So the thermodynamic meaning of heat should be obligatory.

    10

  • #
    Joe Lalonde

    Jo,

    I finding more and more evidence that NASA is just a corrupted about science and are using models rather than actual measurements for many areas.

    As for the thermodynamic laws, they are based on bad science.
    They do not include: Centrifugal force from a rotating planet, density of materials to transfer heat (some are more efficient than others), Forward momentum of the solar system, current evidence that the sun uses magnetic fields to align the rotational speeds, etc.

    10

  • #
    bananabender

    @Joanne Nova:
    May 20th, 2011 at 6:10 pm

    Simply discussing the totally unscientific Greenhouse Effect simply gives credence to the absurd concept of AGW.

    Making basic errors of reasoning is far worse.
    It gives alarmists good reason to mock, laugh and distract us from the science that matters.

    No it just makes us look like morons. Discussing the totally non-existent Greenhouse Effect is as absurd as discussing how many angels can dance on pin with a religious fundamentalist. It only encourages Warmists and gives them credibility.

    The Greenhouse Theory is totally discredited 19th nonsense which was exhumed in the 1960s by the environmental movement. By the 1920s meteorologists were fully aware that the climate is primarily controlled by the hydrological cycle.

    10

  • #
    David

    Sorry O/T

    JTI is biased!!
    Bulldust 14
    I just got one through with an answer from (JTI) Jack The Idiot – his reply was quote

    Dave
    Fri 20 May 11 (07:20pm) JTI – I have been informed you are not publishing all the comments sent to you – some are not happy JACK and you are getting a name on the Blog Game.
    P.S. Good article – just your imput has let you down!!!
    Dave
    Jack the Insider
    Fri 20 May 11 (08:20pm) Let me explain this to you, Dave: I’m under no obligation to publish any comment and I certainly won’t publish anything that breaches The Australian’s rules for commenters.</blockquote>

    Unbelievable – that attitude would ban MattyB, John Brookes – free debate etc.
    I don’t think he will publish your comment.

    10

  • #
    Bulldust

    Popeye:

    JtI hasn’t the cahones… he thinks he is so very clever sitting back in his armchair in Canberra, navel-gazing about the Canberra follies. In truth, I used to enjoy his ascerbic wit which smacked of Yes Minister, but when it comes to climate change he is a dyed-in-the-wool warmista.

    Unlike MattB and JB he doesn’t have the gumption to take on critical skeptics. Instead he hides behind a wall of censorship. I am happy to take him on any day, anywhere, any time… he has two chances.

    10

  • #
    michael hammer

    It is abundantly clear that the stumbling block for many of the people here is the thought that the cool atmosphere is heating the surface and that contravenes the second law of thermodynamics. For those of you who feel that way may I point out, this and the previous post are not claiming the cool atmosphere directly heats the surface. The sun heats the surface. What the so called green house gases are doing is to reduce Earth’s heat loss to outer space at some wavelengths. They do this by blocking some of the surface emission travelling out through the atmosphere to outer space. To maintain equilibrium, the heat flow at other wavelegnths has to increase a bit and for that to happen the Earth has to get a bit warmer. The sequence is the heat flow to outer space is reduced so an imbalance is created whereby heat inflow from the sun exceeds heat outflow to space so the Earth starts to warm. As it does so the radiation to space at other wavelengths progressively increases until equilibrium is restored.

    By the way, as stated before, these posts are not suggesting the mechanism has anything whatever in common with an actual green house. We do know, surprising as this might seem to some, the two mechanisms are completely different. We use the term “green hosue gases” because that is a very commonly used and immediately recognised handle. Thats all it is, don’t read any more into it.

    On a third point, I see several comments along the line go “read some physics text books”. Often the content of the comments in which such comments are made makes it abundantly clear the author has little idea of physics themselves. They have read a phrase somewhere, taken it out of context and turned it into an idee fixee. Please consider that the people writing the posts that start these threads have actually studied physics at university level and just maybe they do know what they are talking about. Tell me, would you go to your doctor, listen to his diagnosis and then tell him he is talking rubbish and should go study some medicine? Do you really think that would enhance the consultation?

    The issue uder debate in this thread is not contentious CAGW “science”. It is extremely well known and understood physics and was so long before CAGW came on the scene. The are major falacies in the CAGW hypothesis but this is not one of them.

    10

  • #
    Phillip Bratby

    Michael @31

    You are forgetting the other processes involved in the transfer of energy from the surface. The restoration of the energy balance is such that the effect on temperature is immeasurably small.

    10

  • #
    Bryan

    michael hammer says

    “I see several comments along the line go “read some physics text books”. Often the content of the comments in which such comments are made makes it abundantly clear the author has little idea of physics themselves. They have read a phrase somewhere, taken it out of context and turned it into an idee fixee. Please consider that the people writing the posts that start these threads have actually studied physics at university level and just maybe they do know what they are talking about.”

    Several of the posters went to Universities that took great care with the word HEAT.
    Take the NASA diagram above.
    It is correct

    Compare the diagram in your previous post where you have HEAT moving from a cold surface to a hotter surface.
    Incorrect.

    If I had labelled my diagram like that in a physics test I would have failed.

    10

  • #
    cohenite

    Michael@31: “The sun heats the surface. What the so called green house gases are doing is to reduce Earth’s heat loss to outer space at some wavelengths. They do this by blocking some of the surface emission travelling out through the atmosphere to outer space.”

    What is the duration of that ‘blocking’? I have read of the difference in time between excitation and dexcitation of the CO2 molecule; is this the duration so that if the sun ‘switched off’ the “greenhouse effect” would last, theoretically, for only the difference between excitation and dexcitation? In this ‘thought experiment’ please disregard such sinks as the ocean and soil where, according to Hansen and ilk, AGW/greenhouse heat is stored.

    10

  • #

    I repeat, all this is incompetent theorizing. You refuse to acknowledge that the observed temperatures in the Venus atmosphere, when properly (and very simply) compared to Earth’s atmosphere, show there is no greenhouse effect whatsoever, as promoted in climate science. Fact trumps theory, and the Venus/Earth comparison trumps all of climate theory, and demolishes it. What is inarguable is that CO2 (and every other gas) absorbs infrared radiation, and therefore heats the atmosphere — but it does so by absorbing incident solar radiation, not by absorbing radiation from the planetary surface, and it heats only up to the atmosphere’s ability to hold that heat (as determined by the average molecular specific heat and the density, which increases the deeper you go in the atmosphere — that is why you find the surface temperature decreases with altitude, as you drive into the mountains for example). Always go to the critical, observed facts, rather than waste your time on theory and vain argument that is ignorant of those facts.

    10

  • #
    Neville

    Just another life saving use for that terrible pollutant and at 5% or 128 times the level in the atmosphere.

    But at 5% it can actually help these people suffering from hypocapnia or too little co2.

    http://www.ncbi.nlm.nih.gov/pmc/articles/PMC1872042/

    10

  • #
    KR

    “Technically, strictly, greenhouse gases don’t “warm” the planet (as in, they don’t supply additional heat energy), but they slow the cooling, which for all pragmatic purposes leaves the planet warmer that it would have been without them.” – Jo Nova

    Very well stated, Jo.

    10

  • #
    memoryvault

    Bananabender @ 28

    Congratulations once again on hitting the nail on the head

    As I pointed out in a very early post on the previous thread, there never was an “argument” about the physics of the so-called “greenhouse effect” contravening the Second Law of Thermodynamics until someone challenged the “Trenberth Travesty” that the “extra heat” was “hiding” in the “deep oceans”.

    This caused a backlash from the sceptical side that the atmosphere “heating the oceans” contravened the Second Law of Thermodynamics (which it does). Rather than argue the point (which is impossible) the warmista cultists turned it into a debate about whether the so-called “Greenhouse Effect” of itself contravened the Second Law.

    The tens of thousands of largely meaningless words written here in these two posts are testimony to how successful they were at sidetracking meaningful debate on the subject.

    Yes, regrettably, they see us a “morons”.

    And perhaps rightly so.

    10

  • #
    Jeremy

    Technically speaking, Phonons are “responsible” for heat transfer. Photons are the particle version of electromagnetic waves, which is something different.

    10

  • #

    No it just makes us look like morons. Discussing the totally non-existent Greenhouse Effect is as absurd as discussing how many angels can dance on pin with a religious fundamentalist. It only encourages Warmists and gives them credibility.

    The Greenhouse Theory is totally discredited 19th nonsense which was exhumed in the 1960s by the environmental movement. By the 1920s meteorologists were fully aware that the climate is primarily controlled by the hydrological cycle.

    Righto – these last two posts have shown conclusively and repeatedly that the greenhouse effect doesn’t break the second law of thermodynamics. That doesn’t prove the GHG effect is real, but it shows the argument that it can’t exist because of The-2nd-Law, is wrong. Now if you have some other line of argument, evidence or reason, how about we move past the “second law of thermodynamics argument” so we can discuss the other reason. Moving right along would look like this kind of statement: “OK I see your point, since the sun provides energy, and the entropy of the system increases, it would be possible for greenhouse gases theoretically to heat up the atmosphere. For reasons that have nothing to do with the second law I think the greenhouse effect is …. (insert logical reasoning here that doesn’t involve calling people names).

    Got it?

    10

  • #
    BobC

    Just my 2c:

    1) The clearest statement of the 2nd law here is the one by Feynman (quoted by Phillip Bratby at post #3):

    A process whose only net result is to take heat from a reservoir and convert it to work is impossible.

    2) It is not a violation of the laws of thermodynamics for a cold object to be instrumental in causing a warmer object to further heat up. Example: A large refrigerated mirror is taken to a sunny beach, where it is positioned to reflect sunlight onto a sunbather. Result: The sunbather heats up due to the extra Solar radiation she is receiving (and therefore radiates more IR herself). The temperature of the mirror is not relevant, only it’s specific interaction with Solar radiation (e.g., it is reflective). (Note that this wouldn’t work if the mirror were a blackbody — then it would have to be warmer than the sunbather to have a similar effect due to it’s own radiation.)

    3) CO2 in the upper atmosphere is more like the mirror than the blackbody. CO2 molecules are capable of absorbing IR photons that are of the correct energy to excite one of the internal vibrational states of the molecule. If they do not transfer this energy due to a collision, they will re-emit a photon of the same energy in a very short time. In the thin upper atmosphere, the absorption-emission process is more likely than the energy-partition-collision process. This ability to absorb photons that excite internal vibrational states is independent of the Kinetic Energy (e.g., temperature) of the CO2. Since a CO2 molecule has an equal chance of emitting a photon in any direction, roughly half of the absorbed photons are re-emitted back toward the ground. Thus, the CO2 layer acts essentially as a 50% reflective mirror for those wavelengths.

    4) CO2 also scatters ~50% of the incoming Solar radiation (within the narrow resonant bandwidths). However, since the Sun is quite hot, the vast majority of it’s radiant energy is in the visible band, where CO2 is essentially transparent. The Earth, being much cooler than the Sun, radiates most of it’s thermal energy into the longwave IR band, where the CO2 resonances have a discernable effect.

    5) Because CO2 “reflects” ~50% of the IR photons (within specific narrow bands) back toward the ground, the ground experiences a higher flux of radiation and hence increases it’s temperature slightly to re-establish equilibrium (equal energy in as out) — just like the sunbather.

    This is a weak conversion process being driven by a huge energy source — the Sun. It no more violates the Laws of Thermodynamics than the cooled mirror on the beach, or an electric heater or refridgerator. None of these processes would work without a distant source of energy.

    20

  • #
    BobC

    On the other hand, the funniest statement of the Laws of Thermodynamics are the ones quoted by memoryvault at post # 23:

    FIRST LAW: You can’t win.
    SECOND LAW: You can’t break even.
    THIRD LAW: You can’t ever quit the game.
    ZEROTH LAW: The more things change the more they stay the same.

    10

  • #
    RJ

    @ 41

    5) Because CO2 “reflects” ~50% of the IR photons (within specific narrow bands) back toward the ground, the ground experiences a higher flux of radiation and hence increases it’s temperature slightly to re-establish equilibrium (equal energy in as out) — just like the sunbather.

    Not correct. Backradiation from the earth does not usually increase its temperature slightly. Otherwise a cooler body (the atmosphere) is heating a warmer one (earth). The radiation hits the ground but does not increase the surface temperature

    20

  • #
    RJ

    @38

    Yes, regrettably, they see us a “morons”.

    And how do you know this? And why does it concern you what they think?

    10

  • #
    Bryan

    BobC says

    ….” A large refrigerated mirror is taken to a sunny beach, where it is positioned to reflect sunlight onto a sunbather.” …….

    Reflection is a different process to absorption and emission

    BobC says….”If they do not transfer this energy due to a collision”

    The relaxation time is much longer than the chance of losing the energy by collision.
    10 to the power of ten collisions per second at STP

    Spectrograph’s from satellite looking down show large “bite” around 15um.
    This represents the heating of the atmosphere caused by CO2 absorbing 15um radiation and this energy being passed on to N2 and O2 molecules.
    For downward radiation the probability shifts to longer wavelengths supplied by H2O rather than 15um CO2 band.
    I

    10

  • #

    UPDATE #1 From Michael Hammer

    Michael Hammer has suggested this as a line of reasoning that may help people discuss this. If you don’t agree with the end conclusion (that greenhouse gases can’t warm earth because they are not hotter than Earth), point out exactly which step in the sequence is the one you think is wrong and explain it so we can understand why.

    1. Do you agree that if you stand surrounded by cold objects (say a ring of huge ice blocks) you feel cold?
    2. Do you agree that the colder the object you are surrounded by the colder you feel? eg: if they were blocks of frozen CO2 (dry ice) instead of water ice would you feel colder?
    3. OK now if you have been standing surrounded by extremely cold objects and then move so that you are now surrounded by merely cool objects does the move make you feel less cold than you were before?
    4. In the absence of green house gases the earth is surrounded by an extremely cold object – outer space at 4K (-269C). Green house gases make the atmosphere opaque at some wavelengths. With these in place the earths surface is in effect surrounded by a merely cool atmosphere instead of the truly frigid outer space. Because the surface is now surrounded by a less cold object than it was before it is less cold.
    5. Since warm and cold are opposites, less cold is the equivalent of warmer. Surrounding earth’s surface with an opaque cool atmosphere make it warmer than it would be if exposed directly to the ultra frigid outer space.

    ————————————————————-

    10

  • #

    Thanks for posting my comments, Jo.

    Firstly, I would like to re-iterate that in my paper, “Understanding the Thermodynamic Atmosphere Effect”, I in fact refer to the Laws of Heat Transfer (i.e. thermodynamics) in total, not just the 2nd Law. You may have an earlier pre-edit copy which had the reference to the 2nd Law specifically.

    Secondly, the 2nd Law isn’t even the most important one when it comes to understanding how the atmosphere actually works. Conservation of energy is actually much more important, in terms of the physics. Your diagram from NASA shows just how obscure the 2nd Law can be.

    Lastly, the brief mention of the Laws of heat transfer, around page 6-7 of “Understanding…”, is NOT central in any way to the paper! There are MUCH more important issues discussed within.

    Now to your post:

    “heat transfer can certainly move from a colder to a warmer body if there is some other compensating movement where more heat is transferred from a hotter body to a colder one.”

    This was never disputed…it is unclear why there is so much focus on this issue. What is undisputable is that more heat transfers from the warm body to the cold body and so the warm body has to cool down. That is, a cold body does not raise the temperature of a warmer body. There is too much ambiguity and invitation to equivocation when using terms like “heat transfer” when what is more important is how the temperature is actually controlled. “Heat” does not equal “higher temperature”. 1 Kelvin has “heat”.

    “Voila… whatever heat transfer goes from greenhouse gases to the Earth is more than countered by the heat moving from the Sun to Earth and on to space.”

    And so by extension the GHG’s cannot increase the temperature of the ground. Because the net heat transfer is from the ground (warmest), outward (coolest).

    “Greenhouse gases can heat the Earth as long as the entropy of the whole system increases.”

    Not without an additional source of energy, could they do this. GHG’s do not represent an additional source of energy. Re-emitted, or “back-IR” radiation from GHG’s, do not represent additional energy – this is rather the RESULT of the original Solar energy insolated into the system. The Laws of Thermodynamics, in total, tell you that this energy can’t be replicated for further heating. Never mind the 2nd Law all by itself. The IR and back-IR, etc, is a RESULT of what has already happened…they’re not the cause.

    “Why do you keep ignoring the sun? There is a continuous input of energy into the Earth?”

    This is unfortunate. If my paper would actually be read it can be learned that I spend a GREAT deal of time discussing the Sun and its continuous input into the Earth system. When understood properly, as I qualitatively and partially quantitatively explained, you discover there is no need to postulate a radiative greenhouse theory in the first place because 1) there’s more than enough Solar insolation to explain the ground temperature, and 2) back-IR heat amplification is not supported experimentally – not that we should have ever expected it to be so, if one utilizes all 3 laws of thermo.

    “This is preposterous. God doesn’t make rules like “photons can travel in any direction except the way it came”. Atoms radiate photons randomly and some return in the same direction.”

    Obviously I did not say that. I said there is a restriction on cold bodies raising the temperature of a warmer body. If you surrounded yourself by an entire “death star” of ice – you were placed right in the center of it – ALL of that IR energy from the ice-star, which would be tremendously more energy than you radiate at ~30C (or whatever skin temperature humans are), would not raise your temperature. It could be emitting one-billion times more thermal-IR than you do, and you could be right at the center of the sphere, but it would still not warm you. You would still cool. Even if the “death-star” you were in the center of were 29C, you would still cool.

    “What if we put it this way: Greenhouse gases don’t warm^ the planet but they do create a situation where the Sun can warm the world to a temperature above where it previously was without the gases?”

    The Sun can only warm the planet to whatever its insolation temperature is. If a planet surface is warmer it is because of additional physical contributions, such as adiabatic compression and convection, etc. Any IR energy as a result of these processes are just that – the RESULT. They cannot self-amplify the temperature or energies of the system as they are the consequence of what has already happened, not the cause of what has already happened.
    Also, the discussion of the actual solar insolation and resulting expected temperature can be read about in “Understanding…”. The input insolation upon the Earth is NOT -18C…it is actually +30C on average with a maximum of around +121C, depending on local effects and extinction. It is only the “effective”, or mathematically and theoretically averaged energy OUTPUT of the entire system, as compared to a Black Body, which equates to -18C. This does not mean this is the actual kinetic temperature you should find, nor does it say where you should actually find it even if it was.

    “… it describes reality with a technical accuracy.”

    Well, I disagree 🙂 It isn’t accurate, ENOUGH.

    In regard to Hammer’s post, I will just say this:

    The effect of an atmosphere on a planet is to modulate the surface temperature from the extremes it would otherwise experience, as is the case on the moon compared to Earth. ANY atmosphere, independent of GHG’s, has a thermal capacity and ability to retain heat, and so it will ALWAYS be the case that the presence of an atmosphere keeps the “bottom-of-atmosphere” average temperature higher than the effective mathematical equivalent (as just discussed). If adiabatic effects are added to this then the “bottom-of-atmosphere” temperature can rise even further – even to well above the original Solar insolation, as is the case on Venus.
    So consider then what the strongest GHG, H2O vapour, does in contribution to this effect: it is the best modulator of temperature that we have in our atmosphere. Under the Sun, in the tropics with lots of H20 vapour, the temperature will raise to LESS-than whatever the Solar insolation says it should. For some reason all the H2O doesn’t amplify the temperature. Very telling.
    Then at night, comparing a tropic region to a desert, the tropic region cools by only a fraction of what a desert does. They likely have roughly the same “daily-average” temperature.
    So the strongest GHG actually kept it cooler under the Sun than it should have been, and cooler than a desert, even with all the extra back-IR “heating”, of which a desert experiences very little.
    There’s nothing wrong with being kept warm at night…but there is a significant concern if Solar insolation heating is modulated downwards by GHG’s.
    The only extra heating more CO2 could provide is that contribution to increasing the atmospheric depth and density, such as to increase the physical heating effects.
    I think we agree that ideally, CO2 concentration should be above at least 1000ppmv, in order to sustain the biosphere and improve its productivity. I am not sure this can be attained via fossil fuel burning…but I DO think we should try! Even if it DID warm the planet…well that would be great. The climate has always changed and it is only the intellectually inept, and the physically lazy, who now try to use other-people’s MONEY to avoid doing the work and economic improvements to adapt to it.

    But as it is, the GH postulate is borne out of an incorrect application of the Stefan-Boltzmann equation, and worse, an illogical interpretation of its result. Under such a scenario, a logician EXPECTS non-physical postulates and theories to be borne out of an illogical premise. Applying the Laws of Thermodynamics, and trying to figure out how the 2nd Law applies, is secondary to this more fundamental physical mistake. Though, again, we should expect to find an inability to agree on the correct application under such a scenario.

    We (the Slayers of the Sky-Dragon) will be publishing a new paper in the coming weeks which quantitatively will put this issue to rest…if logic is allowed back into the paradigm. We will also outline (and carry out) the proper physics experiment which anyone can perform, to actually prove or disprove the thesis on either side – such is the way science actually works.

    20

  • #
    mkelly

    “In reality, you put on a blanket and your temperature goes up.”

    Nope my body temp is 98.6 F and does not increase unless I have an ailment. The rate at which I lose heat to the surroundings changes. I suspect this is what you meant.

    10

  • #
    Nullius in Verba

    I have a deep pond of clear water with a perfectly black liner at the bottom. It is a sunny day. Sunlight shines through the water and is absorbed by the liner. Being a black body, the liner radiates at a rate dependent on its temperature, but this radiation is absorbed almost immediately by the water – within a few millimetres. The water itself radiates, and this radiation is again almost immediately reabsorbed. If radiation were the only mechanism for heat transfer, and we maintained conditions until it reached steady state, what would be the temperature distribution of the water in the pond?

    The top surface of the water – a layer a few millimetres thick – must radiate as much energy upwards as the pond receives from the sun. It radiates the same amount downwards, so is continually losing energy at about twice the rate of solar heating. Where does this energy come from? From the layer below – which must be radiating two units up, to keep the top layer’s temperature constant, and two below, by symmetry. It is losing energy at four times the rate of solar heating. Where does it come from? We know that we’re getting one unit from the layer above, so we must be getting three from the layer below. It is now radiating six times the solar heat, three up and three down. Where does it come from? We only get two units from the layer above, so it must get four from below. And so on.

    Radiated power increases linearly with depth, and hence temperature with the fourth root of depth.

    A pond of water contains the pure-radiative greenhouse mechanism concentrated into a tiny space. Every layer receives radiation from the cooler layer above – hundreds of stacked ‘greenhouses’ on top of one another. We know warm water radiates – because we can observe it with an IR camera – and we may suppose it continues to do so even when surrounded by more water. (You could try putting a waterproof IR camera under the water to see if it receives anything.) Back-radiation exists.

    10

  • #
    BobC

    Bryan (@45):
    May 21st, 2011 at 12:32 am
    BobC says

    ….” A large refrigerated mirror is taken to a sunny beach, where it is positioned to reflect sunlight onto a sunbather.” …….

    Reflection is a different process to absorption and emission

    Yeah, that was sort of the point of the example. Absorption and re-emission of photons that excite vibrational resonances in CO2 molecules is a different process than blackbody absorption and radiation also. Like the mirror, it is not dependent on temperature.

    BobC says….”If they do not transfer this energy due to a collision”

    The relaxation time is much longer than the chance of losing the energy by collision. 10 to the power of ten collisions per second at STP

    First, we’re not talking about CO2 at ground level (STP), but in the stratosphere. Differential spectroscopy of the atmosphere from satellites shows that line broadening is essentially non-detectable at those altitudes. Hence collisions have negligible effect there.

    Second, line broadening due to collisions smears out the resonances, but does not change the total energy absorbed and re-emitted, so my comment about collisions is mostly irrelevant, as the vast majority of collisions are elastic and don’t change the internal energy of the molecules. (If this weren’t so, CO2 would not follow the gas law.)

    11

  • #
    RJ

    @46

    5. Since warm and cold are opposites, less cold is the equivalent of warmer. Surrounding earth’s surface with an opaque cool atmosphere make it warmer than it would be if exposed directly to the ultra frigid outer space.

    But only due to an reduced cooling rate.

    It you say it makes it warmer people might then assume (as many I know do) that GHGs further warm the surface. Even above the maximum from the sun only. Or that backradiation can increase the temperature of an object with a steady heat source. Or that a blanket can increase our temperature (rather than just reduce cooling) or ice can warm use up due to backradiation etc.

    If we say for example -the planet average surface temperature is higher because GHGs reduce the surface cooling rate. Don’t use warmer without qualifications.

    Its best to be precise otherwise wrong assumptions will be made. Especially by people who want to understand but have little time to understand this topic. I stress now a slower cooling rate at the surface not a warmer surface when I explain the impact of GHGs.

    10

  • #
    BobC

    RJ:
    May 21st, 2011 at 12:21 am

    The radiation hits the ground but does not increase the surface temperature

    Mind elaborating on exactly what happens at the molecular level here? In what way is a photon from a cold object different from a photon with the exact same energy from a hot object?

    10

  • #
    Bryan

    Joanne Michael Hammer at 46

    The process you describe is essentially insulation.
    Heat can travel by three methods

    Lets say Earth surface is at 288K and deep space at 3K
    1. Radiation from atmospheric gases(back radiation) can provide radiative insulation by sending back some energy to planet surface.
    2. Conduction air molecules leaving Earth surface have RMS speeds of around 500m/s.
    If the air molecules left and never came “back” then this energy would be lost from Earth forever.
    However the air molecules do come back with a reduced speed thus returning some energy back to the Earth surface.(Backconduction).
    3. Convection we would all would agree is the biggest heat transfer mechanism in the atmosphere but as hot air rises so colder air falls.
    So we also have back convection returning energy to the surface.
    All three methods of heat transfer insulate the Earth surface from a chilly “would otherwise be”.
    This is the broad outline.

    Now dealing with the “CO2 greenhouse effect”.
    If the CO2 partially retards the radiative loss at night then it will retard the IR component of Solar radiation during the day.
    The net effect could well be overall cooling of the Earths atmosphere.
    Hottel and Schack have made measurements of the radiative properties of CO2 and have concluded that the effects are very small at atmospheric temperatures.
    The clouds produce much more radiation than CO2 and H2O in gaseous form.

    10

  • #
    mkelly

    UPDATE #1 From Michael Hammer

    All gases absorb heat. Gases are very good dissipaters of heat. N2 and O2 absorb heat at the surface via conduction and rise via convection. So aren’t all gases GHG since we would have a “heated” atmosphere even if it was only N2 and O2.

    10

  • #
    RJ

    @ 48

    Back-radiation exists.

    But does anyone claim it does not. And isn’t backradiation simply radiation given off by a object that is heated by another object. So radiation from the earth that travel towards the sun is backradiation isn’t it. It will not heat the sun though.

    10

  • #
    Nullius in Verba

    “It will not heat the sun though.”

    It will slow the rate at which the sun loses heat, and increase its temperature by a tiny amount though.

    10

  • #
    RJ

    Mind elaborating on exactly what happens at the molecular level here? In what way is a photon from a cold object different from a photon with the exact same energy from a hot object?

    Not all photons are the same. And photons from a colder object do not warm a warmer one

    This was covered on the last thread refer 407 on

    http://joannenova.com.au/2011/05/why-greenhouse-gas-warming-doesnt-break-the-second-law-of-thermodynamics/#more-14778

    10

  • #
    Alistair

    An awful lot of guff has been written about GHG warming here, including by myself although I have used the usual weasel words and self-questioned so I looked it up to check.

    This a good analysis: http://wattsupwiththat.com/2010/08/05/co2-heats-the-atmosphere-a-counter-view/

    The requirement for local thermodynamic equilibrium means the following:

    1. absorption of IR by CO2 is balanced by equal and opposite emission of IR from all directions, the practical result of Prevost’s Law of Interchanges and Kirchhoff’s Law.

    2. there can be no heating of the atmosphere by that transiently absorbed vibrational energy.

    3. back radiation is impossible.

    4. anyone who claims anything different hasn’t studied the physics.

    20

  • #
    BobC

    Bryan:
    May 21st, 2011 at 2:23 am

    If the CO2 partially retards the radiative loss at night then it will retard the IR component of Solar radiation during the day.
    The net effect could well be overall cooling of the Earths atmosphere.

    Except for this fact: The peak of the Sun’s output is in the visible where CO2 is transparent. Hence CO2 scatters a very small proportion of the Sun’s energy input to the Earth. However, the peak radiative power from the Earth is in the deep IR, 30 times the wavelength of the visible, due to it’s much cooler temperature than the Sun. Hence CO2 can scatter a significantly larger proportion of the Earth’s total radiated energy than the Sun’s.

    10

  • #
    RJ

    @ 55

    and increase its (the suns) temperature by a tiny amount though.

    I don’t agree. It will make no difference whatsoever. Photons from a colder body can not heat a warmer one.

    post 97

    http://joannenova.com.au/2011/05/why-greenhouse-gas-warming-doesnt-break-the-second-law-of-thermodynamics/#more-14778

    There is no experimental (nor theoretical, in truth) framework which supports the idea of radiative self-amplification of temperature after insolation from another hotter source. I will spare details, but we will be experimentally proving this simple reality in due course. There is no such thing as conductive self-amplification of temperature; nor is there such a things as radiative self-amplification of temperature. Both conduction and radiation are modes of heat transfer and obey said laws of heat transfer. Radiation cannot conveniently get around those laws, as many try to claim.

    10

  • #
    DavidA

    Bryan solar radiation sits in the visible light spectrum so there isn’t any incoming IR from the sun in the day. The frequency of the radiation output by a warm body is a function of its temperature and bodies as hot as stars output visible light.

    10

  • #
    BobC

    Alistair:
    May 21st, 2011 at 2:33 am

    The requirement for local thermodynamic equilibrium means the following:

    1. absorption of IR by CO2 is balanced by equal and opposite emission of IR from all directions, the practical result of Prevost’s Law of Interchanges and Kirchhoff’s Law.

    You’re absolutely correct, if you are considering a blackbody. However, there is no requirement for a mirror, for example, to be in thermodynamic equilibrium with the radiation it is reflecting, nor for a CO2 molecule to have any particular kinetic energy before it can absorb or re-emit photons that excite it’s internal resonances.

    2. there can be no heating of the atmosphere by that transiently absorbed vibrational energy.

    I agree, to first order anyway.

    3. back radiation is impossible.

    Then, what do you call the photons that CO2 absorbs, then re-emits (due to excitation of vibrational energy states) when they happen to head back toward the Earth?

    4. anyone who claims anything different hasn’t studied the physics.

    This isn’t a valid argument.

    10

  • #
    Nullius in Verba

    “Photons from a colder body can not heat a warmer one.”

    If photons from a cold body are absorbed by a warmer one, they increase its energy. The photons carry energy, the colder body loses energy by emitting them, the warmer body must gain the same energy by absorbing them.

    10

  • #
    Alistair

    BobC: ‘Then, what do you call the photons that CO2 absorbs, then re-emits (due to excitation of vibrational energy states) when they happen to head back toward the Earth?’

    The necessary requirement of Kirchhoff’s Law for thermal equilibrium [it applies to all frequency intervals].

    ‘This isn’t a valid argument.’

    Yup, you’re right but it made me feel better!

    10

  • #

    RJ,

    Not all photons are the same. And photons from a colder object do not warm a warmer one.

    This is flat out wrong. Photons do not know where they’ve come from. Photons do not pass harmlessly through objects cooler than their source.

    You are right in saying that not all photons are the same. They may differ in frequency (i.e. the quantum of energy they represent). This in turn affects which particles, atoms, or molecules will or will not absorb or emit the photon.

    But it has nothing whatsoever to do with temperature or heat. Nothing at all.

    Now, if you are talking about a huge mass of photons, then you can talk in aggregate. Given two masses of the same substance, so that both will emit and absorb in the same frequency, you can safely say that the warmer mass will emit more and absorb less than the cooler mass, so that the net effect will be the transfer of heat from the warmer mass to the cooler mass, through radiation.

    But in that process, there will be photons that are emitted by particles in the cooler mass, and are absorbed by particles in the warmer mass. It will simply happen more often the other way around, so that the net effect is for the warm mass to pass heat to the cooler mass.

    10

  • #
    RJ

    @ 64


    But it has nothing whatsoever to do with temperature or heat. Nothing at all.

    ? This seems a very strange comment

    Are you saying for example photons are the same from the sun as from the earth.

    10

  • #
    RJ

    But in that process, there will be photons that are emitted by particles in the cooler mass, and are absorbed by particles in the warmer mass.

    But these photons will not having a warming impact. At least according to the slayer book. And others including Postma in the previous thread as noted above (if I understand him correctly).

    10

  • #
    KR

    Alistair @ 57

    1. absorption of IR by CO2 is balanced by equal and opposite emission of IR from all directions, the practical result of Prevost’s Law of Interchanges and Kirchhoff’s Law.

    In the absence of collisions, which can transfer that energy elsewhere.

    2. there can be no heating of the atmosphere by that transiently absorbed vibrational energy.

    Absorbed IR increases the vibrational and electron energy levels in the absorbing molecule. Now, relaxation time for CO2 after absorption (before emission) averages 1 millisecond, while at sea level pressure there’s a collision with another molecule ever 1 nanosecond. That means that an excited CO2 molecule collides with ~1000 other molecules before it can emit, raising the average energy of the atmosphere.

    This means that energy gets transferred to/from the surrounding O2, N2, argon, etc, warming the atmosphere. The CO2 molecule won’t emit unless it’s at that energy state for a millisecond or more, which means that CO2 emission is proportional to the temperature of the atmosphere, not the amount of IR absorbed.

    Your statement is flatly wrong, contrary to observations. And if your hypothesis is contradicted by the observations, it’s time for a new hypothesis.

    3. back radiation is impossible.

    It’s quite well observed and measured for something that is “impossible”. You are incorrect.

    4. anyone who claims anything different hasn’t studied the physics.

    It’s statements like these that cause climate scientists to dismiss skeptics – nonsense and, quite frankly, insults. If you push views like this you will be ignored as a crank. You’re entitled to your own opinions, but not your own facts.

    Jo Nova – Regular posters know that I often disagree with you on various topics. On this one, however, I must applaud your willingness to face off with those skeptics holding fixed opinions contrary to the data.

    I really want to see skeptics at the table – presenting opinions and hypotheses supported by facts. A one-sided debate isn’t a good idea. But promoting nonsense means you won’t be taken seriously.

    10

  • #
    RJ

    @ 67

    I really want to see skeptics at the table – presenting opinions and hypotheses supported by facts.

    But only if they support your beliefs.

    10

  • #
    Alistair

    KR: you have not considered the thermodynamic requirement of the equipartition of energy. That means the proportion of excited CO2 molecules is solely a function of absolute temperature so absorption and re-emission is exactly neutral.

    I too was seduced by kinetic arguments until I went back to my fundamental thermodynamics.

    Climate Science relies for its justification of ‘back radiation’ the incorrect mathematics of Arthur Milne who used the wrong boundary condition when he solved the PDE for IR absorption in the atmosphere. Miskolczi shows it vanishes when you correct the maths.

    10

  • #
    KR

    RJ @ 66

    But in that process, there will be photons that are emitted by particles in the cooler mass, and are absorbed by particles in the warmer mass.

    But these photons will not having a warming impact.

    Then what, pray tell, happens to the energy carried by those photons? Your statement violates the first law of thermodynamics, conservation of energy.

    The IR absorption spectra of the Earth’s surface is almost that of a blackbody in IR, about a 98% chance of absorbing any IR photon that hits it. The atmosphere radiates in IR, therefore the surface absorbs 98% of the photons striking it from the slightly cooler atmosphere.

    Why? Because photons do not carry ID tags.

    The second law of thermodynamics is a statistical law – in total, more energy leaves the warmer object than the cooler object. In exchange, net (total) energy moves from warmer to cooler. But the rate of net energy depends on the differences in temperature – and a cool object nearby will cause less net energy transfer away from (slower cooling of) the warm object than a cold object (like outer space).

    10

  • #
    Nullius in Verba

    “But these photons will not having a warming impact.”

    Photons are all the same in the sense that they all carry energy. The energy is lost by the originating system – irrespective of its current temperature – and is gained by the absorbing system – again, irrespective of its current temperature.

    If a body is at a higher temperature, then more photons will be emitted from it and each one will carry more energy, but they all carry energy away, and that energy ends up in whatever absorbs them, whatever its temperature might be.

    “At least according to the slayer book.”

    And how about according to reality?

    Is the slayer book correct? Sceptics shouldn’t take the answer for granted, just because it argues against AGW. Every photon carries positive energy from the emitter to the absorber. Warm bodies emit photons at a rate and of a size dependent only on their own colour and temperature, and if other bodies absorb those photons the energy they carry ends up in the absorbing body. There’s nowhere else it can go. If the slayer book says otherwise, the slayer book is wrong.

    None of this means that the conventional explanation is correct, it just means that this particular claim isn’t the real reason it is wrong. (If you think about my first post above, with the pond, you might be able to see the real problem with it.)

    10

  • #
    Bryan

    Bob says

    …..”Except for this fact: The peak of the Sun’s output is in the visible where CO2 is transparent. Hence CO2 scatters a very small proportion of the Sun’s energy input to the Earth. However, the peak radiative power from the Earth is in the deep IR, 30 times the wavelength of the visible, due to it’s much cooler temperature than the Sun. Hence CO2 can scatter a significantly larger proportion of the Earth’s total radiated energy than the Sun’s.”……

    Are you still on about reflection?
    Most discussion centres around absorption and emission
    What type of scattering are you referring to.
    Solar radiation is about 47% or so in the IR bands.

    10

  • #
    KR

    Alistair @ 69

    Thermal absorption and emission spectra are equivalent when an object is at thermal equilibrium – not changing in temperature.

    That doesn’t mean that the same molecule will emit that energy, though, just that in the statistical case as many will be absorbed as emitted. And if you increase the IR into a volume of atmosphere, that atmosphere (all of it, not just CO2) will warm until the emission spectra increases to balance it.

    Miskolczi’s work has many issues, all of which are rather off-topic here.

    I will note again that “back-radiation”, or rather the thermal radiation from the atmosphere that strikes the surface, is a very well observed phenomena. See this link for an example. Flatly denying facts like these, well, really doesn’t improve your chances of convincing anyone who is well informed about the physics.

    10

  • #
    KR

    Bryan @ 72

    Solar radiation below 4 micrometers (what is normally considered longwave radiation) is about 0.5% of the spectra. See the spectra here.

    At visible/UV bands Rayleigh scattering is an influence.

    10

  • #
    KR

    RJ @ 68

    I really want to see skeptics at the table – presenting opinions and hypotheses supported by facts.

    But only if they support your beliefs.

    No – only if they match the facts. I’m willing to change my opinions given sufficient evidence. Are you?

    I’m not willing to accept any statement that supports my beliefs, not matter how weak, without testing against reality (that’s confirmation bias). What about you?

    10

  • #
    Alistair

    KR: DLR exists but it has to be balanced by ULR for thermal equilibrium.

    The principle of equipartition of energy means that when a CO2 molecule absorbs a photon, from any direction, another molecule emits one, also in any direction.

    10

  • #
    Alistair

    PS: an idea just popped in. The reason why clouds warm at night is because when you have liquid water they unlike single phase air can store energy at constant temperature via the latent of heat of evaporation as the temperature of the local air increases slightly in LTE with that water.

    But it’s not greenhouse heating!

    10

  • #
    BobC

    RJ:
    May 21st, 2011 at 3:06 am
    @ 64
    This seems a very strange comment
    Are you saying for example photons are the same from the sun as from the earth.

    That is correct. Photons have no identities, only frequencies (hence, energies) and directions of travel. Both the Sun and the Earth emit photons over an infinite range of wavelengths. Look at this plot of blackbody curves from Wikipedia. (Also see here, for a more analytical description.)

    Note that, while the peak wavelength and total power change with temperature, any object above absolute zero will emit photons of every wavelength.

    Any two photons with the same wavelength are identical in every respect (except, perhaps, for their direction of travel).

    10

  • #
    KR

    Alistair @ 76

    The principle of equipartition of energy means that when a CO2 molecule absorbs a photon, from any direction, another molecule emits one, also in any direction.

    Statistically, when a mass at thermal equilibrium receives photons, it emits the same number. But not on an individual photon/photon basis! Not instantly! You don’t get a photon instantly kicked out one side when another comes in. Statistically, conservation of energy requires it.

    DLR exists but it has to be balanced by ULR for thermal equilibrium.

    No – incoming energy (convection, latent heat, radiation) must be balanced by outgoing energy (convection, latent heat, radiation) if a mass is at thermal equilibrium. There is no requirement that the balance of energy exchange be the same – and it is not, in the general case.

    Also note that net energy (heat flow) goes from the sun to the surface to the atmosphere to space (with a side track of sun, atmosphere, space). The surface sends more net energy to the atmosphere than the atmosphere returns.

    I suggest you do some reading up on the physics, Alistair.

    10

  • #
    CHIP

    I don’t know that much about physics (hardly anything) but I’d like to offer my thoughts on this subject. I think where some people are getting confused is that they are applying the second law of thermodynamics to a closed-system and, of course, in a thermodynamically closed-system a cold body could obviously never increase the temperature of a warmer body. However the earth is an open-system and is continually receiving new radiant-energy from the sun which is then emitted from the surface into the atmosphere where it is then reradiated multidirectionally by greenhouse gases, upwards and downwards proportionately. Some photons will make it back down to the earth’s surface to heat it further (albeit this heating I imagine would probably be immeasurably small) and the rest will escape harmlessly into space. Where is the conflict with the second law of thermodynamics in that? I see none. The idea that heat cannot flow from a cold body to a warmer one would be correct if we were dealing with a closed-system, but we are not. And as has been noted before, it applies only to net energy flows. All bodies above -273° emit radiation. To increase the temperature of a system surely you just need to either increase the input or slow the output of energy (as Joanne says). Maybe I have this all skew-whiff, in which case, I would welcome correction.

    10

  • #
    Alistair

    PPS: the logical inference is that the gas phase of the atmosphere is almost instantaneously in thermal equilibrium at all points and the only energy storage is via the phase transition of water, usually combined with convection.

    What we also have is a percolation problem. Miskolczi is part way there but more needs to be understood.

    it seems that back radiation is a primitive belief system of people with too little faith in or too little knowledge of science!

    10

  • #

    Okay, how about this:

    The Greenhouse Effect Simplified Diagram

    This is a very simple diagram of energy in and out.

    Energy in from the sun (yellow) equals 4 units.

    The earth is warmer than it should be (288K instead of 255K in real life, or 5 units instead of 4 units in our simplified case), so it has a temperature of 5 and emits 5 units of energy. Orange represents the energy that escapes unimpeded into space, while red represents the energy intercepted by the atmosphere.

    The atmosphere is cool, with a temperature of 2, and emitting 2 in our simple case, but because of its position in the system, it emits both “up” and “down” (or “in” and “out”).

    Note that of the 5 units emitted by the earth, 3 make it out into space uninterrupted. The atmosphere is transparent to this radiation. The other 2 are absorbed by the atmosphere, and then radiated back, 1 up, 1 down.

    Note that everything balances.

    The earth-atmosphere system as a whole received 4 units in and sends 4 units out.
    The atmosphere receives 2 units in and sends 2 unites out.
    The surface of the earth receives 5 units in and sends 5 units out.

    Note also that net heat is passing from the warmer object (the earth) to the cooler object (the atmosphere). The atmosphere is not warming the earth. On the contrary, the earth is warming the atmosphere. Heat flows from warm to cold, but that flow is not uni-direction in part, only in aggregate.

    And yet the earth has a temperature of 5 units, not 4.

    10

  • #
    RJ

    @ 70

    Then what, pray tell, happens to the energy carried by those photons? Your statement violates the first law of thermodynamics, conservation of energy.

    Here’s my response on the other thread @ 413

    OK I think I have found the answer to my question above in the slayers book ch 18 Claus Johnson

    An ideal blackbody absorbs all incoming radiation and remits all absorbed energy below cut off.

    Conservation of energy requires absorbed frequencies above cut-off to be stored in some form, more precisely as heat energy thus increasing the temperature of the blackbody.

    So the radiation would initially be absorbed but this would not raise the temperature of the warmer body.

    10

  • #
    KR

    RJ @ 83 – “the radiation would initially be absorbed but this would not raise the temperature of the warmer body.”

    Then where does this energy go? The ninth dimension? Into the Tardis?

    Again – your statement violates conservation of energy. The energy absorbed must be somewhere – and that somewhere is in vibrational and orbital energy, also known as increased temperature.

    Really, RJ – photons have an energy but no ID cards.


    Here’s a couple of questions for you, RJ:

    If a 15C object (like the ground) has a 98% change of absorbing a 10 micrometer photon, and is struck by a 10 micrometer photon from an ice cube, what is the probability of absorption – (hint, 98%).

    If a 15C object (like the ground) has a 98% change of absorbing a 10 micrometer photon, and is struck by a 10 micrometer photon from a pot of boiling water, what is the probability of absorption – (hint, 98%).

    10

  • #
    RJ

    KR @ 75

    I’m willing to change my opinions given sufficient evidence.

    Here’s chapter 18 from the slayers book. In PDF format. It expands on my post @ 83. Please read with an open mind though.

    http://www.slayingtheskydragon.com/en/sample-chapters/98-computational-blackbody-radiation

    10

  • #
    BobC

    Sphaerica (Bob) @82:
    May 21st, 2011 at 4:29 am

    Okay, how about this:

    The Greenhouse Effect Simplified Diagram

    There is nothing unphysical about this at all — energy trapping cavities are extremely common and uncontroversial:

    1) All resonant devices, from optical etalons to organ pipes, to kids on a swingset can achieve higher internal energy density than the driving external energy density, by storing the energy temporarily.

    2) Your diagram is exactly analogous to the “heater in a blanket” argument we’ve heard here, if you substitute the incoming solar energy for the electrical wires bringing power to the heater, and consider the atmosphere as the insulator.

    It is also conceptually similar to keeping warm by wearing a down jacket. The jacket is definitely colder then your body, but keeps you warm nevertheless. Your body has an internal heat source, but the Sun is injecting energy into the Earth through a short-wave transparent window in the atmosphere, so it amounts to the same thing.

    10

  • #
    BobC

    Bryan:
    May 21st, 2011 at 3:25 am

    Are you still on about reflection?
    Most discussion centres around absorption and emission
    What type of scattering are you referring to.

    Absorption of a photon, followed by emission of a photon of the same energy IS a form of scattering.

    10

  • #

    BobC,

    Exactly.

    The point is that the diagram clearly shows that the cooler atmosphere is not heating the warmer surface. Energy exchange is taking place in both directions, but the net is always in the right direction (from warm to cool), in keeping with the second law of thermodynamics, and yet the surface of the earth is warmer than it would be without an atmosphere (and specifically one with greenhouse gas properties).

    20

  • #
    KR

    RJ @ 85

    Sadly, that did not take long.

    A theoretic blackbody, as they state, is such that:

    “…all frequencies are being absorbed.” (emphasis added)

    The misstatement comes a couple lines later, with:

    “Conservation of energy requires absorbed frequencies above cut-off to be stored in some form, more precisely as heat energy thus increasing the temperature of the blackbody.” (emphasis added)

    This is incorrect – conservation of energy requires that all absorbed photons of all frequencies be stored in some form, more precisely as heat energy increasing the temperature of the blackbody.

    Then:

    “A blackbody thus absorbs and emits frequencies below cut-off without getting warmer, while absorbed frequencies above cut-off are not emitted but are instead stored as heat energy increasing the temperature.”

    Completely, utterly false.

    All photons carry energy. All absorbed photons add to the energy of the blackbody. The blackbody (at equilibrium) will radiate a thermal spectra with a power equal to the total absorbed energy. There is no separation between absorbed/warming and absorbed/radiated – conservation of energy holds, and the total energy absorbed must be emitted – not identical photons, but an amount carrying power equal to what is absorbed at all frequencies. Please read the black body Wiki – it’s a decent overview.

    Next, Clas Johnson attempts to supplant quantum mechanics, introducing a function simply to separate energy into coherent waves (emitted) and incoherent (warming) frequencies. Aside from the hubris of supplanting quantum mechanics (the basis of so much, including the computer I’m writing upon) due to a dislike of the results, it is noteworthy to point out that emitted thermal radiation is incoherent, contradicting this mechanism.

    Here’s a clear counterexample for you, RJ: CO2 lasers (in the IR) burning through materials – found all through industry. Surely by Clas’s physics all that light would be below the cutoff, each photon too low in frequency, and just re-emitted, rather than burning holes in stuff? Hmm???

    The rest is more of the same…

    Given this example of Clas Johnson’s physics, I will have to say that I’m quite inclined to dismiss the entire book as bull****. This section certainly is.

    10

  • #

    RJ,

    I glanced at the chapter. I’ve never seen such drivel in my life. You are going to be and stay hopelessly lost as long as you use that book as a source of “knowledge” about science. We’re not even talking climate science here, but just plain, ordinary physics.

    10

  • #
    KR

    RJ

    I completely agree with Sphaerica (Bob) on this topic. This book is nonsense, pure and simple. Please – find a textbook or something, and learn some physics from that.

    You’re basing your opinions on a physics that Alice in Wonderland would find hallucinatory – you’re not going to get far with that.

    10

  • #
    RJ

    KR

    Did you note this section from my post above.

    Please read with an open mind though.

    10

  • #
    RJ

    @ 89


    All photons carry energy. All absorbed photons add to the energy of the blackbody.

    Only if a cooler body can heat a warmer one. And it can not. Its a false belief that some will not let go. But the tide is turning and quickly.

    10

  • #
    BobC

    Alistair:
    May 21st, 2011 at 4:26 am

    PPS: the logical inference is that the gas phase of the atmosphere is almost instantaneously in thermal equilibrium at all points and the only energy storage is via the phase transition of water, usually combined with convection.

    So, radiation in transit is not stored energy, but heat in transit is? What a surprise to optical physicists who have been utilizing radiation in transit storage (in etalons) for 150 years!

    it seems that back radiation is a primitive belief system of people with too little faith in or too little knowledge of science!

    (There you go again, with these ‘fun’ non-arguments.)

    So, if I had enough “faith” in science, I would not automatically assume that radiation scattered ‘back’ toward the first emitter was ‘back radiation’, but would have a choice of other obfuscatory labels for it.

    What would you label the radiation emitted from the atmosphere toward the ground in Sphaerica (Bob)’s model @ 82? And why would any label change the effect it has of increasing the internal flux (and hence internal energy storage)?

    10

  • #
    Bryan

    It seems odd that the only people who seem happy with Michael Hammer two posts are rampant CAGW advocates like KR.
    He is having a field pretending he knows some physics.

    He is insulting sceptics in every post and now he will decide who are “approved sceptics” whose ideas can be tolerated as long as they dont conflict with IPCC guidelines.

    For the record KR admits he knows very little about thermodynamics and there is little reason to doubt him.
    A number of sceptic posters from the original thread are missing.
    No doubt fed up with the KR tactic of constantly repeating the same nonsense hoping repetition might somehow make readers give up out of sheer boredom.

    People who are in any doubt about the real physics should read the earlier thread where Leonard Weinstein, Postma and others made their contribution as there has been very little new insights presented here.

    10

  • #
    BenAW

    UPDATE #1 From Michael Hammer

    4. In the absence of green house gases the earth is surrounded by an extremely cold object – outer space at 4K (-269C)

    Never been there, but afaik outer space is an allmost perfect vacuum. Don’t think you can assign a temperature to “nothing”. 4K (2,75K?) is probably the Big Bang backgroundradiation?
    I understand a big problem for NASA is to shed excess heat from spacecraft, because radiation is the only way, and isn’t very efficient in doing so.

    10

  • #
    BobC

    Bryan:
    May 21st, 2011 at 8:20 am

    It seems odd that the only people who seem happy with Michael Hammer two posts are rampant CAGW advocates like KR.

    Bryan, I don’t see anything wrong with Hammer’s posts (based on my degrees on physics, math and optical engineering). On the other hand, I’m not ready to say he is perfectly correct (or, if he is, what he calculates is relevant to the actual climate system of the Earth).

    I still think CAGW is BullPuckey, however. CAGW hinges on the existence of never-ever seen positive feedbacks in the climate system — which, if they existed, would have rendered the Earth uninhabitable a billion years ago. It does not rest on the exact physics of GHGs.

    10

  • #
    Bryan

    BobC says

    …”Bryan, I don’t see anything wrong with Hammer’s posts (based on my degrees on physics,”..

    So which university taught you that heat can flow spontaneously from a colder surface to a warmer surface?

    10

  • #
    KR

    RJ

    Yes I read that line. I also read the chapter you linked to.

    While its important to keep an open mind, its equally important that it not be open at both ends.

    Clear violations of extremely basic physics… That is a sign of a book that isn’t worth reading. In this case, not worth printing.

    10

  • #
    L.J. Ryan

    Claes Johnson?…Is he the one whom claims insulating a -18C radiator will raise it’s temperature to 15C? Claes Johnson?…Is he the one whom makes contradictory statement about the atmosphere forcing only slowing loss, then says 240 W/m^2 solar input is increased to 390 W/m^2 as a result of the various interactions in the system (reflection, absorption, greenhouse effect, etc.).

    Oh wait Claes Johnson doesn’t reserve such foolish tripe…ownership of this revolutionary thinking belongs to KR and Sphaerica (Bob). Talk about hubris.

    10

  • #
    Craig Thomas

    Like Venus, Bob?

    10

  • #

    OOps Posted this to the wrong discussion first

    An Interesting Approach here:

    http://www.biocab.org/Density-of-Energy-in-Atmosphere.html

    Which contains some graphs and calculations which Leads the Author to this conclusion:

    “The warmhouse effect, or “greenhouse” effect, is not caused by the gases composing the atmosphere, but by the inversion of the decline of sensible heat flux with respect to the decline of the thermal energy density in the atmosphere. As the sensible heat flux increases as the energy density diminishes, the warmhouse effect happens. Conversely, as the sensible heat flux decreases simultaneously with a decrease of the energy density, the warmhouse effect terminates. Therefore, any increase of the sensible heat flux in the atmosphere follows from an increase in the incident solar energy on the Earth’s surface; consequently, any increase of the incident solar radiation on the Earth’s surface implies an increase of the energy density of any mass of air. Otherwise, the warmhouse effect, or “greenhouse” effect, would be impossible since energy is neither created nor destroyed, but only transformed.”

    Obviously you have to have an atmosphere for this to happen. The guy’s first language is not English so the wording of the conclusion is a bit fuzzy and ambiguous. Just excuse this and read the article before commenting on its substance.

    Love to see some well reasoned discussion on this one Jo.

    10

  • #

    And here is another one to ponder:

    This is an experiment where the researchers were experimenting to optimize a sun powered cooker but found you could also make it cool by pointing to a different place in the sky. Point it to the sun it cooks point elsewhere it can cool.

    http://solarcooking.org/research/McGuire-Jones.mht

    Oh boy this can’t be right – lets see, didn’t we learn here: The sun heats the ground, the hot air rises, it radiates back to the ground, the ground gets hotter, the hot air rises, it radiates back to the ground, the ground gets hotter…… MMmm, must be Utah liquor laws.

    10

  • #
    Kevin

    Mr. Hammer, with respect,

    You do not have a comprehensive understanding of how heat/radiation flows through any system. Also, if you paid any significant amount of money for your “thermodynamics” training you should immediately seek a full refund of your money.

    You wrote;

    “What the so called green house gases are doing is to reduce Earth’s heat loss to outer space at some wavelengths”

    Any engineer that has actually worked on “thermal management” (yes, this is a real engineering specialty) will tell you that is is IMPOSSIBLE TO REDUCE HEAT LOSS, if they are at all worth their salary they will inform you that it is only possible to SLOW (i.e. insulation) or HASTEN (i.e. substituting steel with copper, or heat pipes) the speed at which heat flows through a system. All heat is eventually LOST, it is only a question of how long it takes to LOSE it, and can it be replaced faster than it is LOST. ONLY if you can replace it faster than you lose it can you maintain a higher temperature.

    Sorry to be rough, but you obviously have not designed any system that attempts to maintain a controlled temperature at a location. I have, and your observations are from my experience totally flawed.

    Cheers, Kevin.

    10

  • #
    DavidA

    Whacky Physics Experiment # 87

    We get 1 million lasers and direct them all at one point on a body who’s initial temperature is above the absorption threshold of the laser’s photons.

    Despite having 1 million laser beams focussed on one spot there is no warming because no individual photon exceeds the absorption threshold. We can crank this up to 2,3 10,100 million, 1 trillion, infinity lasers and there is still no warming.

    The body is cooling and eventually it cools to a level where photon absorption occurs and then BOOM the energy of 1 million lasers is absorbed all at once.

    (or is it? each absorbed photon raises the temperature, when temperature exceeds the threshold absorption is stopped, equilibrium is maintained, maybe. rather than pinpoint we coat a sphere with laser beams… Whacky Physics Experiment # 88)

    10

  • #
    DavidA

    Kevin are you maybe getting just a bit too hung up on semantics?

    ” reduce Earth’s heat loss ”

    If heat loss is slowed then heat loss is reduced over the course of time. For example, over the course of a night overall heat loss is reduced if the atmosphere slows the rate of heat loss.

    10

  • #
    KR

    DavidA @ 105

    Thanks for that post – still laughing! I tried the same in @89, but many folks seem not to recognize a reductio when struck over the head with it.

    Jo Nova – good luck with this crowd. You may need it…

    10

  • #
    DavidA

    KR amongst this crowd it’s a better to have more thumbs down than up!
    \

    [I hear you DavidA, but I ticked your #106 up – sorry:-) ! — JN]

    10

  • #
    IAmDigitap

    It is just as easy as pie to check if there’s any additional heating due to CO2.

    CO2 has risen in the past years. If optical telescopy shows no additional, chartable rise in distortion in the atmosphere, all your claims about how entropy blinked, and a well mixed, highly turbulent frigid bath heating a warm rock are where they were when you tried it at first: the waste bin.

    HEAT on GAS is DIRECTLY CORRELATED to MOTION of that GAS’s ATOMs.

    IF THAT GAS isn’t MORE ENERGETIC there is NO MORE HEAT in the ATMOSPHERE, and since the optical telescopy field hasn’t had people come up with the proof of additional turbulence,

    and since the I.R. TELESCOPY FIELD hasn’t come trotting out THEIR proofs of rising I.R. pollution: all that claim about “I HAZ DUN CALCULATED ENTROPY DON’T WORK and a FIFTEEN BELOW BATH warmed a HOT ROCK” is made as evident to those who can’t count, as to those who can.

    There can BE no overall heating by the atmosphere, and NO JOANN less cooling is NOT heating, no matter how upset people get, that they can’t make HEAT transfer in REVERSAL of E.N.T.R.O.P.Y.

    Here’s how you can check if someone works transmitting, capturing, and analyzing atmospheric electromagnetic radiation: if they work in my field they don’t believe in a G.H.G. Effect because THE EFFECTS of RADIATION are STRAIGHTFORWARD.

    No matter HOW many NON RADIATION SPECIALISTS CALCULATE MAGIC GAS,

    THERE is a R.E.A.S.O.N. YOU CAN NOT FIND an INSTRUMENT on EARTH to MEASURE said MAGICAL PROPERTY.

    Because you have been DECEIVED.

    10

  • #
    IAmDigitap

    You must always ask yourself joann if there are instruments that pick up some magical force claiming to reverse entropy – not to mention, gravity.
    Joann are you aware that gravity is the sole driver of photonic energy on gas, climbing upward from the earth, ever higher?
    This effect is derived from g.r.a.v.i.t.y.
    To make a photon emit downward while it is furiously dragging an entire gas molecule, up
    Is bizarre of itself: on it’s face you should have been suspicious, then.
    Then when they told you they calculated a frigid cooling bath was heating a rock submerged, you should have been suspicious, then.
    Then when they told you there isn’t an instrument in the world that can measure it,
    T.h.e.y. L.i.e.d. To y o u joann, because all you have to do, is look at the stars twinkling over your head.
    If that twinkling distortion has grown along with g.h.g. In the atmosphere, you’re on.
    If not you’re done because heat on gas is motion.
    P.e.r.i.o.d.
    Not “i already made up my mind” not “yea but!”
    If there is no more motion there is no more heat and if there is no more heat when there is more g.h.g. Co2
    Then you have been deceived by fools who thought their pressure would make you admit to a 15 below fluid bath
    Heating
    a warm rock.
    Ask yourself joann how it is that you got snookered into believing calculations that show you entropy reversed, and instead of thermal energy spreading out to seek equilibrium,
    Charges move from less concentrated, to more.
    Joann if you want to know if there’s a g.h.g. Effect, ask yourself why this magically unmeasureable effect doesn’t even make the stars twinkle more at night.
    You know that heat on gas means motion.
    Why aren’t the people who magnify atmospheric heat anomalies thousands of times through optical instruments,
    Coming forward with reports of ever more motion?
    There’s ever more co2 joann.
    So why not ever more motion?
    Because : you’ve been conned by people who thought they calculated the end of e.n.t.r.o.p.y. And not just entropy but gravity.
    Because if that radiation has turned around those gas atoms and shot downward, it had to have some way to know which way ‘down’ was, and these people have simply gotten away with telling you that
    Photons on gas in gravity move down, and that photons on gas in gravity cause z.e.r.o. Motion of that gas.
    Wow so now you believe in the magical gas that heats up but doesn’t move more.
    Joann never: not ever: side with people who tell you they calculated entropy being reversed.
    That’s not ever going to win.
    N.ot
    o.nce
    n.ot
    e.ver.
    Think about it.
    No instrument on earth?
    Reversal of gravity?
    Reversal of entropy itself??????
    Yea right and i’ve got a magic bridge to sell you that you can’t see, can’t measure, but it’s there. The fact your car fell trying to drive on it just shows how little you know about bridges????????????
    Oh palease.

    10

  • #
    L.J. Ryan

    Joanne Nova and Michael Hammer 46

    4. In the absence of green house gases the earth is surrounded by an extremely cold object – outer space at 4K (-269C). Green house gases make the atmosphere opaque at some wavelengths. With these in place the earths surface is in effect surrounded by a merely cool atmosphere instead of the truly frigid outer space. Because the surface is now surrounded by a less cold object than it was before it is less cold.

    5. Since warm and cold are opposites, less cold is the equivalent of warmer. Surrounding earth’s surface with an opaque cool atmosphere make it warmer than it would be if exposed directly to the ultra frigid outer space.

    re 4) This assumes no other energy/temperature inputs exists. Pressure has a very well understood, uncontroversial relationship with temperature. To the extent CO2 plays a role in warming the earths surface, it is CO2 additive mass. That aside, you supposition has a larger hurdle to clear.

    re 5) If you stand amongst ice blocks, you body struggles to maintain 37 C. The temperature loss surrounded by CO2 blocks will be faster. On this we agree. However, if you were to move from blocks of CO2 to blocks of H2O your body would not increase beyond 37 C. Similarly, the earth input energy 240 W/m^2 confers -18 C. Larger hurdle The earth struggles to maintain -18 C with no GHG…now introduce GHG, the earth struggles less to maintain -18 C, it does not however shoot up to 15 C.

    10

  • #

    Joseph Postma has replied but it was caught in the spam and only just released at way back at #47. I’ve copied his whole reply to the bottom of the post (Update #2) and replied in line. See the update.

    I want to simplify the argument, and isolate one point – the second law – so we can hopefully move on to other points that may be valid.

    10

  • #

    This is a topic visited on Roy Spencers Blog site.

    http://www.drroyspencer.com/2010/07/yes-virginia-cooler-objects-can-make-warmer-objects-even-warmer-still/

    I believe the following is the killer argument on this issue, But would someone like to muddy the water, by telling us that vectors have both magnitude and direction and both the emitting gas and the surface would be directionally noisy and suggest a calculus that resolves the potential impulse leakage problem if there is one.

    Irwin Hasenwinkle says:
    July 30, 2010 at 6:38 AM

    Dear Dr. Spencer,

    In your post of July 23rd , “Yes Virginia Cooler Objects Can Make Warmer Objects Even Warmer Still”, you made the following statements: “…radiation flow involves energy flow in both directions. “ and “Can Energy Flow Uphill? In the case of radiation the answer to that question is, yes”. With all due respect, I think these comments are not correct and suggest the following for your consideration:

    The Poynting Vector characterizes the power of any electromagnetic wave, including infrared radiation. The Poynting Vector is the cross (or vector) product, ExH, where E and H are the associated electric and magnetic field vectors.

    Assume two plane-polarized electromagnetic sinusoidal waves propagating in exactly opposite directions. If each wave is considered separately, the scalar magnitudes of the Poynting Vectors for each at any given point in space is [1]

    P1=(E1^2)(cos^2(wt))/n

    And

    P2=(E2^2)(cos^2(wt))/n

    Where:

    E1, E2 are the peak magnitudes of each of the two electric fields in Volts per meter.
    n is the intrinsic impedance in ohms.
    w is the frequency in radians per sec.
    t is time in sec
    P1, P2 are the values for power in Watts per square meter

    The direction of each Poynting Vector can be shown to be in the direction of propagation.

    The questions are: If both waves occupy the same space, can one treat these two Poynting Vectors as if they are completely independent so that energy flows both directions? If so, what is the net energy flow? If these waves were truly independent, would not the net energy be the vector sum of the respective Poynting Vectors?

    But wait, the electric and magnetic field components of the two waves are also vectors. Hence, the actual value of the electric or magnetic field at any point in space is the vector addition of the individual field values. For electric fields that are exactly opposite, this value is E1 – E2. The scalar value of the Poynting Vector for this resultant field is:

    P2=((E1 – E2)^2)(cos^2(wt))/n

    This result is clearly not the same as P1 – P2, the vector addition of the individual Poynting Vectors, since E1^2 – E2^2 does not equal (E1 – E2)^2.

    Note that the direction of the latter Poynting Vector is based on cross product of E and H, the vector sums of the individual electric and magnetic field vectors. It has but one direction!

    Which is correct? In my opinion, the Poynting Vectors (i.e. power flow) calculated from individual electromagnetic waves cannot be treated independently. This is analogous to the superposition theorem in circuit theory where total current flow of a linear circuit is calculated by considering each voltage source in the circuit separately and then by summing the results. One may not, however, sum the losses produced by each of these calculated currents to find the total power dissipated in the circuit since power losses are based on the square of the current. Similarly, Poynting Vectors are based on the square of the electric field.

    Thus, in the case where a hot body and a cold body both emit electromagnetic radiation, the direction of electromagnetic energy flow will be from the hot to the cold body since the hot body will produce a stronger electromagnetic field. Energy does not flow both ways. That being the case, energy cannot “flow uphill” and the second law of thermodynamics remains intact.

    [1] Skilling, H. H., Fundamentals of Electric Waves, p135, John Wiley and Sons Inc., 1948

    10

  • #

    Re: Jo’s comment #18

    It gives alarmists good reason to mock, laugh and distract us from the science that matters.

    Hi Jo, in science one must never fear being mocked or ridiculed or going against the “consensus”. In fact progress in science more often than not requires this; surely?

    When it comes to electromagnetic radiation the “speed of heat flow” is isn’t slowed down by the presence of other, colder objects or even of warmer ones.

    The most that can happen, for example, to change your temperature is to have a warmer object warm you and then you will increase in temperature and emit EMR energy proportional to T^4. Thus, heat loss is never ever slowed down in the case of EMR.

    Stuff just mutually radiates to each other. That’s the natural state of the Universe. You can’t get energy from that.

    The EMR that is lost from an object occurs at a rate, and transfers heat at a rate, only related to its temperature and not to the temperature of other objects. Period.

    10

  • #
    RJ

    Another great response by Joseph. And I look forward to the next paper.

    10

  • #
    cohenite

    Yes, KR, is on a role, but he ignores the hard tasks; I am still waiting for a proper response to my queries at 402 and 474 in the previous thread:

    http://joannenova.com.au/2011/05/why-greenhouse-gas-warming-doesnt-break-the-second-law-of-thermodynamics/#comments

    And at 34 on this thread. More pertinently and following on from CO2ISLIFE @114 I post this excellent explanation of the 2nd Law and heat transfer for KR’s edification.

    Microstates and the Outer Space

    To properly talk about microstates, we need that any amount of matter is present in a given medium. We cannot talk about microstates if we have not, at least, one Hydron (H+) in a given medium.

    A microstate refers to any initial of final configuration of the energy in a given system.

    The Second Law of Thermodynamics, although initially was derived from the observation of thermal processes, has been proven to be acting on every level of energy exchange between two or more systems.

    Initially, the Second Law was described in terms of the directionality in the flow of the energy in transit (a process function), which depends on the states of the systems involved in the exchange of such energy in transit. The Second Law clearly specified that the work only can be done by a higher energy density system on a lower energy density system and not the opposite.

    However, with the advent of Quantum Physics, the scientists wondered whether this Law was valid at the quantum level or not. The answer to this question was given heuristically through the calculations of Maxwell, Boltzmann and Gibbs. The heuristic character of the calculations vanished when those hypotheses were later confirmed by experimentation.

    In consequence, the definition of the Second Law was amplified to include its influence on the quantum level and not only on those process functions where heat and work were implied.

    This shift was important because it defined the real concept of entropy and detached it from contextual derivations. For example, now we know that the fundamental concept of entropy has nothing to do with disorder, movement, complexity or heat “content”, but with the configurations that the energy adopts in a given system and the directionality of the energy exchange.

    Entropy is now defined as the natural trend of the energy to flow towards the system or systems with a higher number of available microstates.

    Let us say that two systems permit six configurations of the energy. One of them, let us say the system A, has four “occupied” configurations and only two available configurations. The other system, or system B, has only one “occupied” configuration and five available configurations. According to the Second Law of Thermodynamics, the energy will flow spontaneously from the system A to the system B and never the opposite.

    Perhaps, you are wondering if the energy could flow from B to A during the process. The answer is no because two systems implied in an energy exchange process cannot adopt the same configuration at once, although any system could adopt any configuration.

    To calculate the number of microstates that a system can adopt, we resort to the following formula:

    Nms = N! / (n1! * n2! * n3! …)

    Where Nms is the number of available microstates (Maxwell-Boltzmann Number), N is the number of particles, and n is the number of particles in a determined occupied microstate. For example, we have a system A that have six particles from which four are in the microstate 0E, one is in the microstate 3E and one is in the microstate 5E. The number of available microstates for system A is:

    Nms = 6! / (4! * 1! * 1!) = 720 / 24 = 30

    Then, 30 is the number of available microstates for this system.

    Let us consider a system B with the same number of particles (six) and the same number of levels of energy, i.e. six, but where each particle is occupying a level of energy, i.e. one particle at level 0E, one particle at level 1E, one particle at level 2E, etc. The solution is as follows:

    Nms = 6! / (1! * 1! * 1! *1! * 1! * 1!) = 720 / 1 = 720

    This system offers more available microstates, that is, more configurations to be adopted by the energy in a radiation process. Therefore, the radiation will flow from system A, with 30 available microstates, towards system B, with 720 available microstates.

    What about the outer space, where there is only one particle per cubic meter? Is it possible that it has more available microstates than the massive Earth?

    All the particles in the deep space are in their basic configuration, that is, there are no particles occupying any level of energy, but only high speed protons, therefore:

    Nms = 6! / (0!) = 720 / 1 = 720

    Consequently, the radiation trajectory will be always from the Earth towards the deep space, the most efficient sink of radiation of any kind. Notice that it has nothing to do with temperature, disorder, complexity, etc.

    Kevan Hashemi asked Cohenite if a particle at 300 K will or will not emit photons. Any particle at 300 K is at its fundamental energy state, i.e. its available microstates will be higher than those of any particle with a temperature above 300 K. Such particle won’t radiate, but it will absorb energy.

    By Nasif S. Nahle, Scientific Research Director at Biology Cabinet Mexico

    10

  • #
    Phillip Bratby

    I agree entirely with Joseph Postma’s post @47. Physics rules. End of story for the theory of the “greenhouse effect”.

    10

  • #
    Louis Hissink

    I’ll second Phillip Bratby’s summation and the following link adds more to the argument by going back to the times of Fourier and those who subsequently misinterpreted him.

    http://greenhouse.geologist-1011.net/

    There is no such thing as a greenhouse gas effect. Period.

    10

  • #
    Nullius in Verba

    “The Sun can only warm the planet to whatever its insolation temperature is.”

    How about if you use a magnifying glass?

    10

  • #
    Louis Hissink

    Nullius,

    A magnifying glass of a scale large enough to affect the thermal state of the earth in toto?

    Incidentally your pond analogy above is a bit wet – the black liner doesn’t heat up by radiation but by convection – radiation only warms things up in a vacuum.

    Which breakfast weeties brand do you get your science from?

    10

  • #
    Phillip Bratby

    Louis Hissink
    Thanks for that link. It gives a good historical summary of where the science behind the “greenhouse effect” all went wrong.

    10

  • #
    Nullius in Verba

    Louis,

    I was asking about the physics of magnifying glasses. Postma isn’t claiming it applies to the whole earth either – just the bit out in the sun.

    “the black liner doesn’t heat up by radiation but by convection – radiation only warms things up in a vacuum.”

    That’s nonsense. Radiation only heats things if they’re in a vacuum?!

    Ad hominem comments about expertise in physics or its lack detract from the credibility of your argument. They detract exponentially more if you make basic errors in the same post you use them.

    10

  • #
    Louis Hissink

    Jo,

    I missed your concluding paragraph to Joseph’s statement.

    “JN: Joseph, you can’t quantitatively put to a rest an argument of reason which is not based on logic. The reasoning underpins everything. Maybe CO2 doesn’t heat the Earth (for other reasons unrelated to this post), but if that’s the case — it’s not anything to do with “Laws of Thermodynamics”. Can you at least acknowledge that in the Sun-Earth-Atmosphere system it is possible that energy flows which are always going from the hot sun to cold space, could be changed by a colder entity (GHGs) which would allow a warmer entity (Earth) to get even warmer, given that a third source (the Sun) is continually adding energy?”

    I’m not sure about the idea that a cold space could be changed by an even colder entity (GHG) is valid, let alone logical. The fact is that the GHG’s are warmer than space, so your proposition that they are colder is not supported physically.

    The argument is strictly rhetorical, not physical, since physically we know that cool bodies cannot warm hotter ones, but the argument is pursued on the basis that there is measured downwelling IR in the atmosphere, and that this is believed to be solely due to radiating gases.

    If I have 2 bodies A and B, A being a hot body, and B being space, then there will exist a temperature profile that decreases from A to B. If I then interpose a third body C between A and B, and make it hotter than B, but cooler than A, then will C affect the temperature of A? More importantly will the presence of C affect the cooling rate of A, meaning A will cool slower than if C were absent?

    Only under one circumstance – if C itself is an additional energy source.

    As C can be considered to be a GHG, then the thermal profile A to B, and the profile A-C-B will be identical, but not if C adds energy itself.

    The principal error made in the GHG idea is that a gas behaves as an electrical capacitor, that it can store energy – no gas can – its thermal state is instantaneously transmitted to its neighbours.

    But the crux of your argument seems to rely on the assumption that GHG are sources of additional energy, when in reality they are not.

    10

  • #
    Louis Hissink

    Nullius,

    Nonsense? Transfer of energy within solid matter is via Brownian motion – but transfer between the sun and earth is via radiation since there is no matter in between.

    And my criticism remains.

    10

  • #
    Nullius in Verba

    Louis,

    Brownian motion applies only to fluids (strictly, particles suspended in fluids), not solids.

    I agree that energy transfer between the sun and Earth is by radiation, but that doesn’t imply that energy transfer by radiation only occurs in a vacuum. The liner of the pond receives its energy input from radiation. What happens to it then is another matter – and the issues with that were actually the point I intended to make – but given the conditions as stated, that’s what follows.

    Your criticism remains – so far as I can see – that “radiation only warms things up in a vacuum”. How is that a criticism? Or did you mean your criticism that because a magnifying glass does not heat the whole Earth, there’s no point in discussing how it can use solar radiation to exceed Postma’s maximum?

    10

  • #
    Louis Hissink

    A slight logical error in the earlier post, but I’ll wait for anyone to pick it up.

    10

  • #
    Louis Hissink

    Nullius,

    The pond liner does not warm up from radiation – it is warmed by convection, or if a solid or liquid, conduction. Brownian motion is only applicable to gases. The idea that it warms from the incidence of radiation is an impression of what some believe is occurring.

    That the pond ultimately receives its energy input from radiation is strictly correct, but to then use that fact to support a proposed greenhouse mechanism is problematical.

    Postamas’ maxium temperature? Haven’t raised that.

    10

  • #
    RJ

    radiation only warms things up in a vacuum.

    This statement is a bit misleading though isn’t it. If the earth is in a vacuum what about objects on the earth. Am I in a vaccuum?

    10

  • #
    Louis Hissink

    RJ,

    Hardly misleading – you are part of the earth-system, so unless you believe you are a satellite or disembodied in space, you cannot be in a vacuum.

    10

  • #
    Nullius in Verba

    “The pond liner does not warm up from radiation – it is warmed by convection,”

    OK, just for the sake of argument – suppose we stop convection by filling the pond with saline solution of varying density – the densest at the bottom, so that normal convection is inhibited. By your argument, the bottom of the pond would not be warmed?

    10

  • #
    Nullius in Verba

    Oh, and Brownian motion applies to liquids as well. Brown was studying pollen suspended in water.

    10

  • #
    RJ

    Hardly misleading – you are part of the earth-system, so unless you believe you are a satellite or disembodied in space, you cannot be in a vacuum.

    I’m confused then. If I’m in the sun doesn’t the sun’s radiation warm me up.

    10

  • #
    Bryan

    Nullius in Verba says

    “The Sun can only warm the planet to whatever its insolation temperature is.”

    How about if you use a magnifying glass?

    I say;
    Nullis I had always thought that you were a serious thinker.
    How can you justify a post like that?

    Its the kind of juvenile point scoring that KR indulges in.
    It can be easily rebutted but whats the point?
    It doesn’t move the topic of the second law along any further forward.

    10

  • #
    Baa Humbug

    Electro Magnetic Radiation travel in WAVES. Interaction of EMR Waves with molecules is nothing like micro marbels (photons) whizzing around at light speed being swallowed up and spat out by molecules.

    A molecule ALREADY EXCITED BY SHORT WAVES CANNOT BE FURTHER EXCITED BY LONGER WAVES.

    10

  • #
    Nullius in Verba

    Bryan,

    It has a serious point. Postma’s calculation of a maximum temperature is based on solar radiation coming from only one direction. The effect of a magnifying glass is to redirect radiation from a wider range of directions. If one were to extend this to input sunlight from every direction – using parabolic mirrors as well, say – the temperature would rise to the actual theoretical maximum which is the temperature of the surface of the sun – about 6000 C.

    The claim that the sun can only warm the planet to whatever its insolation temperature is seems to be a significant part of Postma’s thesis. I had hoped that someone would respond seriously by saying that it works by receiving radiation from a wider part of the sky, that the radiative input is based on the average temperature over the entire 360×180 degree view, from which we could move on to the idea that other radiation from the rest of the sky could also have a warming effect on an object at the surface.

    But all I get is that because there isn’t a magnifying glass big enough to fry the entire Earth, the question can be dismissed, and that radiation doesn’t heat things unless they’re in a vacuum. What am I supposed to do with that?

    The topic of the second law isn’t going to move forward. You won’t get people to explicitly concede that they were wrong. The best you’ll get is that they’ll start trying to avoid the subject, and distract with spurious nit-picking of other points. But if you want to try to move things forward, please do. Where to next?

    10

  • #
    Bryan

    Nullis

    You I and Postma and most readers know that if you collect parallel rays of light over an area a lens will focus them to almost a point and in this case… “temperature would rise to the actual theoretical maximum which is the temperature of the surface of the sun – about 6000 C.”.

    But if someone posts a short reply they cannot put in endless caveat’s to cover every obscure interpretation.

    10

  • #
    BobC

    Bryan:
    May 21st, 2011 at 8:56 am

    BobC says

    …”Bryan, I don’t see anything wrong with Hammer’s posts (based on my degrees on physics,”..

    So which university taught you that heat can flow spontaneously from a colder surface to a warmer surface?

    Any two blackbody radiators in view of each other will exchange energy. There is no magical rule that prevents radiation from the cooler one from being absorbed by the warmer one, adding energy to it. The energy flows go both ways. (My university wasn’t Hogwarts, so didn’t teach magical rules.)

    Although the energy goes both ways, because of the properties of blackbodies (the warmer they are, the stronger they radiate), the NET energy flow will be from the warmer to the cooler.

    10

  • #
    Nullius in Verba

    Bryan,

    None of us can. We put up a short comment, wait for the inevitable queries, and fill in the gaps as required.

    I don’t know, though, that Postma or most readers do know – or at least, have thought of it that way. If Postma knows, why did he claim that the maximum temperature achievable with solar radiation was around only 100 C? Given some of the statements being made here, it’s clear that there are many misunderstandings.

    I thought that Louis might be on to something, but the way he was expressing it was a bit scrambled. Trying to get him to clarify was an attempt to make progress – by small steps. But it’s quite difficult when he seems more interested in discussing my choice of “breakfast weeties brand” than the physics.

    10

  • #

    Almost all of this hooha is beside the point. The cusp of the matter is the alarmist claim that added GHG will cause catastrophic warming and, because of that pending catastrophe, we MUST stop the technological revolution by stopping the use of fossil fuels. It is even more significant that the global political class is happily planning a global unaccountable totalitarian governance for which we are to have no choice but to submit.

    In face of this, it is irrelevant to show that there might be a slight slowing of cooling or a slight but likely immeasurable increase in temperature caused by blah blah blah…. The issue is the prophesy and plan that must be dealt with.

    Fact: the earth, biosphere, atmosphere, sun system is massively complex made up of many interacting process with many of those processes being highly non-linear to chaotic.

    Fact: the behavior of any one process in isolation does not represent the behavior of the self same process in context let alone of the entire system.

    Fact: we must know each process within the system an the nature of it’s interactions with all other processes in minute detail to be able to compute the the behavior of the system.

    Fact: There are process we know and understand, there are process we know and sort of understand, there are process we know about and don’t understand, AND ,most importantly, there are uncounted processes we don’t know about and don’t understand.

    Fact: we know even less about the interactions among the processes than we know about the individual processes that make up the system.

    Conclusion: we don’t really know what we are talking about. We at best only know bits and pieces of the puzzle and a even less about how they go together. THIS is true for both sides of the argument.

    Now what can we really say one way or the other about the alarmist claim?

    Fact: The AGW catastrophe has not yet occurred because we are here arguing about it.

    Fact: There is massive evidence from the last century that unaccountable totalitarian governance IS a catastrophe and that making it bigger and more pervasive will not make it better.

    Question: Is there any credible real world evidence that the proposed AGW catastrophe can happen AND, if it can happen, that we can do anything about it for real?

    Question: What can we do about the very real but pending catastrophe of unaccountable totalitarian governance?

    If we can’t answer these questions and actually do something about them, there is no point in arguing about anything.

    10

  • #
    Bryan

    BobC

    I asked
    So which university taught you that heat can flow spontaneously from a colder surface to a warmer surface?

    You replied

    Any two blackbody radiators in view of each other will exchange energy. There is no magical rule that prevents radiation from the cooler one from being absorbed by the warmer one, adding energy to it. The energy flows go both ways. (My university wasn’t Hogwarts, so didn’t teach magical rules.)

    I agree with your points above however it says nothing about HEAT!
    Heat is the net flux difference between the radiative energy streams.
    It is unidirectional
    From the higher temperature body to the lower temperature body.

    I think that unfortunately Michael Hammer used the word Heat for the radiation from the colder surface.
    Although I’m sure he didn’t intend it, it has the implication of contravening the second law of thermodynamics.

    If the topic had been more general I would probably have let it pass with a “I know what he means” benefit of the doubt.
    However the topic is the Second Law and we should try to get it right.

    10

  • #
    bananabender

    The atmosphere is heated by it’s own gravity according to the Ideal Gas law: PV=nRT.

    Gravity acts as piston compressing the air below. As the air is compressed it is heated. This heating is exactly the same as what happens on the compression stroke of diesel engine.

    The adiabatic lapse rate is due to decreasing pressure with altitude.

    The temperature of the atmosphere at most altitudes can be predicted fairly accurately (+/-5%) in degrees Kelvin just by knowing the pressure. However due to convection and other heat transfer processes between the atmosphere and oceans this doesn’t always work eg in the tropopause.

    The Ideal Gas Law also explains the temperatures of every planet in the solar system: Mercury and Mars are cooler than expected due to lack of atmosphere (Mars surface pressure is only 0.006Bar). Venus, Jupiter, Uranus and Neptune are all warmer than expected due to dense atmospheres.

    10

  • #
    Nullius in Verba

    “Almost all of this hooha is beside the point. The cusp of the matter is the alarmist claim that added GHG will cause catastrophic warming and, because of that pending catastrophe, we MUST stop the technological revolution by stopping the use of fossil fuels.”

    I agree. But to achieve any pushback on that, you have to persuade intelligent onlookers that we know what we’re talking about, and the alarmists don’t. We won’t do that if we use the wrong arguments, especially obviously wrong arguments. That’s why it’s important that we do understand the physics, and we don’t make easily refuted claims that certain parts of the theory are wrong when they’re not.

    If you make it obvious that the accuracy of the details doesn’t matter, so long as it refutes AGW, it just looks like shut-eyed ‘denial’ rather than scientific scepticism. That’s a big and very dirty stick for the warmists to beat you with.

    I’ve already explained several times that the standard “back-radiation” mechanism is wrong, but it isn’t because it breaks any of the laws of thermodynamics, it isn’t because back-radiation doesn’t exist, it isn’t because cold objects can’t emit IR that gets absorbed by warmer objects, or that warm objects can’t absorb it, or that absorbed IR doesn’t cause heating. If we can agree not to keep repeating the “breaks the 2nd law” stuff, or at least come up with an explanation as to why you think it does that accords with real physics (either proposed or actual), then maybe Jo will be able to move on to the real reasons why the back-radiation explanation is wrong.

    We’re all on the same side.

    10

  • #
    Nullius in Verba

    “The temperature of the atmosphere at most altitudes can be predicted fairly accurately (+/-5%) in degrees Kelvin just by knowing the pressure.”

    You also have to know the average altitude of emission of radiation to space. Otherwise you just get a slope with no intercept.

    10

  • #
    BobC

    DavidA @ 87:

    What in the world are you talking about? Do you have a reference for the phenomena you are calling the “absorption threshold?” Such a concept is missing from any analysis of blackbody characteristics I have ever seen. (See Here, for example.)

    DavidA:
    May 21st, 2011 at 1:33 pm

    Whacky Physics Experiment # 87

    We get 1 million lasers and direct them all at one point on a body who’s initial temperature is above the absorption threshold of the laser’s photons.

    Despite having 1 million laser beams focussed on one spot there is no warming because no individual photon exceeds the absorption threshold. We can crank this up to 2,3 10,100 million, 1 trillion, infinity lasers and there is still no warming.

    The body is cooling and eventually it cools to a level where photon absorption occurs and then BOOM the energy of 1 million lasers is absorbed all at once.

    10

  • #
    BobC

    Bryan:
    May 21st, 2011 at 11:16 pm

    If the topic had been more general I would probably have let it pass with a “I know what he means” benefit of the doubt.

    That’s a pretty good rule to follow — without it, the argument becomes semantic (this thread: Exhibit #1).

    10

  • #

    BobC — DavidA is being satirical… not serious.

    10

  • #
    Bryan

    Nullius in Verba says its incorrect to say

    ……”that absorbed IR doesn’t cause heating.”……..

    Could you give reasons why a colder passive object can HEAT warmer radiating surface.

    What is your definition of HEAT

    10

  • #

    Louis, think of it this way:

    Energy flows (greatly simplified)

    SUN —-> Earth —–> Air —–> Space
    Hot —-> Warm —–> Cool —-> Cold

    What happens to EARTH if there is a block in the final stage:

    SUN —-> Earth —–> Air + GHG –|—> Space

    The net flow of energy is still warmer to cold. The break —|— is only partial, a temporary delay, a bottleneck…

    How can Earth keep losing heat at the same rate?

    10

  • #

    Nullius in Verba @ 143: But to achieve any pushback on that, you have to persuade intelligent onlookers that we know what we’re talking about, and the alarmists don’t.

    The alarmists pretend they know what they are talking about. Are we simply to pretend harder or do we actually have to know what we are talking about and be able to prove it?

    I don’t think it is a simple matter of persuasion or communicating an alternate hypothesis better. I think we actually have to know what is critical to know and be able to demonstrate its truth with obvious clarity. Simply being better at making hand puppets and reading a canned script won’t cut it.

    10

  • #
    Nullius in Verba

    “Could you give reasons why a colder passive object can HEAT warmer radiating surface.”

    I’m only talking about half the radiation exchange, and so I’m dividing the exchange of heat in the same way. Emitting radiation cools an object, absorbing radiation heats it. The actual change of temperature will depend on the balance of the two – and a warmer radiating surface will be cooled by emission more than it is heated by absorption, and vice versa for the colder object.

    The second law applies only to the net combination of all heat flows, but some people still seem to think that the only way it can happen is if it applies to all the individual components of the balance, and that IR emitted from a cold object can have no effect on a warmer absorbing object.

    I agree, it’s difficult to keep terminology both precise and comprehensible, and sometimes I may slip.

    10

  • #
    Nullius in Verba

    “I think we actually have to know what is critical to know and be able to demonstrate its truth with obvious clarity.”

    Agreed.

    10

  • #
    BobC

    CO2ISLIFE (@ 114): Re: Your argument about Poynting vectors.

    I totally agree that Poynting vectors (being vectors) can be added, and can add up to zero. I think, however, that parts of your argument are somewhat misleading. Let me give an example:

    Imagine two EM wave trains (with identical frequencys) propagating head on at each other, each with equal power. Each has a Poynting vector associated with it describing the quantity and direction of energy flux of the waves. At the point where they intersect, these vectors add to zero (and, you have a standing, not traveling, wave).

    Does this mean that an absorber, placed at the intersection point, would not see any energy flux? Obviously not, as it would be heated from both sides. The fact that the two Poynting vectors add to zero (indicating zero net energy flow) is not relevant to the experience of a potential absorber at the intersection.

    10

  • #
    cementafriend

    114 has a good explanation. Others include Jo and Michael Hammer base their arguments on photons but these do not exist see this from Nobel Physics prize winner W E Lamb http://www-3.unipv.it/fis/tamq/Anti-photon.pdf, another physicists expanding Lamb’s idea http://arxiv.org/abs/1009.5119 and an experienced engineer Xavier Borg explaining antennae & focusing http://www.worldsci.org/pdf/abstracts/abstracts_5711.pdf
    The claims of back radiation measurements have been refuted many times by engineers who make the instruments, know how they are programmed (no intrument can directly measure radiant energy) and the errors involved.

    10

  • #

    Joanne @ 149,

    Can you actually demonstrate that the so called break actually exists and that its effect is at all significant in magnitude and duration?

    Can you show that its effect is measurable in context of the huge thermal inertia of the rotating earth system and the constantly shifting relative position of the earth with respect to the sun? Especially while taking into account ALL the other natural variations within and outside of the earth atmosphere system.

    If it can neither be demonstrated nor measured, how is it important for either side of the argument?

    10

  • #
    Bryan

    Nullius in Verba says;
    “The second law applies only to the net combination of all heat flows, but some people still seem to think that the only way it can happen is if it applies to all the individual components of the balance, and that IR emitted from a cold object can have no effect on a warmer absorbing object.

    I agree, it’s difficult to keep terminology both precise and comprehensible, and sometimes I may slip.”

    I would rewrite the above as;
    The second law applies only to the net combination of all energy flows and this combination is called heat, but some people still seem to think that the only way it can happen is if it applies to all the individual components of the balance, and that IR emitted from a cold object can have no effect on a warmer absorbing object.

    The effect of a passive object near an object with a power source is to insulate or reduce the flow of heat from the hotter object

    10

  • #
    Nullius in Verba

    Bryan,

    A reasonable summary. I don’t disagree.

    10

  • #
    Bryan

    Nullius in Verba, from previous posts I have the impression that your views on the climate mechanics are pretty similar to those of Leonard Weinstein.
    Leonard with some reservations seems to think that Postma paper is sound.
    Have you read his paper?

    10

  • #
    Nullius in Verba

    Bryan,

    Yes, Leonard and I agree on the mechanism.

    Postma’s paper has some good bits in it, and most of the actual mechanism is buried in there, but at the same time there are too many things that are not quite right, ranging from minor details to some of the more important bits. They jar a bit as one reads. It would make a good basis from which to start, but only if we were to go through it systematically and negotiate a significant number of alterations. I don’t think, given the impasse we seem to have reached on even minor components of it, that anything so comprehensive is likely to happen.

    I’d say it was probably a document worth people reading at some point in their investigations, for the ideas it introduces, but everything then has to be examined carefully before being included or rejected. It’s not a good summary for a beginner, or as a statement of ‘the’ sceptic position, and I definitely wouldn’t rely on it to persuade physicists to take sceptics more seriously. It’s a worthwhile paper, but it shouldn’t be oversold.

    10

  • #
    Roy Hogue

    Greenhouse made simple:

    Given:

    1. A can into which marbles are delivered at the constant rate of 100/minute

    2. A hole in the bottom of the can through which marbles can escape at a rate determined by the depth of marbles in the can (they have to squeeze through a rubber diaphragm)

    The marbles will increase in depth until they can escape at exactly 100/minute, at which point the depth becomes constant.

    Now I throw a monkey wrench into the works by catching some of the escaping marbles and throwing them back into the can. What happens? The depth of the marbles must increase until they can escape at the rate of 100/minute plus whatever rate I’m tossing some of the escaping marbles back into the system.

    Have I introduced any new marbles to the system? No. Does the level in the can rise? Yes.

    marbles = energy (heat)
    source of marbles = the sun
    depth of marbles in the can = temperature of the earth
    marbles I throw back into the can = energy radiated back at the earth from CO2, H2O, etc.

    This is what happens. It’s all that happens. It’s necessary for this to happen as long as there is anything that can send an escaping photon back toward the surface — second law or any other law notwithstanding. Jo has it right.

    —————-

    Of interest:

    If my can has a small diameter it takes only a few marbles to reach the equilibrium depth. But if it has a large diameter it takes many more marbles to get to the same place. I call this out because it illustrates that heat is not temperature and temperature is not heat. I won’t point a finger at the offender but this mistake is made in at least one comment in this thread and I’ve seen it elsewhere.

    And now you get to throw darts at me if you want to. 😉

    10

  • #
    BobC

    Roy Hogue @160:

    Nice analogy.

    10

  • #
    Bryan

    Roy Hogue

    At the point you start feeding the marbles back stop feeding them in(100per minute) for a moment to see what is happening.
    We notice that the level(temperature) is now dropping
    Now divert some from bottom back in.
    The level(temperature) is now dropping more slowly.
    Now start the 100 per minute again, the level rises to new equilibrium level(temperature).

    Your model shows the effect of insulation.

    That is if the power supply continues at a constant rate insulating an object causes its temperature to increase to a new higher equilibrium temperature.

    10

  • #
    Bryan

    Roy Hogue

    One problem with your model is that you have to supply WORK to lift marbles to top.
    This means that the diverted marbles are now entering at the same “temperature” as the original 100min.

    I’m afraid the Second Law strikes again!

    The insulation effect is more like slowing down the rate of return of the diverted marbles.

    10

  • #
    Roy Hogue

    Bryan @162,

    I agree. So you’ve no argument from me.

    However, my analogy dealt only with radiated energy, which, after all, is what this climate catastrophe nonsense has always been about.

    10

  • #

    Can you actually demonstrate that the so called break actually exists and that its effect is at all significant in magnitude and duration?

    Lionell, I’m trying to break this down to one single point. This is about the principle. The absorption spectra have been shown in a lab lots of times. Whether its effect is significant in the atmosphere is another question, and is something I’ve asked myself. But if skeptics are going to say that the greenhouse effect can’t possibly work due to the second law, then as a point of principle I feel we need to discuss it. I can see no legitimacy for the statement. I think the skeptical world is more effective if skeptics don’t put forward lines of reasoning that are incorrect.

    10

  • #
    Roy Hogue

    Bryan @163,

    Now you’re distorting my intent. A physical analogy of the process involved is always going to have some shortcoming.

    I might as well deal explicitly with the rest of the scenario. At night the sun isn’t delivering marbles to the can. But they’re still escaping at a rate dependent on their depth and the depth will decrease. But if I keep putting some of the escaping marbles back in at the top the rate of decrease will be slower than if I didn’t do that. In the morning the depth will be greater than it would have been if I hadn’t intervened.

    10

  • #
    BobC

    cementafriend:
    May 22nd, 2011 at 12:37 am

    no intrument can directly measure radiant energy

    What is, then, that bolometers and pryometers are measuring?

    10

  • #
    Bryan

    Roy Hogue:

    …”Now you’re distorting my intent. A physical analogy of the process involved is always going to have some shortcoming.”…..

    No, I’m noticing that your model is not bad at all!
    To get lower temperature marbles(escaping from bottom back up to higher temperature marbles(top) requires work as the second law predicts.

    At night the can(atmosphere) gets no radiation from the Sun.
    However it gets radiation(marbles) from the surface and returns some.
    The atmospheric understanding of downwelling long wavelength radiation is best studied in night time conditions as there is no solar contribution to complicate things.
    Good luck with developing your model to represent the surface temperature and the atmosphere.

    10

  • #
    Roy Hogue

    Good luck with developing your model to represent the surface temperature and the atmosphere.

    Best I quit before Jo takes away my marbles.

    10

  • #
    KR

    BobC @ 145

    DavidA was being a bit satirical, but both his post and mine at @89 pointed out one of the major flaws of the “Slaying” book. Namely, that photons below a particular temperature dependent frequency are simple re-emitted, and photons above that frequency are absorbed heating the object.

    Yes, I’m serious, that’s what the book (in the Clas Johnson chapter 18) claims. In total violation of the behavior of blackbodies both experimental and theoretic.

    Industrial CO2 cutting lasers are a perfect counterexample to this foolishness.

    10

  • #
    KR

    cohenite @ 117

    I didn’t reply to you on the previous thread because your first post was (to me) a bit incomprehensible – I couldn’t tell what your objection was, and your second listed post is something I agreed with. The lapse rate is a function of both adiabatic expansion/compression and other factors such as convection, UV absorption, radiative effects, etc.

    “Kevan Hashemi asked Cohenite if a particle at 300 K will or will not emit photons. Any particle at 300 K is at its fundamental energy state, i.e. its available microstates will be higher than those of any particle with a temperature above 300 K. Such particle won’t radiate, but it will absorb energy.”

    That would be, um, incorrect. All matter with a temperature greater than absolute zero emits thermal radiation. Disprove that, and a Nobel prize awaits.

    As to thermodynamics, as Jo and Michael Hammer correctly state, there are no violations by the radiative greenhouse effect. All objects above absolute zero emit thermal radiation, but the sun is warmer than the Earth is warmer than the atmosphere is warmer than outer space – net (difference) of energy flow is quite correct. Rates of energy flow are determined by temperature and the (related) ability of each part of this to lose energy – the insulating atmosphere prevents the Earth from cooling as much as it would without such an atmosphere.

    10

  • #
    KR

    Bryan @ 134

    I’ve tried to be polite – but you have yet to produce any point supported by evidence, any relevant physics. All you have done is to be insulting. Ad Hominem is a logical fallacy, and proves exactly nothing.

    If you do have anything relevant and meaningful to say, I’ve yet to see it.

    10

  • #
    RJ

    DavidA was being a bit satirical, but both his post and mine at @89 pointed out one of the major flaws of the “Slaying” book. Namely, that photons below a particular temperature dependent frequency are simple re-emitted, and photons above that frequency are absorbed heating the object.

    I think the authors might know a bit (or a lot) more about this than you KR. And logic is on their side as well

    And what do you think about the comment at 154

    10

  • #

    RJ,

    cementafriend’s interpretation of Lamb’s paper is wildly misguided. It doesn’t say what he thinks it says (i.e. that photons don’t exist). Lamb is instead arguing over the nuances of what model to use (conventional photon, which includes a messy wave-particle duality, or the quantum theory of radiation) to use to properly think about and describe photons.

    10

  • #
    Bryan

    KR says

    …”Bryan If you do have anything relevant and meaningful to say, I’ve yet to see it.”…

    Well I pointed out that you are quite happy about gross errors in measuring instruments.

    The CO2 contribution to atmospheric radiation that we should all be worried about you said was around 1.5W/m2.

    I pointed out that an official ARM report say that errors of 17W/m2 could be happening with some recorded values in the literature.

    I would have thought that should cause you some concern!

    10

  • #

    RJ,

    I think the authors might know a bit (or a lot) more about this than you KR…

    Appeal to authority fails.

    10

  • #
    KR

    RJ @ 173

    I agree, there are some fascinating references in @154. The first denies quanta in characterizing photons – however, I work with photon counting devices, and that energy is quantized. The last invokes “virtual sources” at arbitrary distances behind the actual radiators in order to maintain quanta packet strength – with the unspoken requirement of supraluminal (faster than light) information transfer to create such a ‘virtual source’.

    Fascinating – but I cannot take them seriously. As I said before, I don’t take any source for granted; they must be critically examined for consistency with themselves (which Clas violates) and with known measured physical behavior.

    So, for my experiments, RJ:

    * Take a heater (with a fixed input power, no thermostat), measure it’s temperature in a cold room. Then insulate it, and measure the heater core temperature again. If it fails to rise, I owe you a decent beer, and the world owes you a Nobel if you can explain it.

    * How can CO2 lasers (emitting low frequency photons) manage to cut through multiple materials? By the physics described in Clas Johnson’s chapter, where photons below a critical frequency are re-emitted, and only those above that frequency heat an object, that can not happen. Either Clas is spinning fantasies, or the cutting laser industry is impossible.

    Well, RJ? Can you produce results different than I have predicted? Show that cutting IR lasers are not functional, or that insulating a heater core with a fixed input power does not raise it’s temperature by reducing cooling?

    10

  • #
    RJ

    @ 177

    * Take a heater (with a fixed input power, no thermostat), measure it’s temperature in a cold room. Then insulate it, and measure the heater core temperature again. If it fails to rise, I owe you a decent beer, and the world owes you a Nobel if you can explain it.

    I’m surprised that experiments like this have not been done. Say with a oil fired heater

    First compare the oil temperature surrounded by air with a cold then a warmer air temperature. Does the oil temperature change?

    Then surround the heater with a vacuum with an outer container filled with air and then with CO2.

    I would have thought these experiments would be easy to do. And fairly cheap to set up.

    10

  • #
    Bryan

    KR says

    …….”How can CO2 lasers (emitting low frequency photons) manage to cut through multiple materials? By the physics described in Clas Johnson’s chapter, where photons below a critical frequency are re-emitted, and only those above that frequency heat an object, that can not happen. Either Clas is spinning fantasies, or the cutting laser industry is impossible.”….

    You must be getting a bit desperate.
    The theme is about the second law of thermodynamics and the atmosphere.
    Yet the only evidence you bring up to support your IPCC position is the Laser which is a quantum device and works on principles that have nothing to do with equilibrium thermodynamics.
    Stay on topic please.

    10

  • #
    Nullius in Verba

    How does food absorb microwaves in a microwave oven?

    Magnetron radiation isn’t coherent.

    10

  • #
    L.J. Ryan

    Nullius in Verba, KR:

    How does food absorb microwaves in a microwave oven?

    How can CO2 lasers (emitting low frequency photons) manage to cut through multiple materials? By the physics described in Clas Johnson’s chapter, where photons below a critical frequency are re-emitted, and only those above that frequency heat an object, that can not happen. Either Clas is spinning fantasies, or the cutting laser industry is impossible.

    Microwave ovens and CO2 laser incorporate atmospheric forcing? Are you two indulging in the beer KR owes Bryan?

    Nullius in Verba are the microwave emitted a higher energy then the food contained within the oven?

    CO2 laser…KR I doubt you understand how a laser works. Was it the CO2 that suckered you in? Are “photons” emitted via CO2 laser a higher energy then surface being cut? You suggesting CO2 lasers pertain to atmospheric condition has as much validity as CO2 freezer or CO2 fire extinguishers. If you need primer on lasers, let me know.

    10

  • #
    L.J. Ryan

    KR

    You insist on this wrapped radiator test as verification of GHG physics. I trust you realize you’re alone with delusion…or maybe not. Others whom agree with KR, please put yourself on record, make a quick affirming KR supposition:

    * Take a heater (with a fixed input power, no thermostat), measure it’s temperature in a cold room. Then insulate it, and measure the heater core temperature again. If it fails to rise, I owe you a decent beer, and the world owes you a Nobel if you can explain it.

    10

  • #
    cohenite

    KR; everyone is picking on you but really, your comment at 171 is classic fudging; I gave you that article on microstates and the relative capacities of different surfaces to absorb and emit radiation and you wilfully misinterpret what Nasif said: ie;

    “Kevan Hashemi asked Cohenite if a particle at 300 K will or will not emit photons. Any particle at 300 K is at its fundamental energy state, i.e. its available microstates will be higher than those of any particle with a temperature above 300 K. Such particle won’t radiate, but it will absorb energy.”

    That would be, um, incorrect. All matter with a temperature greater than absolute zero emits thermal radiation. Disprove that, and a Nobel prize awaits.”

    The key points in Nasif’s article are these:

    “1 A microstate refers to any initial o[r] final configuration of the energy in a given system.

    2 In consequence, the definition of the Second Law was amplified to include its influence on the quantum level and not only on those process functions where heat and work were implied.

    This shift was important because it defined the real concept of entropy and detached it from contextual derivations. For example, now we know that the fundamental concept of entropy has nothing to do with disorder, movement, complexity or heat “content”, but with the configurations that the energy adopts in a given system and the directionality of the energy exchange.

    Entropy is now defined as the natural trend of the energy to flow towards the system or systems with a higher number of available microstates.”

    What Nasif said, although badly worded and English is not his first language, is entirely consistent with his assumptions. Would you care to comment on those?

    10

  • #
    BobC

    Sure, I’ll go on record Ryan. If KR’s experiment didn’t work, then putting on a coat wouldn’t keep you warm in cold weather.

    A somewhat safer experiment would be to use one of those immersion heaters you use to brew a single cup of tea. The last one I had had a warning not to plug it in if it wasn’t in water, as it could get hot enough to melt down. Apparently, it couldn’t lose energy fast enough in air to stay at a safe temperature.

    Here’s one you should be able to do without anything you can’t find in the house: Use a heating pad as the source and a fever thermometer to read the temperature. Try it just laying on a table with the thermometer on top. After you reach a stable reading, put a heavy coat on top of it and read it again in a few minutes.

    L.J. Ryan:
    May 22nd, 2011 at 2:45 pm

    KR

    You insist on this wrapped radiator test as verification of GHG physics. I trust you realize you’re alone with delusion…or maybe not. Others whom agree with KR, please put yourself on record, make a quick affirming KR supposition:

    Information is not produced by arguing, Ryan, but by observation — do the experiment.

    And if KR is right, so what? The calculations of the radiative GH effect give a 1 deg increase for doubling CO2. Actual measurements in the real world (al la Idso) show that negative feedbacks reduce this to less than 0.5 deg. Measurements of atmospheric CO2 concentrations show a remarkably steady growth of ~0.5%/year for the last 50 years, despite anthropogenic emissions growing 8-fold in the same time — hence there is no reason to expect this rate to suddenly change.

    The net result is that we can expect the world to get about 0.5 deg warmer over the next 140 years, independent of human activity. Oh wow, oh wow — what a crisis.

    10

  • #
    Paul

    Joanne Nova:
    May 22nd, 2011 at 12:13 am

    Louis, think of it this way:

    Energy flows (greatly simplified)

    SUN —-> Earth —–> Air —–> Space
    Hot —-> Warm —–> Cool —-> Cold

    What happens to EARTH if there is a block in the final stage:

    SUN —-> Earth —–> Air + GHG –|—> Space

    The net flow of energy is still warmer to cold. The break —|— is only partial, a temporary delay, a bottleneck…

    How can Earth keep losing heat at the same rate?

    Just in case this hasn’t been answered already, I’ll propose an answer myself, though I’m not an expert in the physics of heat transfer just an enquirer after the truth of the matter. I’m a little late to this thread.

    We are not talking about insulation materials or the presence of colder ‘bodies’ in conjunction with the surface of the earth. we are talking about the thermal properties of a mixture of gas, with varying proportions of some molecules that, beside the normal thermal properties of molecules in gaseous form, have the additional property of being able to interact with radiation by absorbing a photon of electromagnetic energy, mostly in the long-wave band although some do so in the short-wave band. And, more specifically, we are talking about the possible effect on near-surface-level atmospheric temperatures that might result from a change in the proportion one of those molecules in the atmosphere, namely CO2, that may have resulted from the use of fossil fuels.

    “The break –—|—— is only partial, a temporary delay, a bottleneck …”

    When an molecule of CO2 is struck by a photon of long-wave EM radiation, the result may be that the photon is absorbed, depending on the direction that the molecule was moving in, its temperature, etc, and if the photon is absorbed the consequent effect is that the two oxygen atoms vibrate around the carbon atom at the ‘centre’. This is a different behaviour from what happens when the photon collides with a molecule of nitrogen or of oxygen since these are not so constructed as to be able to enter this vibrational state.

    What happens next, also dependent on a number of factors, is that the photon is re-emitted more or less instantaneously and the molecule returns to its state of lower vibration, or the molecule collides with another molecule and loses some of its energy in the collision, meaning that it is less able to emit a photon. Some energy therefore is imparted to the general body of the atmosphere as heat and some energy is now emitted at lower energy levels and longer wave lengths presumably at a higher altitude.

    Without an experiment to ascertain the delay involved in the transmission of EM energy through the atmosphere it is hard to quantify that delay, but, knowing the speed at which photons move I would imagine that it is measured in milliseconds. At any rate, the energy that is ‘delayed’ by the CO2 molecules in the atmosphere by being transformed into heat and then re-emitted at higher altitude, lower temperature and at longer wave lengths would have to be of very insignificant proportion relative to the amounts of incoming and outgoing energy.

    One aspect of this question that should settle the matter, to my way of thinking, is that the higher atmosphere should be warming, according to this theory, faster than the temperature at the surface. However the ‘hot spot’ in the tropics has never been found and so the empirical evidence in support of this aspect of CAGW conjecture is missing.

    Paul

    10

  • #
    L.J. Ryan

    Correction to 182:

    make a quick post affirming KR supposition:

    10

  • #

    And once the energy leaves the earth, it gets absorbed in the first few feet of the atmosphere. Almost none of this heat can reach the Earth in backscatter. 100% leaves the Earth, reaches say 6 feet and is fully absorbed. Each portion of the 6 feet of atmosphere has essentially the same CO2 content. More radiation is absorbed in the first inch than the second inch and more in the second than the third all the way to extinction. If the one inch of atmosphere can absorb 100 units, 50 units on average get sent back towards earth, but the extinction distance is less for 50 units than it would be for the 100 original units. Now, when it gets scattered, more energy will be absorbed in the first half inch back towards Earth than in the second, leaving nearly no photon energy left by the time it transfers the full inch back. If the one inch of atmosphere can absorb 100 units, 50 units on average get sent back towards earth, but the extinction distance is less for 50 units than it would be for 100 units. This same concept works for the first inch and 6 feet and all the way to the top of the atmosphere.

    I was somewhat convinced at someones argument about the silvering of a thermos to prevent heat loss. I was thinking that 50% of the energy could make it back to the Earth, but I no longer think that is possible. The difference here is that in a thermos, the space is a vacuum, and thus there is no backscattering, the silver simply reflects the photons back, they never escape and are never absorbed. In the air, the photons are absorbed and sometimes are re-emitted and other times the energy is converted into physical motion.

    10

  • #
    Paul

    DavidA:
    May 21st, 2011 at 2:44 am

    Bryan solar radiation sits in the visible light spectrum so there isn’t any incoming IR from the sun in the day. The frequency of the radiation output by a warm body is a function of its temperature and bodies as hot as stars output visible light.

    Oh, but the Sun does radiate in the long-wave frequency, just not as much energy as in the shorter frequencies. So the atmosphere is heated from above, during the day, both by the absorption of short-wave radiation by ozone molecules and by the absorbtion of long-wave radiation by CO2, H2o, clouds, dust particles, etc.

    Paul

    10

  • #
    BobC

    My last prediction in #184 should have included the caveat: “If nothing else changes”. The Sun is obviously in a different state than it was 50 years ago, leading many to wonder if we are heading into another Little Ice Age. My opinion is that the chance of the CO2 “climate signal” being even detectable over the next 100 years is slim.

    10

  • #

    Do you agree that if you stand surrounded by cold objects (say a ring of huge ice blocks) you feel cold?
    Yes, because the energy of the air around me absorbed into the cold objects near me.
    Do you agree that the colder the object you are surrounded by the colder you feel? eg: if they were blocks of frozen CO2 (dry ice) instead of water ice would you feel colder?
    Yes, because less of the energy of the air around me is absorbed into the cold objects near me.
    OK now if you have been standing surrounded by extremely cold objects and then move so that you are now surrounded by merely cool objects does the move make you feel less cold than you were before?
    Again, air temperature is lower, if you do not control for physical air temperature at my position, then you are simply saying cold air feels colder. You lose the argument.
    In the absence of green house gases the earth is surrounded by an extremely cold object – outer space at 4K (-269C). Green house gases make the atmosphere opaque at some wavelengths. With these in place the earths surface is in effect surrounded by a merely cool atmosphere instead of the truly frigid outer space. Because the surface is now surrounded by a less cold object than it was before it is less cold.
    And science already has answers to this question with no need at all of greenhouse gas theories. Adiabatic lapse rate. The air close to the surface will be warmer than the air at the top.
    Since warm and cold are opposites, less cold is the equivalent of warmer. Surrounding earth’s surface with an opaque cool atmosphere make it warmer than it would be if exposed directly to the ultra frigid outer space.
    Yes, but it does not have to be opaque, just simply capable of absorbing energy physically from the planet surface and transporting it upwards. No magical beans needed, not mysterious gasses required, just simple everyman’s physics.

    If you take a room and surround it with containers that have invisible walls to radiation, (|) that looks like this…
    |vacuum|liquid nitrogen|vacuum|20 feet of pure CO2|vacuum|room with specific energy production|vacuum|20 feet of pure CO2|vacuum|liquid nitrogen|vacuum|
    And
    |vacuum|liquid water|vacuum|20 feet of pure CO2|vacuum|room with specific energy production|vacuum|20 feet of pure CO2|vacuum|liquid water|vacuum|
    What would be the difference in temperature between the two rooms?
    Then do the same thing and replace the 20 feet of pure CO2 with 19 feet then 18 feet … 1 inch, filling the remainder of the 20 feet minus actual width of CO2 with additional vacuum distance.

    Record the differences and get back to me.

    00

  • #

    #153 BobC

    Does this mean that an absorber, placed at the intersection point, would not see any energy flux? Obviously not, as it would be heated from both sides. The fact that the two Poynting vectors add to zero (indicating zero net energy flow) is not relevant to the experience of a potential absorber at the intersection.

    Assuming that the intersecting object is cooler than the two emanating, background sources the rate of heat loss from the sources is unaltered whether the intersecting object is there or not.

    Paradoxical as this seems, there is no heating from other objects that are at or below the critical threshold of equal temperature.

    This is because light doesn’t consist of marbles of energy moving from point A to B necessarily raising the temperature of B; imparting momentum. (Thanks #154 cementafriend and #135 Baa Humbug). Light can cancel and interfere, which corpuscles of light “photons” can’t do. (Thanks comment #114, CO2ISLIFE).

    If this energy cancellation wasn’t true you could sit in your lounge room and get warmed up to a higher temperature by the TV and the lounge and the walls and the carpet backradiating all of that nasty energy which “must” have some effect. But, you know what? It doesn’t warm you at all!

    00

  • #
    Alistair

    Just had a quick look at new posts. It’s truly amazing that one poster criticised my physics [not my first subject, which is really physical chemistry but from metallurgy] and claimed ‘back radiation’ can be explained like a Fabry-Perot Etalon.

    Well, that’s an interferometer whereby energy at particular wavelengths is cancelled out and you can get enhanced reflection. When I was taught physics, it was with always supported with experimental verification. If we do have a FPE above our heads, it has to be from a variation in the composition of the air in a precisely controlled manner and there’s no evidence of it.

    Or maybe, it’s the clouds? If it is, it’s nothing to do with GHGs. Indeed, I am now arguing that because gases obey the law of Equipartition of Energy, once a CO2 or other GHG molecule absorbs an IR photon, all that does is to slightly perturb the local thermodynamic equilibrium and another molecule, which was going to emit the same energy photon anyway, restores it.

    Think on this: unlike in a solid where you explain refractive index by the slight delay before re-emission, in gas there is no ‘delay’. Thus, the cascade of energy from the Earth’s surface to space has no delays from change in GHG composition. All that happens is that you measure the specific absorption, in reality followed by exchange of that energy with all the other molecules in a totally random manner.

    I must admit I’m still unhappy about my understanding of this specific absorption. This is because we are looking at a snapshot in time. However, I am fairly convinced that the only stored energy in the atmosphere has to be in aerosols and in the case of water droplets, the local latent heat energy as humidity changes.

    00

  • #
    mullumhillbilly

    Wow, its going to be hard to stay polite here, but what the hell, this is too important to let the wheat stay mixed with the chaff.

    Mike Hammer & Jo, you had it nailed in the previous thread. The 2nd law is not being violated by AGW theory, and I am astonished at the number of posts, (incl I must say from prev heroes of this blog), which continue to miss two simple points about NET flow of energy, and the difference between heat content and temperature.

    Mike and Jo, thanks for having the fortitude to bear all these inanities with patience and still fight the good fight, the one where empirical reason and science prevail. Bouquets to BobC, Sphaerica, Nullus in Verbia et al for keeping their wits about them. Its a sad day for this blog when I’m giving X’s to stalwarts such as LionelG and BrianV, and ticks to pro-AGW posters like Mattb and KR, (respect to you KR for staying polite and engaged and on-topic.)

    In all 4 parags of Jo’s headline article of this thread, I see J.Postma is talking about TEMPERATURES, whereas Mike and understand that a discussion of 2nd law of Thermodynamics will be about heat (energy) not temperature. Temperature of a sparse gas can be very high, even though little heat is contained.

    update: I see Postma@47 has now corrected this misunderstanding of the difference between H and T, but persists in making silly statement like “And so by extension the GHG’s cannot increase the temperature of the ground”. Strawman !…What is so difficult to understand in Mike and Jo’s posts about reduced rate of heat loss, ie that its NOT “heating” per se. ??

    Slayers of the sky-dragon are merely tilting at windmills; their hearts might be in the right place, but those quixotic pretensions are setting the skeptic position up for mockery. Yes, they laughed at Einstein, and they laughed at Darwin, but they also laughed at Bozo the clown !! I agree wholeheartedly with Jo @165 “I think the skeptical world is more effective if skeptics don’t put forward lines of reasoning that are incorrect.”

    So I agree that CAGW is flawed for a range of reasons, but saying that AGW theory is in violation of the 2nd law is incorrect and will harm the credibility of skeptics.

    00

  • #
    Bryan

    mullumhillbilly says;

    …”So I agree that CAGW is flawed for a range of reasons, but saying that AGW theory is in violation of the 2nd law is incorrect and will harm the credibility of skeptics.”…..

    But there is no such a thing as a single orthodox AGW theory.
    There are several versions of it.
    Some make claims that violate the second law, most do not.

    Lack of a proper definition of HEAT for instance can lead sceptics to inadvertently violate the second law themselves.
    Expressions like ..”the heat from the colder object partly balances…..”implies just such a violation.

    Loose use of words like HEAT… will harm the credibility of skeptics.

    What is your definition of HEAT?

    00

  • #
    RJ

    mullumhillbilly @ 193

    I have been very impressed with the Postma posts and the slayers book. I suggest you read them (again if you have already done so) with a open mind and a clear head.

    00

  • #
    Bryan

    I said at 194
    There is no such a thing as a single orthodox AGW theory.
    Its equally true that there is no single sceptic position.

    The slayers book for instance contains an input from Claes Johnson.

    Now Claes is engaged criticising;

    1. AGW
    2. The existence of photons.

    I have a lot of sympathy for the project that Claes is engaged in but I feel that he has much more work to do to convince mainstream physicists of its merits.
    For instance I doubt if Gerlich and Tscheuschner would endorse his views.

    So its possible to support 1 and not 2, or 2 and not 1, or neither.

    00

  • #
    RJ

    @ 196

    I have a lot of sympathy for the project that Claes is engaged in but I feel that he has much more work to do to convince mainstream physicists of its merits.

    Merits of what. The existence of protons (or otherwise). And criticising what version of AGW

    And G+T would not endorse his views regarding what in particular?

    00

  • #
    mullumhillbilly

    Noting the difference between heat and temperature, it seems possible to me that the JN/Hammer explanation of the 2nd law, GISS/NASA rising “global” surface temp trends, and Trenberth’s “missing heat” may all actually be consistent. Here’s how.

    Supposing CO2 intercepts Gaia’s (Earth’s)long wave nocturnal IR at near-ground, briefly slowing the escape of some wavelengths in the first ~50m above ground at night (~saturation height or less). This would mainly happen during the early part of clear evenings.

    Slowing the escape of radiant energy is synonymous with lowering the rate of heat loss, and as I understand it, this is the “orthodox” explanation for GHG warming.

    NASA/GISS/IPCC AR4 all concur in predicting that the largest component of future temp increases is coming from slight night warming,in winter, at near-polar latitudes.

    But what is the actual result? A short duration, early evening, near-ground integral of temperature over time (lets call them degree-seconds) which is higher number compared to when [C02] is lower. (see graph here). That fattened section of the diurnal-T curve and slightly higher overnight minimum can present as an increase in the average temperature for the day, (thence by summation for the year for that location , for the region, and the globe). Temps measured at 9am, daily max or 3pm are unaffected, but consider what would happen with overnight minimums or early evening (eg 9pm) records.(see graph link above).

    BUT a slight increase in early evening temperature in near-polar latitudes, mainly in winter, does not constitute “global climate change” !!! Maybe this could change some plants and insect distributions at the margin, but will it change rainfall, drought, hurricanes? Nada, zip, nyetski, and thus no need to invent (or slay) non-existent sky-dragons.

    Why not “climate change”? Well,the heat energy gain (as evidence by the delayed temperature reduction in early evening) is only temporary. The warmed near-ground air will move upward due to convection (interestingly this convection itself can also increase recorded night temp as a result greater mixing in otherwise still inversion layers of night air). So convective air movement and NET radiative transfer still carries the delayed/retained heat towards the tropopause, and thence it radiates to space. I haven’t got the calcs, but based on what I’ve read elsewhere on this site from George (“CO2isnotEvil”) I would readily accept that the heat balance is conserved overnight. Therefore the 2nd Law is observed, and there is no “missing heat”. But miraculously and concurrently with energy balance, we have an increase in early evening temperatures, and this (apart from spurious corrections, and badly sited weather stations etc) could be what is scaling the average upward, leading to the reported higher GISS “global” temperatures. The trick is to consider the integral over time; all the average temperatures I’ve seen used are static points, comparing standard times of day eg 9am, 12 noon 6pm etc, and/or daily min/max.

    So, again I agree with Jo that some GHG warming will occur from 2x[CO2], but its relatively minor and unimportant in it’s effect on “climate”. But its not zero.

    Fortunately satellites and radiosondes have not been confused by the early evening’s warming affect on “global average temperature”. The spectral emission profiles show little or no statistically defensible trend.

    Bryan@194,196 “Some make claims that violate the second law,”… well those would be fanciful speculations not deserving of serious consideration, and would probably not be falsifiable by reasonable empirical tests. “Now Claes is engaged criticising; .. 2. The existence of photons.” Well OK, I’m looking forward to the Nobel… I can keep an open mind on that, but Okham’s razor prevails in the meantime.

    00

  • #
    Nullius in Verba

    L.J. Ryan, #181,

    Microwave ovens and IR lasers are – obviously – relevant to this question of whether low energy photons can be permanently absorbed by warmer objects. I note that you don’t answer the question, but instead find a (totally spurious) reason not to answer.

    “are the microwave emitted a higher energy then the food contained within the oven?”

    No. Microwaves are characteristic of temperatures down around 5-10 K. Even frozen food is at temperatures around 250 K. Similarly, IR is characteristic of room temperatures, not the white-hot steel surface being cut. Quite obviously, warm objects can absorb ‘cold’ photons.

    #182,
    It’s not a full ‘test’ of greenhouse physics, but it does happen.

    cohenite, #183,
    1. a microstate doesn’t just refer to energy. It can refer to momentum, angular momentum, position, or any other microphysical variable.
    2. The 2nd law works just the same for quantum mechanics ass classical – introducing it is an unecessary complication.

    Entropy isn’t a trend, it’s a quantity. And even interpreting energy in terms of numbers of microstates, the GHE mechanism still doesn’t violate the second law. Net heat is still flowing from warmer to colder.

    Paul, #185,

    “Without an experiment to ascertain the delay involved in the transmission of EM energy through the atmosphere”

    You’re right that the delay is short, but it’s not relevant because that’s not how they say it works anyway. “Delay” is another of these words with multiple everyday meanings. The reason the interception matters is that the intercepting gas is colder than the surface, so when it re-emits upwards it does so at a lower rate, with lower-energy photons. (Energy balances because it is also emitting down, and in a real atmosphere, convecting.) If the absorbing atmosphere was hotter than the surface, then more greenhouse gases would lower the temperature.

    The theorised hot spot is due to the water vapour feedback changing the lapse rate – it’s nothing to do with energy being trapped on the way out.

    It’s not easy for me to explain it briefly. See my first post on the previous thread for a link to longer explanation.

    00

  • #
    Bryan

    RJ

    To disprove the existence of Photons is much harder than any aspect of the climate science controversy.

    Is there some experiment to show that photons don’t exist?
    Is there some natural phenomena that photon theory contradicts?

    G&T are very much in the mainstream of orthodox physics and in their publications accept and use radiation transmission through photons.

    However the point I was making was you can disagree the IPCC advocates and also Claes no photon physics at the same time.

    00

  • #
    Nullius in Verba

    astonerii, #187,

    If IR energy got absorbed within the first few feet, IR cameras wouldn’t work.

    #190,

    “And science already has answers to this question with no need at all of greenhouse gas theories.”

    The adiabatic lapse rate – which only applies to an actively convective atmosphere – only tells you what the gradient of the line is, it doesn’t tell you the intercept. To find the intercept, you need to know the average altitude of emission of radiation direct to outer space, because it has to be the right temperature there to radiate all the heat the Earth gets from the sun. The higher that altitude, the warmer the surface. If the atmosphere was entirely transparent to IR, all the heat would be emitted from the surface, at -18 C, and the atmosphere would be even colder than that (at the adiabatic lapse rate) as you went higher.

    The ALR is an essential part of the explanation, but is not the whole explanation. GHGs can still play a role.

    Stylo, #191,
    “But, you know what? It doesn’t warm you at all!”

    When you sit there at 37 C = 310 K, you radiate 5.67e-8 *310^4 = 523 W/m^2 into your surroundings – roughly a kilowatt. If you did not get energy back from your living room, you would have to burn food at that rate to maintain your temperature. The actual rate is about 100 W, because you only need to supply the difference between input and output. Be thankful for your walls and furniture – it keeps you alive!

    mullumhillbilly, #193,

    Thanks for the bouquet!

    Bryan, #194,

    Using “heat” in an everyday rather than technical sense is wrong, but a forgiveable error, and correcting it distracts from dealing with more serious misunderstandings.

    RJ, #195,

    You have said that before. As mullumhillbilly has apparently read the thread, he presumably knows. Have you, likewise, read our posts in response with an open mind?

    00

  • #
    Louis Hissink

    Joanne Nova:
    May 22nd, 2011 at 12:13 am
    Louis, think of it this way:
    Energy flows (greatly simplified)
    SUN —-> Earth —–> Air —–> Space
    Hot —-> Warm —–> Cool —-> Cold
    What happens to EARTH if there is a block in the final stage:
    SUN —-> Earth —–> Air + GHG –|—> Space
    The net flow of energy is still warmer to cold. The break —|— is only partial, a temporary delay, a bottleneck…
    How can Earth keep losing heat at the same rate?

    Jo

    The block that is proposed has to be a different physical phase (like clouds) to slow the energy escaping the earth. If the block is clouds, sure the heat loss slows down. If it is a solid like glass, then that is the mechanism of a greenhouse, and heat loss also slows down.

    But if the block is a radiating gas it won’t block the heat leaving the earth, rather it will accelerate the heat loss as it transmits the escaping energy not only via Brownian motion, but also by radiation. As temperature drops so does the energy so radiated and this in itself raises come interesting conundrums. Another error is assuming that the earth is source of energy in this circumstance – it isn’t since it is losing energy while sun, on the hand, continually emits energy.

    The interesting remark made by Professor Gert Venter, recounted by Astronomer Hilton Ratcliffe, that injecting CO2 into his various greenhouse experiments causes the atmosphere inside the greenhouse to drop in temperature, (And Venter is a world authority on greenhouse horticulture) seems to be substantiated with other observations that the GMT hasn’t risen in step with CO2. Both of these physical facts falsify the AGW hypothesis.

    It also means that the CO2 backward radiating phenomenon is misunderstood. As the solar tap is turned off, neither the earth or its gaseous coating will be receiving energy and both objects will, from a lack of input, drop in temperature. For CO2 to trap heat, something which a gas cannot do by the way, is has to be receiving a source of energy, but the earth is not in this case a source of energy, just as the N2 molecule next to a CO2 molecule cannot be a source of energy either.

    This does not mean that I dismiss the measured down welling IR that almost everyone believes comes from atmospheric radiating gases. Rather because the AGW hypothesis has been falsified by measurement and observation, then the scientific conclusion is to then look elsewhere for the source of the down welling IR since it cannot be coming from CO2.

    00

  • #
    RJ

    @ 201,

    You have said that before. As mullumhillbilly has apparently read the thread, he presumably knows. Have you, likewise, read our posts in response with an open mind?

    If mullumhillbilly has then maybe he could be a little more specific. What are these quixotic pretensions?

    And yes I have read this thread. I don’t know and do not pretend to. But for clarity and meeting the common sense test the slayers book and Postma’s paper and posts are impressive. I look forward to the slayers next paper which I’m sure will move this debate forward once again.

    00

  • #
    RJ

    Brian @ 200

    To disprove the existence of Photons is much harder than any aspect of the climate science controversy.

    Is there some experiment to show that photons don’t exist?
    Is there some natural phenomena that photon theory contradicts?

    G&T are very much in the mainstream of orthodox physics and in their publications accept and use radiation transmission through photons.

    However the point I was making was you can disagree the IPCC advocates and also Claes no photon physics at the same time.

    But does it matter. Isn’t the key point whether either photons or whatever further heat a warmer body if the energy comes from a colder body. Is it a net energy factor or is it that photons have no impact whatsoever

    I tend to believe radiation from a colder body has no impact whatsoever. They do not arrive and warm up the body. They arrive and have no impact and leave. So a huge flow of photons from ice will not and can not warm us.

    Its irrelevant really to the GHG backradiation theory if photons exist or not.

    00

  • #
    Nullius in Verba

    Louis, #202,

    “The block that is proposed has to be a different physical phase (like clouds) to slow the energy escaping the earth.”

    Why?

    “rather it will accelerate the heat loss as it transmits the escaping energy not only via Brownian motion, but also by radiation.”

    Why? Do you mean that because there are two mechanisms that it inevitably goes faster? How can the atmosphere transmit energy faster than the surface supplies it?

    “Another error is assuming that the earth is source of energy in this circumstance – it isn’t since it is losing energy while sun, on the hand, continually emits energy.”

    How does the fact the Earth is losing energy mean it cannot be a source of energy? What sort of source can supply energy without losing it?

    “injecting CO2 into his various greenhouse experiments causes the atmosphere inside the greenhouse to drop in temperature”

    So? The temperature does not vary significantly from top to bottom of the greenhouse, so the radiation up and down balances and has no effect. All heat lost is via other mechanisms of heat transfer, so messing about with radiative transfer by injecting CO2 likewise has no effect. (This is much like my pond of water example.) Since the greenhouse being filled doesn’t change the average altitude of emission to space noticeably, it doesn’t falsify the atmospheric GHE, either. All it proves is that the mechanism keeping real greenhouses warm is not the radiative GHE.

    “is has to be receiving a source of energy, but the earth is not in this case a source of energy,”

    You’re using “source” in two different senses, here. It’s not a source in the sense that heat is internally generated, but it is a source in the sense that stored energy is being supplied to its surroundings.

    “This does not mean that I dismiss the measured down welling IR that almost everyone believes comes from atmospheric radiating gases.”

    Good.

    “the scientific conclusion is to then look elsewhere for the source of the down welling IR since it cannot be coming from CO2”

    That doesn’t follow from what you said. Even if we take your conclusions as stated, all you’ve said is that the gas in the atmosphere can’t slow the heat loss, you haven’t said anything about why it can’t emit IR downwards. (And don’t forget most of the downwelling radiation comes from water vapour, anyway.)

    It is quite possible to reject CAGW and reject the back-radiation mechanism as an explanation for greenhouse warming without asserting that it breaks the 2nd law, or that GHGs don’t radiate, or that the radiation isn’t absorbed, or whatever. You are not being asked to convert to out-and-out warmydoom belief.

    00

  • #
    Bryan

    Nullius in Verba

    Lasers
    A high enough concentration of photons of any wavelength can cause heating.
    It does not follow the path of equilibrium thermodynamics
    The heating is achieved through compression (shock waves).

    Microwave oven.
    Cold matter emits blackbody spectra in the microwave region of the spectrum. The cosmic microwave background radiation is 3K for instance.
    How can a microwave oven can heat up a bowl of soup?
    It isn’t accomplished by microwave photons but rather by phase variation which causes dialectric molecules (primarily water in this case) to constantly re-orient with the magnetic phase. Friction then (plus choice of resonant frequency) does the actual heating.
    The plastic bowl is quite cool.
    Microwave radiation can’t raise the temperature of anything that isn’t within a couple degrees of absolute zero.

    LED lights
    These are readily available with a colour temperature of 5000K.
    Yet they are cool to touch.
    Once again the device does not take the thermal route for its operation.
    It is a quantum device.

    All three devices above do not depend on thermionics for their operation.
    To try to describe some thermal phenomena thing like the atmosphere through them is a pointless distraction!

    00

  • #
    Paul

    Alistair: @70
    May 21st, 2011 at 3:18 am

    Climate Science relies for its justification of ‘back radiation’ the incorrect mathematics of Arthur Milne who used the wrong boundary condition when he solved the PDE for IR absorption in the atmosphere. Miskolczi shows it vanishes when you correct the maths.

    Thanks for the reference to Miskolczi. I Googled that and found a YouTube with a brief introduction to his ideas. Miskolczi`s New Greenhouse Law if any one is interested in a quick introduction.

    Then, following the URL shown near the end of the video I visited his own web site at “http://miskolczi.webs.com/” where you can read his paper : —

    THE STABLE STATIONARY VALUE OF THE EARTH’S GLOBAL AVERAGE ATMOSPHERIC PLANCK-WEIGHTED GREENHOUSE-GAS OPTICAL THICKNESS

    by Ferenc Miskolczi

    Published in: ENERGY & ENVIRONMENT
    SPECIAL ISSUE: PARADIGMS IN CLIMATE RESEARCH
    VOLUME 21 No. 4 2010, AUGUST

    ABSTRACT

    By the line-by-line method, a computer program is used to analyze Earth atmospheric radiosonde data from hundreds of weather balloon observations. In terms of a quasi-all-sky protocol, fundamental infrared atmospheric radiative flux components are calculated: at the top boundary, the outgoing long wave radiation, the surface transmitted radiation, and the upward atmospheric emittance; at the bottom boundary, the downward atmospheric emittance. The partition of the outgoing long wave radiation into upward atmospheric emittance and surface transmitted radiation components is based on the accurate computation of the true greenhouse-gas optical thickness for the radiosonde data. New relationships among the flux components have been found and are used to construct a quasi-all-sky model of the earth’s atmospheric energy transfer process. In the 1948-2008 time period the global average annual mean true greenhouse-gas optical thickness is found to be time-stationary. Simulated radiative no-feedback effects of measured actual CO2 change over the 61 years were calculated and found to be of magnitude easily detectable by the empirical data and analytical methods used. The data negate increase in CO2 in the atmosphere as a hypothetical cause for the apparently observed global warming. A hypothesis of significant positive feedback by water vapor effect on atmospheric infrared absorption is also negated by the observed measurements. Apparently major revision of the physics underlying the greenhouse effect is needed.

    There is another introduction to his ideas at http://jennifermarohasy.com/blog/2009/05/the-work-of-ferenc-miskolczi-part-1/ at the end of which is a download link for the paper in PDF format.

    Basically what he is saying is that the optical depth of the earth’s atmosphere has remained constant over the period of greatest increase in atmospheric carbon dioxide, leading to the conclusion that the optical depth is a constant and that the increase in atmospheric CO2 will result only in a corresponding decrease in atmospheric water vapour, cancelling any increase in the ‘Greenhouse effect’.

    Maybe Miskolczi’s ideas give credence to what Joanne Nova has been contending for, that the ‘back radiation’ is real. However, at the same time, he shows that it is an irrelevancy to the issue of the temperature near the surface of the earth, which is really what all the fuss is about.

    Surely this makes the discussion on this thread entirely academic and beside the point. QED.

    Paul

    00

  • #
    cohenite

    Nullius@199; thanks for the comment; the microstate and entropy relationship is the view of Nasif which I refered to @117, the salient bits appear to be:

    “This shift was important because it defined the real concept of entropy and detached it from contextual derivations. For example, now we know that the fundamental concept of entropy has nothing to do with disorder, movement, complexity or heat “content”, but with the configurations that the energy adopts in a given system and the directionality of the energy exchange.

    Entropy is now defined as the natural trend of the energy to flow towards the system or systems with a higher number of available microstates.”

    The end result of this entropy ‘trend’ is expressed by Zeroth’s law. If we assume that the system is governed by maximum entropy production [MEP] then MEP will be governed by the isotopic nature of backradiation, the basis of the GHE, so that a Zeroth’s condition will be achieved, with increasing CO2, as described by a simple limiting sum geometric series: Sn=a/1-r where a=1 and r=0.5; or 1/n^2 for n=1 to infinity is 2. In otherwords, the temperature or heating effect of a state of 2XCO2 will be achieved at infinity.

    The point here is that, even accepting the consistency of the GHE with the 2nd law, the actual heating effect is negligible.

    00

  • #
    cohenite

    Paul@207; I have been a fan of Miskolczi; his theory is given a stern critique here:

    http://scienceofdoom.com/2011/05/15/the-mystery-of-tau-%e2%80%93-miskolczi-%e2%80%93-part-five-equation-souffle/

    The thing with M though is that his theory is based on data; the theory may be an Ansatz which unusally follows the data but even if it fails the data remains; and at this stage M’s predictions have been far more successful than AGW modeling.

    00

  • #
    Nullius in Verba

    “It isn’t accomplished by microwave photons but rather by phase variation which causes dialectric molecules (primarily water in this case) to constantly re-orient with the magnetic phase.”

    Calling photon absorption “phase variation” doesn’t make it not photons. Photons come in, they interact with charges applying EM forces to them, the charges move around as a result, and their movement generates a back field that cancels out the original photon. In other words, the photon has been absorbed. That’s just the classical picture of how photon absorption works.

    The plastic bowl is cool because it is transparent.

    “Microwave radiation can’t raise the temperature of anything that isn’t within a couple degrees of absolute zero.”

    Do you seriously propose to stand up and tell the general public this is what climate sceptics believe, and expect them (or other physicists) to regard us as a credible alternative?

    00

  • #
    Nullius in Verba

    “If we assume that the system is governed by maximum entropy production”

    Why?

    “then MEP will be governed by the isotopic nature of backradiation”

    Do you mean “isotropic”?

    Iff you want to calculate the heating effect of isotropic radiation from each layer, look up my “water pond” example above, or my “shells” model in the previous thread.

    “The point here is that, even accepting the consistency of the GHE with the 2nd law, the actual heating effect is negligible.”

    If you accept the basic assumption of the shells model, that radiation is the only significant mode of heat transfer, then the heating effect is not negligible. The surface of the Earth would be at an average of 60 C.

    But because we live in a convective atmosphere, the back-radiation effect is overruled by convective transfer. *If* the atmosphere was non-convective, then back-radiation *would* result in a higher surface temperature with no violation of the 2nd law. But because the atmosphere is convective, the mechanism is entirely different, and all the above discussion has absolutely no bearing on it. You can’t prove that the GHE has no effect by arguing against the wrong explanation.

    Since you said “even accepting the consistency of the GHE with the 2nd law” we might actually get a chance to move on to that, once everyone else has agreed too. The back-radiation version of the GHE is wrong, but *not* because it violates the 2nd law. It’s wrong because it ignores the effect of convection.

    00

  • #
    Alistair

    207: Miskolczi also calculates that the no amplification climate sensitivity of CO2 is c. 0.5K. There’s a second claim of this in the literature.

    00

  • #
    Bryan

    Nullius in Verba says on microwave cookers which he feels are relevant in climate science discussion.

    “Do you seriously propose to stand up and tell the general public this is what climate sceptics believe, and expect them (or other physicists) to regard us as a credible alternative?”…..

    I said

    Microwave oven.
    Cold matter emits blackbody spectra in the microwave region of the spectrum. The cosmic microwave background radiation is 3K for instance.
    How can a microwave oven can heat up a bowl of soup?
    It isn’t accomplished by microwave photons but rather by phase variation which causes dialectric molecules (primarily water in this case) to constantly re-orient with the magnetic phase. Friction then (plus choice of resonant frequency) does the actual heating.
    The plastic bowl is quite cool.
    Microwave radiation can’t raise the temperature of anything that isn’t within a couple degrees of absolute zero.

    These devices do not depend on thermionics for their operation.
    To try to describe some thermal phenomena thing like the atmosphere through them is a pointless distraction!

    Well read this and judge for yourself
    http://www.gallawa.com/microtech/howcook.html

    00

  • #

    Nullius in Verba@201 ]

    Are you seriously going to argue that there is not an extinction rate for specific wavelength radiation in our atmosphere? Do you seriously think that there are any original CO2 or H2O photons that make it from ground to outer space with no intermediary steps along this path? It has been studied, it has been observed and it has been recorded that in our atmosphere, the radiation wavelengths and quantity that CO2 absorbs is absorbed fully within the first 6 feet of atmosphere. What an IR camera sees is the last bit of radiation re-emitted as well as the radiation wavelengths not absorbed by the atmosphere.

    If the atmosphere can absorb (A) units of radiation going upward in any specific distance (Y), when it is re-emitted, assuming a 100% re-emit rate, (A/2) units on average and theory point downward(ish). Those (A/2) units headed downward(ish) now will be absorbed at the same rate as the 1,000,000 units and will be absorbed before they reach the specific distance (Y). This is giving benefit that the direction is specifically downward, which only a tiny part will be, the vast majority of the photons re-emitted will be headed in different vectors requiring far more travel than specific distance (Y). Even playing the greenhouse gas theory 100% with the rules the extremists of the pro AGW theory propose, the net effect at surface is net 0 increase in back radiation energy. ]

    This leaves you with the unbelievably difficult task of showing that a low specific heat molecule such as CO2 that is in minute quantities 390ppm or .039% of the atmosphere, can absorb what energy is provided by blackbody Earth radiation and then in turn physically warm the atmosphere by a measurable amount. I do not buy it. At temperatures similar to Earth of today, the amount of CO2 in the atmosphere was 20 times as high as it is today. 1, 2, 4, 8, 16. If the warming ability then was the same as claimed today, then the minimum higher temperature would have been +5 degrees. But the temperature was essentially the same. Let us not even argue about the ice ages and the fact that during most of them in the past the CO2 level going in, during and coming out was higher than it is today. greenhouse gas theory at every single instance where empirical evidence is used fails. The only place that it succeeds in increasing the temperature of the Earth is in the parallel universes created in computers. Every other place, it fails.

    Yes, back radiation from a cold object can increase the energy of warmer object, hurray. Now back to reality, the photons emitted from the Earth in practice essentially none again reach the Earths surface.

    00

  • #
    Nullius in Verba

    “These devices do not depend on thermionics for their operation.”

    Did you really mean to say that?
    http://hyperphysics.phy-astr.gsu.edu/hbase/waves/magnetron.html

    Yes, please do judge for yourselves.

    00

  • #
    Nullius in Verba

    “Are you seriously going to argue that there is not an extinction rate for specific wavelength radiation in our atmosphere?”

    I’m going to argue that the extinction length for thermal IR from the surface is longer than 6 feet, yes.

    But if you want to know the effect of the radiative GHE in a medium where the extinction length is on the order of millimetres, than see my solar pond example above.

    00

  • #
    Alistair

    216: the practical measure of extinction length for IR in the atmosphere can be seen in the phenomenon of the mirage.

    00

  • #
    Bryan

    Nullius in Verba

    “These devices do not depend on thermionics for their operation.”

    Did you really mean to say that?

    The point I was making was that the thermal route as understood in the atmosphere was not involved.
    They incorporate a hot plate to release electrons which are manipulated in an EM field.
    We could therefore use an old style TV to tells us something about the atmosphere.
    Claes Johnston would also be happy to discuss this area as we need no photons are necessary for the explanation.
    We are firmly back in Rayleigh-Jeans formula territory.
    Has the discussion on microwave cookers proved a pointless distraction – YES.

    00

  • #
    Nullius in Verba

    #217,

    That’s conduction/diffusion.

    00

  • #
    Nullius in Verba

    #218,

    A claim has been made that for each temperature there is a threshold photon energy such that warm bodies cannot absorb photons with lower energy than the threshold. The microwave oven is a counter-example to that claim. Microwave photons, which you would claim cannot be absorbed by anything much above absolute zero, are absorbed by food close to room temperature. Claim falsified.

    That the photons are emitted by a different mechanism is irrelevant. That the classical approximation is closer for lower frequencies is irrelevant. Radiation corresponding to extremely low temperatures is absorbed very easily by matter at high temperatures, heating it. The claim that they’re not is wrong. The claim that this threshold effect falsifies the radiative GHE is incorrect.

    At what point can we move on? The 2nd law objections are invalid. There are far better objections we could be raising. But instead we waste hours and hours going over it again and again. And every time I make some slight progress in persuading other scientists to take a look at climate scepticism more seriously, it takes them about 5 minutes for them to run into this stuff and for all my efforts to get blown out of the water.
    I’m tired of it happening.

    But it just goes on, and on, and on.

    00

  • #
    Bryan

    Nullius in Verba

    You have completely ignored the actual mechanism here.
    The laws of thermodynamics were built around equilibrium thermodynamics.
    The microwave oven certainly does not fit in here.

    The mechanism causes phase variation between the electric and magnetic fields which in turn causes dielectric molecules (primarily water in this case) to constantly re-orient with the magnetic phase. Friction then (plus choice of resonant frequency) does the actual heating.
    I have not come across any description such as a steady constant absorption of microwave photons as you seem to imply.
    I still say the microwave cooker has nothing in common with atmospheric effects.

    You say

    …”At what point can we move on? The 2nd law objections are invalid.”….

    In what sence are “2nd Law objections” invalid?
    This makes no sense.
    If any proposed explanation involves 2nd law violation it is the proposal that is invalid.

    In the previous thread someone proposed that the atmosphere was like a heat pump extracting heat from the above the troposphere and depositing it at the Earth surface.
    Explanations of that type are not unusual and so must be analysed.

    00

  • #
    Nullius in Verba

    “The laws of thermodynamics were built around equilibrium thermodynamics.”

    The laws of thermodynamics apply universally, not just at equilibrium. Otherwise we would just have to make sure things were not at equilibrium to be able to break them.

    “The mechanism causes phase variation between the electric and magnetic fields which in turn causes dielectric molecules (primarily water in this case) to constantly re-orient with the magnetic phase.”

    Yes. And photon absorption by atoms is when an electron spinning around an atom is forced to constantly reorient with the electric field in a higher frequency wave. If the photon frequency matches the natural frequency at which the electron spins around the atom, they can interact. It’s just the classical version of EM absorption. The particle and wave pictures are both approximations to the quantum picture. The wave picture may be more accurate at microwave frequencies, but I can assure you that molecular vibrations are very much at a scale where quantum effects are important, and transitions are still quantised. The universe is entirely run according to quantum physics.

    “If any proposed explanation involves 2nd law violation it is the proposal that is invalid.”

    And if any proposed explanation does not involve 2nd law violation, then claims that it does are invalid.

    “In the previous thread someone proposed that the atmosphere was like a heat pump extracting heat from the above the troposphere and depositing it at the Earth surface.”

    Yes, I think that might have been me – although it wasn’t supposed to be an explanation, and isn’t talking about the radiative GHE. Moving on to analyse it would be great, but we can’t get there until we get over this 2nd law/absorption thing for the pure radiative effect.

    00

  • #
    Bryan

    Nullius in Verba says

    “The laws of thermodynamics apply universally, not just at equilibrium. Otherwise we would just have to make sure things were not at equilibrium to be able to break them.”

    A number of the laws specifically are based on, not only general thermal equilibrium, but also local thermal equilibrium – like Kirchoff’s Law for instance.

    Any explanation I have read about microwave ovens specify a friction effect that causes heating.
    I think its pretty clear that microwave ovens have nothing in common with the atmosphere.
    If you have to extend to such exaggerated examples to make your point I conclude that you have very little evidence to support your opinion.

    Nullius in Verba

    “And if any proposed explanation does not involve 2nd law violation, then claims that it does are invalid.”

    Who is making these invalid claims?

    In the previous thread someone proposed that the atmosphere was like a heat pump extracting heat from the above the troposphere and depositing it at the Earth surface.

    Nullius in Verba
    “Yes, I think that might have been me”

    Well I didn’t actually think it was you, however if there is no evidence of a giant pump in the sky then back to the second law or a suitable excuse from you.
    Nullius in Verba
    “Moving on to analyse it would be great, but we can’t get there until we get over this 2nd law/absorption thing for the pure radiative effect.”

    Well you will just have to accept that you have not convinced me of either and you still have the problem of the giant unexplained pump in the sky!

    00

  • #
    Richard111

    Look up Wien’s Law. Peak temperature for a body radiating at 15 microns is 193.2K or -80C. The energy contained in 15 micron photons are equivalent to radiation from a body at -80C.

    00

  • #
    Alistair

    There is no heat pump in the sky. This is because the law of Equipartition of Energy means that all GHGs do is to increase the optical depth of the atmosphere to certain bands of IR energy but with no extra thermalisation [another thermally-excited molecule re-emits the quantum of energy in a random direction to maintain statistical equilibrium].

    Heat is stored in clouds directly in droplets and indirectly as you get a bit more evaporation, hence latent heat energy. An increase of CO2 would, because it causes more absorption, lead to an increased probability of that energy capture. That local change in lapse rate has to be compensated for in the upper atmosphere where it is known that water vapour concentration has been decreasing to give constant IR optical depth.

    This is Miskolczi’s physics but he doesn’t describe the mechaniism.

    00

  • #
    Nullius in Verba

    “A number of the laws specifically are based on, not only general thermal equilibrium, but also local thermal equilibrium – like Kirchoff’s Law for instance.”

    Yes, there are such laws, but they’re only special cases of the more general laws. They’re not given numbers.

    “Any explanation I have read about microwave ovens specify a friction effect that causes heating.”

    They’re quite right. And when absorption of blackbody radiation leads to heating, that’s caused by ‘friction’ too. The radiation causes the electrons to spin, the spinning electrons interact with electrons in nearby molecules giving both molecules a kick, which sets the whole process off. How did you think blackbody absorption led to heat?

    “Who is making these invalid claims?”

    Good question. Hands up anyone who still thinks the radiative GHE mechanism violates the second law?

    “Well you will just have to accept that you have not convinced me of either and you still have the problem of the giant unexplained pump in the sky!”

    Yes, I know. And believe me, it’s not a surprise to me. I did think Jo was being unduly optimistic in thinking that she just had to host a couple of discussions and it would all be sorted out.
    The Hadley cell heat pump will have to wait for another day.

    00

  • #
    Alistair

    Ah, the Hadley cell heat pump; that’s different.

    Question: do you consider the indirect creation of latent heat energy a form of greenhouse heating?

    00

  • #
    Reed Coray

    “Technically, strictly, greenhouse gases don’t “warm” the planet (as in, they don’t supply additional heat energy), but they slow the cooling, which for all pragmatic purposes leaves the planet warmer that it would have been without them.” – Jo Nova

    Jo, they (greenhouse gases) may slow cooling; but if by cooling you mean decreasing the rate that energy is lost to space, the effect is only temporary. Specifically, if the rate of energy input to a system always exceeds the rate of energy output by the system, the difference in energy must be going somewhere. Some of it may assume forms not directly related to temperature–e.g., chemical energy storage as happens in photosynthesis. Some of the energy difference, however, will almost surely manifest itself as a rise in temperature of the system that experiences the net input of energy. As long as the net increase in energy exists, the temperature will rise. By definition, for energy-rate equilibrium to exist, the rate energy is input to a system must equal the rate energy leaves the system. If we restrict energy flow into and out of a system to be radiation (no convection, conduction, or loss of mass), the temperature of the system always has an effect on the rate energy leaves system but may or may not have an affect on the rate energy enters the system–e.g., blackbodies radiate energy proportional to the fourth power of their temperature, but absorb energy independent of their temperature. If a system (say the Earth and its atmosphere) receives energy from an external source (say the sun) in energy-rate equilibrium is changed in such a way that (a) the rate of energy loss is decreased and the ONLY way to increase the rate of energy loss is to raise the system temperature, then the temperature will rise until energy-rate equilibrium is re-established. If the input rate of energy is unchanged, the rate of cooling before the system change will be equal to the rate of cooling after the system change–not immediately, but eventually.

    Thus, I agree with you in spirit. By placing a “cooler” object next to a “hotter” object, in the presence of a thermal energy source, the temperature of the “hotter” object can be changed and the change may be to increase the temperature of the “hotter” object. However, I make two comments. First, the minor objection that I don’t like the terminology “an atmosphere slows cooling”–because the “slowing” is transient. Although much more complicated, I like the terminology “an atmosphere may alter the behaviors of both energy absorption and energy loss in such a way that the change in energy absorption is small but the rate of energy loss is affected in a way that requires a temperature increase to re-establish energy-rate equilibrium.”

    Second, assuming a fixed input rate of energy, it is not inconceivable that the presence of an atmosphere might result in a decrease in system temperature. Consider for example, a uniform-temperature, isolated, blackbody sphere with an internal thermal energy source of W watts in the vacuum of cold space. The energy-rate equilibrium temperature, TE, of the sphere will be directly proportional to the fourth root of the ratio of (a) “W” to (b) the surface area of the sphere. In the case where the “atmosphere” is the same material as the sphere, by adding an atmosphere the area of the sphere is increased with a resulting decrease in the sphere surface temperature when energy-rate equilibrium is re-established. In the case of the Earth receiving energy from the Sun, you can correctly argue that an increase in the size of a blackbody sphere will result in an increase in “received” energy. However, my point is that the presence of an atmosphere may (likely will) induce thermal affects other than “absorption of electromagnetic bands of outgoing radiation.” For this reason, I am not ready to accept the premise that the presence of “greenhouse gases” in the Earth’s atmosphere must result in a temperature increase of the Earth’s surface. I admit the possibility that the premise is true, but I’m a long way from being convinced.

    Thank you for an excellent blog,

    00

  • #
    KR

    Paul @ 207

    Basically what he is saying is that the optical depth of the earth’s atmosphere has remained constant over the period of greatest increase in atmospheric carbon dioxide, leading to the conclusion that the optical depth is a constant and that the increase in atmospheric CO2 will result only in a corresponding decrease in atmospheric water vapour, cancelling any increase in the ‘Greenhouse effect’.

    But – atmospheric water vapor has increased ~4% since 1970, along with the increases in CO2. This does not hold up to reality.

    00

  • #
    Nullius in Verba

    “Question: do you consider the indirect creation of latent heat energy a form of greenhouse heating?”

    Mmmm. Tricky. It depends how you choose to define it.

    If we choose to define the GHE as anything that moves the lapse rate around – up, down, steeper, or shallower – with a resultant change in surface temperature as “greenhouse heating”, then yes. But latent heat isn’t really a ‘creation’ of energy, and as with GHGs, the mechanism isn’t a simple ‘adding more energy to make it get hotter’ sort of “heating”, so I’d be a bit wary – especially in light of the terminological issues we’ve already seen – in calling it that.

    That would be better discussed after properly discussing the lapse rate mechanism, though. And Jo wanted us to get the 2nd law business sorted out first. So I consider it off-topic for now.

    00

  • #
    Alistair

    KR: there’s a recent report of stratospheric water vapour decreasing by 25%, having increased from the 1970s, also Miskolczi showed that 61 years’ radiosonde data showed a decrease of water vapour.

    00

  • #
    KR

    . Alistair @ 231

    “…there’s a recent report of stratospheric water vapour decreasing by 25%”

    Trenberth 2005 shows 1.3±0.3% per decade over the oceans.

    Zhai 1997 – Increases of precipitable water over China.

    Nedoluha 1998 – entitled “Increases in middle atmospheric water vapor as observed by the Halogen Occultation Experiment and the ground-based Water Vapor Millimeter-wave Spectrometer from 1991 to 1997”

    Can you point me to your reference? That doesn’t match with any of the data I’ve seen.

    00

  • #
    Paul

    It all boils down to whose data you are going to trust.

    The CAGW camp have blotted their copy book so many times in regard to making their data and methods easily available for verification, combined with sneaky corruption of the data, invalid statistical analysis of the data along with suppression of the real peer-review process that I take anything that emanates from that camp with a bucket full of salt.

    As for me and many others, the recent non-warming period, going on for more than a decade, puts the fear of catastrophic global warming firmly to rest.

    It’s instructive that none of the models relied on by the IPCC had any predictive power regarding this stasis although such cyclical patterns are very evident in the past temperature records. It is also instructive that the long-range weather forecasts based on the same modelling have become nothing more than a sick joke and subject to well-deserved derision whereas long-range weather forecasts based upon scientific and historical data, but ignoring the supposed ‘Global Warming’ conjecture, have had a better than 85% accuracy rate, and improving.

    Why there are two threads now, discussing the second law of thermodynamics, I don’t understand. On the former thread I have posted my corrected understanding of how the presence of CO2 alters the passage of long-wave radiation through the atmosphere and conclude that it has no effect at all.

    Given the above, the amount of hot air on these two threads has probably had more effect on the temperature of the near-earth atmosphere than any increase in ‘carbon’!

    Paul

    00

  • #
  • #
    cohenite

    Nullius@211; good of you to pick up the misspelling of isotRopic!

    You ask why should MEP apply to all energy and heat transfers; some of this discussion has implied that entropy may have more than one speed; I think it was SoD who suggested that the 2nd law says entropy cannot be reduced but had nothing to do with increases of entropy; with respect to him I think that misses the point that all natural systems are constrained by entropy at one level with no capacity to either increase or decrease the rate; you may have a number of factors or processes operating similtaneously, in the nature of forcings and feedbacks, but all of those have the same ‘rate’ of entropy; perhaps it would help to consider the Constructal Law:

    http://wattsupwiththat.com/2009/12/27/the-unbearable-complexity-of-climate-2/#more-14585

    Speaking of SoD I see you participated in the discussion on pressure vs radiative transfer and which establishes the lapse rate:

    http://scienceofdoom.com/2010/08/16/convection-venus-thought-experiments-and-tall-rooms-full-of-gas/

    The previous discussion also didn’t resolve the issue:

    http://scienceofdoom.com/2010/06/22/venusian-mysteries-part-two/

    But you are wrong when you say this: “If you accept the basic assumption of the shells model, that radiation is the only significant mode of heat transfer, then the heating effect is not negligible.” The heating effect from radiation in an isotropic model will be governed by the limiting sum geometric series; how could it not?

    This doesn’t mean I don’t agree with you about convection dominating radiative transfers; my point is that in a situation where there is no convection the radiative process will not cause excessive heat build-up.

    00

  • #
    KR

    cohenite @ 234

    Thanks for the references – I’ll take a look.

    cohenite @ 235

    I’ve read up on MEP some time back, due to a previous discussion on another blog (Skeptical Science). Maximum Entropy Production appears (still rather an open question) to apply to how fast a system attains a minimum ordered energy (maximum entropy) state, but it does not affect the final state reached.

    00

  • #
    John Brookes

    Nullius in Verba@230 says that discussion of the lapse rate should be another topic.

    Yes, please Jo, could we have a post on the lapse rate? I have only a tenuous grasp of this part of the science, and need it put in nice simple terms.

    00

  • #
    Kevin

    David a wrote (@ comment #107)

    “For example, over the course of a night overall heat loss is reduced if the atmosphere slows the rate of heat loss.”

    EXACTLY my point, ONLY IF THE “GREENHOUSE EFFECT” can slow the rate of heat loss (i.e. the “speed of heat”) in a significant amount relative to the speed of heat arriving (the SUN rising each day) can a higher “equilibrium temperature” exist.

    So the “GE” effect slows the speed of heat through the system by a few hundred milliseconds (maybe), but every 86 million milliseconds the “heat batteries “of the Earth get a MASSIVE recharge (i.e. the sunrise).

    It all comes down to the speed at which heat flows through the system. Unless the “GE” can change that in a significant way (like a sizeable portion of a day, say 12 hours, or 43 million million milliseconds) the “Greenhouse Effect” is doing nothing but causing us to waste a TON of money studying it, and trying to alleviate it’s alleged affects.

    Cheers, Kevin.

    00

  • #
    Bobc

    I keep seeing this stuff about photons:

    Bryan: (@200)
    2011 at 8:20 pm

    RJ

    To disprove the existence of Photons is much harder than any aspect of the climate science controversy.

    Is there some experiment to show that photons don’t exist?
    Is there some natural phenomena that photon theory contradicts?

    G&T are very much in the mainstream of orthodox physics and in their publications accept and use radiation transmission through photons.

    Thought I might summarize one of my graduate lectures on optics (the ones where I wax philosophical — my students love it, as they don’t have to remember anything)

    First, though: I am very much in the mainstream of orthodox optical engineering, and in my publications I have accepted and used radiation transmission through rays of light. There is no need, however, to wonder if rays can be shown to exist — they are blatantly mathematical abstractions used to simplify (greatly!) the calculation of wave propagation through optical systems sufficiently larger than the wavelength of light.

    If I ever asked what would happen if a ray of light struck an atom, however, I would have passed “through the looking glass” — a ray, being a mathematical abstraction, cannot ‘hit’ an atom, a real object. This would be a severe logical “catagory error”.

    Back to photons: There is a vast body of theory and experiment on the generation, propagation, absorption, and everything about Electromagnetic Waves. This theory (whose crown jewels are Maxwell’s Equations) describe electric and magnetic fields and waves in exquisite detail, allowing us to design and construct the electrical and electronic devices that our civilization is built upon. Although there have been many innovations since then, the theory was relatively mature at the end of the 19th century.

    There was just one little problem with it.

    Although the spectrum of heated blackbody radiators could easily be measured, all attempts to derive it from EM theory failed. Every attempt resulted in the nonsensical conclusion that any object above absolute zero in temperature would lose all its energy in a devastating burst of gamma radiation. This problem was called the “Ultraviolet Catastrophe”.

    Max Planck (whose University advisor told him not to go into physics, as “Everything has already been worked out, except for a few little details”) went into physics anyway, and decided to try to work out this “little detail”. After much trial and error, Planck found that if he made the apparently arbitrary assumption that EM waves could only interact with matter in discrete chunks of energy, the size of which was proportional to the wave’s frequency — and further carefully selected the value of said proportionality constant (now called Planck’s Constant), he could calculate blackbody spectra accurately. Fortunately, peer review was rarely used back then, so this totally bizarre result got published. This is generally accepted as the begining of Quantum Mechanics.

    Inspired by Planck’s success, Einstein used these assumed “quanta” to explain the photoelectric effect, for which he received the Nobel Prize. (That’s right, it wasn’t for Relativity.)

    After that, things went nuts and no one was heard to say anything like Planck’s advisor said for the next 50 years or so.

    Now, we know beyond all doubt that Planck’s assumption is true — we can even see these interactions with our own eyes by using sintillation screens. These interactions look so much like the interactions of particles that we have given the assumed particles a name — “Photons”.

    However, as to the “Theory of Photons” — it remains exactly where it was when Planck initiated it — EM waves interact with matter in discrete quanta of energy. Period. Full Stop.

    Let me be clear: We have NO theory that can describe a “Photon” — we can’t say what it’s physical size is, where it is, how it propagates, reflects or refracts, how it is created or destroyed. No, the theoretical calculations that do that all deal with waves.

    Of course, we all talk about photons propagating. (I also talk about rays propagating, so be warned.) You can even make up rules about photons propagating, etc — as long as you don’t violate any of the actual wave properties. (Just as rules of “ray propagation” through optical systems are devised so as to approximate wave propagation.)

    Photons, however, are in our mind. In the real world what you have is: “EM waves interact with matter in discrete chunks of energy”.

    00

  • #
    BobC

    Stylo (@191):
    May 22nd, 2011 at 4:36 pm

    Paradoxical as this seems, there is no heating from other objects that are at or below the critical threshold of equal temperature.

    If this energy cancellation wasn’t true you could sit in your lounge room and get warmed up to a higher temperature by the TV and the lounge and the walls and the carpet backradiating all of that nasty energy which “must” have some effect. But, you know what? It doesn’t warm you at all!

    But it does in fact warm you, as is obvious in Montana when it’s -40C outside.

    Here is an experiment you can try out for yourself, Stylo, the next time it’s cold enough where you are. Sit next to an insulated outside wall long enough to find the comfortable amount of clothing to wear. Then go sit next to a large window — you will definitely feel colder. Neither the wall nor the window is warmer than your body — but the wall is warmer than the window and hence will radiate more IR.

    If you ever have the pleasure of living in a home with radiant floor heating, you will discover that, by heating the floor to a moderate temperature (say 80 degrees F. or so — still well below body heat) you can be very comfortable with a much lower air temperature than possible in a house where only the air is heated. Houses that use this heating method generally are built so that much of the IR emitted by the floor is reflected from the ceiling, increasing the IR flux in the room and hence the comfort level of the occupants.

    Hey, does this sound a lot likt the “Radiant Greenhouse Effect” or what? And, you can experience it yourself! Here is a company that sells radiant floor tiles — put them in your bathroom, you’ll like it!

    00

  • #
    Bryan

    Bobc: thanks for the post and you say;

    ..”Photons, however, are in our mind. In the real world what you have is: “EM waves interact with matter in discrete chunks of energy”.

    As Ive said above I would like to see Claes Johnston restore causality but I feel he still has some distance to go.
    For the moment photon exchange is the mainstream physics interpretation of radiative transfer.
    I was watching a Feynman video recently and he was saying(1960’s) that they could detect a single photon.

    00

  • #
    Bryan

    Nullius in Verba:

    I was a bit puzzled yesterday about where you were going with the microwave oven thing.
    Its clearly a resonance-friction effect because then you move significantly away from the frequency used the effect disappears.
    However the discussion reminded me of an exchange I had with scienceofdoom so Ill put the same question to you.
    Do you think that 1000Joules of blackbody radiant energy centred around say 2um is thermodynamically equivalent to 1000Joules of blackbody radiant energy centred around say 20um

    a) In terms of the First Law of Thermodynamics
    b) In terms of the Second Law of Thermodynamics.

    00

  • #
    Alistair

    KR 232: Solomon’s paper [ http://www.sciencemag.org/content/327/5970/1219.abstract ] shows a fall in stratospheric water of c. 10% since 2000 has contributed a fall in the rate of increase of temperature due to global warming by c. 25%.

    Sorry to mix this up.

    However, I do not believe any of the theories of GHG warming are valid because they all break fundamental statistical thermodynamic principles and it may be only because IR absorption measurements uses IR absorbing containers that you see warming!

    In the container-less atmosphere, what matters is second order intermolecular [Van der Waals] interaction effects and recent work shows CO2 with O2 should actually hav a negative amplification factor for GHG warming [ C R Anderson is involved]. You get to this from experimental observations, not theory, classical physics in action.

    This game is not over by any means and the scientific horsepower being exerted to get toe the truth dwarfs the intellectusl back-water which is climate science.

    00

  • #
    RJ

    BobC @ 239 AND 240

    Thanks for these posts

    Re this point though

    Hey, does this sound a lot likt the “Radiant Greenhouse Effect” or what

    Is this statement misleading.

    Would this radiation be at the same wavelength as the backradiation from the atmosphere. Or a higher energy wavelength that can cause warming

    So is it more like radiation from the sun (a warmer body). Or radiation from the atmosphere (a colder body).

    This is a question BTW as I do not know.

    00

  • #

    RJ,

    Would this radiation be at the same wavelength as the backradiation from the atmosphere. Or a higher energy wavelength that can cause warming

    This is a non-sensical statement.

    That is to say, any energy can cause warming (except that some radiation has too much energy to cause warming). Radiation doesn’t need to be “high energy” to warm, but rather it needs to be of the correct wavelength to be absorbed by the substance being warmed and translated into vibrational or rotational energy.

    For example, X-rays and gamma-rays are very, very high in energy, and yet they don’t tend to warm anything. They pass harmlessly through most objects, with the occasional random and very damaging absorption. I say damaging because these photons are so high in energy that they don’t make the target molecule vibrate or rotate. They instead work at the subatomic level and change the energy level of electrons, perhaps even causing one to be ejected, which damages the stability of chemical bonds and destroys (alters) the molecular state. Gamma rays are so high level they actually can work on the nuclear and particle level, causing nuclear reactions (i.e. changes in the protons and electrons in the nucleus of an atom).

    Next up in energy is the ultraviolet, for which many molecules are transparent except a handful of important ones in the atmosphere (O2 being a very important one, since it results in blocking UV). But UV is again so high in energy that it tends to damage the molecules that absorb it (i.e. it works not by making a molecule vibrate or rotate, but instead by exciting or even ejecting electrons, resulting in chemical reactions that change the nature of the substance). This is more likely to happen with more complex molecules (i.e. solid objects), although the common reaction of O2 + UV –> O3 (ozone) daily saves us all from death.

    Next up is visible light. Solid objects tend (with rare exceptions) to be opaque to visible light, while the atmosphere is transparent. But the energy in this light is in the range to heat (vibrate the molecules of) the solid objects that absorb it.

    Next up in infra red. This is where things get tricky. Solid objects (unless they are very, very hot, like glowing embers) do not tend to emit visible light. They shed their heat (molecular vibrational energy) by emitting in the infrared. This can in turn be absorbed by various molecules in the atmosphere, depending on their chemistry/geometry/physics. This is what we’re discussing.

    You can’t break radiation into “warmer” and “cooler”. You can profile the energy emitted by a substance, based on its heat and nature (e.g. the sun is hot, and emits primarily in the visible spectrum), and you can profile how a particular substance will deal with a particular wavelength, but you can’t really profile a particular wavelength as “hot” or “cool”.

    In particular, the thought that higher energy wavelengths are the ones that can cause warming is incorrect.

    [Note that this discussion is of course abbreviated as much as possible, and as such both leaves information out, or plays a little loose with some details to avoid in getting bogged down in complexities. It’s just meant to be a broad overview, to present the concept of the range of energies and their varying effects on the molecular, atomic and sub-atomic levels.]

    Would this radiation be at the same wavelength as the backradiation from the atmosphere.

    If by that, do you mean “is it in the infrared?” then yes. But the range of wavelengths we are talking about for H2O and CO2 are going to be a very specific subset of that full range emitted by the tiles.

    00

  • #

    Typo in above comment: obviously “protons and electrons in the nucleus” should say “protons and neutrons in the nucleus”.

    00

  • #
    BobC

    Hi RJ;

    When I said:

    Hey, does this sound a lot likt (sic) the “Radiant Greenhouse Effect” or what?

    I was being a little flippant. However, I was talking about IR radiation from a radiantly heated floor being reflected from the ceiling, which will increase the radiant flux in the room and hence the comfort level of the occupants. This is standard arcitectural design methodology for radiantly heated rooms. This does have some similarities to the radiant greenhouse effect being discussed here.

    And yes, I’m talking about IR radiation from the heated floor which will be significantly below body temperature. If you heated the floor to body temperature, it would seem uncomfortably hot, as you would not be able to lose heat by radiation (or rather, you would be receiving as much as you lost) and would have to resort to sweating to keep from overheating.

    Here’s a thought experiment:

    Assume you are floating inside a (distant) spherical shell of some material, and that material is at absolute zero – it is not radiating anything at all. Your body, however, is at a temperature of approximately 37 deg C, so is radiating electromagnetic waves in blackbody spectrum characteristic to that temperature. (Note that all blackbodies radiate — and absorb — in all wavelengths. The intensity and peak wavelength is dependent on the temperature.)

    Hence, you are losing energy at a certain rate that your metabolism must make up for in order to maintain your body temperature.

    Now suppose that we heat the distant shell to a temperature above absolute zero, but still below your body temperature. It is now radiating as a blackbody, but with a spectrum shifted toward the IR, with respect to your emissions, since it is cooler than your body. Now you are not only radiating energy, but also absorbing it. You are still losing energy, since you are hotter than the shell, but your net loss is slower.

    Hence, your metabolism doesn’t have to work as hard to maintain your body temperature — you are more comfortable.

    Now, suppose we raise the temperature of the shell to that of your body. The shell is now radiating the same spectrum as you are. This is a problem, as you are now absorbing radiant energy as fast as you are emitting it and your net (radiant) heat loss is zero. Since your metabolism can’t go to zero (as long as you’re alive, that is) your body temperature will rise. This could be dangerous, except Humans can also shed heat by exuding water from their skin pores which absorbs energy as it evaporates (e.g., “sweating”).

    Note that, in all these scenarios, the direction of net heat flow is from the hotter object to the cooler — the second law is upheld. The rate of heat flow, however, is dependent on the difference in temperatures, going to zero as the difference does.

    Note that the distant shell does not have to “heat” you to cause you to be warmer — since you have your own energy source (your metabolism) all it has to do is slow down the rate at which you lose heat.

    (Did you know that Humans and Horses are the only animals that sweat? How strange is that? Growing up on a ranch, I always thought horses were the animals most like humans mentally.)

    RJ:
    May 23rd, 2011 at 6:51 pm

    BobC @ 239 AND 240

    Thanks for these posts

    Re this point though

    Hey, does this sound a lot likt the “Radiant Greenhouse Effect” or what

    Is this statement misleading.

    Would this radiation be at the same wavelength as the backradiation from the atmosphere. Or a higher energy wavelength that can cause warming

    So is it more like radiation from the sun (a warmer body). Or radiation from the atmosphere (a colder body).

    This is a question BTW as I do not know.

    00

  • #
    KR

    cohenite @ 234

    Again, thanks for the references.

    It looks like stratospheric H2O has decreased over the last decade, after increasing between 1980-2000. The Wentz 2007 reference states that:

    “Rather, the observations suggest that precipitation and total atmospheric water have increased at about the same rate over the past two decades.”

    In that regard I’d like to point you to this list of water vapor feedback observation papers – there’s about 18 references there, too many to summarize in this post. Water vapor indeed does appear to increase with warming.

    So, despite variations in rates, both water vapor and CO2 have increased at various points in the last 30 years (1980-2000 by cohenite’s references, for example). Hence Miskolczi’s claim that increases in CO2 are balanced by a decrease in H2O, maintaining constant optical density of the atmosphere, is false. Cancellation of CO2 effect in this fashion simply does not occur.

    00

  • #
    RJ

    Bob @ 245

    Thanks for the reply

    This is a non-sensical statement.

    That is to say, any energy can cause warming (except that some radiation has too much energy to cause warming). Radiation doesn’t need to be “high energy” to warm, but rather it needs to be of the correct wavelength to be absorbed by the substance being warmed and translated into vibrational or rotational energy.

    Any energy can cause warming. OK but is this misleading. Can energy from a cold body heat a warmer one. I know you think it can but others do not agree.

    So I still think the statement by BobC is misleading. He is I think saying it is like the GHG greenhouse effect. When it is more like the impact of IR radiation from the sun.

    You can’t break radiation into “warmer” and “cooler

    But what about for an object break it into has a warming impact or has no impact (warming or cooling). And this impact could vary depending on the objects temperature.

    O2 being a very important one, since it results in blocking UV

    Is this right. Or is it O3 + UV –> O2+O. This is a question as I thought I read somewhere it was O3 that stopped UV light not O2.


    e.g. the sun is hot, and emits primarily in the visible spectrum

    Do you mean visible light. I though the % split was around

    UV 11%
    Visible 40%
    Infa-red 49%

    Is this right or have I misunderstood.

    00

  • #
    RJ

    Bob

    Sorry it is O2 as well as O3. (But then what was the problem with the ozone breakdown. Surely if O3 can be produced from O2 it would simple produce more from O2. Although this is probable not the place to discuss this)

    00

  • #
    BobC

    Bryan:
    May 23rd, 2011 at 5:22 pm

    I was watching a Feynman video recently and he was saying(1960′s) that they could detect a single photon.

    Even your eyes can do this, when they are dark-adapted. Since EM waves interact with matter in discrete units of energy, called “quanta”, when we can detect a single quanta of interaction it is common to say that we have “detected a single photon”.

    What we have never done is observe a photon in flight. It is not even certain that that is a meaningful statement, or that photons represent anything real other than the final interaction of a wave with matter.

    They certainly are a convenient mental concept, however, as long as you don’t get too carried away and hypothesize properties contrary to the known facts of EM theory.

    00

  • #
    RJ

    BobC @247

    Note that, in all these scenarios, the direction of net heat flow is from the hotter object to the cooler — the second law is upheld. The rate of heat flow, however, is dependent on the difference in temperatures, going to zero as the difference does.

    Yes. But the question is are you (BobC) and Bob (sphaerica) right. Or is the slayers view the correct one. So the energy flow is from hot to cold. But the energy from cold to hot does not have a warming impact

    So when discussing heat flow there is no need for net. Because no heat whatsoever flows from cold to hot. Just potential energy that can not be released in an object with a higher vibrational state.

    Hopefully I have expressed the slayers viewpoint correctly.

    00

  • #
    RJ

    Sorry it should be

    so the energy flow is from hot to cold and also cold to hot

    00

  • #
    BobC

    RJ @ 252:
    So when discussing heat flow there is no need for net. Because no heat whatsoever flows from cold to hot.

    Hopefully I have expressed the slayers viewpoint correctly.

    This would be true if “heat flow” is defined as “net energy flow”. Then, by definition, no heat flows from cold to hot.

    Does it seem reasonable to you that the rate of “heat flow” should be, among other things, a function of temperature difference? That is, if a hot object is losing heat to a cold object, the rate will be faster if the temperature difference is large and slower if the temperature difference is slow. (And zero net energy flow — “heat flow” — if there is no temperature difference.)

    Given that condition, it is easy to see that the presence of a cold object may indeed reduce the rate that a hot object loses heat. Consider an object at 100C next to an object at 0 deg C — a 100deg C difference (imagine bricks). There will be a rate of heat flow from the hot brick to the cold one. (The hot brick may be losing heat in other directions as well, but we are going to leave that constant.)

    Now, increase the temperature of the cold brick to 50 deg C. The temperature difference between the hot and cold bricks is now only 50 deg C, so the rate of heat flow from the hot brick to the cold brick must decrease. Since the hot brick’s heat loss in all other directions remains constant, it must be true that the presence of the less cold brick has caused the hot brick to lose heat more slowly than when it was next to the more cold brick. If effect, the less cold brick has acted as “insulation” in that it caused the heat loss to be slower, compared to the loss when next to the more cold brick.

    BTY: It is not necessary to postulate selective absorption behavior to get this exact behavior from blackbodies:
    1) An idealized blackbody will absorb any photon at any energy, gaining energy in the process.
    2) A blackbody will emit photons at an intensity and with a spectrum defined by Planck’s distribution. The key property of this spectrum (for our purposes) is that: The hotter the body, the more energy it radiates.

    Given these two properties, you can see that any two blackbodies will always be exchanging energy, but the NET energy flow (“heat flow”) will always be from the hotter one to the cooler one.

    A “Blackbody” is an idealized object, but many real objects (like a tungsten light filament) come close.

    00

  • #
    mkelly

    BobC:
    May 24th, 2011 at 2:11 am

    (And zero net energy flow — “heat flow” — if there is no temperature difference.)

    BobC if T1-T2 in your statement above leads to a zero on the right side of the equation then there is a zero on the left side. If the left side is W/m^2 or J/sec/m^2 and a joule is a unit of energy there is no energy of any kind being exchanged.

    00

  • #
    BobC

    mkelly:
    May 24th, 2011 at 3:04 am

    BobC if T1-T2 in your statement above leads to a zero on the right side of the equation then there is a zero on the left side. If the left side is W/m^2 or J/sec/m^2 and a joule is a unit of energy there is no energy of any kind being exchanged.

    Let’s take an example: Say the T1 represents the energy flow from object 1 to object 2, and T2 represents the energy flow from object 2 to object 1. (Use whatever units you desire — say Watts).

    T1-T2 then represents the net flow of energy. If T1-T2 = 0, that doesn’t imply T1=T2=0, only that T1=T2. They could both be 100 W, for example.

    00

  • #
    BobC

    I think that many of the posters here have been exposed to some controversial material on physics, without a background in standard physics. Without the knowledge of what we know and don’t know (and, particularly, how we know it), it is difficult to analyze new arguments, such as are apparently in the “slayer” book. If these arguments are filling in gaps in our knowledge, that is one thing — if, however, they are making claims that have been falsified by 100 years of experimental science, that is entirely different. Without the general physics background, it is difficult to know which.

    I would like to suggest the “Physics for Dummies” series of books as an excellent way to gain the standard background information. Don’t be put off by the name — the books are intelligently written and are for people smart and motivated enough to want to improve their knowledge of the subject. Unlike textbooks, which can assume a daunting amount of knowledge just to be able to follow the first several pages, these books make no assumptions about the reader’s background except general intelligence and motivation.

    RJ:
    May 24th, 2011 at 12:12 am
    BobC @247

    Yes. But the question is are you (BobC) and Bob (sphaerica) right. Or is the slayers view the correct one.

    00

  • #
    KR

    BobC @ 257, others

    I agree, BobC, it’s hard to judge any argument without sufficient background.

    I would actually recommend the “Cartoon Guide To Physics” as well. This series is pretty good, very clear, and well worth reading.

    00

  • #

    RJ,

    On ozone, all atmospheric reactions are more complex than they seem. It’s really closer to O2 + uv –> 2O, then O2 + O –> O3 + heat, but in reality a lot more goes on, because there are other competing reactions, other reactants, etc.

    On thermodynamics, you seem to be hung up on the slayer’s book interpretation that there are actual laws at the quantum level that prevent heat from ever flowing from a warm to cool object, in any circumstances.

    I must emphasize that this is flat out false, and is contradicted by every single text or piece of published literature except for the slayer’s book. You are giving “equal footing” about predictions of the end of the world to the haggard, unwashed man on the street corner.

    Consider what must happen for this to be true. You’ve danced around it. Basically, the target of the radiation must know the source, or perhaps the radiation itself must carry some sort of thermal signature so that the destination knows whether or not to absorb that photon. Quantum theory gives each photon momentum (direction and speed) and an energy quantum (wavelength/frequency), and nothing more. You need another quality, a “thermal origin signature,” but you need it to fit a conceptual model for which there is no observational evidence whatsoever (meaning an experiment that shows that warmer bodies will ruthlessly ignore appropriately energized photons from inappropriately warm sources).

    But take a step back. Look at one single molecule in a vacuum. For now, let’s put it way out in space, without another molecule anywhere within miles.

    If that single molecule is not rotating or vibrating, it’s rather hard to assign it a temperature (because it doesn’t exist in a “volume”), but you could try to do so based purely on its translational, kinetic energy and some assumed volume.

    Now hit it with a photon from a star. The star was obviously hot, so the molecule absorbs the photon and starts vibrating. It’s now hotter. If another photon of the same wavelength comes by, it’s ignored (no matter how hot the source), because the molecule has already absorbed radiation in that quantum. It can’t do so twice.

    A little time goes by, and it emits that photon — in effect it cools. It is now free to absorb another photon, from another (or the same star).

    Now expand this model. You no longer have one molecule, but untold trillions of them, in a finite space, perhaps trapped there by gravity. Now the volume of molecules has a measurable temperature. Due to equipartition of energy, the energy states are also someone equally divided among the molecules. Some are spinning, some vibrating, some both, some neither. Some have more translational energy, some less. Some have more total kinetic energy (in your terms, those are “warmer”), some less.

    But they are constantly colliding with each other. With each collision, they trade energy. Some stop rotating/vibrating, some start, some move faster, some more slowly. The combinations are endless, and it is happening untold gazillions times every second.

    On average, each molecule has the same energy, but on an individual basis, each molecule is constantly changing energy levels. On an individual basis, each molecule is getting warmer or colder (if it were really appropriate and possible to measure the temperature of a single molecule). But the temperature of the body is constant.

    And yet, in your model, there would have to be a distinction. How do the molecules within the volume behave when they run into each other? If a molecule is going fast (is hotter) than another, does it refuse a photon from a slower, but vibrating molecule? Are there rules within this horde of molecules about which can and cannot accept what from each other? How can you reconcile your (or slayer’s) laws with behavior within such a complex system?

    Now enter that photon from that far off, distant, burning star. Some molecules are already vibrating in that frequency, and cannot absorb the photon. Some are not.

    It is your position that regardless of this random (but statistically equal) distribution of energy among the molecules in the volume, that if their total temperature is higher than that of the stellar source of this photon, that none of them can absorb this photon. Somehow, the molecules in this particular volume have all agreed that they are a family, and none of them will accept photons from strangers who are “cooler” than they are. This despite the fact that within the volume there are undoubtedly molecules on the extreme “cool” side, not moving particularly fast, not vibrating, not rotating, but part of a larger volume of molecules that are “warmer” than the source of the photon.

    Or take another case. Let’s say a molecule absorbs this energy from the star. Is that particular molecule now warmer than another molecule, vibrating with the same energy, that got it’s vibration from a different, cooler star? What temperature will the volume of molecules now use to decide which photons to ignore?

    It’s all Alice in Wonderland physics. None of it makes sense.

    There are basic, simple, quantum rules. Each photon has an energy level. Each molecule has a limited number of energy levels (states) that permit it to absorb photons of the right wavelengths. On an individual, case by case basis, each molecule follows these rules.

    When applied, in aggregate, to a very large number of molecules in a volume, statistically, they will tend towards obeying the Second Law of Thermodynamics. They will not gain more heat from a colder body, because statistically, on average, the warmer body will emit more, and absorb less, than the colder body. This is unchanging and without question. This is not a problem.

    But on the molecular level, some molecules emit, and some absorb, and there’s nothing that prevents some radiation from the molecules in the cooler volume from being absorbed by molecules in the warmer volume (as long as the net exchange is warm to cool, which it always will be, by the mere advent of statistics and averages — throw a coin enough times, and you get equal heads and tails).

    You need magic and Alice’s looking glass to create a scenario where this is not the case. If the slayer’s book is saying this, then it belongs right next to the other Lewis Carrol books on the shelf.

    So, given that on the molecular, quantum level, any molecule can absorb or emit radiation, based on the quantum rules, it is more than possible for a simple mechanism like greenhouse gas theory to take effect.

    On the other hand, without such a mechanism, it is impossible for the temperature of the surface of the earth to exceed 255˚K (-18˚C) without violating The First Law of Thermodynamics. That is to say, if the earth is at 288˚K, it is radiating heat appropriate to a body at that temperature (around 390 W/m2). Yet the earth only receives 240 W/m2 from the sun. There’s a 150 W/m2 deficit to be made up.

    Greenhouse gas theory says that the deficit is made up by radiation from the cooler atmosphere. The earth warms the atmosphere, but part of that energy is radiated back at the earth itself. The warmer earth warms the cooler atmosphere, so the net change follows the second law of thermodynamics, but some energy returns to earth, maintaining the temperature balance.

    Without this… where does the 150 W/m2 of energy come from? How do you avoid violating The First Law of Thermodynamics without greenhouse gas theory?

    00

  • #
    RJ

    @ 259

    Consider what must happen for this to be true. You’ve danced around it. Basically, the target of the radiation must know the source, or perhaps the radiation itself must carry some sort of thermal signature so that the destination knows whether or not to absorb that photon.

    No. The photon is absorbed but because the photons is from a cooler object (with a lower vibrational state) it has no warming impact and is just emitted a IR.

    Hopefully I have got the slayers viewpoint right on this. I do not know the answer but just hold both viewpoints. But logically (I know) I can not see how the slayers viewpoint is not right. Otherwise a colder object is heating a warmer one which I have difficulty accepting even if it is radiation and is claimed to only apply to net transfers of heat.

    Without this… where does the 150 W/m2 of energy come from?

    Have you read the Postma paper. The energy comes from the sun and only from the sun. To claim that energy can magically leave earth and more can come back seems complete and utter nonsense.

    If this is what you are claiming. In this respect I agree with the slaying and Postma viewpoint 100%.

    00

  • #
    RJ

    @ 258

    I agree, BobC, it’s hard to judge any argument without sufficient background.

    I would actually recommend the “Cartoon Guide To Physics” as well. This series is pretty good, very clear, and well worth reading.

    Or flawed background knowledge that someone is strongly attached to and will not let go of.

    I’m not saying it applies to you but it might based on you reaction to the slayers chapter. You seem fixed in your viewpoint. I try to stay humble even though it is difficult at times. Even in areas where I think I have a lot more knowledge than on this one.

    00

  • #
    RJ

    And here is another viewpoint from post 114

    I did not really understand this post but think this is in a photon do not really exist universe.

    Thus, in the case where a hot body and a cold body both emit electromagnetic radiation, the direction of electromagnetic energy flow will be from the hot to the cold body since the hot body will produce a stronger electromagnetic field. Energy does not flow both ways. That being the case, energy cannot “flow uphill” and the second law of thermodynamics remains intact.

    00

  • #
    KR

    RJ @ 261

    As I noted earlier in this thread, I’m not willing to take anything as correct without a critical examination. Clas Johnson’s physics statements are simply wrong. His (re)definition of a blackbody does not match anything in physics or reality.

    His separation of photons into high frequency warming/low frequency emitted is nonsense. I’ll note that if you warm an object with electricity or a flame, it will radiate the characteristic spectra – that’s a function of temperature, not specific photons striking. And absorptivity (easily measured, incidentally) is a function of the material, not temperature. There is no such separation in reality – absorbed photons add energy in proportion to their wavelength, there is no such cut-off.

    And I do have quite a bit of experience in this field – in particular with photon counting cameras.

    Given Johnson’s rules, CO2 lasers could not cut (as the photons are of too low a frequency to warm anything, and would be immediately re-radiated), microwave ovens could not cook (as microwaves are characteristic of 3-10K temperatures, which your average kitchen food is much warmer than). And (to put it a bit less politely) all pigs would be fed and ready to fly.

    Having an open mind does not mean uncritically accepting nonsense. It means evaluating new ideas based upon the evidence, upon consistency, upon explanatory power, etc. Johnson’s chapter fails all the way around.

    Have you tried the electric blanket experiment yet???

    00

  • #
    RJ

    KR @ 263

    As I noted earlier in this thread, I’m not willing to take anything as correct without a critical examination. Clas Johnson’s physics statements are simply wrong. His (re)definition of a blackbody does not match anything in physics or reality.

    This just illustrates my point re a totally closed mind

    I know little on the CO2 laser beam. But here is something on it.

    The laser beam discharged by the formerly excited nitrogen atoms is in the infrared part of the spectrum. The laser produced in this method is powerful enough to cut many substances and also destroy many others. Because it’s in the infrared, it has a very high wavelength, around 10.6 micrometers. As a continuous laser, this laser beam is the most powerful in production.

    Read more: How Do CO2 Lasers Work? | eHow.com http://www.ehow.com/how-does_4899566_co-lasers-work.html#ixzz1NDJPV4dB

    00

  • #
    Louis Hissink

    Hmm, a black body doesn’t actually exist, it’s a mental construct to aid analysis.

    00

  • #
    Bryan

    KR

    It seems that you have very little evidence to support your point of view of climate science when you have to rely on contrived claims about machines.
    Why not throw in the refrigerator while your at it.
    Try to describe the Sun/Earth/atmosphere interactions through the real changes that we can measure.
    This is the path taken by Postma and G&T

    Lasers
    A high enough concentration of photons of any wavelength can cause heating.
    It does not follow the path of equilibrium thermodynamics
    The heating is achieved through compression (shock waves).

    Microwave oven.
    Cold matter emits blackbody spectra in the microwave region of the spectrum. The cosmic microwave background radiation is 3K for instance.
    How can a microwave oven can heat up a bowl of soup?
    It isn’t accomplished by microwave photons but rather by phase variation between the electric and magnetic fields which causes dialectric molecules (primarily water in this case) to constantly re-orient with the magnetic phase. Friction then (plus choice of resonant frequency) does the actual heating.
    The plastic bowl is quite cool.
    If the frequency changes by any significant amount the effect no longer exists
    Microwave radiation can’t raise the temperature of anything that isn’t within a couple degrees of absolute zero.

    LED lights
    These are readily available with a colour temperature of 5000K.
    Yet they are cool to touch.
    Once again the device does not take the thermal route for its operation.
    It is a quantum device.

    All three devices above do not depend on heating matter up by steady absorption for their operation.
    To try to describe some thermal phenomena thing like the atmosphere through them is a pointless distraction!

    00

  • #
    KR

    RJ @ 264

    From “Slayers”:

    “The temperature of the blackbody determines a cut-off frequency for the emission, which increases linearly with the temperature: The warmer the blackbody is, the higher frequencies it can and will emit. Thus only frequencies below cut-off are emitted, while all frequencies are being absorbed.
    A blackbody thus can be seen as a system of resonators with different eigen-frequencies which are excited by incoming radiation and then emit radiation. An ideal blackbody absorbs all incoming radiation and remits all absorbed radiation below cut-off.
    Conservation of energy requires absorbed frequencies above cut-off to be stored in some form, more precisely as heat energy thus increasing the temperature of the blackbody.”
    (emphasis added)

    So – he’s stating that radiation above the thermal cut-off heats the object, below the cut-off just gets re-emitted and has no effect on temperature.

    Radiation at 10.6 micrometers is smack in the thermal emission range for something at room temperature. Hot metal, on the other hand, is radiating in the visible range – CO2 radiation thus (by Johnson’s logic) could not heat metal in that fashion, as the CO2 wavelengths are below the mythic “cut-off”.

    Exactly the same holds for the microwave oven, which is much easier to observe in the home environment. Microwaves are emitted as thermal radiation from objects between 3-5 Kelvin, whereas even a frozen roast is around 250 Kelvin. By his logic a microwave oven could not warm a frozen roast.

    In reality, all absorbed photons add energy. Low frequency photons (or EM quanta, as BobC more exactly noted) add less energy than high frequency photons, but they all have energy, and absorption adds energy to the object. CO2 lasers and microwave ovens pump a lot of low frequency photons into objects (which absorb them), warming them.

    Back to the topic of the thread: the rate of cooling for any object is determined by it’s temperature, presence/absence of insulation, and the temperature of it’s surroundings. If an object receives a fixed input power, but the insulation increases, it’s going to warm up until it’s again losing as much energy as it receives – that’s basic conservation of energy.

    As I stated before, RJ, it’s important to have an open mind, but not one open at both ends. Please apply some critical thinking – this chapter by Johnson flatly contradicts observed reality. How can you not recognize this?

    00

  • #
    BobC

    RJ; let me give you some examples of energy amplification by storage. Try to keep an open mind, and see the parallels with what we’ve been talking about.

    1) A Fabre-Perot optical etalon is two partially reflecting mirrors set parallel to each other. (Here is a discussion on Wikipedia.) In particular, let’s consider an etalon from Research Electro-Optics here in Boulder that uses mirrors that are 0.999 reflective. As the Wiki article describes, at certain resonant wavelengths virtually all the light incident on such a device will (after a short delay) pass through it, seemingly unimpeded.

    To make the example specific, let’s assume you point a laser emitting one of these resonant wavelengths at the etalon, and the laser’s power density is 1 mW/mm^2. This is about 1000 W/m^2, or approximately the irradiance of sunlight at high noon on a cloudless day in the desert. After the transient period ends (several microseconds) we arrange to measure the power density before, after, and inside the etalon (between the two mirrors, that is). Here is what you see:

    Before the etalon: This is just the laser beam, with an intensity of 1000 W/m^2

    After the etalon: This is also nearly the same intensity as the incident beam, minus a few hundredths of a percent scattering loss from the mirrors. (REO’s mirrors are very good.)

    Inside the etalon: Here’s the surprise — the light intensity between the two mirrors is ~ 1,000,000 W/m^2, 1000 times the outside intensity (and 1000 times the intensity of noon sunlight).

    where does [this] energy come from?

    Well, from the laser, of course. During the short transient period I referred to, the light intensity was building up inside the etalon due to multiple reflections back and forth between the mirrors. (Conceptually, this is the same as the time delay it takes to fill a reservoir after you close the spill gates.) If you measure the length of this transient period you will find that it is roughly the time required for light to bounce back and forth between the two mirrors 1000 times.

    Now let’s imagine a desert flea wandering over the mirrors. First, the flea walks into the incident beam on the outside of the first mirror. The flea might get a little warm, but this is the same intensity it sees at noon in desert sunshine, so it will be OK.

    Then, the flea walks around the edge of the mirror, and continues into the beam inside the etalon. This time the results are not so good. At an intensity of 1,000,000 W/m^2 (1000 times desert sunlight), the flea disappears in a puff of greasy smoke (probably ruining the expensive REO mirrors 😉 )

    Now, look back at Sphaerica (Bob)’s Simplified Diagram at post # 83, and notice the similarities between the etalon and his diagram of the Earth-Atmosphere system. In particular, the back-radiation from the atmosphere plays the part of the light reflected within the etalon and also increases the intensity bewteen the Earth and atmosphere without affecting the intensity outside.

    The etalon I described:
    a) Does not violate any laws of physics. (In fact, it is analyzed using the laws of physics.)
    b) These devices have been made and used for > 150 years (although not at REO’s level of precision).
    c) If the physics in the slayer book doesn’t allow the existence of this device, then the slayer book physics is wrong.

    00

  • #
    KR

    Bryan @ 266

    Lasers emit photons – each adds some energy to the target, and the very high number of photons from a laser sums up to adding a large amount of energy.

    Microwaves emit photons at low wavelengths. The molecular configuration of water is a good absorber of these wavelengths (much as CO2 as a dipole molecule is a good absorber of IR), and hence heats from the absorption – energy is transformed from EM to vibrational and rotational modes of the water molecules. Plastics are essentially transparent to microwave frequencies, as O2 is transparent to IR, and hence does not get warmed.

    LED’s have a ‘color temperature’ related to the peak emission of a thermally radiating filament or bulb. However, as nearly monochrome emitters they are only radiating in the visible frequencies determined by their available electron drops, not a full spectra thermal emission, and are thus radiating a lot less power. And heat.

    Your use of these three as ‘counterexamples’ is a clear indication to me that you are not terribly familiar with the physics involved. Photons carry energy, absorbing a photon increases the energy of the absorbing object – it’s really that simple.

    And as Jo and Michael have noted, the radiative greenhouse effect does not violate the second law of thermodynamics, no matter how far afield you wish to take discussions of quantum mechanics, dipole moments, etc.

    00

  • #
    Bryan

    KR
    You take the biscuit for reversing reality.

    I try to drag you back to topics familiar and useful for understanding climate science through the laws of equilibrium thermodynamics.
    Your reply about microwaves etc is pointless as well as wrong, but back to the climate science and the second law.

    There seems to be a theme in your rambling recent posts concerning the frequency and energy involved in two way radiative interactions.

    Your posts reminded me of an exchange I had with scienceofdoom so I will put the same question to you.
    Do you think that 1000Joules of blackbody radiant energy centred around say 2um is thermodynamically equivalent to 1000Joules of blackbody radiant energy centred around say 20um

    a) In terms of the First Law of Thermodynamics
    b) In terms of the Second Law of Thermodynamics.

    00

  • #
    Paul

    BobC:
    May 23rd, 2011 at 11:38 pm

    Now, suppose we raise the temperature of the shell to that of your body. The shell is now radiating the same spectrum as you are. This is a problem, as you are now absorbing radiant energy as fast as you are emitting it and your net (radiant) heat loss is zero. Since your metabolism can’t go to zero (as long as you’re alive, that is) your body temperature will rise. This could be dangerous, except Humans can also shed heat by exuding water from their skin pores which absorbs energy as it evaporates (e.g., “sweating”).

    Note that, in all these scenarios, the direction of net heat flow is from the hotter object to the cooler — the second law is upheld. The rate of heat flow, however, is dependent on the difference in temperatures, going to zero as the difference does.

    Note that the distant shell does not have to “heat” you to cause you to be warmer — since you have your own energy source (your metabolism) all it has to do is slow down the rate at which you lose heat.

    So, let me check if I understand this.

    1) As the temperature of the ‘shell’ is increased, your body loses heat at a slower rate.

    2) When the shell is at the same as or greater temperature than your body, your body will sweat and increase its rate of heat-loss.

    3) In every case the flow of heat is from the warmer to the cooler body, upholding the second law of thermodynamics.

    My questions are : —

    1) If the reduction of the rate at which your body loses heat is caused by the shell radiating at a higher temperature, doesn’t that mean that, applying the analogy, the atmosphere must attain a higher temperature in order for the earth to lose its heat at a slower rate?

    2) Since the earth’s surface contains a lot of water, which will be evaporated at a greater rate as the surface increases in temperature, wouldn’t that also tend to keep the surface temperature moderated to below that which would occur without the presence of water?

    3) Since the IR absorption effect does not increase the temperature of the atmosphere but increases the internal vibratory and/or rotational energy of the excited molecules rather than their velocity, does this represent an additional electromagnetic energy field with a force that is the equivalent of that which is represented by temperature?

    4) What is the capacity of the atmosphere to contain this energy that is absorbed by the IR absorption effect [similar to but differing from its thermal capacity] and by how much would it increase if the proportion of atmospheric carbon dioxide were doubled?

    5) What would be the relative quantities of energy flowing between the sun and the earth’s surface, the earth’s surface and the atmosphere and the atmosphere and outer space, before and after the doubling of atmospheric CO2?

    6) Once the atmosphere has reached its capacity to absorb IR radiation at what rate would the atmosphere shed energy to space compared with the rate at which the earth is radiating its IR radiation?

    In other words, can you quantify the energy flows and provide the basis for setting up an experiment whereby these quantities can be measured and verified or disproved. Failing that this seems to be nothing but metaphysics and not strictly in the field of science.

    To clarify the analogy of a blanket keeping us warm at night, my understanding is that this process works by preventing the movement of air around our bodies. This leads to the production of a small layer of still air around our bodies. Now that heat-loss through convection is prevented our bodies will continue to lose heat mainly through radiation. This radiation will be absorbed by the surface of the blanket which will, by conduction, raise the temperature of that small layer of air thus reducing the rate at which our bodies radiate heat.

    Question : Do you maintain, in the blanket analogy, that the heat-loss is due only to the ‘back-radiation’ of the surface of the blanket, or does the layer of warmed air contribute also to the reduction in heat-loss?

    The reason for asking all these questions is that I understand that, although we are led to expect a warming of the higher levels of the atmosphere in the reported results of all the GCM models, following this AGW conjecture, this ‘signature’ has not been observed in the real atmosphere. Hence, if the causative factors have failed to materialise how can they be ‘causing’ the increase of temperature at the surface?

    Paul

    00

  • #
    cohenite

    KR@248; I don’t think you can make your conclusion that Miskolczi’s comments about water decline balancing CO2 increase is incorrect; you focused on the Wentz paper about increased precipitation and ignored the Pierce and Soloman papers.

    Of equal interest is the research into evaporation which shows a decline:

    http://pielkeclimatesci.wordpress.com/2009/11/11/pan-evaporation-trends-and-its-relation-to-the-diagnosis-of-global-warming-comments-on-a-new-article-by-roderick-et-al-2009/

    And the Franks paper which shows that increased evaporation does not necessarily follow from increased temperature:

    http://www.agu.org/pubs/crossref/2009/2009GL040598.shtml

    Franks explains why here:

    http://www.australianclimatemadness.com/2009/11/shock-murray-darling-warming-not-due-to-co2/

    In respect of your link to the Soden and Dessler page on SH; I’ll stick with Paltridge thank you and did you know that in the overlapping part of the spectrum increased CO2 decreases the emissivity of H2O? So much for water feedback!

    00

  • #

    RJ,

    …because the photons is from a cooler object (with a lower vibrational state)…

    But as I explained, at the molecular level, some molecules are vibrating (or rotating, or both), and some aren’t. In fact, in a gas, the main contributor to temperature is translational energy… how fast are the molecules whipping through space. Some are vibrating/rotating because of equipartition of energy, but this is different from a solid where there is no relative motion, and all heat is encapsulated in vibrations.

    So… in my case, if a molecule is already vibrating, it can’t absorb another photon at that energy level. In your case, the entire body of gas can’t absorb a photon if some of them are already vibrating. If this is not your position, then what happens to a single molecule that is not vibrating, and is struck by a photon of the right energy, in a volume of gas that is at a higher temperature than the source of the photon? What stops that molecule from absorbing the energy, even though the molecule itself is at a lower temperature (by your definition).

    Otherwise a colder object is heating a warmer one…

    But as I’ve explained, the 2nd Law is a macroscopic law that cannot be applied, ad infinatum, to every level of existence. This is one of the marvels of quantum mechanics, that classical theory breaks down at the quantum level.

    A colder volume of gas will not heat a warmer one, but individual molecules do not strictly have “temperatures,” they have energy, and energy can be imparted from a lower energy object to a higher energy object through radiation. That you cannot accept this does not make it false. It simply represents an area where your experience and conceptual approach is hindered by a “law” that you’ve been taught and are now trying to apply in an inappropriate context.

    The energy comes from the sun and only from the sun. To claim that energy can magically leave earth and more can come back seems complete and utter nonsense.

    But this is not at all what I said. I said most leaves, and some comes back. Not more. Not even all. The earth heats the atmosphere, because the atmosphere is cooler, but an exchange takes place. 390 up from the surface, 150 back down to the ground. The net is 240 up (what a coincidence!), so the surface is in fact heating the cooler atmosphere. But it’s a two way street. The Second Law of Thermodynamics governs the net exchange. It is not a one way valve.

    Okay, so in your case (if mine is magic): 240 W/m2 come from the sun. You’ve said so, you insist on it, and I agree 100%. It is the only source of energy in the system. 390 W/m2 leave the surface of the earth. We know this because the temperature of the earth is 288˚K so theory demands it, but beyond this it is an easily measured quantity. There is no debate about this.

    So if 240 W/m2 comes in, and 390 W/m2 leaves, where is the other 150 W/m2 coming from?

    I haven’t read all of Postma’s paper, but I cannot find any place where he explains how the earth can heat to 288˚K without greenhouse gas theory. He goes on and on (incorrectly) about why GHG can’t work, but does not explain how one can find an extra 150 W/m2 without magically defying The First Law of Thermodynamics.

    As to your comment that energy can “magically leave earth and more can come back,” that’s not what I said, or GHG theory says. You are misrepresenting the situation.

    390 W/m2 of energy leave the surface of the earth. 240 W/m2 escape into space, while 150 W/m2 come back to the surface. The 240 that escape into space exactly balance the 240 coming in from the sun, meaning that the temperature of the planet as a whole stays constant. The 150 W/m2 directed back at the surface adds to the 240 W/m2 arriving from the sun, bathing the surface in 390 W/m2 and balancing the 390 W/m2 that is radiated, thus keeping the surface at 288˚K.

    Now you show me your math, using any old numbers you want, for how 240 W/m2 arrives from the sun, and 390 W/m2 leave the surface (as it must for an object at 288˚K)… yet the surface of the earth does not cool, and the atmosphere does not boil away, and energy is not created by magic, defying The First Law of Thermodynamics in order to abide by The Second.

    00

  • #

    Paul,

    If BobC doesn’t mind my interjecting:

    1) If the reduction of the rate at which your body loses heat is caused by the shell radiating at a higher temperature, doesn’t that mean that, applying the analogy, the atmosphere must attain a higher temperature in order for the earth to lose its heat at a slower rate?

    Yes, the atmosphere is warmer than it would be without greenhouse gases. It is not warmer than the surface of the earth, but it is warmer than a world without greenhouse gases (except at the top, where greenhouse gases actually work in reverse to cool the atmosphere, but that is a logical and observationally supported extension of GHG).

    2) Since the earth’s surface contains a lot of water, which will be evaporated at a greater rate as the surface increases in temperature, wouldn’t that also tend to keep the surface temperature moderated to below that which would occur without the presence of water?

    More water is evaporated, and hence humidity is higher than in a world without GHGs. Heat transport through evapotranspiration/latent is part of the equation, and Trenberth estimated it’s contribution at roughtly 80 W/m2. Another 17 W/m2 is transported up through thermals (convection). This compares with 396 W/m2 that leave the earth through radiation.

    So the answer to your question is yes, more evaporation, but not nearly enough to actually moderate temperatures (that particular change in the equation is relatively minimal), especially since once the moisture gets into the atmosphere, it is a positive GHG feedback.

    3) Since the IR absorption effect does not increase the temperature of the atmosphere but increases the internal vibratory and/or rotational energy of the excited molecules rather than their velocity, does this represent an additional electromagnetic energy field with a force that is the equivalent of that which is represented by temperature?

    The answer is no, but you have a mis-statement in your question. The IR absorption effect does increase the temperature in the troposphere. When a molecule absorbs a photon and begins vibrating/rotating, it is far more likely to pass that energy on in a collision, and the energy can be transferred as vibrational, rotational, kinetic, or some combination there of. The reverse can also happen (i.e. a collision excites a molecule into vibrating, and that molecule could be able to emit a photon before another collision “steals” the vibration).

    But we’re talking about net effect, over the course of uncountable molecules, photons and collisions. The energy absorbed through IR is translated into heat in the atmosphere (as well as more radiation, because the atmosphere is higher). The end result is more IR in = more heat, and correspondingly more IR out (until the system is in balance).

    4) What is the capacity of the atmosphere to contain this energy that is absorbed by the IR absorption effect [similar to but differing from its thermal capacity] and by how much would it increase if the proportion of atmospheric carbon dioxide were doubled?

    I’m not sure that I’ve ever seen anyone break the numbers out, but of course the temperature change will vary at different points in the atmosphere, both by latitude and altitude. There’s no easy answer to this question, and I’m not even sure where to find it.

    What is important about GHG is that emissions will always be 240 out, to balance the input of the sun. It will be slightly less as the planet warms, but once equilibrium is reached, it will balance (plus or minus natural variability, of course, day to day and year to year). What will mostly change is the altitude from which that 240 is lost. As the planet warms, it will emit that 240 (or, from another point of view, 240 will escape) from a higher altitude.

    5) What would be the relative quantities of energy flowing between the sun and the earth’s surface, the earth’s surface and the atmosphere and the atmosphere and outer space, before and after the doubling of atmospheric CO2?

    Always 240 in, 240 out. Generally, for how much more from the surface, the number I’ve seen is 3.7 W/m2 per degree, so if doubling of CO2 by itself raises temperatures 1˚C, then the output from the surface would go from 390 (or 396, depending on what numbers you use) to 393.7. Add more for positive feedbacks. But what’s relevant is that the back radiation will amount to the same, so that it will always be 240 in, 240, and the surface will always emit whatever radiation is appropriate to the temperature of the surface (use the Stefan-Boltzman law to compute this, if you wish), and the atmosphere will radiate back the difference (if not, the system is not in equilibrium, and the temperature of the surface will continue to change in accordance with the imbalance, until equilibrium is achieved).

    In fact, thinking about it, you might be able to use this to compute the new temperature of the atmosphere as a whole (using Stefan-Boltzman again) to answer your question #4.

    6) Once the atmosphere has reached its capacity to absorb IR radiation at what rate would the atmosphere shed energy to space compared with the rate at which the earth is radiating its IR radiation?

    Again, see the answer to 5. The sun always puts 240 in, and in equilibrium (i.e. once the earth reaches the temperature dictated by the mechanics of the system) it will be 240 out at the top of the atmosphere. In between, the amount radiated from the surface to the atmosphere will be set by the equilibrium temperature for the surface, and the Stefan-Boltzman Law. How much is radiated back by the atmosphere is going to be the difference between that and the temperature of the surface (after you factor in evapo-transpiration and convection, and reflection of inbound radiation by both clouds and surface features).

    Google Trenberth’s energy diagram for more and more precise numbers for the various and sundry ins/outs.

    00

  • #
    Paul

    It’s all Alice in Wonderland physics. None of it makes sense.

    That is because you continue to do what is inadmissible, mixing the micro-quantum state and the macro-observable state contrary to logic and knowledge of even simple statistics.

    Here is an example that everyone should be able to understand that will illustrate the fallacy of arguing from the micro to the macro state.

    Take a population of people interacting with each other’s neighbours. Give them the rule to live by that they are to share their money with their neighbours equally. Now, whenever they meet, they add their money to a common pool, then divide it equally between themselves. The entire world will tend unidirectionally towards an equal supply of money. While small localities of inequality, in terms of the overall average, will remain while the system is tending towards equilibrium, these equalities will tend to lessen, not increase, with the passage of time. The arrow of time is unidirectional, according to the law of entropy which is closely associated with the second law of thermodynamics.

    Now add a perturbation to the system, whether in equilibrium or not, by giving one person a finite additional amount of money from outside the system. The net effect will be that the amount of money available to the entire system has increased, by a finite, measurable amount, and the individual who received that additional money now has more than the average amount of money.

    So the next time he meets a neighbour, they share their money and divide it between them. Now both have more than average money but the total amount of money available to the system has not changed, in this subsequent sharing process which will continue until no one has more than the average.

    My conclusions are : —

    1) No one has to know what the average amount of money in the system is, they only need to pool their money with each neighbour that they meet and divide the sum between themselves equally.

    2) The average amount of money is given by the total supply of money divided by the number of people.

    3) To argue for an increase in the inequality of money in a localised neighbourhood, when each transaction takes them away from inequality, is to argue a statistical impossibility.

    4) If someone travels, by plane, to a distant locality, and the new locality has less money, on average, than the one they left, then the process of sharing will temporarily increase the average money held in the new locality. Conversely, if the new locality has more money, on average, than the one they left, the process of sharing will temporarily decrease the average money held in the new locality.

    5) To increase the overall average money held by each person in the system there must be an increase in total money available in the system and that increase has to come from outside the system.

    Applying that analogy to the earth’s atmosphere, it is clear that photons have no idea about the average energy levels in the surface of the object from which they came, whether that of the sun, that of the earth or from the atmosphere itself, but that lack of knowledge does not affect the direction of the overall transfer of energy since in every transaction the energy is shared. The arrival of a photon from the surface of the sun to the surface of the earth will increase the amount of energy of the earth-atmosphere system. The sharing of this new energy between molecules in the earth-atmosphere system will not add any energy to the system.

    The claim of the CAGW conjecture, it would seem, hinges on the claim that the cooler atmosphere is back-radiating energy to the surface. However, for this to be so, the ‘rule of thumb’ that neighbouring molecules tend to share their energy-levels equally, has to be over-ridden by the power of radiation from upper levels in the atmosphere to the surface of the earth. It is this claim, that energy at low frequencies, can be transmitted from a cooler body to warm a warmer body, that is the central claim that has to be refuted. Without a higher degree in physics I can see enough to throw this claim into doubt even if I do not have the ability to disprove it mathematically.

    I would contend that you cannot infer this macro-radiation system from the interaction of individual photons that are being absorbed and re-emitted every few micro-millimetres. If this radiation process can run ‘up-hill’ over the range of several kilometres, against the down-hill localised processes of sharing energy levels equally on contact, then the laws of statistics have had to be suspended!

    It is like claiming that you can always expect to throw 100 heads and no tails in a row of 100 throws! The micro-world tells you that you can expect that result once in a few million trials [I’m not sure of the actual probability but it can be calculated]. But to go from claiming that this result can happen in that rare instance to claiming that you can therefore equally expect to throw a billion heads and no tails in a row out of a billion throws is patently fallacious.

    In the absence of a valid statistical argument supporting this unexpected flow of energy, there has to be another mechanism by which it could exist. I have seen or read nothing that supports such novel mechanism.

    Paul

    00

  • #
    Kevin

    BobC wrote (in comment #268);

    “let me give you some examples of energy amplification by storage.”

    And then gives a nice little example of an optical Etalon.

    However the Etalon is in NO WAY (ALSO NO HOW, NEVER EVER, etc.) an example of energy amplification.

    Sorry to break your little bubble, but your explanation of what happens inside an etalon is flawed in too many respects to state them all here. But I will just give you two;

    If you reconcile the units (supplied by you) an amplification of “an intensity of 1000 W/m^2” into “~ 1,000,000 W/m^2” is an example of POWER AMPLIFICATION, NOT an example of ENERGY AMPLIFICATION. I know from experience that it is easy to confuse power (Watts) with energy (Joules, BTUs, etc.) But you really need to pay attention to those units, it can totally mix up an analysis or prediction.

    Secondly, what happens inside the etalon is that the laser energy constructively interferes at a very small portion of the spacing between the mirrors and the power is indeed amplified by up to 1000x in a good quality etalon. However at all of the other locations between the mirrors the energy is equal to ZERO. So if the distance between the mirror is 1000 units, NO power is present in 999 of those locations, and all of the power is in one location, hence the POWER AMPLIFICATION. Same amount of power in less space = more power/intensity at that location.

    This is why etalons are rare optical elements that have very few practical uses. Their main usful purpose is to “lock” the wavelength of a laser because they show very high throuhput at a certain wavelength (the 99.999% you refered to) and reduce the throughput at other wavelengths. If they could amplify energy we could just tune them for the peak wavelength of the Sun and put solar cells inside of them. That way all our energy problems would be solved……

    Cheers, Kevin.

    00

  • #
    Paul

    The etalon I described:
    a) Does not violate any laws of physics. (In fact, it is analyzed using the laws of physics.)
    b) These devices have been made and used for > 150 years (although not at REO’s level of precision).
    c) If the physics in the slayer book doesn’t allow the existence of this device, then the slayer book physics is wrong.

    There is only one problem that I see with your argument, and that is that you have extrapolated a laboratory experiment, with exceedingly finely tuned conditions, to the atmosphere where none of those conditions exist.

    To be precise, the qualities of atmospheric carbon dioxide have been experimentally ascertained and they are so far from approximating a blackbody, they are that of a very poor absorber and emitter of radiation.

    Take a look at the figures given in this discussion on Jennifer Marohasy’s blog : —

    Total Emissivity of the Earth and Atmospheric Carbon Dioxide: A Note from Nasif S. Nahle

    Conclusions

    In the introduction I asked: What is the total emissivity of carbon dioxide?

    In this note I have calculated the real total emissivity of the atmospheric carbon dioxide at its current partial pressure and instantaneous temperature to be 0.002.

    Clearly carbon dioxide is not a nearly blackbody system as suggested by the IPCC and does not have an emissivity of 1.0. Quite the opposite, given its total absorptivity, which is the same than its total emissivity, the carbon dioxide is a quite inefficient – on absorbing and emitting radiation – making it a gray-body.

    Accepting that carbon dioxide is not a black body and that the potential of the carbon dioxide to absorb and emit radiant energy is negligible, I conclude that the AGW hypothesis is based on unreal magnitudes, unreal processes and unreal physics.

    Where now is the comparison between mirrors that are 0.999 reflective and the atmosphere of earth?

    Surely this proves that the Greenhouse Gas Warming conjecture is just that, with the actual effect being so close to zero that it will never be measurable.

    Paul

    00

  • #
    KR

    Bryan @ 270

    1000 Joules from a 2 micron centered source, and 1000 Joules from a 20 micron centered source?

    I’m going to assume that you mean 1000 Joules/second, or 1000 Watts.

    The 2 micron centered source will be at ~1450K, the 20 micron centered source at ~140K.

    So, aside from the fact that the 140K source will have to be 11,500 times larger (or take 11,500 times longer) to radiate the same power as the 1450K source, well, Joules are Joules. As to the 1st and 2nd laws, each source must receive 1KW as input in order to radiate 1KW as output, and net energy flows will depend on what else is around them.

    And what, pray, is your point?

    00

  • #

    Paul @277

    Exactly right. But something I have been thinking on above and beyond that argument is that for specific wavelengths CO2 absorbs 100% of the radiation available, if even 100% of those photons are re-emitted only 50% at most get pointed back towards the earths surface. Those 50% are not going to make it back the same distance that the 100% made it upwards. Thus, it is not just simply close to 0, it is in fact 0.

    00

  • #
    KR

    Paul @ 277

    The point is that you are not understanding the physics. “…that the potential of the carbon dioxide to absorb and emit radiant energy is negligible” It is not, as a realistic view of the numbers would point out. CO2 represents ~25-30% of the total greenhouse effect. It is the only major component that is non-condensing, that doesn’t react to temperature changes by precipitating or evaporating (as water does, hence water acts as a feedback, not a driver). Claiming that it has negligible effect without numbers is rhetoric, not science.

    OK, as a matter of personal opinion, I consider this topic to have reached the point of diminishing returns.

    This really brings home the point of the original Michael Hammer posts – that if you push a silly idea, you will not be taken seriously. Does your microwave oven work? According to Clas Johnson, it shouldn’t! Cancellation of electromagnetic fields going in opposite directions (@191)? Then if I’m lit by a 100W bulb, and a friend is across the room lit by a 40W bulb, he should be invisible because the higher energy from my side of the room cancels him out?

    There have been multiple points of view (not hypotheses, because most included no criteria by which they could be invalidated) pushed in these two threads – the vast majority of which are contradicted by facts, by the behavior of the world around us.

    If your neighbor attempts to convince you to trim your bushes in a certain way, shift your walk into a particular pattern, because he is absolutely convinced that the lawn gnomes of the neighborhood will go to war otherwise – would you take his views seriously? By pushing such non-physical hypotheses you are putting yourself in the position of that neighbor.

    This is the core of the Common Sense fallacy. To quote from this, one of my favorite sites on the subject:

    Common Sense:

    unfortunately, there simply isn’t a common-sense answer for many questions. In politics, for example, there are a lot of issues where people disagree. Each side thinks that their answer is common sense. Clearly, some of these people are wrong.

    The reason they are wrong is because common sense depends on the context, knowledge and experience of the observer. That is why instruction manuals will often have paragraphs like these:

    When boating, use common sense. Have one life preserver for each person in the boat.

    When towing a water skier, use common sense. Have one person watching the skier at all times.

    If the ideas are so obvious, then why the second sentence ? Why do they have to spell it out ? The answer is that “use common sense” actually meant “pay attention, I am about to tell you something that inexperienced people often get wrong.”

    Science has discovered a lot of situations which are far more unfamiliar than water skiing. Not surprisingly, beginners find that much of it violates their common sense. For example, many people can’t imagine how a mountain range would form. But in fact anyone can take good GPS equipment to the Himalayas, and measure for themselves that those mountains are rising today.

    – Don Lindsay

    Sufficiently complex questions (and climate certainly qualifies) are not amenable to “Common Sense” opinions, because your common sense has not experienced global heat distributions, moist air lapse rates, graybody thermal spectra, and the like.

    Opining without knowledge leads to your opinion being dismissed.

    00

  • #
    BobC

    Paul:
    May 24th, 2011 at 11:57 am

    There is only one problem that I see with your argument, and that is that you have extrapolated a laboratory experiment, with exceedingly finely tuned conditions, to the atmosphere where none of those conditions exist.

    Don’t weasel Paul: What I was trying to do was answer the dogmatic argument that ANY energy amplification was IN PRINCIPLE impossible. You, and others, were arguing here that it was impossible for the radiant flux between the atmosphere and the Earth to be higher than the radiant flux supplied by the Sun and radiated away at the top of the atmosphere. The etalon example demonstrates that one can get a significantly increased internal flux by storing the energy. In the etalon, it is sent back and forth between mirrors — in the Earth-atmosphere system, some energy is recycled between the atmosphere and the Earth by being absorbed and re-emitted.

    If you STILL are going to argue that it’s impossible, you need to explain why the Earth is significantly warmer than a blackbody would be, given the Solar insolation.

    If you want to use made-up physical laws to answer this, then you need to explain why the phenomena predicted by these “laws” contradicts 200 years of experimental results.

    And CO2 a blackbody? Who here has argued that? Good God man, CO2 exhibits a LINE SPECTRUM. It is absorbing and emitting due to narrow and sparse internal resonances. This is the total antithesis of a black body.

    I’m tired of wrestling with jello. Go educate yourself.

    Or not. Not my concern.

    … with the actual effect being so close to zero that it will never be measurable.

    You may be right about that — but you’ll never convince anyone with any knowledge of science with the arguments you’ve been making. Read Idso’s paper to see how it’s done.

    And Kevin @ 276: You don’t know Jack about optics. Why don’t you stop pretending and read a book?

    00

  • #
    L.J. Ryan

    BobC: 184 240 247 268

    184
    A coat can only slow loss to a MAXIMUM of your body temperature. If a body could double in temperature with the aid of a highly reflective, highly insulating jacket, as proposed by GHG physic, imagine the fatalities…do the lawyers know of this cash-cow. Or maybe just maybe radiative quantities don’t add as GHG physics requires.

    240, 247
    GHG physics proposes terrestrial radiation of 240 W/m^2 is “forced” to 390 W/m^2 via the cooler/lower energy atmosphere.

    You propose floor radiation of 240 W/m^2 is “forced” to 390 W/m^2 via the cooler/lower, radiatively insulated ceiling.

    Five winters, circulating water 85 F, I yet to realize such performance. Maybe you can help me pinpoint the problem? According to GHG physics, 60 F water pumped through the distribution manifold, should confer a sweltering 127 F…what a conundrum. I need to cool the water which warms my home. Or maybe just maybe radiative quantities don’t add as GHG physics requires.

    268

    Inside the etalon: Here’s the surprise — the light intensity between the two mirrors is ~ 1,000,000 W/m^2, 1000 times the outside intensity (and 1000 times the intensity of noon sunlight).

    A Photon Capacitor, all you need is a DeLorean and Shazam , you’ve got yourself a time machine.

    Seriously though, according to your calculations 1,000,000 W/m^2 bouncing around the inside the etalon means the outside is dissipating 1000 W/m^2…(.001 absorption). These little cavities (with an area << smaller then a m^2) must be white hot if not molten mess. Or maybe just maybe radiative quantities don't add as GHG physics requires.

    00

  • #

    BobC @ 281

    in the Earth-atmosphere system, some energy is recycled between the atmosphere and the Earth by being absorbed and re-emitted.

    How does it warm the Earth?

    And CO2 a blackbody? Who here has argued that? Good God man, CO2 exhibits a LINE SPECTRUM. It is absorbing and emitting due to narrow and sparse internal resonances. This is the total antithesis of a black body.

    And these are limited in quantity. Once emitted from the physical surface of the Earth, how do they return? What path allows an even less level of narrow and sparse wavelength energy to penetrate back the physical surface of the Earth? If 100 units goes up, at most 50 units points back down, how do the 50 units get past the same amount of CO2 that absorbed 100 units on the way up? There is no cycle between CO2 and Earth surface.

    If you STILL are going to argue that it’s impossible, you need to explain why the Earth is significantly warmer than a blackbody would be, given the Solar insolation.

    Because the Earth is a body with an Atmosphere. The point of which the blackbody temperature would be reached would be somewhere higher than the physical surface and the top of the atmosphere. At this point, adiabatic lapse rate controls the temperature at the Earth’s surface along with ocean heat capacity.

    00

  • #

    Paul, 277,

    You are so very, very close to the truth. All you have to do is to stop trying to focus on one answer, and open your mind to the correct answer.

    The claim of the CAGW conjecture, it would seem, hinges on the claim that the cooler atmosphere is back-radiating energy to the surface. However, for this to be so, the ‘rule of thumb’ that neighbouring molecules tend to share their energy-levels equally, has to be over-ridden by the power of radiation from upper levels in the atmosphere to the surface of the earth.

    No. The greenhouse effect hinges on exactly what you are saying, that neighboring molecules do tend to share their energy levels equally (on average). They do so completely at random, in random directions, with no central control or intelligence whatsoever. They don’t know how hot the source of the photon was, and they don’t care. They don’t know if they’re emitting a photon towards a warmer or cooler object, and they don’t care.

    It’s all random. And in this random world, some goes up, some goes down, some gets lost in collision and converted to heat.

    It is in RJ’s/slayer/Postma’s world that molecules become disciplined socialists, very carefully following a preordained party rule to only share wealth (heat) with poorer (cooler) molecules. But it doesn’t work that way individually. Any molecule can emit, and any molecule that isn’t already excited can absorb. In a random environment it does work that way in net effect over the entire system. So warm objects always have a net heat exchange to cooler objects, but this does not mean that no energy can ever flow from a cooler object to a warmer one (i.e. less energy must flow from cool to warm than from warm to cool).

    By the way, your monetary analogy was absurd, because it began with an impossibility that has no equivalent in our discussion… an infinite supply of money. Your analogy begins by violating the First Law of Thermodynamics, which coincidentally is also violated if you do not accept greenhouse gas theory (see below).

    In the absence of a valid statistical argument supporting this unexpected flow of energy,…

    But that’s just it, you don’t have a valid statistical argument that supports your case! And you don’t have an explanation for where the missing energy comes from to heat the surface above 255˚K.

    Imagine 1,000 rubber balls, all being shot at the Parthenon. One would expect some to go between the columns and get through, some to rattle around and fall to the floor, and some to bounce straight back.

    In your world, not a single rubber ball bounces back, because they know that’s where they came from, and it’s not allowed. It violates the Second Law of Rubberdynamics.

    Can you not see that your position requires both magic, a complete lack of statistics, and most importantly a violation of the First Law of Thermodynamics.

    I will put it to you as I put it to JR. How in the world do you find 150 W/m2 of energy if the surface of the earth, at 288˚K, is emitting 390 W/m2 but the sun is only supplying 240 W/m2.

    It’s simple math. 240 in, 240 out at the top of the atmosphere. 390 out from the surface of the planet. Where did that other 150 go? And why hasn’t the planet cooled into a ball of ice with only 240 W/m2 coming in (unless the earth is magically creating energy at the surface to make up the difference)?

    00

  • #
    Kevin

    To BobC who wrote;

    “And Kevin @ 276: You don’t know Jack about optics. Why don’t you stop pretending and read a book?”

    Bob, with respect, I do in fact know quite a bit more than “jack” about optics, having graduate degrees in both optics and electrical engineering, plus several decades of practical electro-optic engineering experience, including work on imaging satellites that are currently orbiting the Earth and returning images of the surface. There are these little equations known as Maxwell’s equations which do a very good job of describing how electromagnetic radiation flows through a system. I have studied them and mastered them. They apply to everything from Ultraviolet light to the AM radio band.

    You are in fact quite free to believe that an optical etalon demonstrates “energy amplification”, but you would look less the fool if you could at least get your units correct, (Hint: Watts are Power, NOT ENERGY).

    Cheers, Kevin.

    00

  • #

    By the way, anyone done a study in the deserts and found out what the change in night time temperatures has been since the rise from the supposed 280ppm CO2 to the new 390ppm CO2 level? I am guessing that deserts no longer are cold at night due to the massive increase in back radiation.

    00

  • #

    L.J. Ryan,

    GHG physics proposes terrestrial radiation of 240 W/m^2 is “forced” to 390 W/m^2 via the cooler/lower energy atmosphere.

    Same question to you. 240 in. 390 out. Where are you getting the missing 150 from without a greenhouse gas effect? How are you doing it without violating The First Law of Thermodynamics.

    00

  • #

    Sphaerica (Bob): @284

    In your world, not a single rubber ball bounces back, because they know that’s where they came from, and it’s not allowed. It violates the Second Law of Rubberdynamics.

    Actually, you are right and wrong here. Yes, some will bounce back, but they will never again get back to the gun that fired them. There is not enough energy left in the system to allow it. Get rid of gravity, as outside of massive entities near the mass of a star or black hole, photons (balls) are unaffected. What goes up in this case does not need to come back down, ever.

    00

  • #

    astonerii,

    Deserts cold at night due to a lack of moisture (you know, H2O, that other greenhouse gas). Without it, deserts lose heat rapidly at night (due to a lack of the GHG effect), They are also hot during the day because they are unable to lose heat during the day through evaporation.

    In addition, the increase in water vapor in the atmosphere of a warming world is not uniform. The same effects of the Hadley Cells that deprive today’s deserts of moisture continue to operate, however the Hadley Cells are growing in size and strength with the warming climate (predicted by theory and observed in fact), and with them the deserts of the world will grow as well… not because of more heat, but because of larger bands north and south of the equator that are starved of moisture.

    00

  • #

    astonerii,

    Really? Are you serious? A photon emitted straight at the ground will not have enough energy to reach the surface of the earth? Every photon emitted from the atmosphere must go up and out because it offends your sensibilities for them to go down?

    Do you have any idea how silly you make skeptics look with comments like that?

    00

  • #

    astonerii,

    I’m picturing all of these photons whizzing at the ground at the speed of light (gee, where’d a phrase like that come from). Suddenly, they all turn into Wiley Coyote, pull out Acme rocket packs or “Air Brakes” or “Don’t Break the Second Law of Thermodynamics” signs, then stop in midair, reverse, and shoot up out of the atmosphere.

    The world is saved from climate change by Loony Toons Physics.

    Hooray.

    00

  • #
    Matt b

    “Actually, you are right and wrong here. Yes, some will bounce back, but they will never again get back to the gun that fired them.”

    This is not correct. A ball propelled against a solid object is perfectly able to bounce back further than from where it was propelled. The game of squash/racketball would be quite different if you were correct.

    00

  • #

    Sphaerica (Bob): @289 @290 @291 and Mattb @292

    Process:
    1) Energy arrives at the Earth from Sun.
    2) Earth Warms
    3) Earth radiates
    4) Some parts of the radiation are absorbed into CO2 through which they come into contact with.
    5) Dependent on the amount of radiation, all the available radiation gets absorbed in a finite distance from the Earth’s surface.
    6) Some of this radiation absorbed can be re-emitted, lets just call it 100% just to make you idiots look better.
    7) Randomly half of this radiation will be emitted towards the surface of the Earth, now fully 50% less total radiation available, but the exact same level of CO2 and the exact same distance to reach the Earth, here gravity has no impact on speeding or forcing these (balls) back down to the ground, (are you feeling a bit wiley e coyote ridiculous at the moment?). Thus, less photons, same obstacle course, failure to penetrate to depth.
    8) Of that 50% half will again be sent towards the Earth and again fail to penetrate the distance, this will continue until there are 0 photons left. Out of all the photons sent by Earth towards outer space, essentially 0 will make it back to the Earths surface, and then only those that got blown off course through convection currents down closer to the surface. Sans outside force (wind) (reflective surface) (absorbed in the absolute first distance from the Surface of the Earth) radiation absorbed by CO2 never again has opportunity to warm the surface.
    9) Now, educate me on the physics I am missing that allows those photons I have shown do not reach the Earth do in fact reach the Earth.

    00

  • #

    astonerii,

    So if the photons can’t make it back through the oh-so-thick atmosphere, how did they make it out to begin with?

    Also… I note that you do everything with hand-waving and broad, unquantified statements. Ever tried doing any math? Ever tried putting actual substance behind your thought experiments, or do you figure that science is so easy, you can just eyeball things and you’re sure you’ll come up with the right answer?

    Hint: If you do the math, you don’t get the answer you were looking for (which, I suspect, is part of the reason you don’t do the math).

    00

  • #
    L.J. Ryan

    Sphaerica (Bob): 287

    GHG physics proposes terrestrial radiation of 240 W/m^2 is “forced” to 390 W/m^2 via the cooler/lower energy atmosphere. is not a question

    However, I’ve answered your question a number times.

    Sphaerica (Bob), aside from your question do otherwise agree with my overarching point i.e. Maybe just maybe radiative quantities don’t add as GHG physics requires.

    00

  • #

    Sphaerica (Bob): 294

    Do it, your the one trying to prove a theory.

    your rules:

    1) CO2 absorbs certain wavelength photons and re-emits those same wavelength photons.
    2) Half gets sent back towards Earth.
    3) The photon hit the Earth’s surface, the surface warms.
    4) Global warming.

    I think my theory stacks much better than yours does. As your theory states that the radiation is only absorbed when heading towards space, but when headed towards the Earth’s surface, they transform Wiley E Coyote style such that they are invisible to the CO2.
    Radiation in the wavelengths that CO2 absorbs has an extinction rate (a distance in which 100% of available specific wavelength radiation is absorbed.)
    Once absorbed, in order for global warming to occur the radiation has to make it back to the Earth’s Surface.
    Because only half get sent back towards the Earth, and that the distance to extinction has been reduced due to less total energy, they cannot penetrate back to the surface.
    No global warming.

    So, prove to me that photons are so attracted to the Earth’s surface than they are to the CO2 molecules between them and the Earth’s surface that they looneytoons alter reality to by pass the CO2 in their path.

    00

  • #

    Silver lined Thermos. The inner lining radiates energy at x temperature, the radiated energy is reflected by the silver lining, has x temperature, but because the inner lining just sent y energy, its new temperature is <x, photon temperature x can replace lost energy of item with <x temperature.

    Silver lined thermos has a vacuum between silver lining and blackbody inner lining. Thus there will be no loss of temperature as happens in the atmosphere through a myriad of different mechanisms.

    I was nearly convinced of the thermos argument, but upon closer examination, it does not prove the idea that cooler objects can heat warmer ones, or slow the speed of energy loss. I still think the photons will be absorbed from cooler bodies into warmer bodies, but the silver lined thermos is not evidence of this.

    00

  • #

    Why are multi-pane windows vacuum sealed when it would be far easier to seal them with CO2 to reduce heat loss? Questions, questions. I think someone could make a mint by taking this idea to fruition. Or better yet, maybe this would be a good place to get rid of those new refrigerant by products that are orders of magnitude more effective than CO2.

    00

  • #
    CO2ISLIFE

    A little theoretical adventure:

    This article from Australian Physicist John Nicol is a really important early work in the understanding of the effects of atmospheric CO2.

    Here John derives from first principles just how CO2 gets and deals with captured energy, and just how quickly it unloads its molecular excitation to surrounding atmospheric gasses.

    Here he shows us that all an increasing CO2 content will do is to lower the height above the ground where what CO2 there is transfers its captured heat energy to surrounding gas molecules which immediately adiabatically rise in height and are replaced by colder molecules.

    The total amount of energy available for division between capture, distribution and re-radiation is set by the available energy from the sun radiating from the earth’s surface and aportionment has little to do with increasing atmospheric CO2 content:

    http://www.middlebury.net/nicol-08.doc

    You will note this model allows Back Radiation but this becomes an invariant with rising CO2 levels and equates to a fixed part of the initial radiation.

    If one disregards the back radiation feedback in equations 26,27,28 of this model due to the overwhelming electromagnetic pressure of ground radiation this paper is still an intellectually stimulating discussion of the quantum processes useful to both camps of opinion on back radiation reality.

    I’ve quickly summarized this article according the posting rules for links so please no blog space wasting semantic arguments over the reader bait.

    Having lit blue touch paper exit stage right…….

    00

  • #
    Alistair

    299: an interesting analysis somewhat mirroring my own. Let’s consider this thought experiment again: I’m on the beach, the air temperature is 25°C and as it’s windy, the sand temperature is 30°C. I then erect a windbreak and because I reduce local convection, the sand temperature rises to 45°C so the sum of convection and radiation remains constant. If ‘back radiation’, defined as extra heating of the Earth’s surface as part of that IR is absorbed by GHGs were true, I will have increased global warming without having to go through the GHG route, and that’s impossible.

    Read this article by one of the US’s top physicists: http://www.firstthings.com/article/2011/05/the-truth-about-greenhouse-gases

    ‘The frightening warnings that alarmists offer about the effects of doubling CO2 are based on computer models that assume that the direct warming effect of CO2 is multiplied by a large “feedback factor” from CO2-induced changes in water vapor and clouds, which supposedly contribute much more to the greenhouse warming of the earth than CO2. But there is observational evidence that the feedback factor is small and may even be negative……..

    Indeed, the computer programs that produce climate change models have been “tuned” to get the desired answer. The values of various parameters like clouds and the concentrations of anthropogenic aerosols are adjusted to get the best fit to observations. And—perhaps partly because of that—they have been unsuccessful in predicting future climate, even over periods as short as fifteen years…….In my judgment, and in that of many other scientists familiar with the issues, the main problem with models has been their treatment of clouds, changes of which probably have a much bigger effect on the temperature of the earth than changing levels of CO2.’

    My view is that the direct GHG effect is much smaller than claimed because if it existed at the claimed level, it would violate the law of Equipartition of Energy [the basis of the 2nd Law of Thermodynamics]: thermalisation is a red herring; any slight direct GHG effect, which in the case of CO2 interacting with O2 means a negative feedback, has to come from Van der Waals’ issues.

    The problem is that climate science is full of people who haven’t had a good enough education to realise that the experimental data of over a century enshrined in standard texts on Thermodynamics, have within them the data which, when put together by properly trained scientists, disprove some basic tenets of climate science. That the fake science has survived in the literature is because of the broken peer review process. The way out is to set up a supervisory management team for the IPCC of properly qualified scientists so the junk can be ejected.

    00

  • #
    Bryan

    KR says “And what, pray, is your point?”

    I said

    1000 Joules from a 2 micron centered source, and 1000 Joules from a 20 micron centered source?

    KR cannot handle Joules so he changes my question to

    …….”I’m going to assume that you mean 1000 Joules/second, or 1000 Watts.

    NO – I meant Joules!!!!!!!

    So here is MY question again

    Do you think that 1000Joules of blackbody radiant energy centred around say 2um is thermodynamically equivalent to 1000Joules of blackbody radiant energy centred around say 20um

    a) In terms of the First Law of Thermodynamics
    b) In terms of the Second Law of Thermodynamics.

    KR says “And what, pray, is your point?”

    My point is that you peddle pseudoscience.

    You had a number of posts (involving bizarrely microwave ovens) about frequency and energy of radiant energy absorption in the atmosphere, yet you avoid answering my question.

    The question I put can be answered within the frame of equilibrium thermodynamics which is an area microwave ovens most certainly cannot.

    I know its a bit unkind of me to ask you a question on thermodynamics since you have admitted you know very little about the subject.
    However thinking about the answer might afford a learning opportunity for you!

    My guess is however you will once again evade the question.

    00

  • #
    Bryan

    Sphaerica (Bob) says

    …..”I haven’t read all of Postma’s paper, but I cannot find any place where he explains how the earth can heat to 288˚K without greenhouse gas theory. He goes on and on (incorrectly) about why GHG can’t work, but does not explain how one can find an extra 150 W/m2 without magically defying The First Law of Thermodynamics..”……

    It appears that you haven’t read ANY of his paper.

    Page 13 Maximum possible blackbody Sun facing Earth point = 121C
    Realistic value Sun facing Earth point after albedo and extinction(equator) = 50C (page 14)
    Average Earth System at 5Km = -18C page 11
    Thermodynamic calculation involving lapse rate with full equation derivation = 14.5 C for average Earth surface Page 21.

    00

  • #
    Bryan

    KR says to Paul

    …….”The point is that you are not understanding the physics. “…that the potential of the carbon dioxide to absorb and emit radiant energy is negligible” It is not, as a realistic view of the numbers would point out. CO2 represents ~25-30% of the total greenhouse effect. It is the only major component that is non-condensing, that doesn’t react to temperature changes by precipitating or evaporating (as water does, hence water acts as a feedback, not a driver). Claiming that it has negligible effect without numbers is rhetoric, not science.”…….

    Well KR nows your chance to use the numbers you claim to know so well.

    What heating effect would you find inside a volume of air the size of a large greenhouse say 30cubic metres on a KT average day?

    Note just the volume of air in the greenhouse size (just a size no greenhouse there).

    I would say its almost zero.

    Get your calculator to work out what 25% of almost zero is!!!

    00

  • #
    BobC

    Kevin:
    May 24th, 2011 at 1:46 pm

    To BobC who wrote;

    “And Kevin @ 276: You don’t know Jack about optics. Why don’t you stop pretending and read a book?”

    Bob, with respect, I do in fact know quite a bit more than “jack” about optics, having graduate degrees in both optics and electrical engineering, plus several decades of practical electro-optic engineering experience, including work on imaging satellites that are currently orbiting the Earth and returning images of the surface.

    Well then, if you are all that experienced, I withdraw my statement about “not knowing Jack”. However, it is blindingly evident that you don’t have a clue about etalons.

    Then, here’s a little review for you on how etalons work. Your explanation of the interior fields was both incoherent and totally wrong. (Hint, a single frequency standing wave cannot add up to an impulse function — obvious if you remember anything about Fourier Transforms.)

    And drop the red herring about units: What I wanted to show was the the intensity inside the etalon was multiple times larger than the intensity outside (Intensity = W/m^2 = J/s/m^2), which is what was being claimed here as impossible IN PRINCIPLE.

    00

  • #
    Mark D.

    KR says @ 280 (to Paul)

    The point is that you are not understanding the physics. “…that the potential of the carbon dioxide to absorb and emit radiant energy is negligible” It is not, as a realistic view of the numbers would point out. CO2 represents ~25-30% of the total greenhouse effect. It is the only major component that is non-condensing, that doesn’t react to temperature changes by precipitating or evaporating (as water does, hence water acts as a feedback, not a driver).

    KR with all due respect, I’m perfectly willing to read your posts about the “Second Darn Law” because you have mostly stayed on subject with the “well understood” physics. With respect to the above paragraph though, NO FREE RIDE! You haven’t yet had perfect success explaining how carbon dioxide COULD cause warming you don’t get to say “CO2 represents ~25-30% of the total greenhouse effect. There is lots more ground to cover in the warming theory for example what about saturation? The molecules of CO2 at the puny .039% of the atmosphere will all will have absorbed photons and become excited. After that has happened all other photons will pretty much have a free ride.

    Beyond that, you have a lot of explaining on how any potential effect CO2 has on nearby warming isn’t completely negated by convection and water vapor.

    While you are at it explain how all your confidence in the power of CO2 isn’t meeting any of the warmist global temperature projections.

    Claiming that it [CO2] has negligible any effect without numbers is rhetoric, not science. INDEED!

    PS a few of you here are new enough that you might not know that BobC is very much a skeptic of AGW. Just because the “well understood” isn’t so well understood doesn’t mean it’s wise to attack an ally.

    00

  • #

    L.J. Ryan,

    However, I’ve answered your question a number times.

    Sidestep, because you can’t answer the question. It’s a huge hole in your position, and you can’t fill it. Point to the posts that answer the question.

    Do it, your the one trying to prove a theory.

    Sidestep, because you can’t answer the question. You are the one who is defying modern science with a better theory — in this case, those who believe in modern science include the host of the site, Jo Nova, as well as climate scientists like Dr. Spencer, Dr. Lindzen, Dr. Choi, Dr. Christy, Dr. Pielke Sr., Dr. Muller, Dr. Curry, and many more. They all ascribe to greenhouse gas theory. You don’t.

    I’ve clearly explained how GHG works, and does not violate the 2nd law. The only response has been vague sort-of hand waving.

    I’ve also explained how without GHG, you must violate the 12 law for the earth to be as warm as it clearly is. No one ever replies. The silence is deafening.

    Bryan,

    Page 13 Maximum possible blackbody Sun facing Earth point = 121C
    Realistic value Sun facing Earth point after albedo and extinction(equator) = 50C (page 14)
    Average Earth System at 5Km = -18C page 11
    Thermodynamic calculation involving lapse rate with full equation derivation = 14.5 C for average Earth surface Page 21.

    While his explanations and derivations are bizzare, those equations are standard, basic fare in climate science. None of the equations on those pages explains why the temperature of the earth is 288˚K. To get that, you have to move on to page 21, where does something very simple.

    He creates an equation to compute temperature based on altitude, since for obvious physical reasons the earth is warmer at the surface, and temperature drops as you get higher into less dense air.

    He then uses the known temperature of the earth, 288˚K, and says that since that is the average temperature of the earth, it must also be the average for the equation of height vs. temperature. Okay, we’ll accept that. With this he computes the midpoint, and says everything above is cooler, everything below is hotter.

    When you get to the surface, he gets 14.5˚C. Perfect!

    Did you see what he did? He ran you through a complex serious of equations and explanations, from page 1 to page 21, to get to a point where he computed the earth’s temperature at 14.5˚C.

    But all he did there was to take the tables for adiabatic lapse rate, turn them into an equation, and work backwards. His argument amounts to the observation that temperature decreases with altitude (surprise!) and to use that to compute the temperature of the surface of the earth by using the observed temperature of the surface of the earth.

    But he didn’t need pages 1 to 20. He only needed page 21.

    Look at it another way.

    Gases are not simply hotter because they are more dense. Yes, if you compress a volume of gas, the temperature will rise. But you can’t keep compressing it forever. Once you stop compressing it, it’s going to cool. But the earth isn’t cooling. So how does that much energy keep getting further down, to the surface, to keep the temperature from dropping?

    Gravity isn’t magic, but Postma is a great illusionist. Postma used sleight of hand to focus you on the Second Law of Thermodynamics, to distract you from the fact he was violating the First Law of Thermodynamics. He used pages and pages of equations, to reach the point where he slipped in the clincher, a simple calculation of a reasonable temperature for the earth’s surface, but one which is merely based on taking the existing observed values of the system without actually deriving anything.

    He never explains how this magic works. That’s the hallmark of a great illusionist. Create enough distractions that the mark never notices the obvious, the fact that nothing really happened at all.

    Magic!

    But where did the energy come from? No matter what he calculates, 240 W/m2 will only generate a temperature of 255˚K (-18˚C). Period. If you want a higher temperature, you have to get the energy from somewhere.

    Where is it coming from? Explain it! Stop dodging and state how Postma or the rest of you can live on a planet at 288˚K without violating The First Law of Thermodynamics.

    00

  • #
    BobC

    And Kevin: An etalon stores energy — it doesn’t create it. I think that should be obvious to anyone who read my post, as I explicitly state that several times.

    00

  • #
    BobC

    L.J. Ryan (@ 282):
    May 24th, 2011 at 1:39 pm

    Seriously though, according to your calculations 1,000,000 W/m^2 bouncing around the inside the etalon means the outside is dissipating 1000 W/m^2…(.001 absorption). These little cavities (with an area << smaller then a m^2) must be white hot if not molten mess. Or maybe just maybe radiative quantities don't add as GHG physics requires.

    Wow Ryan! You’ve just “proven” that etalons with Q’s of 10^3 – 10^6 are physically impossible! You better inform REO quickly — they have been selling them for 20 years.

    But of course, with egotistical solipsists, flacky mental “analysis” trumps the real world everytime.

    Follow me carefully here: The input to the device is 1000 W/m^2. If 1000 W/m2 was being dissipated, there would be nothing coming out.

    Oh, and since 1000 W/m^2 is approximately the intensity of the noon sun in the dessert (as I stated in the post you didn’t quite read), totally absorbing this input would be similar to putting a piece of black painted glass in noon sunshine. Maybe your experience is different (chemically enhanced?), but I haven’t noticed such things turning into a “molten mess”.

    00

  • #
    Bryan

    Sphaerica (Bob) says

    “Gases are not simply hotter because they are more dense.”

    Quite correct
    I agree some posters seem to have a simplistic Charles Law mistake.

    They are hotter because of the Sun heating the Earth system which is mainly the surface.
    The way I looked at it prior to the Postma article was a bottom up approach (and probably not that different to your view).

    Earth surface heats atmospheric layer => convection => clouds + radiation to space(highly simplified)

    An analysis of the kinetic theory of gases in a gravitational field and the adiabatic expansion of an ideal gas gives the same temperature profile.
    A single gas particle going vertically up tells most of the story KE (hence kT) changing to gravitational PE
    The other route was a thermodynamical derivation such as Postma page 21
    Both these routes are compatible and give the same results

    The IPCC advocates derived the -18C and then contrasted this with bare rock Earth to get their 33K “greenhouse effect”.

    Postma said the average effective emission height is 5km for the (Earth+atmosphere) system.
    The bottom up approach matches the top down approach.

    Where the Postma analysis would break down is if someone proved that the average effective emission height is NOT 5km for the Earth system.

    But no Postma critic has contested this point.
    What I can’t see is any significant greenhouse heating.

    00

  • #
    L.J. Ryan

    Bob C 308

    You claim 1,000,000 W/m^2 is internal to etalon. You claim the surface is .999% reflective.

    Follow me here Bob: 1-.999=.001. .001 X 1,000,000 W/m^2 = 1000 W/m^2…if not dissipated as heat what happens to the 1000 W/m^2?

    2)Are you saying the reflective inefficiency (.001) is the intentional…it’s the output?

    3)Wouldn’t the RD department REO like to increase etalon efficiency to .9999% or better yet .99999%?

    If the loss is not intentional and is dissipated as heat, a need to know variable is surface area.
    4)Being the rep for REO, what is the surface area of your etalon?

    In reference to my other point on 282: Should I not expect your “expertise” on getting 60 F radiant floor to warm my house to 127 F? Will we see trial lawyers soliciting cases for those injured by garment malpractices? Or maybe just maybe radiative quantities don’t add as GHG physics requires.

    00

  • #
    BobC

    L.J. Ryan:
    May 25th, 2011 at 12:41 am

    Bob C 308

    You claim 1,000,000 W/m^2 is internal to etalon. You claim the surface is .999% reflective.
    Follow me here Bob: 1-.999=.001. .001 X 1,000,000 W/m^2 = 1000 W/m^2…if not dissipated as heat what happens to the 1000 W/m^2?

    2)Are you saying the reflective inefficiency (.001) is the intentional…it’s the output?

    Ryan, do you have trouble reading, or are you just a little dumb?

    1) I postulated a reflectivity of 0.999 A reflectivity of .999% would be just under 1% reflection.

    2) There are 3 things that happen to light striking a partially reflective mirror: part of the light is reflected, part is transmitted and part is scattered/absorbed.

    You are making the mindless assumption that the light that is not reflected is absorbed, and hence there is no transmitted light. How, then, is any light supposed to get into the cavity?

    These, and the rest of your questions show 1) A lack of reading ability, 2) Unwillingness (or inability) to follow reference links and educate yourself, and 3) Plain stupidity (intentional or congenital).

    If you can manage to come up with an intelligent question, then I might answer it (assuming I even check in on this nuthouse again).

    00

  • #
    KR

    L.J., RJ, Bryan, MarkD, etc.

    I would suggest taking a look at The Discovery of Global Warming. This is an excellent reference documenting the historic development of the radiative greenhouse gas theory, starting with the summary.

    At this point the discussion has really circled – repeated statements of current theories versus repeated “Just So Stories” (Physics that don’t allow microwave ovens to work? Lasers heating via ‘shock waves’? Claiming minimum energy/microstates for objects well above 0K? The whole ‘greenhouse effect violates 2nd law’ nonsense?). No progress…

    I’ve spent quite a bit of time here attempting to convey what I know of the observations and theories, but it’s not my job to teach Atmospheric Physics 001. I would encourage everyone to look further into the topic, and learn a bit – “common sense” only applies where you have actual experience, and will lead you astray if you extend it into areas you don’t know very well.

    Finally – as has been stated before by Jo, Michael Hammer, myself, and others – pushing hypotheses that are just plain wrong leads to your opinions being discounted as nonsense. If you want to be taken seriously, you’re going to have to step up your game.

    Adieu.

    00

  • #

    Bryan,

    An analysis of the kinetic theory of gases in a gravitational field and the adiabatic expansion of an ideal gas gives the same temperature profile.

    No, you’ve fallen into a trap.

    Consider this. We start with a gas at a given, homogeneous temperature around a planet. Gravity sucks the gas down, compressing it closer to the surface. That part of the gas closer to the surface is warmer than the cooler, higher gas that is less dense. The gas hasn’t actually gained energy, though, it has simply been forced into a smaller volume/higher pressure near the surface, so the temperature there goes up. Correspondingly, the temperature in the area that is now less dense, higher up, goes down.

    Think of it, for simplicity, in two layers, a cool, rarefied top layer, and a warm, dense lower layer. Both of these must emit and cool, but the lower layer must emit faster because it is warmer.

    But we have a system that approximates the earth, in a similar state (warm lower atmosphere at 288˚K, cool upper atmosphere at some temp below 255˚K).

    Now turn on the sun to keep things at that temperature. But for that lower atmosphere to maintain its temperature, it must receive at least enough radiation to balance the 390 W/m2 that it is emitting. If not, it is going to cool. You got it to 288˚K to start with by compressing it with gravity, but that only works once.

    As I said, Postma created an equation which matches our observations (i.e. adiabatic lapse rate), but then left it there with the presumption that mere density raises temperature and keeps it there without any further input of the energy required to maintain it.

    If these two masses, dense, warm low and rarefied, cool high, do not equalize their temperature, then you are violating the second law of thermodynamics. You’re basically saying that gravity traps heat.

    He’s tricked you by making it seem like this is the way things are supposed to be, but by leaving out some very important considerations, like explaining how mere gravity can maintain that temperature gradient.

    Both these routes are compatible and give the same results.

    But they don’t. Postma’s equations simply describe the environment, without in any way explaining it. And if you try to explain it, you have to find 390 W/m2 at the surface to maintain the temperature. There is no getting around that.

    Postma is either violating the First Law of Thermodynamics, and creating energy from nothing, or else he is violating the Second Law of Thermodynamics, by trapping heat at the surface and refusing to allow it to cool… for no reason other than the fact that observations show that the surface is warmer.

    00

  • #
    Mark D.

    Sphaerica (Bob): @313

    Postma is either violating the First Law of Thermodynamics, and creating energy from nothing, or else he is violating the Second Law of Thermodynamics, by trapping heat at the surface and refusing to allow it to cool… for no reason other than the fact that observations show that the surface is warmer.

    Or is he talking about the real world where there is a dynamic process with continual uplifting of moist air masses, together with compression as this air is replaced with dry (after moisture is condensed at altitude) air masses? A nearly continual Moist Adiabatic change upon rising and a nearly continual Dry Adiabatic change upon the falling of air masses. Thus mimicking (or is it accounting for) all the heat you have decided is cause by CO2?

    00

  • #
    Bryan

    KR

    Your weazle way out fooled nobody.
    You started a discussion and got pinned down on several topics.
    You give a very bad account of the type of person that advocated “settled consensus science”.
    No wonder the “team” avoid any contradictory viewpoints.
    Their theories don’t stand up to scrutiny.

    00

  • #
    Mark D.

    Sphaerica (Bob):

    To elaborate on the above; the Sun supplies the energy to cause uplift, gravity operates the return stroke of this “engine”.

    When the cooled (on average drier) air is pulled by gravity down to lower elevation, the Dry Adiabatic lapse rate will warm at a rather large rate. I invite you to provide any actual measurements worldwide that account for and quantify this very large heat movement.

    00

  • #
    Mark D.

    KR, don’t get me wrong, I appreciate the amount of effort you’ve put in here. I don’t necessarily agree with all your points but some of what you have typed is interesting if not helpful. Other times you have visited Jo’s site haven’t been anywhere near this productive and you shouldn’t leave without at least the impression that there can be discussion.

    00

  • #
    L.J. Ryan

    BobC: 311

    Personal insults Bob, sounds as if I’m insulting your relgion not your bull**** physics.

    You claimed radiative quantities are additive. You said a ceiling acts like the atmosphere warming a room beyond input. You said a jacket, by means of back radiation warms a body. You claimed the etalon you sell, store 1,000,000 W/m^2. When pressed for clarification, you insult and dodge.

    So please Bob C answer the following questions, reveal my stupidity:

    1)If not dissipated as heat what happens to the 1000 W/m^2?

    2)Are you saying the reflective inefficiency (.001) is the intentional…that is,it’s the output?

    3)Wouldn’t the RD department REO like to increase etalon efficiency to .9999% 99.99% or better yet .99999%? 99.999% or best yet 100%?

    If the loss is not intentional and is dissipated as heat, a need to know variable is surface area.
    4)What is the surface area of your etalon?

    5)With 20 years at REO, maybe you can save me the calculation and let us all know the exterior temperature of an etalon containing 1,000,000 W/m^2?

    6)A floor radiating 240 W/m^2 with a well insulated, highly reflective ceiling should, according to GHG physics, begin radiating 390 W/m^2. Or maybe the ceiling need be matte black…emulating blackbody. Does 390 W/m^2 happen instantaneously or should I get one of those GHG physic jackets? If they still legal?

    Or maybe just maybe radiative quantities don’t add as GHG physics requires.

    00

  • #
    Mark D.

    KR, with links at 312 says

    L.J., RJ, Bryan, MarkD, etc.

    I would suggest taking a look at The Discovery of Global Warming. This is an excellent reference documenting the historic development of the radiative greenhouse gas theory, starting with the summary.

    Thank you for the links, I’ll take a look.
    In the meantime how about you take a look here:
    http://joannenova.com.au/global-warming/climategate-30-year-timeline/
    for a little insight about what the skeptic sees.

    00

  • #
    Bryan

    Sphaerica (Bob)
    1.Start with the Sun heating the Earth surface(Average 15C).
    2. Air molecules hitting the surface leave with the characteristic temperature of the surface.
    3. Pick one air molecule moving vertically without colliding.
    4. Convert air molecules KE(3kT/2) into PE as it works against gravity also involving PV expansion of gas formula.
    5. This gives you the adiabatic lapse rate.
    6. Exactly the same result is found by thermodynamics = -g/Cp or – 9.8K/km This formula and its thermodynamic calculation is an accepted part of serious climate science.
    All the energy comes from the Sun.(bottom up approach)
    All energy accounted for.
    All this I knew before reading Postma

    Now Postma contribution is to say that measured from space it appears that the effective radiating temperature is -18C and the effective average radiating altitude is 5km.
    By picking the existing well known average lapse rate figure you arrive back at 1.

    All Postma says is that the space calculations match up with the observed surface temperature of 15C.

    00

  • #

    MarkD,

    First, Postma didn’t say how. He just threw out some equations, and expected everyone to just accept it. He does not discuss how the gradient is maintained, he just says it’s there, and because it’s there, no greenhouse effect is needed. This translates into “because the results of the greenhouse effect exist, and I can quantify that with an equation that doesn’t use the word greenhouse, then the greenhouse effect doesn’t exist.” It’s silliness.

    I also don’t understand your mechanism. You’re moving warmer air up. Doesn’t that warm the upper atmosphere? And why would sinking air heat more than rising air cools, when it’s just the same mechanism in reverse? Sounds like a perpetual motion machine to me. Then you add the idea of moisture. So the moist air goes up, and the dry air goes down. But this transports heat upwards, not downwards (see evapotranspiration and latent heat).

    I’d warn against “thought experiments” without hard numbers. It’s too easy to presume one thing happens as much (or as little) as you’d like it to to make your theory work.

    …the Sun supplies the energy to cause uplift, gravity operates the return stroke of this “engine”.

    Gravity is not a perpetual motion machine. The only source of energy is the sun. And no matter how you try to use it, you have 240 W/m2 to use. You need some mechanism that transports heat from the upper atmosphere to the lower, to maintain the gradient.

    Postma’s paper is about the premise that any such mechanism violates the Second Law of Thermodynmics. Now you’re proposing exactly such a mechanism, but one which uses convection instead of radiation. So how does that validate Postma’s paper?

    If you don’t mind my saying so, you’re grasping at straws, using vague concepts, trying to conjure any way that greenhouse theory would not be valid. Does that strike you as a “skeptical” approach to science?

    …the heat you have decided is cause by CO2?

    First, the heat isn’t “caused” by CO2. Second, I haven’t decided it. Modern science has, and you and those who refuse to accept it are far, far outside of the mainstream.

    00

  • #
    KR

    Mark D. @ 317

    Thanks, I’ve been trying to stay reasonable and on topic.

    However, when posters earnestly espouse ‘physics’ that would invalidate microwave ovens, or state that “lasers heat via shock waves”, and then these very same posters sincerely tell me amidst insults that I don’t understand the physics (insert sound of me slapping my forehead), well, there’s not much of a discussion going on.

    At that point those posters are discussing beliefs, not science, and I make a point not to argue beliefs – it’s just annoying all around.

    00

  • #

    I only check about 1 in 20 comment replies to my email…the comment at 313 (Sphaerica (Bob)) finally made me write back in:

    “Sphaerica (Bob): As I said, Postma created an equation which matches our observations (i.e. adiabatic lapse rate), but then left it there with the presumption that mere density raises temperature and keeps it there without any further input of the energy required to maintain it.

    He’s tricked you by making it seem like this is the way things are supposed to be, but by leaving out some very important considerations, like explaining how mere gravity can maintain that temperature gradient.”

    For Goodness’ sake. No tricks intended.

    It is the LAWS of physics, and thermodynamics, as represented IN that adiabatic equation, which specifies that there HAS to be a temperature gradient in a gravitational field. The equation isn’t “created to match that”. It comes straight out of basic physical principles! I mean come on man…you might as well ask how does gravity maintain a constant acceleration near Earth’s surface.

    I can’t answer your last question for you if you don’t even know or understand highschool physics.

    The sunlight comes in and heats the ground surface, at an average of +30C (303K) and potential maximum of +121C; the air convects and rises to great heights, expands and cools, then comes back down and compresses and heats. If convection is strong enough and the atmosphere is deep/dense enough and given a thermal capacity, by the time the air gets back down to the ground it can be heated to an even higher temperature..

    “Sphaerica (Bob): But they don’t. Postma’s equations simply describe the environment, without in any way explaining it.”

    What do you THINK equations are for, and do? They describe the environment because they describe what’s real – which is basic physics. What you’re saying is that d = v*t only “describes” the distance traveled by an object on velocity “v” over time “t”, but doesn’t “explain” it. THAT’S how basic the adiabatic equation is. The equivocation you present is priceless sophistry.

    “Sphaerica (Bob): And if you try to explain it, you have to find 390 W/m2 at the surface to maintain the temperature. There is no getting around that.”

    See above. +30C solar input, ~+121C max input, resulting convection effects, QED.

    How do YOU propose to explain it? By NOT specifying any actual physical mechanism supported by basic physics equations, and just stating a desire that radiative energy can spontaneously amplify itself in an atmosphere? That’s not just NOT a better explanation, it is fictional.

    “Sphaerica (Bob): Postma is either violating the First Law of Thermodynamics, and creating energy from nothing”

    Alright…I’ll keep this brief. The GH effect creates energy from nothing and violates ALL the laws of thermodynamics. The adiabatic equation and ACTUAL physical solar insolation are fundamental basic physics equations and reality, and provide all the physical mechanisms necessary. Add in additional energy from Coriolis forces and possible geothermal contributions and there’s more than enough energy to explain the ground temperature…not that basic thermo and physics doesn’t do it already.

    “Sphaerica (Bob): or else he is violating the Second Law of Thermodynamics, by trapping heat at the surface and refusing to allow it to cool… for no reason other than the fact that observations show that the surface is warmer.”

    I thought it was the GH Effect that “TRAPS” heat at the surface and keeps it from cooling? LOL! You’re saying this is impossible now? The adiabatic distribution allows it to cool just fine…there’s NO restriction on the ability to cool!
    And it’s not just observations that confirm the adiabatic distribution effect, it is fundamental physics and basic logic that the bottom of the atmosphere should be warmer than the ensemble system average.

    00

  • #
    RJ

    Joseph Postma @ 323

    How do YOU propose to explain it? By NOT specifying any actual physical mechanism supported by basic physics equations, and just stating a desire that radiative energy can spontaneously amplify itself in an atmosphere? That’s not just NOT a better explanation, it is fictional.

    Thanks Joseph for the great work. People like Sphaerica will find it hard to let this belief go (radiation energy can amplify itself) but it is such a silly belief as well as being completely unscientific that it will happen.

    How did a belief like this continue unchallenged for so long? I guess many or maybe even the majority of scientists are not great scientific thinkers after all but just accept what they are told.

    00

  • #

    I know, you’re right Hans! I know. In any case, here is another:

    “First, Postma didn’t say how. He just threw out some equations, and expected everyone to just accept it. He does not discuss how the gradient is maintained…”

    I don’t know if you actually have ANY physics training whatsoever…certainly not even high-school physics. The gradient does not need to be “maintained” – that’s a completely senseless way of stating the physics. The gradient is the natural, expected distribution of temperature of a compressible gas in a gravitational field. Asking why gravity maintains a denser atmosphere near the surface than at altitude is the same question you’re asking. Can you answer that?

    “He does not discuss how the gradient is maintained, he just says it’s there, and because it’s there, no greenhouse effect is needed.”

    Actually, the physics DOES explain why there is a gradient, and yes you are correct, it explains why no greenhouse effect is needed, especially when you also add in the actual average insolation of +30C with a maximum of up to +121C.

    “This translates into “because the results of the greenhouse effect exist, and I can quantify that with an equation that doesn’t use the word greenhouse, then the greenhouse effect doesn’t exist.” It’s silliness.”

    Sophistry…thanks for the example. http://dictionary.reference.com/browse/sophistry

    “I also don’t understand your mechanism.”

    Finally some actual science out of you…admitting you don’t know something. That’s the first step to knowledge…if your ego can handle it.

    “You’re moving warmer air up. Doesn’t that warm the upper atmosphere?”

    Yes, it is called convection. This is exactly what it does.

    “And why would sinking air heat more than rising air cools, when it’s just the same mechanism in reverse? Sounds like a perpetual motion machine to me.”

    This is a very good question, and must have a physical answer. Energy and temperature can’t just create more and more of itself without some physical mechanism introducing the additional energy. (Reflected/re-emitted IR is NOT additional energy by the way.) But the problem isn’t on Earth anyway, as the ground-air temperature (+15C average) is LESS than the solar insolation average of +30C. The question is more interesting for Venus. However, the adiabatic distribution is still a fundamental and expected property of its atmosphere, like on Earth, and so is the likely cause. Obviously, at Earth and Venus, there is enough solar insolation to maintain a gaseous atmosphere. Once that atmosphere is maintained, more things can start to happen.
    Move Venus to Pluto and it would have hardly any atmosphere at all, and all the gases would be frozen out on the surface. But move Venus back to where it should be, and it will “re-grow” its atmosphere to the same depth and density it has today. The ENERGY is supplied by the Sun, but the temperature at the surface of Venus naturally develops due to the crushing density and adiabatic distribution. Once Venus’ atmosphere is “full grown”, as it is now, it doesn’t even absorb enough radiation IN THE FIRST PLACE to explain its surface temperature…therefore there needs to be a physical explanation for it, such as the adiabatic distribution – a result of basic and fundamental physics – rather than inventing some silly postulate that radiation can amplify itself passively without additional energy input.

    “The only source of energy is the sun. And no matter how you try to use it, you have 240 W/m2 to use.”

    This is incorrect. You have +30C to use (478 W/m2), on average, but a maximum average of about +87C (952 W/m2) over an area 33% larger than North America, about 45% of the diameter of the Earth. The whole error starts with thinking that 240W/m2 is the input…it is NOT. The input is MUCH more intense than this. Please read the ENTIRE paper at the link.

    “You need some mechanism that transports heat from the upper atmosphere to the lower, to maintain the gradient.”

    You’re mixing things up. The gradient is natural and would exist without ANY vertical mixing whatsoever. Vertical mixing actually slows the gradient as heat is transported from the surface UPWARDS, by convection. If air from altitude comes down to WARM the surface, it can’t if it is COLD! It can only if it has been compressed and heated to a higher temperature.

    “Postma’s paper is about the premise that any such mechanism violates the Second Law of Thermodynmics.”

    No, it isn’t, and the paper is NOT about the 2nd Law. This is a completely incorrect summary. Please read the entire thing, and get the copy at the link above which is the correct version.

    “Now you’re proposing exactly such a mechanism, but one which uses convection instead of radiation. So how does that validate Postma’s paper?”

    Because one is supported by basic physics and is real, and is observed, and the other satisfies neither.

    “If you don’t mind my saying so, you’re grasping at straws, using vague concepts, trying to conjure any way that greenhouse theory would not be valid. Does that strike you as a “skeptical” approach to science?”

    I have already exposed your sophistry in this and my previous post. Ta-ta…
    It is PERFECTLY skeptical to pursue the flaws of AGW, consider the possibility that the GHE might be incorrect, study it for while, then conclude that it likely is. That’s called science and skepticism.

    “First, the heat isn’t “caused” by CO2. Second, I haven’t decided it. Modern science has, and you and those who refuse to accept it are far, far outside of the mainstream.”

    Ahhhh aaaand HERE we have it…the collectivist social-network peer-pressure junior-high approach to science. What happened to your “skeptical approach to science”? We see it was only a facade.
    We DO NOT CARE how many friends you have on Facebook, we DO NOT CARE what all your friends think. We DO NOT CARE for the mainstream. That we are outside of all that indicates our success and our leadership.

    Back to infrequent attention from me…I see that the situation is being handled by true skeptics and scientists very well. Thank you all for the support and filling in the gaps while I am busy doing other work.

    00

  • #

    Oops, my comment to “Hans” won’t make sense at all at 325. Just ignore it.

    00

  • #
    BobC

    L.J. Ryan:
    May 25th, 2011 at 3:57 am

    BobC: 311

    Personal insults Bob, sounds as if I’m insulting your relgion not your bull**** physics.

    So please Bob C answer the following questions, reveal my stupidity:

    OK Ryan, just this once.

    1)If not dissipated as heat what happens to the 1000 W/m^2?

    If it’s not dissipated as heat, then it just passes through the etalon. (Duh!)

    2)Are you saying the reflective inefficiency (.001) is the intentional…that is,it’s the output?

    Had you bothered to check the Wiki page I linked you would have known that Fabry-Perot etalons are made with partially reflective mirrors. I specified the reflectivity; The difference between that and 1.0 is the transmission (less a very very small amount of scatter/absorption loss. I said REO’s mirrors were good.)

    But, of course, you couldn’t be bothered to spend 1 minute reading about a device you had obviously never heard of before. Instead, you just made a bunch of ignorant assumptions. Part of intelligence, L.J., is knowing what you don’t know.

    3)Wouldn’t the RD department REO like to increase etalon efficiency to .9999% 99.99% or better yet .99999%? 99.999% or best yet 100%?

    Beats the hell out of me, L.J. Maybe you should ask them — give them the benefit of your keen insight.

    If the loss is not intentional and is dissipated as heat, a need to know variable is surface area.

    Well, no. I gave the intensity, which is power/area. Since heat dissipation also goes as the area, the area cancels out of any calculation about heating.

    4)What is the surface area of your etalon?

    I don’t own one. Check with REO’s sales department.

    5)With 20 years at REO, maybe you can save me the calculation and let us all know the exterior temperature of an etalon containing 1,000,000 W/m^2?

    There you go with ignorant assumptions again. I have never worked for REO, but I can give you the answer off the top of my head: Room temperature, plus a small fraction of a degree.

    6)A floor radiating 240 W/m^2 with a well insulated, highly reflective ceiling should, according to GHG physics, begin radiating 390 W/m^2. Or maybe the ceiling need be matte black…emulating blackbody. Does 390 W/m^2 happen instantaneously or should I get one of those GHG physic jackets? If they still legal?

    Do I really need to tell you why this is stupid?

    00

  • #
    mkelly

    313Sphaerica (Bob):
    May 25th, 2011 at 3:07 a
    “You’re basically saying that gravity traps heat.”

    No, persons like myself say gravity creates heat via pressure. How do you think stars are born?

    Some amount of the “natural temperture” of earth is caused by the mass of the atmosphere and gravity.

    00

  • #
    Mark D.

    Sphaerica (Bob): @ 321

    I also don’t understand your mechanism. You’re moving warmer air up. Doesn’t that warm the upper atmosphere? And why would sinking air heat more than rising air cools, when it’s just the same mechanism in reverse? Sounds like a perpetual motion machine to me. Then you add the idea of moisture. So the moist air goes up, and the dry air goes down. But this transports heat upwards, not downwards (see evapotranspiration and latent heat).

    OK so who is in denial here?
    You do believe in warm moist rising air masses. (as you’ve stated)
    and you admit to them cooling drying and then falling back to earth.
    Where does the adiabatic lapse rate kick in?

    Do you not believe these air masses have momentum and inertia?
    That once they start moving down they just stop because of warming and begin to rise again? If you are correct then the night time temperature should go how cold?

    I’d warn against “thought experiments” without hard numbers.

    That sounds like where models come in huh?

    OK so you deny the easily demonstrable adiabatic lapse rate and call it more of a “thought experiment” hmmmmmmm…..

    I know it’s complicated but remember since your theory doesn’t meet with observations then I have to make some assumptions. There is more going on in the atmosphere than you or anyone else can measure.

    00

  • #
    CO2ISLIFE

    Now since we’re all agreed that Poynting clinched it and big pressures overcome small pressures when it comes to low down radiation.

    We’ve another argument to settle on why the electromagnetic pressure from sun does not prevent outgoing radiation to space from earth’s atmosphere.

    The escape hatch is thusly made:

    A bit of this:……..

    http://en.wikipedia.org/wiki/Black_body

    Stuffed into here:……….

    http://en.wikipedia.org/wiki/Poynting_vector

    http://hyperphysics.phy-astr.gsu.edu/hbase/waves/emwv.html

    http://www.mike-willis.com/Tutorial/PF3.htm

    And Ze the Vindow, she opens.

    00

  • #
    Paul

    Back to infrequent attention from me…I see that the situation is being handled by true skeptics and scientists very well. Thank you all for the support and filling in the gaps while I am busy doing other work.

    Thanks for popping in and contributing some sanity and objectivity to this ‘discussion’.

    I have started to read again your paper and notice early in the paper you write : —

    “If the source of light is constant, meaning it shines with the same unchanging brightness all the time, then the blackbody absorbing that light will warm up to some maximum temperature corresponding to the energy in the light, and then warm up no further. When this state is reached it is called “radiative thermal equilibrium”, which means that the object has reached a stable and constant temperature equilibrated with the amount of radiation it is absorbing from the source of light. This is distinct from regular “thermal equilibrium”, which is when two objects which are in physical contact eventually come to the same temperature, if they started out at different temperatures. In radiative thermal equilibrium, the object absorbing the light will not come to the same temperature as the source emitting the light, but actually will always be cooler than it because the distance between the two objects reduces the energy flux density of the radiation from the source.”

    Isn’t it just here that the confounding assumption that leads to the nonsense of CAGW comes in, when it is assumed that all the energy absorbed at the surface of the earth is emitted from the surface by means of radiation only?

    The earth’s thermal behaviour is treated as if it were not in physical contact with the atmosphere!

    I think that the proponents of the CAGW theory do not see the fallacy of that assumption because they have already ‘believed’ that, without any Tindall gasses in the atmosphere, the earth would necessarily have to lose all its energy by means of radiation. Where is the proof of that assumption? Where is that assumption even acknowledged by those who make it as their starting point?

    Hence, having made this unjustified and unacknowledged assumption, they calculate the warming effect of Tindall gasses on the near-earth surface air temperature as the difference between the temperature of a planet without any atmosphere and the current temperature near the surface of the earth and attribute it entirely to the ‘Greenhouse Effect’.

    Everything else, with billions of dollars of research applied, is simply to try to justify that assumption and to confirm that belief [and try to persuade us sceptics that we have it all wrong.]

    No account is ever taken by the CAGW Team of the obvious fact that huge chunks of energy are transported from the surface by means of conduction, convection and latent heat of vaporisation, either in their diagrams or their equations. And then they expect us just to ‘believe’ their conclusions because it is based on ‘settled science’ and that there is a consensus in main-stream science about the conclusion! As you so rightly pointed out above, that is post-modern science, not experimental science and thinking that way will produce no improvements to the human condition but will be disastrous on a global scale if allowed to hold sway.

    In radiative thermal equilibrium, the object absorbing the light will not come to the same temperature as the source emitting the light, but actually will always be cooler than it because the distance between the two objects reduces the energy flux density of the radiation from the source.”

    I suppose that applying that to the ‘back radiation’ from the atmosphere, which is at a lower temperature than the surface of the earth, will still apply, meaning that such radiation will not warm the surface at all. This would be in accordance with the well-known law of thermodynamics that a cooler body does not warm a warmer body. Since the CAGW theory claims the opposite, the rule ‘extraordinary claims require extraordinary proof’ applies.

    Paul

    00

  • #
    Kevin

    to BobC;

    So per your posts an eatlon both;

    “Demonstrates Energy Amplfication”

    and;

    “Stores Energy”

    Thusly, I should be able place my eatlon in front of a 1 watt laser and “fill” it with 1000 watts of “energy” because it amplifies energy. Then I can walk across town with my eatlon in my pocket with my 1000 watts of stored “energy” and give to you,

    Is that about how an eatlon works Bob ?

    Do you purchase your eatlons at the unicorn store, Bob ?

    Cheers, Kevin.

    00

  • #
    Kevin

    Whoops, the spell checker got setup to replace etalon with eatlon, but at least it spelled unicorn correctly.

    Cheers, Kevin

    00

  • #
    Paul

    Thinking about my post just above, I am reminded of a game we played as children.

    1) Think of a number between 1 and 10 [any number will do but this keeps is simple enough for children to play the game]

    2) Now double it. [this adds a distraction to hide the crucial next step, the introduction of a hidden ‘assumption’ that contains the ‘answer’]

    3) Now add 6

    4) Now halve what you have.

    5) Take away the figure you first thought of. [This removes whatever the person thought of and leaves only half of what was added at step 3]

    6) And your answer is 3!

    “How did you read my mind”, is the astonished question of the child, the first time this trick is played on them, but they soon wise up once they start trying out the new trick on some other innocent acquaintance.

    This illustrates just how an initial assumption can skew the end result, especially if and when that assumption is not acknowledged and taken properly into account, no matter how sophisticated and complicated the calculations that intervene between the assumption and the conclusion.

    I would call this, “The fallacy of the hidden assumption”. It is very common in so-called science unfortunately.

    Paul

    00

  • #

    Postma,

    …the collectivist social-network peer-pressure junior-high approach to science…

    I couldn’t have said it better myself. Pot, meet kettle. Kettle, meet the Dunning-Kruger with a Galileo complex, who has written a single unpublished paper, and in so doing has, with nore than a touch of arrogance, overturned the work of all of the professional climate scientists around the world… except, of course, he’s only convinced five die-hards on a comments thread.

    Even the host of the site, along with every reputable climate scientist (including Lindzen, Spencer, Christy, Pielke, and more) dismisses at your work.

    Hint: You have taken some very simple concepts, and cling to them like a tree in a storm. When you are faced with difficulties, you dismiss them with a wave of the hand. When the math gets tough, you fudge and use vague terms (convection! gravity! lapse rate!) instead of working out the details and presenting the numbers. Your paper is noticeably lacking in that regard.

    Second hint: Gravity increases temperature by increasing pressure, but it does not add energy, and it does not stop a warm body from radiating. You still haven’t explained (except in the most vague and impossible to test terms) how your system keeps from radiating away all of its excess heat and cooling to 255˚K.

    But I’ll give you a chance. I’ll come back in a year or two, when you’re famous from having published your work in a reputable science journal. I’m sure that you’re so confident in your theories that you’ll have no trouble at all putting it in proper form, submitting it, and seeing it published.

    [And none of that “ooh, they won’t publish skeptical work” nonsense. That’s a cop out. Submit it to E&ESceptical science OOPS that slipped [ed]. They’ll publish anything. [snip] I think Sonja will be very receptive to your work. How about a reference? [ed}

    00

  • #
    Paul

    Sphaerica (Bob):
    May 25th, 2011 at 10:20 am

    Postma,

    …the collectivist social-network peer-pressure junior-high approach to science…

    Fallacy: Ad Hominem . Dismissed as irrelevant.

    Paul

    00

  • #
    Paul

    All the talk about “a cooler body warming a warmer body” I find merely a distraction.

    Here is an experiment that would settle the argument empirically with measured values.

    Take a spherical body, contained in a vacuum and surrounded by a blackened surface that is near to absolute zero. You can approximate this by cooling the container outside with liquid nitrogen.

    Allow an aperture through which light of a given intensity is admitted and allow sufficient time for the internal spherical body to attain its blackbody temperature. This aperture needs to be one which allows the passage equally of short and long wave radiation so that what is measured in the experiment is the effect on the enclosed sphere of the reflection of its long-wave radiation back onto itself.

    Repeat this experiment with similar apparatus but have the interior of the containing vessel coated with a reflective surface. Allow sufficient time for both the reflective surface to reach equilibrium and the contained sphere to attain is new black-body temperature.

    Now compare the temperature of the contained blackbody with and without the surrounding reflective surface.

    What do you think the respective temperatures will be?

    Comment : The internal sphere will absorb energy from the impinging radiation and will warm until it is radiating the same quantity of energy, at a lower energy level, as an approximate theoretical blackbody.

    In the first scenario the long-wave radiation will be from the entire surface of the sphere and there will be nothing to prevent that radiation from escaping.

    In the second scenario, there will be radiation from the entire surface of the sphere, but in every direction other than that of the small aperture where light is admitted that radiation will be reflected back at the sphere. Thus this represents the same kind of situation hypothesised with the Greenhouse Gas Effect on earth.

    In other words, with a reduced ‘aperture’ out of which the earth can radiate its absorbed energy from the sun its surface temperature is postulated to rise.

    Talking about this second situation in terms of photons, one can imagine any photon bouncing around inside the containing, reflective sphere until its trajectory takes it to the small aperture where light is being admitted and only then making its escape.

    My question is, therefore, will the number of photons making their escape through that small aperture be any less than the number of photos that were originally being emitted from the enclosed sphere? My guess would be ‘no’.

    The second question, the important one, is then, will the temperature of the sphere that is surrounded by a reflective surface be greater than that which is surrounded by the blackened surface? My guess, again, would be ‘no’.

    Paul

    00

  • #
    BobC

    Kevin @333:

    Was my critique a little too sharp for you?

    Yours is just dumb.

    I think I’ll have to re-reconsider my opinion of your knowledge.

    Cheers, Bob

    00

  • #

    Repeat this experiment with similar apparatus but have the interior of the containing vessel coated with a reflective surface. Allow sufficient time for both the reflective surface to reach equilibrium and the contained sphere to attain is new black-body temperature.

    much of the reflected energy from the blackbody will be based on a slightly higher temperature than blackbody itself will possess on average when it is returned to the blackbody. Thus, a colder object warming a warming one is not occurring. You should probably think your ideas out a bit longer. Some atoms of the blackbody will be vibrating at higher frequency than others, thus a higher temperature radiation from those atoms or molecules. The mirror simply reflects the radiation, it does not stop it, absorb it and then miraculously send it back at a 180 degree angle from whence it came.

    00

  • #
    BobC

    Paul @338:

    Your experiment seems hard to set up. Also, I didn’t see that you considered the effect of continuing to cool the outer surface with liquid N2 — It won’t be allowed to warm up any, so most of the energy leaving the chamber will do so in the liquid N2 coolant.

    How about a much easier experiment (one I tried 30+ years ago): Sit outside with light clothing on a cold, calm evening after the sun is down until you get chilled. Then loosely wrap yourself in a “space blanket” (aluminumized mylar film). I guarantee you will, almost instantly, feel much more comfortable.

    The mylar film provides negligible insulation value for conduction, but it is quite good at reflecting the IR your body emits back at you.

    If you want to model the rest of your experiment, you can try the same thing with a sheet of black plastic, and compare the experiences.

    00

  • #

    In the second scenario, there will be radiation from the entire surface of the sphere, but in every direction other than that of the small aperture where light is admitted that radiation will be reflected back at the sphere. Thus this represents the same kind of situation hypothesised with the Greenhouse Gas Effect on earth.

    Except the fact that in the green house theory those little reflective mirrors point in all directions at the same time. Thus, the light that passes outward that gets reflected back can just as easily be rereflected back outward again.

    People like you love to argue about how us peons have no idea the complexity of the work they are doing is.

    Here is your idea:
    1) energy from sun penetrates to the Earth’s surface.
    2) Earth’s surface warms up.
    3) Atmosphere absorbs heat from the Earth’s surface while the Earth’s surface radiates energy outward.
    4) Some select parts of the radiation are absorbed by “green house gasses” and are re-emitted in a random direction, sometimes towards the Earths surface.
    5) Your idiocy on display, Radiation headed towards space is always 100% intercepted by “green house gasses” but energy heading back towards the Earth’s surface are always allowed to pass freely back to warm the Planet and cause the world to end.

    Here is mine:
    1) energy from sun penetrates to the Earth’s surface.
    2) Earth’s surface warms up.
    3) Atmosphere absorbs heat from the Earth’s surface while the Earth’s surface radiates energy outward.
    4) Some select parts of the radiation are absorbed by “green house gasses” and are re-emitted in a random direction, sometimes towards the Earths surface.
    5) The radiation heading back towards the Earth who were completely stopped and absorbed by the “green house gasses” in a specific distance, say z, is at about half the total amount as it took to penetrate to distance x with no energy left to warm anything beyond distance x. Therefore, even if half the energy amount of radiation penetrates the full distance x back towards Earth’s surface, the net amount of energy available at Earth’s surface is precisely the same as is available at distance x, 0.

    00

  • #

    The mylar film provides negligible insulation value for conduction, but it is quite good at reflecting the IR your body emits back at you.

    And is awesome at stopping nearly 100% of convection. Not enough variables controlled. Maybe a mylar woven mesh with plenty of ventilation to be similar to clothing convective abilities.

    00

  • #
    Kevin

    To Bobc, just once more for the h—l of it;

    “Was my critique a little too sharp for you?
    Yours is just dumb.
    I think I’ll have to re-reconsider my opinion of your knowledge.”

    Please, by all means go ahead and re-consider your opinion of my knowledge, I considered your knowledge from your first post about “demonstrates energy amplification” and my opinion could not go much lower.

    Enjoy your little unicorn ride there Bob.

    00

  • #
    Hans

    Jo, you stated “Note that the two well known versions of the Second Law both contain the phrase “whose sole result”, meaning that heat transfer can certainly move from a colder to a warmer body if there is some other compensating movement where more heat is transferred from a hotter body to a colder one.”

    I wonder, did you ever take notice of the Second Law before this post because your quick deduction seems just guessing and interpreting it the way that fits you the best.

    Neither Clausius nor Kelvin had heat compensation in mind at all, what in earth would be the use of this. These men are talking about work, that’s what steam engines were used for. Not for compensating heat.

    The Clausius statement like Kelvin’s refers to work, and forbids spontaneous heat flow from cold to hot. Work has to be performed to transfer energy from colder to hotter. Now you want to say that the Clausius statement means that in that case even more heat has to be moved to the cold? So refridgerators always get warmer then?

    The Kelvin statement refers to dissymmetry between work and heat, and work results in an amount of lower quality heat and this means lower temperature.
    Both describe the opposite of what you think it means, and that is that heat transfer has one direction and at the cost of something else (work or entropy). It flows from hot to cold and high to low quality.

    Definitions of heat:

    – The energy transferred from a high-temperature system to a lower-temperature system is called heat.
    – Any spontaneous flow of energy from one system to another caused by a difference in temperature between the systems is called heat.

    And look at the NASA figure you used. Do you see another arrow for the heat transfer in the reverse direction? Don’t you think that molecules from the cold reservoir will hit the left wall and transfer energy to the warm resevoir? Just like photons they don’t know which way to go, so by the same reasoning we should have ‘heat’ going from cold to hot. But have we ever heard of the fact that we can only use a net heat conduction in thermodynamics do we? (nor did we for radiation before GHG theorists invented net heat)

    No, there is just the gradient delta T that tells us the amount of heat being transferred and these two give us delta S and this S is what the Second Law is about.

    00

  • #
    Paul

    BobC: @ 340

    The experiment that I proposed has the effect of isolating all but the issue of interest, the effect of increasing the reflection of outgoing IR radiation back on the emitting object. Having the object enclosed in a sphere cooled to near absolute zero is to simulate the earth suspended in space at 3 degrees, and irradiated by the sun. Adding a reflective surface to the inside of the enclosing sphere is to test the theory that reflected long-wave radiation from the enclosed body back thereon will increase its temperature.

    I know that this experiment is not one that you or I can set up, but in terms of the expenditure so far on an untested and unfalsifiable theory and the reliance on that theory for the restructuring of the global economy, this seems to be a simple way for the theory to be tested, quantified and validated.

    I’ll make a prediction, though, that nobody will perform the experiment, least of all those who want us to believe that we are close to incinerating the world because of our use of cheap fossil fuels.

    As for your alternative experiment I fail to see how it addresses the issue of black-body warming from reflected radiation, all other things being kept unchanged. It is just not sufficiently isolated from other effects to be of any use in teasing out one effect from another, just as in the real world atmosphere.

    Paul

    00

  • #
    Reed Coray

    Construct a sphere of uniform material whose surface area is “A” and whose surface properties approximate an ideal blackbody–i.e., the surface absorbs all incident electromagnetic energy and radiates electromagnetic energy in accordance with Planck’s blackbody radiation law.

    In the exact center of the sphere, hollow out a small spherical region.

    Fill the hollowed out region with radioactive material whose radiation (electromagnetic and/or particulate) is entirely absorbed by the sphere and converted into thermal energy at an assumed rate of “W” Watts for all time. By symmetry arguments, all properties of the sphere are only a function of radial distance from the sphere center. As such, all properties of the sphere surface are the same everywhere on the sphere surface.

    Scenario 1

    Place the sphere with radioactive material in the vacuum of cold space–i.e., space at 0 Kelvin and isolate the sphere from all other matter/energy.

    Wait until the surface temperature of the sphere achieves “steady state” (i.e., quits changing).

    Question: What is the Scenario 1 surface temperature, T_isolation, of the sphere in “steady state”?

    Answer: When the surface temperature ceases changing with time, the difference between the rate thermal energy enters the sphere and the rate thermal energy leaves the sphere must be zero. For a blackbody object in a large enclosed cavity whose walls are maintained at a uniform temperature less than the temperature of the object, the rate of object cooling (i.e., the net rate of object thermal energy loss — “http://www.goiit.com/posts/show/813430/heat-transfer-stefans-law-of-radiation-804248.htm”) is equal to “sigma” times “the object surface area” times “the difference between (i) the fourth power of the “object surface temperature” in Kelvins and (ii) the fourth power of the “wall surface temperature” in Kelvins. In the case of our sphere, the rate energy enters the sphere is “W”. Since in the case of an isolated sphere, there are no “cavity walls” (or equivalently, the “cavity walls” are at zero Kelvin), the steady-state surface temperature, T_isolation, of the sphere in isolation is given by

    T_isolation = [W/(sigma*A)]^0.25

    Scenario 2

    Once the sphere in isolation has achieved steady state, surround the sphere with a spherical shell of blackbody material whose inner surface is everywhere maintained at “T_shell” Kelvins, where T_shell is less than T_isolation; and wait until the surface temperature of the sphere achieves steady state.

    Question: What is the Scenario 2 surface temperature, “T_surrounded”, of the sphere in steady state?

    Answer: For the sphere of temperature T_surrounded enclosed in a cavity whose walls are at a uniform temperature T_shell, the sphere rate of cooling (i.e., the net rate of sphere thermal energy loss) is equal to sigma times A times (T_surrounded^4 minus T_shell^4). The rate energy enters the sphere remains unchanged at W. In steady-state these two energy rates must be equal. Solving for T_surrounded gives

    T_surrounded = [W/(sigma*A) + T_shell^4}^0.25 = [T_isolation^4 + T_shell^4]^0.25.

    The only difference between the two scenarios is the addition of matter at a temperature less than the temperature of the sphere in isolation. Yet the steady-state sphere temperature in the presence of the added “matter” is higher than the steady-state sphere temperature in the absence of that “matter”.

    Anyone who argues that T_surrounded is equal to (or less than) T_isolation must either (a) show an error in the above logic, or (b) question the “Law of Cooling” as stated in the reference.

    If “warming” implies a temperature rise and a temperature rise implies “warming”, since the only change between scenario 1 and scenario 2 was the adding of material at a temperature less than the isolated sphere temperature, it’s hard to argue that the added “cooler” material didn’t “cause” a temperature rise in the “warmer” sphere. If the added material didn’t “cause the temperature rise”, what did? Note: At no time during the transition from scenario 1 steady state to scenario 2 steady state was the net energy flow from the colder “shell surface” to the warmer “sphere surface”. However, it is true that the sphere surface temperature increases.

    00

  • #
    RJ

    Hans at 344

    Just like photons they don’t know which way to go, so by the same reasoning we should have ‘heat’ going from cold to hot. But have we ever heard of the fact that we can only use a net heat conduction in thermodynamics do we? (nor did we for radiation before GHG theorists invented net heat)

    An excellent point. Is saying net energy transfer also incorrect or just net heat transfer.

    Although based on the viewpoint at post 114 (co2islife)

    Energy does not flow both ways. That being the case, energy cannot “flow uphill” and the second law of thermodynamics remains intact.

    Although isn’t this viewpoint based on a no photon world. If photons do exist I assume net energy flow would not be an incorrect expression.

    00

  • #
    Paul

    Reed Coray: @346

    Surrounding a body with solid material is different from surrounding it with a gas.

    The process of interest is not conduction, it is radiation and more particularly the effect of reflecting radiation from the body back to itself, or the supposed ‘back radiation’, on the temperature of the radiating surface.

    I am not much interested in thought experiments. Experience tells me that thought experiments can easily leave things out or otherwise conform to pre-conceived ideas, but the real world of physical experiments does not leave out or distort anything and is the only arbiter of reality.

    If anyone can think of how to set up an experiment to test this back-radiation theory more simply then I am interested, but not really in alternative experiments in other areas.

    Paul

    00

  • #
    Paul

    RJ:
    May 25th, 2011 at 5:59 pm

    Hans at 344

    Just like photons they don’t know which way to go, so by the same reasoning we should have ‘heat’ going from cold to hot. But have we ever heard of the fact that we can only use a net heat conduction in thermodynamics do we? (nor did we for radiation before GHG theorists invented net heat)

    An excellent point. Is saying net energy transfer also incorrect or just net heat transfer.

    Although based on the viewpoint at post 114 (co2islife)

    Energy does not flow both ways. That being the case, energy cannot “flow uphill” and the second law of thermodynamics remains intact.

    Although isn’t this viewpoint based on a no photon world. If photons do exist I assume net energy flow would not be an incorrect expression.

    I don’t think that the issue is whether or not photons exist. The idea of a photon is, as has been pointed out, an aid to thinking, but it is really only an electromagnetic field that propagates in quantised manner – you only get discrete, if small, additions. The discrete field of energy interacts with other fields according to its wave properties, as far as I understand it.

    I have argued extensively that the error being made by some is to talk about the behaviour of a single ‘photon’, as if it could exist independently of the electromagnetic field of which it is part, and then draw conclusions about the entire field based on that supposed behaviour of the single ‘photon’. It is simply inadmissible, from a mathematical perspective, to argue from the micro, quantised sphere of the atom to the macro, observable world of the surface of the earth. I don’t know how else to say it than I already have. A drop of water in a waterfall may splash upwards, against the force of gravity, but the macro flow of water will never be changed or reversed because of that micro event.

    That said, the thought that it is ‘net energy flow’ with some going one way and more going the other way, with a net flow resulting, that whole concept depends upon thinking in terms of photons that don’t ‘know’ what is going on. If, instead, you think in terms of an electromagnetic field of a measurable force, then the greater force overcomes the lessor force and there is a consequent flow of energy from the matter with the higher to that with lower energy levels. There is less opportunity for confusion when looked at this way, I think. Yes, it is still a ‘net’ flow, but there is no confusion about energy flowing from the cold to the hot body, it is always one way, as specified in the second law of thermodynamics.

    Paul

    00

  • #
    RJ

    paul @ 349

    Thanks. Very clear. (I think) I understand now although the term net energy flow maybe should not be used then as it can just confuses. Like saying a jersey or blanket warms the body rather than reduces the rate of cooling.

    00

  • #
    Paul

    RJ:
    May 25th, 2011 at 7:08 pm

    paul @ 349

    Thanks. Very clear. (I think) I understand now …

    I think that the whole strategy behind the Global Warming scare hinges on the ability to confuse almost the entire population!

    You will notice the heated discussion on even this thread about one distinct aspect of the matter with little ‘consensus’ even amongst ‘experts’ [myself not included in that category]. Imagine how little the man in the street really knows about ‘the settled science’ and how easily he can be persuaded that our profligate use of fossil fuels has damaged the atmosphere already for our grand children, great grand children and even more remote generations!

    I was reading one of the official documents on Global Warming from BOM last night [GreenhouseEffectandClimateChange.pdf] and marvel at the clever way they have ‘simplified’ the matter so as to ensure that the message is clear that ‘we done it’ and ‘no cost is too great for the good of the future survival of the species’ but without saying anything that can be tested by the scientific method. Propaganda at its best [or worst].

    By the way, have you notice how the emphasis has now moved, from ‘Global Warming’, to ‘Climate Change’, to ‘Climate Disruption’ and now to ‘Climate Heritage for our great grand children’. We had James Hansen in New Zealand recently and he was emphasising that it is not the current generation that will ‘bear the cost of’ Global Warming, so now they are using our future generations as hostages to force us into the direction that they wish us to take on the use of fossil fuels. They are getting more and more desperate, methinks. [Mind you, some of the more extreme among that crowd would like the world population to drop to less than one billion souls, so it is a bit hypocritical of them to use our grand children this way!]

    Paul

    00

  • #
    RJ

    Paul

    I was reading one of the official documents on Global Warming from BOM last night [GreenhouseEffectandClimateChange.pdf] and marvel at the clever way they have ‘simplified’ the matter so as to ensure that the message is clear that ‘we done it’ and ‘no cost is too great for the good of the future survival of the species’ but without saying anything that can be tested by the scientific method. Propaganda at its best [or worst].

    Its why I now mainly focus on the uselessness of the proposed remedies not the science. I have meet people who just do not want to know about the science but believe we must be harming the planet (the we are sinners reply although they do not directly say this).

    So my argument is the warming is exaggerated but even if it is not the carbon tax (or cap and trade) is useless because it will have no almost impact reducing GW even if the highest IPCC projection is correct.

    So its all now about money nothing more. Money for Govts. Money for traders. Money for land owners. Money for the EU and UN etc that we will all pay for.

    This often works better than a science based discussions.

    00

  • #
    Bryan

    Paul says

    ..”[Mind you, some of the more extreme among that crowd would like the world population to drop to less than one billion souls, so it is a bit hypocritical of them to use our grand children this way!]”

    This is the unspoken rational behind the demonisation of CO2.
    Its the people are rubbish syndrome.
    The “elite” only need enough workers to maintain their lifestyles and any excess just get in the way.

    00

  • #
    Alistair

    RJ: 350. Just don’t buy into the ‘back radiation’ as a heat source argument: it’s simply normal radiative interchange that doesn’t do any useful work. After all, you didn’t seriously think your pullover actually produced its own heat?

    [‘Climate science’ does, apparently choosing to accept Arthur Milne’s 1922 mistake as Holy Writ because it fitted the political mantra despite it being wrong. Add in Sagan and Chandrasekar’s later mistakes which predicted, wrongly, that imaginary cooling from polluted clouds hid the imaginary heating and you can truly understand that the name has been mis-spelt. It should be h o r s e s h i t.]

    00

  • #
    astonerii

    How do infared camera sensors work? They are able to see temperatures below their own temperature are they not? If this is the case, what is the mechanism that allows this? It is purely the magnetic feild of the radiation going through the sensor, or does the sensor absorb the radiation and get something from it?

    00

  • #
    Alistair

    Infrared sensors work by having an IR window and behind that a transducer sensitive to IR, e.g. a pyroelectric substance, or a thermopile.

    The key issue is that they are shielded from radiation coming from the opposite direction. If you want to know the net signal, turn the sensor around by 180° and subtract the second reading from the first.

    00

  • #
    Baa Humbug

    Paul:#349
    May 25th, 2011 at 6:41 pm

    I don’t think that the issue is whether or not photons exist. The idea of a photon is, as has been pointed out, an aid to thinking, but it is really only an electromagnetic field that propagates in quantised manner –

    Quite correct. However, as an aid to thinking, the “visualization” of photons speeding around in straight lines creates it’s own problems when trying to understand radiation.
    Rather like the kindergarden level graphics used by Al Gore in his movie (little black angry CO2 molecules shooting rays of heat back to earth) the use of photons in explaining radiation leads to misunderstanding of the process.

    When used by those who “should know”, it’s downright misleading and a tool to con layman by way of over simplifying.
    Backradiation does not and cannot warm the surface (and that includes slowing down cooling) except in very very isolated circumstances in a very very minor way to a point where it’s just not worth considering.
    Climate science would serve better if it spent it’s time and resources studying the properties of water and convection and conduction.

    00

  • #
    KR

    astonerii @ 355

    “How do infared camera sensors work?”

    There are a number of different designs – one interesting one is an array of micromachined bolometers. These have an absorbing element (such as silicon or vanadium oxide) attached to a heat sink. Photons absorbed by the element change the temperature and thus the electric resistance of the absorber, which gets measured to return an intensity at that pixel.

    Note: while bolometers can be cooled for more sensitivity, they absorb photons with thermal energies well below the temperature of the camera (they are often used uncooled), with each photon absorbed adding energy to the detector and raising it’s temperature.

    00

  • #
    Reed Coray

    Paul (#348)

    Surrounding a body with solid material is different from surrounding it with a gas.

    The process of interest is not conduction, it is radiation and more particularly the effect of reflecting radiation from the body back to itself, or the supposed ‘back radiation’, on the temperature of the radiating surface.

    I agree surrounding a body with solid material is different from surrounding it with a gas.

    You brought up the issue of conduction. In my thought experiment no conduction exists because the small sphere is placed in the vacuum of a cavity (interior of a spherical shell)–i.e., no part of the interior surface of the spherical shell touches any part of the small sphere; and with the exception of the small sphere, no matter exists in the interior of the shell. On the other hand, if the small sphere is surrounded by a gas, thermal conduction (and if a gravitational field exists, thermal convection) will exist between the small sphere and the gas. So the issue of conduction isn’t relative to my thought experiment, but is relevant to a solid material surrounded by a gas.

    Regarding a real-world experiment. Construct a cryostat cabable of providing liquid helium temperatures. Remove all gas from the interior of the cryostat using a vacuum pump. Cool the walls of the cryostat to liquid helium temperature. At this point the quality of the vacuum will make a sudden jump because any non-helium gas (and possibly non-hydorogen gas) in the interior to the cryostat will liquify or solidify onto the interior walls of the cryostate. I’ve seen this happen many times. Inside the cryostat place a small resistor and pass a current through the resistor. Measure the electrical resistance and the temperature of the resistor. Be careful because the electrical resistance of many materials is a function of temperature. If need be, adjust the voltage across the resistor to maintain a constant power dissapation, P (Voltage squared divided by resistance). Monitor the temperature of the resistor until it settles at an unchanging value. Record that temperature. Now raise the temperature of the cryostat walls to liquid nitrogen temperatures. If necessary, adjust the voltage across the resistor to establish the same power dissapation, P. Monitor the temperature of the resistor until it stops changing and record that temperature. I believe you will find the temperature of the resistor will be higher in the “liquid nitrogen cryostat” than in the “liquid helium cryostat.”

    00

  • #
    Reed Coray

    Paul, I forgot to mention that I am a confirmed AGW skeptic. I believe the CAGW premise is a bunch of bunk promoted by many people for various reasons, almost none of which have scientific validity. I’m also a skeptic of the GHG theory not because I don’t believe in “backradiation”, I do; but because the atmosphere and its thermal behavior are complex phenomena, for which radiation is not the whole story. For example, I believe an atmosphere devoid of all “greenhouse” gases will have an impact on the temperature of a solid object orbiting the sun. If true, a “greenhouse effect” will exist in the absence of all “greenhouse gases.” The closest I’ve ever encounted to the definition of the “greenhouse effect” is the statement: “The temperature of the Earth in the absence of an atmosphere will be different than the temperature of the earth in the presence of an atmosphere. That temperature difference is the greenhouse effect.” Using this definition, the “greenhouse effect” has nothing to do with “greenhouse gases,” and therefore why should I be concerned about “greenhouse gases?” Concerned about the atmosphere, maybe; but not specifically greenhouse gases. If someone wanted to make the definition of the “greenhouse effect” to address greenhouse gases, then we can go forward from there. Something like: “The greenhouse effect is the temperature difference between (a) the surface of an object with a non-greenhouse atmosphere, and (b) the surface of the object with the same atmosphere augmented with greenhouse gases.”

    I deplore the issue that a generic atmosphere may alter the temperature of the object’s surface, and for this reason we should be worried about greenhouse gases. It might be logical to conclude we should be worried about the atmosphere in general, but why pick on “greenhouse gases?”

    00

  • #
    astonerii

    KR @358

    That would almost be enough to show that low temp radiation can heat higher temp black bodies. I would nearly say, case closed.

    00

  • #
    BobC

    Paul: (@345)
    May 25th, 2011 at 4:26 pm
    BobC: @ 340
    As for your alternative experiment I fail to see how it addresses the issue of black-body warming from reflected radiation, all other things being kept unchanged. It is just not sufficiently isolated from other effects to be of any use in teasing out one effect from another, just as in the real world atmosphere.

    Well, 2 mil clear mylar does have an R-value of 1 (US units = 1/6 SI units). This is quite low, but not low enough to ignore. One way to improve the experiment would be to do it with both aluminumized and clear mylar sheeting. That way, the insulation factor due to thermal conduction would be the same and the only difference would have to be due to the reflectivity.

    I haven’t done this comparison, but I’ll look for some uncoated mylar sheet (I already have several coated sheets). I think I saw some for home-made storm windows in the local hardware store, but may have to wait for fall for them to stock it again.

    To eliminate the subjective aspect, you could use two boxes with mylar windows — one coated, one uncoated — and measure the temperature inside with a small light bulb as a source. Doing the experiment multiple times, while switching which box has the coated window would remove the effects of differences in box construction and materials.

    If both boxes are set on an insulated pad outside on a clear night, they should both experience a fairly effective blackbody (the sky). Not space, but good enough, especially if both boxes have the same exposure, as that would remove any heat absorbed from the sky as a cause of different temperatures between the boxes.

    My expectation is that you would see a significant temperature difference between the boxes — that it won’t be a subtle effect.

    Commercially, reflective mylar sheet is claimed to be quite effective at reducing heat loss from the home.

    On another note: It’s hard to believe that none of you have experienced the dramatic difference between sitting near a wall vs. sitting near a window when it is very cold outside (-40C). Maybe it just doesn’t get cold enough where you are.

    And despite people’s attempts to ridicule it (ignorantly, since they haven’t experienced it) there is also a dramatic difference in what the comfortable air temperature is between rooms with heated and unheated floors. Baseboard heaters won’t do this, as the heated surface must subtend a fairly large solid angle (4-5 steradians) w.r.t. your body to really show the effect. The heated floor does this with a much lower temperature than baseboard heaters. (Has to be much lower, to prevent overheating the occupants of the room.)

    There is the fact that both IR radiant heating and IR reflective insulation are techniques used in buildings for >50 years and are routinely calculated by HVAC engineers designing building heating systems. Additionally, there are large industries supplying components for these methods. Is all of this fantasy or a fraud? I would hesitate to write off standard industial engineering practices that have meet the test of time and economics because you don’t understand them.

    00

  • #
    Bryan

    KR

    If as usual the IR detector is held at ground level to detect the cold night air there will be more radiation leaving the detector than entering it.
    Would you agree?

    00

  • #
    KR

    Bryan @ 363

    If the detector is warmer than the night air (depending on the emissivity of the detector, of course), you are quite correct, more radiation will be leaving the detector than arriving.

    However, as an imaging system, the absorptive section of each bolometer is facing the aperture, the heat sink is not, and the difference in temperature between the absorber and heat sink is what is monitored. Each IR photon striking the absorber adds energy, raising it’s temperature, changing it’s resistance relative to the unheated portion of the pixel. The absorber can be read if you’re quick – the heat sink is intended to return the pixel to neutral (basically no temperature difference between absorber and heat sink) at a rate close to the frame time, so that the camera can detect motion.

    astonerii @ 361

    Another item in this category is the microwave oven. Microwaves are characteristic of thermal emission of objects around 2-5K, whereas even a deep frozen roast is at ~250K. Low temp radiation can certainly heat warmer bodies.

    00

  • #
    Bryan

    KR says

    ……”If the detector is warmer than the night air (depending on the emissivity of the detector, of course), you are quite correct, more radiation will be leaving the detector than arriving.”….

    I think its a fair assumption that the detector at say 10C will be warmer than the cold night sky <-20C

    KR says

    ……."However, as an imaging system, the absorptive section of each bolometer is facing the aperture, the heat sink is not, and the difference in temperature between the absorber and heat sink is what is monitored."…..

    How on Earth will a Detector at 10C contain a heat sink at less than -20C?
    Your knowledge of heat transfer needs serious revision.

    KR returns to microwaves ovens which he thinks are particulaly relevent to the atmosphere

    ……"Another item in this category is the microwave oven. Microwaves are characteristic of thermal emission of objects around 2-5K, whereas even a deep frozen roast is at ~250K. Low temp radiation can certainly heat warmer bodies."……..

    It has been pointed out to KR before that microwave ovens work because of a resonance/friction effect.
    If microwaveovens worked on a simple absorption effect then they would not require any particular frequency of microwave.
    Once the microwave frequency is changed significantly the microwave oven effect dissappears.

    The other objection to this example is that classical thermodynamics is concerned with equilibrium or near equilibrium situations which the oven is most certainly not.

    The other thing to notice is how little evedence an IPCC advocate has when a contrived interpretation of a machine is required to support an atmospheric theory.

    Have a read at a descrition of how a microwave oven realy works

    http://www.gallawa.com/microtech/howcook.html

    00

  • #
    KR

    Bryan @ 365

    Absorption is absorption, transferring energy into the absorbing object. That’s how microwaves heat: water molecules absorb the microwave energy, converting EM energy into electron level/rotational/vibrational energy, as do silicon/vanadium oxide bolometer materials with IR energy. There is simply no difference in how absorption works at different frequencies other than the energies transferred.

    An IR camera does not need a heat sink at -20C, one at 15C is just fine. That in fact is one of the advantages of the microbolometer cameras – they are capable of imaging IR at room temperature.

    The actual measurement is made between a bit of material at 15C and one that – due to the absorption of IR photons – is currently slightly warmer than 15C, and hence has a different resistance. The heat sink is there simply to remove that energy difference, resetting the pixel, for the next frame of the camera image.

    Image detection is done by detecting the energy of IR photons. If they were incapable of transferring energy to the detector, such detectors could not work – yet they do. This provides a simple Reductio ad absurdum proof that low energy photons do indeed add energy to warmer objects.

    Bryan, it’s clear to me from your comments that you do not understand electromagnetic absorption by materials – I suggest you read up a bit. Unless you demonstrate some actual knowledge in this field, I’m going to consider your posts rather meaningless.

    00

  • #
    KR

    Bryan @ 365

    A small excerpt from the reference you provided:

    “Microwaves penetrate and are absorbed by some substances, primarily food products. As the energy penetrates the food, its power is gradually absorbed, or lost, to each successive layer of molecules.” (emphasis added)

    Microwave energy is absorbed by the molecules of the absorbing object, transferring power (energy) to it. Isn’t that exactly what I said earlier?

    Low energy photons transfer energy to warmer objects. The 2nd law of thermodynamics is a statistical rule – given thermal energy transfer between two objects, the warmer object will transfer more energy than the cooler, and net energy flow will be from warmer to cooler object.

    A microwave oven electrically generates large numbers of low energy photons (not thermal emission, mind you) – and these low energy photons warm the food.

    00

  • #
    Bryan

    KR

    You obviously did not read the description below from the article

    ……”Now, consider that the actual frequency of the RF energy used in microwave ovens is 2450 million cycles per second! Moreover, consider that within the course of one of those cycles, the molecules would actually change their direction (polarity) twice – once for the positive half-cycle and once for the negative half-cycle. This red-hot rate of vibration causes tremendous friction within the food, and – just as rubbing your hands together makes them warm – this friction produces heat.”……..

    Friction plus the resonance effect causes the microwave cooker to function just as I said.
    Not a simple absorption effect as you seem to imply otherwise it would not have to select a very particular frequency.

    Do you understand what resonance is if not, I will give you some references to look up.

    00

  • #
    Bryan

    KR

    Now that we have pointlessly examined lasers and microwave overs perhaps we can return to the topic of this thread atmospheric thermodynamics and the second law

    So here is MY question again

    Do you think that 1000Joules of blackbody radiant energy centred around say 2um is thermodynamically equivalent to 1000Joules of blackbody radiant energy centred around say 20um

    a) In terms of the First Law of Thermodynamics
    b) In terms of the Second Law of Thermodynamics.

    The question I put can be answered within the frame of equilibrium thermodynamics which is an area microwave ovens most certainly cannot.

    My guess is however you will once again evade the question.

    00

  • #
    KR

    Bryan @368

    The “resonance” explanation for microwaves is incorrect, although sadly often (mis)used. See this article on how a microwave works.

    The molecular configuration of an absorber affects what wavelengths it interacts with – water is a good absorber (non-resonant) of 2.45 GHz microwaves, CO2 is a good absorber of 6 micron and 15 micron IR, etc. A dipole moment is very important in this absorption – providing what is essentially an antenna that matches the incoming EM wavelengths.

    Absorbed energy then raises electron levels, inducing vibrational and rotational motion (i.e., heat).

    More specific examples include fluorescent molecules (which I work with on a daily basis), where a very specific bandwidth of EM is absorbed, raising electron orbitals, partially dissipated as heat, and re-radiated as a band drop to a lower orbital of a particular energy difference (quanta). You have to carefully control energy levels to avoid overheating your samples.

    Again – it’s clear to me that you need to read up more on electromagnetic absorption by materials. Your statements on this subject have been, sorry to say, nonsense.

    00

  • #
    KR

    Bryan

    Joules are Joules -> 1000 = 1000.

    I will note that the wavelengths of thermal radiation are important, both from a concentration factor (10^4 more concentrated at 2um than 20um, a big influence on rate of energy transfer per area) and due to the absorptivity of whatever object they impact. But Joules are Joules; they need to be conserved (1st law), and net energy flow between pools of energy (objects) will be from high/warm to low/cold (2nd law).

    Now – kindly get to the “gotcha” you obviously have in mind.

    00

  • #
    Bryan

    KR says

    …..”Again – it’s clear to me that you need to read up more on electromagnetic absorption by materials. Your statements on this subject have been, sorry to say, nonsense.”

    Readers will note that he has danced around but not answered a very straightforward question which I will remind you is

    Do you think that 1000Joules of blackbody radiant energy centred around say 2um is thermodynamically equivalent to 1000Joules of blackbody radiant energy centred around say 20um

    a) In terms of the First Law of Thermodynamics
    b) In terms of the Second Law of Thermodynamics.

    The question I put can be answered within the frame of equilibrium thermodynamics which is an area microwave ovens most certainly cannot.

    My guess is however you will once again evade the question.

    00

  • #
    Bryan

    KR

    Its obvious that an IPCC proponent like yourself is obsessed with radiative transfer at the expense of the other methods of heat transfer.
    However you dont even get that right.

    Notice how KR describes the sensor on an IR camera.

    …..”The actual measurement is made between a” bit of material at 15C” and one that – due to the absorption of IR photons – is currently slightly warmer than 15C, and hence has a different resistance. The heat sink is there simply to remove that energy difference, resetting the pixel, for the next frame of the camera image.”…..

    Notice the breathtaking stupidity of his understanding

    The sensor is a “bit of material at 15C” it is detecting the cold sky at -10C.

    Now most educated people would say that the ” bit of material at 15C” would radiate more radiation than it receives from the -10C sky.
    That is it would get slightly cooler!!

    But no according to KR the sensor is now

    “is currently slightly warmer than 15C”

    Its almost unbelievable but KR once again proposes a violation of the second law!!!!!

    Then wallowing in his own ignorance KR ends the post by

    ……..”Bryan, it’s clear to me from your comments that you do not understand electromagnetic absorption by materials – I suggest you read up a bit. Unless you demonstrate some actual knowledge in this field, I’m going to consider your posts rather meaningless”….

    00

  • #
    BobC

    365Bryan: (@365)
    May 26th, 2011 at 5:02 am

    How on Earth will a Detector at 10C contain a heat sink at less than -20C?
    Your knowledge of heat transfer needs serious revision.

    Bryan, you’re just amazing.

    You don’t understand how something can be done, so instead of doing some research on it, you just pronounce that it’s impossible and furthermore, anyone who knows that it can be done is ignorant.

    Here’s a little video of a BMW’s thermal imaging camera (optional on M5 and 7-class) imaging 10-12 micron IR as it heads into Las Vegas at 80 mph. I’ve designed lenses (germanium & silicon) for these cameras. The detectors have microbolometers for pixels and can easily image things significantly colder than the camera. They work just like KR explained: The heat sink doesn’t need to be colder than what is being imaged (impossible to arrange on the front of a car in the desert, anyway), they just have to be at a fixed temperature.

    I’ve also worked on single-pixel sensors for the Air Force that were designed to “image” the air temperature outside of the slipstream of a super-sonic jet at very high altitude. (That’s the only way you can read the true air temperature (< -40C oftentimes) because the shock wave heats up all the air near the jet.)

    The fact that you can't understand how these devices work doesn't mean they don't exist. (Heard of Solipsism?)

    You not only need to upgrade your general knowledge, you need to learn how to do it.

    00

  • #
    Bryan

    BobC

    It appears that you and KR would rather talk about anything other than the atmosphere and the second law.

    Here is a more technical description of the operation of the microwave oven

    http://www.lsbu.ac.uk/water/microwave.html

    It is clearly not about simple absorption of microwaves.

    It adds nothing to our understanding of the atmosphere.

    Read post 373 above of how your radiation expert KR defies the second law
    You can buy into that rubbish if you like.

    00

  • #
    Bryan

    BobC

    It appears to me that your powers of comprehension are rather limited.

    You had to be told several times about how the lapse rate works before it “sunk in”.
    Perhaps even yet your not quite “with it”.

    Now pyrgeometers pyrometers IR thermometers and IR cameras of course they will give an indication of what they are pointing at.

    Read again any post I have made on the topic with a bit more care.

    These devices may have different sensors such as thermistors in a balanced wheatstone bridge, thermocouples, thermopiles or whatever.

    But if the object is at a higher temperature than the sensor the detection current or voltage will be reversed if the object was at a lower temperature than the sensor.

    00

  • #
    Paul

    Reading up on the Stefan-Boltzmann law I am interested to read the following : —

    Temperature of the Earth

    Similarly we can calculate the effective temperature of the Earth TE by equating the energy received from the Sun and the energy transmitted by the Earth, under the black-body approximation:

    \begin{align} T_E & = T_S \sqrt{r_S\over 2 a_0 } \\ & = 5780 \; {\rm K} \times \sqrt{696 \times 10^{6} \; {\rm m} \over 2 \times 149.598 \times 10^{9} \; {\rm m} } \\ & \approx 279 \; {\rm K} \end{align}

    where TS is the temperature of the Sun, rS the radius of the Sun, and a0 is the distance between the Earth and the Sun. Thus resulting in an effective temperature of 6°C on the surface of the Earth.

    The above derivation is a rough approximation only, as it ignores any temperature differences and changes on the planet and also the greenhouse effect. However, the result does not change if the Earth is a grey body rather than a black body because any change in both the absorption and the emission will cancel each-other out.

    The IPCC[3] and other sources[4] present an effective temperature of 255 K (−18 °C), which can be found by considering that the Earth’s albedo is 0.3, meaning that 30 % of the solar radiation that hits the planet gets scattered back into space without absorption. The energy from the Sun is thus multiplied by 0.7 to account for this reduction in absorption. Even though the absorption is reduced, perfect blackbody emission is assumed, leading to this lower equilibrium temperature.

    However, compared to the 30% reflection of the Sun’s energy, a much larger fraction of long-wave radiation from the surface of the earth is absorbed or reflected in the atmosphere instead of being radiated away, by greenhouse gases, namely water vapor, carbon dioxide and methane.[5][6] Since the emissivity (weighted more in the longer wavelengths where the Earth radiates) is reduced more than the absorptivity (weighted more in the shorter wavelengths of the Sun’s radiation), the equilibrium temperature is higher than the simple black-body calculation estimates. As a result, the Earth’s actual average surface temperature is about 288 K (14 °C), rather than 279 K.

    Did you get the classic ‘bait and switch’ deception technique that the IPCC are using here?

    So, going by main-stream science, not disputing the back-radiation, but merely applying the Stefan-Boltzmann law appropriately, gives a difference of 8 °C in the estimated average surface temperature of the earth, with and without the presence of Greenhouse gasses. The IPCC estimate of 33 °C is therefore four times greater than an the estimate should be according to the assumptions given.

    Absent the flaunted 33 °C, where now is the correct estimate of a small increase in atmospheric carbon dioxide even according to their assumptions?

    If the ‘Establishment’ cannot get even that most basic and simple calculation right, how much should we rely on their models?

    This is just one more piece of evidence that we should expect the increase in near-earth atmospheric temperatures to increase by a vanishingly small amount for a doubling of atmospheric carbon dioxide from any source. Never mind that even using all the available reserves of coal, oil and gas would not result in such doubling.

    Paul

    00

  • #
    L.J. Ryan

    Bob C. 327

    Indulge me here Bob, my confusion arises from your self contradictory “answers”.

    You said the REO etalon is 99.9% reflective. You said 1,000,000 W/m^2 are store within the cavities. I assumed the 0.1% not reflected was absorbed and therefore dissipated as heat. You said 0.1% was transmitted as the output…but then hedged your answer by saying If it’s not dissipated as heat, then it just passes through the etalon. (Duh!). I then asked if the tenth percent inefficiency was intentional. You never answered.

    1) So is the .001 of 1,000,000 W/m^2…1000 W/m^2 the output (transmitted) or is it loss (absorbed)?

    If the loss is not intentional and is dissipated as heat, a need to know variable is surface area. If area is unknown and loss is heat, then you can make no prediction of surface temperature. Since you can not provide the you assumption. You said, Well, no. I gave the intensity, which is power/area. Since heat dissipation also goes as the area, the area cancels out of any calculation about heating. Bob this last statement, is nonsensical.

    2)How long does take for the etalon to accumulate a 1,000,000 W/m^2?

    3)Can the output be closed as to store the 1,000,000 W/m^2 flux within the etalon?

    00

  • #
    L.J. Ryan

    Bob 327

    I asked:A floor radiating 240 W/m^2 with a well insulated, highly reflective ceiling should, according to GHG physics, begin radiating 390 W/m^2. Or maybe the ceiling need be matte black…emulating blackbody. Does 390 W/m^2 happen instantaneously or should I get one of those GHG physic jackets? If they still legal?

    You replied:

    Do I really need to tell you why this is stupid?

    I agree, my question is stupid, idiotic shows a complete lack understanding of radiant physics…and Bob C it is a scenario you conceived (240) to describe GHG physics. It is much like the endorsement(184) of KRs wrapped radiator test. For those following along, KR and Bob C believe a radiator (say cast iron-hydronic) filled with 65 C water, will warm itself do to own reflection…assuming it’s wrapped in a IR reflective insulation.

    So yes Bob C, please tell me why my bolded question/scenario is stupid.

    00

  • #
    astonerii

    Well, I learned alot on this thread.

    As for Jo Nova’s question on if greenhouse gases can warm the Earth radiation, the answer is yes, as respect to back radiation the answer is no. With the qualification that the energy input has to come from a place other than the Earth’s Surface. Once it starts the journey outward, the opportunity for the energy from the Earth to return is just not there. It leaves, and half always continues moving outward, as it tries to return back to Earth, only continually diminishing amounts remain, until extinction just above the surface of the Earth.

    00

  • #
    KR

    Bryan @ 372 – Re: 1000 Joules

    I did answer your question – see @ 371.

    Now kindly either get to the “gotcha” you’ve been apparently sitting on, waiting for me to answer, or stop wasting folks time with bad physics.

    You clearly do not understand photon/matter interactions, despite being pointed to some references on the question multiple times; you continue to espouse ‘physics’ that directly leads to blatant contradictions with facts, and insult people with more knowledge (such as BobC) when corrected. I would suggest using some of your time and learning something about the subjects you wish to discuss.

    00

  • #
    Bryan

    KR

    Readers will note that you have danced around, but not answered a very straightforward question which I will remind you is.

    Do you think that 1000Joules of blackbody radiant energy centred around say 2um is thermodynamically equivalent to 1000Joules of blackbody radiant energy centred around say 20um

    a) In terms of the First Law of Thermodynamics…….. Yes or No
    b) In terms of the Second Law of Thermodynamics…….. Yes or No

    See, I have made it even easier for you.

    Once you have answered the question add any further comment

    My guess is however you will once again evade the question.

    00

  • #
    Bryan

    KR

    Thank you very much for your answer on 366.
    Read carefully my reply on 373.

    Nullis in Verba doubted that any IPCC proponent would actually propose to defy the second law of thermodynamics.

    Your 366 analysis made exactly that claim.

    Its good to have you as an example of the dangers of sloppy interpretation of the radiative properties of CO2 and H2O.

    What would we do without you!!!!!!

    00

  • #
    KR

    Bryan @ 382

    I did answer your question, quite clearly, in @371. 1K Joules is 1K Joules. As energy, they follow the 1st and 2nd laws of thermodynamics, conservation and net energy flow – yes and yes.

    I also pointed out that thermal radiation centered at 2um and 20um will have some differences in terms of concentration (I try not to ignore factors of 10^4, I would consider that sloppy).

    So – energy is energy, although in terms of rates and concentrations blackbody thermal radiation at those two peaks differ.

    You’ve yet to make a point – I’m going to ignore any other postings on this particular side-track until and unless you do. I won’t be holding my breath…

    Bryan @ 383

    Your reply on @373 demonstrates to everyone reading that you do not understand the physics. Your “Just So Stories” attempting to explain the function of basic EM devices like microwaves and cameras are simply, and completely, wrong.

    End of discussion on that matter for me, Bryan. Not only are you wrong, you fail to recognize it, and refuse to learn. It’s not my job to pound reality into your head.

    00

  • #
    KR

    L.J. Ryan @ 379

    I will ask again – have you tried the heating pad experiment BobC recommended? It’s quite easy.

    * Turn on a heating pad, set it on a table with a fever thermometer under the edge, wait a while, read the thermometer.

    * Put a blanket over it (doesn’t have to be a thick one), wait a while until the blanket warms up.

    * Check the thermometer. It will be warmer.

    Constant energy comes in, the rate of energy loss/temperature declines, and the only thing that can change is the accumulation of energy in the heating pad until the rate of energy loss equals the rate of energy gain.

    For those following along, KR and Bob C believe a radiator (say cast iron-hydronic) filled with 65 C water, will warm itself do to own reflection…assuming it’s wrapped in a IR reflective insulation.

    And there’s the misstatement, L.J. – you’re describing a fixed system. I described one with constant energy coming in, not your strawman

    The heating pad experiment is a simple reality check, L.J.; ante up.

    00

  • #
    Alistair

    Bryan and KR [383, 373, 366]]

    This is how a bolometer works: http://www.evilprofessor.co.uk/bolometer-theory/

    The sensor loses heat by radiation until the exchange of radiative energy with the [generally] cooler sky equals the conductive heat transfer from the heat sink to the sensor. The resistance of the strip then gives its temperature and through calibration this is then converted to the temperature of the radiation source [the sky].

    To imagine that what happens is that ‘back radiation’ heats up the bolometer strip illustrates very well the fundamental lack of physics in some who espouse the failed CAGW hypothesis.

    00

  • #
    Bryan

    Alistair

    Thanks for the extensive note on the bolometer.
    My version is from a text book…. Thermal Physics by C J Adkins page 91.
    The semiconductor detector forming the fourth resistance in an in balance Wheatstone bridge.
    When the detectors resistance changes(down for higher object temperature, up for lower object temperature) a current is created which is scaled as desired.
    Obviously the current changes direction depending on the object temperature being above or below the detectors initial temperature.

    00

  • #
    Bryan

    My question to KR was;

    Do you think that 1000Joules of blackbody radiant energy centred around say 2um is thermodynamically equivalent to 1000Joules of blackbody radiant energy centred around say 20um

    a) In terms of the First Law of Thermodynamics…….. Yes or No
    b) In terms of the Second Law of Thermodynamics…….. Yes or No

    KR replies at last

    “I did answer your question, quite clearly, in @371. 1K Joules is 1K Joules. As energy, they follow the 1st and 2nd laws of thermodynamics, conservation and net energy flow – yes and yes.”

    Its just as I feared poor KR has no idea about the second law.

    The correct answer is Yes for the first law but NO for the second.

    The radiation from the 2um BBCentred 1000J is of a much higher QUALITY.
    That is in a given situation it will be capable of being turned into useful work with a much higher efficiency.

    KR did you ever study the Carnot cycle?
    If you work through the appropriate section in a physics textbook all will be clear.
    We are here to help you if you find it a bit of a struggle.

    00

  • #
    L.J. Ryan

    KR 385

    Assuming you can maintain constant current input and constant resistance, temperature will remain constant. Heating pads have “sloppy” control circuitry…low accuracy and wide hysteresis. That is, “HI” temperature will very from day to day.

    On the other hand KR, supplying a hydronic radiator with a constant water temperature, has less control feedback issues. So no KR, the rapped radiator, as you proposed (177), is not a fixed system and is not a strawman. What you GHG physics champions don’t like is reconciling a low beginning temperature with adding radiative quantities. IR within a freezer set to -18 C should quickly accumulate, boiling your ice cubes posthaste. So contrary to reality, a lower temperature freezer, according to GHG physic, will actually cook it’s contents where a freezer set to 0 C will merely warm it’s contents…see a lower set temp means more run time, more run time means more introduced IR. Dammed freezer industry.

    00

  • #
    Alistair

    Bryan: 387.

    You must realise that a Wheatstone bridge simply sets the current measurement side. the real calibration is the temperature difference between the heat sink [constant temperature presumably by a Peltier cooler] and the sensor through the known thermal conductance. It is that which is the calibrated part of the system.

    00

  • #
    Bryan

    Alistair

    I agree with your points but I was more interested in indicating a real physical difference in the sensor when a colder object or warmer object was being detected.

    00

  • #
    KR

    L.J. Ryan @ 389

    So – no, you’re not going to run the experiment. Sorry to hear that.

    Now, if your radiator has continuous flow of 65C water through it, in a cold room the surface of the radiator will be somewhat below 65C (cooled by the room). The radiator if insulated won’t get warmer than 65C – because if it did the water from the boiler would act as a refrigerant.

    Try the experiment with the heating pad. Or get one of those ‘tea warmer’ coils, and run it in air instead of water (I suggest outside on a sidewalk). See what happens when the rate of cooling changes.

    Your “Slayers” physics violates conservation of energy; these simple experiments will demonstrate what actually happens.

    Bryan @ 388

    You conveniently ignored my second paragraph:

    “I also pointed out that thermal radiation centered at 2um and 20um will have some differences in terms of concentration (I try not to ignore factors of 10^4, I would consider that sloppy). “

    So you’ve distorted what I’ve said. Fail.

    Alistair @ 390

    Actually, in most microbolometer cameras there is no active cooling, and the detector is around room temperature – the heat sink only provides conduction to the case, and air cooling from there. Absorption of photons on the detector changes the temperature of that leg of the bridge relative to the other (reference) side, and that difference is what is detected.

    Many of these cameras aren’t calibrated, incidentally, other than zeroing out any differences on a per-pixel basis between the detector and reference sides of the bridge.

    Over, and out – I have other things that require my attention. Enjoy, folks.

    00

  • #
    Bryan

    KR or “indiarubber” man now backtracks!

    Do you think that 1000Joules of blackbody radiant energy centred around say 2um is thermodynamically equivalent to 1000Joules of blackbody radiant energy centred around say 20um

    a) In terms of the First Law of Thermodynamics…….. Yes or No
    b) In terms of the Second Law of Thermodynamics…….. Yes or No

    KR replies at last

    “I did answer your question, quite clearly, in @371. 1K Joules is 1K Joules. As energy, they follow the 1st and 2nd laws of thermodynamics, conservation and net energy flow – yes and yes.”

    When he said “yes” to point (b) he didnt really mean “yes” or did he?

    Who knows?
    He can change his opinion every half hour.

    When KR cannot honestly stick to a point and debate it to a rational conclusion there is no hope that he will leave the darkness of self imposed ignorance.

    00

  • #
    L.J. Ryan

    KR 329

    Now, if your radiator has continuous flow of 65C water through it, in a cold room the surface of the radiator will be somewhat below 65C (cooled by the room). The radiator if insulated won’t get warmer than 65C – because if it did the water from the boiler would act as a refrigerant.

    I agree KR! Unfortunately, your newly found insight contradicts that other poster…ah whats his name…oh yeah, KR.

    See KR the water supplies the radiator (earths surface) with a constant power input. The IR emitted by the radiator to the room (atmosphere), can not be re-radiated or reflected (insulation) to increase temperature of radiator (earths surface).

    Lets make it more obvious KR. Add glycol to water allowing our radiator to function at a more obvious temperature. Given the following conditions lets re-examine our agreed upon scenario:

    radiator temp -18C (earths surface)
    room temp -273C (atmosphere prior to turning on radiator)

    The IR emitted by the radiator (earths surface) to the room (atmosphere), can not be re-radiated or reflected (insulation) to increase temperature of radiator (earths surface).

    Your welcome KR.

    00

  • #
    IAmDigitap

    KR’s sure he’s going to calculate the vanishing of entropy, because he owns a calculator. Well, I W.O.R.K. in the ATMOSPHERIC ELECTROMAGNETIC TRANSMISSION, CAPTURE, & ANALYSIS business, and

    IF THERE was any G.H.G. EFFECT, then WHEN THAT HEAT: HIT THOSE GAS ATOMS: T.H.E.R.E. W.O.U.L.D. HAVE TO BE MORE A.N.G.U.L.A.R. M.O.M.E.N.T.U.M. called M. O. T. I. O. N.

    and there’s N.O.T.

    So: THANK YOU, all those of you who DO NOT, HAVE NOT EVER, AND NEVER WILL, PARTICIPATE in the DISTRIBUTION, CAPTURE & ANALYSIS of ELECTROMAGNETIC RADIATION through the ATMOSPHERE

    for trying to tell us HOW WE CAN SPEAK of W.H.A.T. I.T. IS W.E.

    not YOU

    do for a LIVING…

    but WE’LL STICK with the LAWS that HELPED US CREATE this ELECTRONIC SPACE AGE we BUILT,
    U.S.I.N.G. those D.E.F.I.N.I.T.I.O.N.S.
    and it might not seem like we KNOW WHAT WE ARE DOING

    but THERE’S a REASON YOU CAN’T FIND A.N.Y. RECORD ANYWHERE of ANY ELECTRONIC ENGINEERS, INSTRUMENT ENGINEERS, or A.N.Y. KIND of COMMUNICATIONS FIELDS, ENDORSING this V.O.O.D.O.O.

    IF THAT HEAT IS THERE
    as there MUST BE since CO2 is RISING

    THERE MUST BE MORE MOTION in THAT GAS

    and OPTICAL & INFRA-RED TELESCOPY would have been S.C.R.E.A.M.I.N.G.
    about it
    FOR

    Y.E.A.R.S.

    They’re N.O.T.

    Because there I.S. no such THING as A.N.Y: watch my words:

    A.N.Y.

    “G.H.G. EFFECT”

    of A.N.Y. kind WHATEVER.

    If there WAS, we in the ATMOSPHERIC RADIATION BUSINESS would have POINTED it OUT to you in the INSTRUMENTS we BUILD to MEASURE A.L.L. THINGS, more than less.

    I’m not ASKING, I’m TELLING: the DEFINITION of HEAT on GAS is MOTION

    and SINCE the OPTICAL TELESCOPY and INFRA RED TELESCOPY FIELDS have NOT REPORTED RISING MOTION

    having that motion MAGNIFIED HUNDREDS and THOUSANDS of TIMES

    then IT’S not THERE.

    And the reason we knew it didn’t exist when HANSEN LIED to CONGRESS, is that WE KNEW ALREADY HE WAS LYING ABOUT THE SENSITIVITY of INSTRUMENTATION AT THE TIME:

    and as I said: IF THERE WAS ANYTHING THERE,

    it COULD NOT HIDE:

    HEAT
    on GAS
    is
    MOTION.

    NO INCREASING MOTION hence DISTORTION in OPTICAL TELESCOPY
    then
    NO
    HEAT.

    That means NO
    EFFECT

    because FOR SURE, the FALSELY NAMED G.H. Gases H.A.V.E. BEEN G.R.O.W.I.N.G.

    bu

    00

  • #
    Mark

    IAmDigitap:

    For heaven’s sake, go and buy a new keyboard.

    00

  • #
    Mark D.

    Aww c’mon Mark just read it in your best robot voice…..

    00

  • #
    Steve

    If anyone can think of an experiment wherein an equal mass of CO2 replaces N2 in an IR spectrometer, but the replacement doesn’t show an increased absorption of IR radiation, let me know.

    If anyone can think of an experiment wherein an increase in absorbed IR radiation per unit time doesn’t result in an increase in the temperature of the gas, let me know.

    If anyone can think of an experiment wherein a 15 degree surface beneath a 5 C atmosphere (at equilibrium) doesn’t increase in temperature if that atmosphere’s temperature rises to 7 C, let me know.

    The sun radiates energy on any point on the earth for approximately half of the day. The atmosphere radiates energy on any point on the earth for 24 hours of the day. The surface of the earth must either radiate or reflect the sum total of radiation received in that 24 hour period, otherwise it will rise in temperature forever. Therefore, one would expect that, eventually, the total radiation/reflection of the surface within a 24 hour period would exceed the solar radiation directly upon the surface within that 24 hour period. At equilibrium, surface radiation emitted = (absorbed solar radiation) + (absorbed atmospheric radiation).

    There is one way to increase the atmospheric temp without increasing the surface temp. This involves switching solar radiation that was previously heating the surface directly (visible light) to heating the atmosphere instead. Dark aerosols, such as soot, do the trick. Total solar radiation incident upon the earth system remains the same, but the absorption occurs higher up. The atmosphere must still emit just as much as it absorbed, but now only half of that blocked solar energy, in the form of IR radiation, is directed towards the surface (atmosphere radiates up and down). Net effect on surface = cooler.

    00

  • #
    BobC

    IAmDigitap fails the Turing Test.

    00

  • #
    Paul

    Steve:
    May 28th, 2011 at 4:52 am
    If anyone can think of an experiment wherein an equal mass of CO2 replaces N2 in an IR spectrometer, but the replacement doesn’t show an increased absorption of IR radiation, let me know.
    If anyone can think of an experiment wherein an increase in absorbed IR radiation per unit time doesn’t result in an increase in the temperature of the gas, let me know.
    If anyone can think of an experiment wherein a 15 degree surface beneath a 5 C atmosphere (at equilibrium) doesn’t increase in temperature if that atmosphere’s temperature rises to 7 C, let me know.

    And if anyone can demonstrate that the earth-atmosphere system can be modeled by such laboratory experiments, let me know. This is at the crux of the controversy – extrapolation from small laboratory experiments, where conditions are closely confined so as to isolate the one effect under observation, to the atmosphere where conditions are free to change in an almost infinite number of ways – and then saying “the science is settled”.

    The sun radiates energy on any point on the earth for approximately half of the day. The atmosphere radiates energy on any point on the earth for 24 hours of the day. The surface of the earth must either radiate or reflect the sum total of radiation received in that 24 hour period, otherwise it will rise in temperature forever. Therefore, one would expect that, eventually, the total radiation/reflection of the surface within a 24 hour period would exceed the solar radiation directly upon the surface within that 24 hour period. At equilibrium, surface radiation emitted = (absorbed solar radiation) + (absorbed atmospheric radiation).

    I think that you meant to say “the total radiation/reflection of the surface within a 24 hour period would tend towards equality with the solar radiation” since it is clear that what you have said is an impossibility.

    However I would still take askance at your suggested formula which takes account of only two out of many mechanisms whereby heat is transferred from the surface at any specific locality.

    Instead of your “At equilibrium, surface radiation emitted = (absorbed solar radiation) + (absorbed atmospheric radiation).” I would propose,

    At equilibrium, surface radiation emitted plus surface heat transferred by (conduction + convection + advection + latent heat of vaporisation) = (absorbed solar radiation) + (absorbed atmospheric radiation).

    It is clear that your formula relies on all those other means of heat transfer remaining constant, which seems to me to be violated by daily experience. On a daily basis one may easily observe the sequence of events as incoming radiation increases. At sunrise the amount of incoming radiation increases dramatically but surface temperatures respond slowly, especially if the surface is wet. Then, as the surface slowly responds by increasing in temperature convection currents begin and the air-temperature near the surface remains almost constant despite the massive increase in incoming radiation. Eventually, some time in the early afternoon, the air temperature will reach a maximum but convectional air currents will continue until some time in the early evening.

    All the above will be highly dependent on the associated cloud cover. As moist warm air rises it will cool due to the transfer of some of the heat energy to potential energy and the water vapour will condense, releasing further heat energy into the drier air, causing it to further ascend and at the same time forming clouds which now reflect a substantial amount of incoming radiation back into space, with a very substantial cooling effect at the surface.

    To ignore these easily observable phenomenon and attribute to a laboratory experiment the power to explain the climate on earth seems to me fanciful thinking at the best. When it is proclaimed by so-called scientific authorities I hardly know what to call it.

    There is one way to increase the atmospheric temp without increasing the surface temp. This involves switching solar radiation that was previously heating the surface directly (visible light) to heating the atmosphere instead. Dark aerosols, such as soot, do the trick. Total solar radiation incident upon the earth system remains the same, but the absorption occurs higher up. The atmosphere must still emit just as much as it absorbed, but now only half of that blocked solar energy, in the form of IR radiation, is directed towards the surface (atmosphere radiates up and down). Net effect on surface = cooler.

    You are jumping the gun a bit by proposing a solution to your non-problem before even getting close to proving that it exists outside of your speculative thought processes.

    Paul

    00

  • #
    RJ

    Steve @398

    If anyone can think of an experiment wherein an equal mass of CO2 replaces N2 in an IR spectrometer, but the replacement doesn’t show an increased absorption of IR radiation, let me know.

    But so what.

    If anyone can think of an experiment wherein an increase in absorbed IR radiation per unit time doesn’t result in an increase in the temperature of the gas, let me know.

    Have you read the comments above?

    If anyone can think of an experiment wherein a 15 degree surface beneath a 5 C atmosphere (at equilibrium) doesn’t increase in temperature if that atmosphere’s temperature rises to 7 C, let me know.

    Are you saying a cooler atmosphere will further heat a warmer surface. That a warmer body can in fact heat a warmer one?

    At equilibrium, surface radiation emitted = (absorbed solar radiation) + (absorbed atmospheric radiation).

    Back radiation must be excluded from absorbed atmospheric radiation. So really surface radiation emitted = absorbed solar radiation.

    If 100 balls are in a box. 20 are thrown in the air and 10 return. It does not increase the number of balls to 110.

    00

  • #
    Steve

    Paul @ 400: “I think that you meant to say “the total radiation/reflection of the surface within a 24 hour period would tend towards equality with the solar radiation” since it is clear that what you have said is an impossibility.”

    Nope, I meant it exactly as stated. The atmosphere is emitting light (IR wavelengths) down onto the earth’s surface, 24 hours of the day, even at night when there is no solar radiation. The energy of the light that is absorbed must eventually be emitted. That is why the Trenberth energy budget diagram shows 396 W/m^2 of surface radiation, despite showing only 184 W/m^2 of solar radiation incident upon the same surface.

    “This is at the crux of the controversy – extrapolation from small laboratory experiments, where conditions are closely confined so as to isolate the one effect under observation, to the atmosphere where conditions are free to change in an almost infinite number of ways – and then saying “the science is settled”.”

    So you don’t approve of the standard scientific method? Can you name even one of these “almost infinite number of ways” that the basic physics of the atmospheric gases as measured in a lab will change in the “wild” of the atmosphere?

    The crux of the controversy is the degree of warming to be expected for a given increase in CO2, and what harm a particular degree of warming could cause to the human race, not whether or not a greenhouse effect exists at all.

    “It is clear that your formula relies on all those other means of heat transfer remaining constant, which seems to me to be violated by daily experience.”

    To prove that the greenhouse effect exists? Then no. Yes, instead of rising in temperature and emitting radiation, that part of the surface covered in water could have an increased incidence of evaporation instead. Note that this would still be a reaction to increased atmospheric radiation due to the greenhouse effect. Let’s say the clouds do block enough solar radiation to balance the increase in atmospheric temperatures due to CO2. Then it rains and the clouds go away. But the CO2 is still there, so the atmosphere heats up again and the increased evaporation cycle begins again. The hydrological cycle is exactly that – a cycle. It cannot permanently cancel an enhancement of atmospheric heating via more CO2.

    RJ @ 401 “If 100 balls are in a box. 20 are thrown in the air and 10 return. It does not increase the number of balls to 110.”

    That is not an example of a system with a constant energy source. If there is a perpetual rain of balls down onto the box, while also bouncing out of the box, and the equilibrium state snapshot shows “contains 100 balls”, then putting any barrier that allows the same rain down while bouncing a few more balls back will raise the number of balls in the box.

    “Are you saying a cooler atmosphere will further heat a warmer surface. That a warmer [sic] body can in fact heat a warmer one?”

    A warm body next to a cooler body will be the body to experience the cooling. The rate of that cooling depends on the temperature difference. So a planetary surface beneath a 7 degree atmosphere will not cool as much throughout the night as a planetary surface beneath a 5 degree atmosphere. Then morning comes, and with the same solar energy input in, to a surface that hasn’t cooled as much, means higher temperatures the next the day.

    00

  • #
    L.J. Ryan

    Steve 402

    A warm body next to a cooler body will be the body to experience the cooling. The rate of that cooling depends on the temperature difference.

    Yes slows the rate of COOLING. The pertinent question: what is the temperature conferred via solar radiation? The average solar flux 240 W/m^2…the average temperature 255K.

    So a planetary surface beneath a 7 degree atmosphere will not cool as much throughout the night as a planetary surface beneath a 5 degree atmosphere. Then morning comes, and with the same solar energy input in, to a surface that hasn’t cooled as much, means higher temperatures the next the day.

    Cool a 255K surface as slowly as you like, that surface temperature will not warm 33 degrees.

    00

  • #
    RJ

    Steve at 402

    RJ @ 401 “If 100 balls are in a box. 20 are thrown in the air and 10 return. It does not increase the number of balls to 110.”

    That is not an example of a system with a constant energy source. If there is a perpetual rain of balls down onto the box, while also bouncing out of the box, and the equilibrium state snapshot shows “contains 100 balls”, then putting any barrier that allows the same rain down while bouncing a few more balls back will raise the number of balls in the box.

    So the energy comes from the sun (and only the sun) not the atmosphere. If energy leaves the earth and the same energy returns. It will not and can not increase the total energy. The 100 balls example is valid.

    00

  • #
    RJ

    A warm body next to a cooler body will be the body to experience the cooling. The rate of that cooling depends on the temperature difference. So a planetary surface beneath a 7 degree atmosphere will not cool as much throughout the night as a planetary surface beneath a 5 degree atmosphere. Then morning comes, and with the same solar energy input in, to a surface that hasn’t cooled as much, means higher temperatures the next the day.

    Only a higher average temperature

    If the highest surface temperature from the suns energy is 7 degrees. Without GHGs assume it drops to 2 degrees. With GHG to say 4 degrees. The average temperature will be higher as the starting temperature is higher. But if the suns energy is the same the next day the highest temperature will not increase above 7 degrees

    00

  • #
    RJ

    That is why the Trenberth energy budget diagram shows 396 W/m^2 of surface radiation, despite showing only 184 W/m^2 of solar radiation incident upon the same surface.

    This diagram is fiction not science. Extra radiation can not be magically created by CO2 and water vapour in this way.

    00

  • #

    .039% of the atmosphere. Heated up with radiation, adds what percentage of heat to the system?

    While I am starting to get to the point where I think that back radiation can slow the cooling of the planet surface, I am still stuck on the idea that the radiation is limited, it moves at the speed of light, and only a small portion of it, close to 0% can ever be returned to the Earth surface through back radiation, because only 50% gets re-radiated back towards the Earth’s surface, and will have an extinction rate significantly lower than the original radiation headed outward. Because the extinction rate decreases in distance precisely the same as the CO2 increases, only the same finite amount of radiation can be returned to the Earth’s surface, and that finite amount is close to 0. So, 8% of the radiant energy of the Earth is available to CO2 and much of that radiation can also be absorbed by H2O, it ends up being significantly less than 8% of next to 0 added w/m2.

    I just do not see how adding more CO2 increases back radiation. Adding more energy to the Earth’s surface would do this by increasing actual radiation outwards, and a corresponding increase in back radiation. It might warm the atmosphere up based on it’s mass the amount of radiation it can hold for longer than infinitesimally short period of time. The rate at which the radiation is absorbed along with the rate at which 1/2 the radiation can be reabsorbed. That is the key here. Unless half the radiation has larger than half the distance before extinction, then it is impossible to increase the back radiation by adding more CO2, and if it is less than half the distance, then it is actually going to reduce the back radiation to 0.

    So, can anyone with testable arguments state what the equation/ratio is for the absorption rate of CO2 friendly radiation is dependent on the variable of starting energy? That should answer much of this.

    00

  • #

    That is why the Trenberth energy budget diagram shows 396 W/m^2 of surface radiation, despite showing only 184 W/m^2 of solar radiation incident upon the same surface.

    I wonder if this is why they cannot account for the missing energy, because it never officially existed?

    00

  • #
    Paul

    I’ve been out cleaning up the driveway using a hose attachement that acts as a water-blaster by highly concentrating the flow of water. As I worked my mind was mulling over the workings of the atmosphere and I had a couple of interesting lines of thought.

    1) The more concentrated the energy, the more useful work it can accomplish.

    There is nothing new in that, but relating that to the incoming radiation from the sun and the outgoing radiation from the earth’s surface, it is clear that the incoming radiation is more energetic and concentrated and the outgoing radiation is less energetic and more diffused. When someone wants to use the incoming radiation, even to heat water, it is normal to concentrate that energy further using some form of reflector.

    The radiation from the atmosphere and clouds is even less concentrated. It would be very surprising if the diffused, scattered radiation from the atmosphere has even a small fraction of the ability to heat the surface possessed by the direct rays of the sun. Taking into consideration that only a small portion of the earth’s surface-heat is removed by radiation, about 21% of ‘average’ incoming radiation, that would put it at about 10% of the sun’s intensity. So one might conclude that the amount by which ‘back-radiation’ is able to slow outgoing long-wave radiation will have only a very minor effect on surface temperatures.

    File:57911main Earth Energy Budget.jpg Note, by the way, the total absence of ‘back-radiation’ in this depiction of the earth’s energy budget!

    2) The other line of thought, even more significant I think, is that we have been led up the garden path with the analogy of the atmosphere being a blanket that slows the cooling of the earth and keeps it at a higher temperature than it would be without the atmosphere. If we stick with that analogy, then I have to say that it is a very wet blanket and we know just how much warming we would expect from a wet blanket!

    The reality, though, is that the surface, the oceans and the atmosphere work together acting as a giant ‘fly-wheel’ moderating the surface temperatures and keeping them from being much higher by day and much cooler by night.

    The atmosphere, relative to the size of the world, is a very thin ‘skin’ which, intuitively, we would not expect to have great thermal insulating properties. View this image taken from a orbiting satellite, showing the earth’s curvature in relation to the thickness of the atmosphere.
    File:Sunset from the ISS.JPG

    Imagine, if you will, that the earth rotated at the same speed as the moon with one rotation every month, just how high the temperature might rise on the side facing the sun, at high noon, and how cold it might become on the side away from the sun’s warming rays before the dawn? I don’t think that it is any too extreme to suggest that the dark side would be plunged into snow and ice while the light side would be scorched by the blazing sun. With most of the atmospheric water being precipitated there would be no clouds to modulate the night-time temperatures and it would take a considerable time for the day-time warmth to melt the ice and snow. The 24-hour rotation of the earth cannot be ignored in calculating the surface temperatures.

    Of course, we can get some idea about that from studying the moon. What is its maximum, minimum and transitional temperatures where night is turning to day?

    Having thus, in principle, shown that the earth/ocean/atmosphere is always acting to moderate swings in near-earth atmospheric temperatures, keeping day temperatures from soaring and slowing the cooling during the night, where now is the calculation to show that Tyndall gases are the cause of the 33 degree difference between a blackbody temperature and what we actually have?

    Paul

    00

  • #
    Paul

    Steve:
    May 30th, 2011 at 4:19 am
    Paul @ 400: “I think that you meant to say “the total radiation/reflection of the surface within a 24 hour period would tend towards equality with the solar radiation” since it is clear that what you have said is an impossibility.”
    Nope, I meant it exactly as stated. The atmosphere is emitting light (IR wavelengths) down onto the earth’s surface, 24 hours of the day, even at night when there is no solar radiation. The energy of the light that is absorbed must eventually be emitted. That is why the Trenberth energy budget diagram shows 396 W/m^2 of surface radiation, despite showing only 184 W/m^2 of solar radiation incident upon the same surface.

    So 21% of energy received at the earth’s surface is being emitted as long-wave radiation, with 7% by means of sensible heat flux and 23% by means of latent heat flux, according to the energy budget at NASA or File:57911main Earth Energy Budget.jpg. So how is that going to produce more watts of energy from the atmosphere and clouds than from the direct radiation of the sun?

    “This is at the crux of the controversy – extrapolation from small laboratory experiments, where conditions are closely confined so as to isolate the one effect under observation, to the atmosphere where conditions are free to change in an almost infinite number of ways – and then saying “the science is settled”.”
    So you don’t approve of the standard scientific method? Can you name even one of these “almost infinite number of ways” that the basic physics of the atmospheric gases as measured in a lab will change in the “wild” of the atmosphere?

    I don’t think that you thought about what I said much. I do not deny the results of lab experiments or suggest that, when conditions are the same in the atmosphere, that there will be a similar result. What I was alluding to is the obvious fact that the stringent lab conditions will never apply in the real atmosphere. Hence it is not legitimate to apply the results of lab experiments directly to the atmosphere where they have no direct relevance.

    As a case in point, when measuring the absorption of long-wave radiation in the the lab, care must be taken to prevent any energy loss from conduction and convection, yet in the atmosphere we have not only conduction and convection but also energy loss by advection and latent heat of vaporisation. It is illegitimate to assign a value arrived at in the laboratory to still, dry, contained air to the moist, rising, moving currents of air in the atmosphere.

    The crux of the controversy is the degree of warming to be expected for a given increase in CO2, and what harm a particular degree of warming could cause to the human race, not whether or not a greenhouse effect exists at all.

    Pardon me, but I think that you are assuming what you need to prove, or begging the question. When there are a large number of independent variables that describe the processes by which the earth’s surface is warmed by the sun and cooled by the earth-surface/oceans/atmosphere, to take just one of those variables and assign to it a controlling influence on all the others, and then make the surface temperature a function of that one variable, is a statistical and scientific nonsense.

    “It is clear that your formula relies on all those other means of heat transfer remaining constant, which seems to me to be violated by daily experience.”
    To prove that the greenhouse effect exists? Then no. Yes, instead of rising in temperature and emitting radiation, that part of the surface covered in water could have an increased incidence of evaporation instead. Note that this would still be a reaction to increased atmospheric radiation due to the greenhouse effect. Let’s say the clouds do block enough solar radiation to balance the increase in atmospheric temperatures due to CO2. Then it rains and the clouds go away. But the CO2 is still there, so the atmosphere heats up again and the increased evaporation cycle begins again. The hydrological cycle is exactly that – a cycle. It cannot permanently cancel an enhancement of atmospheric heating via more CO2.

    Can you really think that back-radiation from the atmosphere and clouds is increasing vaporisation at the surface? Once again you are begging the question and making atmospheric CO2 the one independent variable, setting the others to constant values and then estimating the supposed effect of increasing that rare atmospheric gas based on those assumptions. Well I have news for you, your assumptions are not valid. Get real and look at what happens in the real atmopshere.

    Paul

    00

  • #
    IAmDigitap

    What we have here, is people who thought they CALCULATED that the FIFTEEN-BELOW ZERO TURBULENT BATH, brought the rock to a certain temperature,

    and they then think they’ve given some indication that higher concentrations of certain gases have changed the level of the temperature of the rock, because they CLAIM FALSELY TO HAVE CHANGED the TEMPERATURE of the BATH:

    but the AVERAGE TEMPERATURE of the BATH has N.O.T. C.H.A.N.G.E.D. as EVINCED by the fact that FROM the BEGINNING, the TEMPERATURES ATTRIBUTED to the OVERALL ATMOSPHERE have not VARIED ONE WHIT.

    Then the can’t-count crew: the same people who think Mannian Statistics is real math,
    and Jones Briffa Extensions to it formed a WHOLE NEW BRANCH of MATHEMATICS:

    the same people who thought Trenberth’s ludicrously childish ‘global radiation budget’ could have POSSIBLY been real,

    Then these people want to tell you that the reason the telescopy, infra-red telescopy, radar, weather radar, and electromagnetic radiation communications fields show: with intruments DESIGNED to DETECT CHANGES in the atmosphere’s CONDUCTANCE and HOLDING of

    electromagnetic radiation

    is because we’re all stupid and they haz calculashuns.

    But THEY FORGET the LAST WORD in ALL SCIENCE: CAN we FIND an INSTRUMENT which WOULD DETECT MORE INFRA-RED IN THE ATMOSPHERE?

    That answer, is Y.E.S.

    We have ENORMOUS banks of thermometers, the readings of which stopped climbing in the mid nineties, and have remained stable.

    We have ENORMOUS banks of OPTICAL TELESCOPIC OBSERVATIONS and PHOTOGRAPHS which effectively HOLD the ATMOSPHERE UP to the LENS of a MICROSCOPE: MAGNIFYING the

    read it: M.O.T.I.O.N. hence H.E.A.T. in the ATMOSPHERE

    HUNDREDS;
    THOUSANDS
    of times.

    And in ALLLLLLLLLLL these years, not ONE student: each semester, looking for that ‘easy A’

    And in ALLLLLLLLLLL these years, not ONE professor, looking for that

    BLOCKBUSTER PAPER –

    And in ALLLLLLL these years, not ONE person looking for that NEXT NOBEL PRIZE:

    went outside, looked up at the stars and said, “hmmm if there’s more heat up there, there has to be more MOTION, and that MOTION can’t be HID by the SCEPTICS: I’LL PROVE the MOTION’S THERE because WE HAVE YEARS and YEARS
    of OPTICAL
    and INFRA-RED TELESCOPY RECORDS

    and we’ll measure the E.V.E.R. R.I.S.I.N.G. MOTION due to H.E.A.T. and PROVE there is a

    GREENHOUSE
    GAS
    EFFECT.

    WHY HASN’T T.H.A.T. HAPPENED?

    Why NOT?

    BECAUSE THEY HAVE ALL THOUGHT OF IT, TOO, AND THE RESOUNDING SILENCE FROM THE VERY GRANT-PAID FIELDS WHICH HAVE THE INSTRUMENTATION to DETECT electromagnetic energy RISE

    shows you that people a WHOLE LOT SMARTER than YOUR ENTROPY-REVERSING patootie

    find S.Q.U.A.T.

    So when you FIND the ARGUMENT that MAKES HEAT ON GAS NOT EQUAL MOTION in the atmosphere

    you come around and SHOW THE INSTRUMENTS PICKING UP the MAGICAL EFFECT you HAVE JUST HAD PROVEN TO YOU DOESN’T EXIST by the fact YOUR FRIENDS WOULD HAVE PROVEN IT DID,

    JUST LIKE I SAID THEY COULD

    and THEN, with your FALSE ARGUMENT, YOUR REAL INSTRUMENT READINGS SHOWING IT THERE, YOU WILL BE IN.

    YOU WILL NO LONGER BE FRINGE SCIENCE.

    Because INSTRUMENT trumps “I DUN FIGUR’d wid muh CALCULATER that ENTROPY DROPPED DEAD”

    EVERY time.

    Now WHEN YOU EXPLAIN WHY NO LIGHT/HEAT/I.R. LIGHT-HEAT MOTION is DETECTABLE by ANY FIELD that WATCHES for these changes MAGNIFIED THOUSANDS of TIMES

    You’ll have an argument.

    WHEN YOU SHOW THOSE INTSTRUMENTS REPRODUCING YOUR PREDICTIONS,

    you’ll have a GREENHOUSE GAS EFFECT.

    Till then you have NOTHING but the desire to prove the laws of physics don’t work, when you’re talking.

    SHOW ME THOSE TELESCOPY RECORDS from the EARLY 1900s TO NOW and SHOW ME the EVER RISING ATMOSPHERIC AGITATION from the

    rising greenhouse gases (a mythical term that describes NO THING)

    creating EVER RISING ATMOSPHERIC AGITATION from

    HEAT
    on
    GAS
    equaling

    MOTION.

    Pfft.

    I can’t believe an entire globe full of people are too dense to look up at the STARS twinkling over their heads, and thinking,

    “Oh yea, if there’s more HEAT there’s more MOTION hence more OPTICAL TELESCOPY DISTORTION.”

    “Oh yea, if there’s more INFRA-RED (heat) there’s more INFRA-RED POLLUTION and the INFRA-RED TELESCOPES of the world WOULD BE SHOWING IT.”

    Nah, it never occurred to you.
    Mmmm Hmmm.

    And IT NEVER OCCURRED to a WHOLE GLOBE FULL of PEOPLE, that

    “Oh yea, the EVER RISING INFRA-RED POLLUTION would mean the FAMOUS dependency of the ELECTROMAGNETIC COMMUNICATIONS field(s) – the various fields who bathe the atmosphere’s curving geometry with radiation, sometimes depending on the specific heat held in the atmosphere to use ‘skip’ transmission capability by bouncing signal off an electromagnetically charged layer of the atmosphere-

    THESE GUYS have more instruments than G O D and in ALL THESE DECADES, we have N.E.V.E.R.

    NOT ONCE

    heard a SINGLE WORD about how “radio amateurs/professionals who use the electromagnetic energy-charge of the atmosphere to propagate certain signals, are
    NOW HAVING to SLIGHTLY SHIFT FREQUENCIES because EVER RISING ATMOSPHERIC HEAT/INFRA-RED has CREATED A SLIGHT DIFFERENCE in TIMING/FREQUENCY of conditions needed for CERTAIN ATMOSPHERIC-HEAT-SENSITIVE CLASSES of TRANSMISSIONS.

    Nah, NOT ONE WORD

    but we’re supposed to sit around here and have people

    who have to have BUTTONS and SWITCHES put on the THINGS WE BUILD so THEY CAN ‘MEASURE’ the UNIVERSE

    tell us that we just don’t really understand, because they learned to flip entropy in mathematical voodoo that would be I.N.S.T.A.N.T. FLUNK in ANY ATMOSPHERIC ELECTROMAGNETIC RADIATION UNIVERSITY TEST ON EARTH.

    Hundreds of additional watts from nothing.

    ABSOLUTELY Z.E.R.O. instrumental capability of even SENSING : all this hoodoo – EVEN WHEN THE INSTRUMENT is effectively a LIGHT MICROSOPE POINTED through the VERY ATMOSPHERE these WANNABES claim is going crazy with the magical effect from the 1800s.

    It’s HOODOO or YOU’LL PRODUCE AN INSTRUMENT that SHOWS it is there.

    And since I already pointed out there are THREE different fields INTIMATELY associated with DETECTING MINUTE CHANGES in ATMOSPHERIC HEAT HOLDING capability:
    and
    TWO of those fields nothing more or less than DESIGNED to CHECK PRECISELY your FALSE CLAIM

    That means all you’ve got is barnyard talk about how “I coulda’ been a atmospheric radiatiashun guy cuz I haz calculator.”

    Explain how these telescopic instruments don’t show your heat or

    SHOW HOW THE LAWS of PHYSICS have CHANGED for YOU so that THE

    H.E.A.T.

    IS
    NOT

    MOTION.

    You’re so easy to trap it’s like putting peanut butter on a dowel over a barrel of oil.

    Every rat who’ comes along thinking there’s a FREE PHYSICS LUNCH winds up still and quiet, floating in that barrel because it DIDN’T RECOGNIZE the IMPOSSIBILITY of DENYING the LAWS of PHYSICS.

    00

  • #
    Mark D.

    Digitap, I think I understand what you are saying (in between the periods). What of the idea that there should be optical distortion if the warmists are correct? Where would I look for more information?

    00

  • #
    Myrrh

    Of course it breaks the 2nd Law – it’s ridiculous. Heat is energy on the move from a hotter to a colder, the colder is taking, the hotter is losing. The only net, the real net here, is between the taking and losing in the flow from hotter to colder.

    When the colder is heated by the hotter it then passes that on to another even colder. It doesn’t send some of it back to heat the hotter from which it got hotter, unless that has now itself become colder.

    This is simple logic.

    What is nonsense here is that “net” has been usurped to say something different.

    The claim is that it is ‘energy’ being exchanged. Energy is not heat. Not all energy creates heat.

    There is a basic error in the AGWScience Energy Budget, which says that ‘Solar’, short wave UV, Visible Light and Near Infrared, convert to heat the surface of the Earth, land and oceans. This is really key to the problem. They exclude Thermal Infrared from the downwelling radiation from the Sun to the Earth.

    So, not only is it ridiculous that AGWScience excludes Thermal Infrared, which is the HEAT we feel from the Sun, travelling from the hotter to the colder, but it claims that Visible Light, creates heat, converts to heat, actually heats the land and seas. How?

    Because they confuse the different properties of Radiation and think every exchange produces heat, but that’s not true.

    Some energy, for example in Visible Light in photosynthesis,is being used to produce a chemical change, converting CO2 and water to sugars. Some energy is gives off Light, the scattering of Visible Light by the molecules of Nitrogen and Oxygen in our atmosphereis not creating heat, but Light, so we have a blue sky.

    Try heating your bath of cold water with Blue Visible Light.

    If there was such a thing as ‘back radiation’ from CO2, we would have had it incorporated into our heating systems by now!

    AGWScience has corrupted all the basics, it is through the looking glass impossible in our physical world. Come back to the Real World.

    00

  • #
  • #

    @ #23

    A better formulation using that “joke” description:
    0th law: You have to play the game.
    1st law: You can’t win.
    2nd law: You can’t break even, except on a very cold day.
    3rd law: There are no days that cold.

    00

  • #

    #413

    Hard to imagine that many fundamental misconceptions in a single post. The interaction between electromagnetic radiation (infrared, visible, ultraviolet, radio, X-ray, gamma, etc.) is complex and is related to the nature of the matter and the energy (i.e., frequency) of the electromagnetic radiation (e=hf). EMR doesn’t “convert” to heat. It may do a variety of things: it may excite thermal motion (“heat”), it may loosen electrons (photovoltaics), it may cause cellular damage (uv sunburn), it may pass through unimpeded (be transmitted), it may be reflected, etc.

    In your second paragraph you say “When the colder is heated by the hotter it then passes that on to another even colder. It doesn’t send some of it back to heat the hotter from which it got hotter, unless that has now itself become colder.” So, how does the colder object know whether or not the hot object has become colder in ordert to decide whether to “send some heat back?” Maybe it’s a thermos (“it keeps the hot stuff hot, it keeps the cold stuff cold, how do it know?”).

    “Some energy is gives off light” doesn’t even make sense. Light (visible), like all EMR, is energy. So if you meant “some energy is light” that’s obvious but meaningless. If you meant “some energy gives off light,” there’s no meaning to be discerned from such a statement.

    The “heating your bath” remark is completely irrelevant.

    00

  • #

    #411

    Quite possibly a world record for nonsense. I hope someone with more time on his or her hands will bother to debunk it because all those capital letters, clever misspellings to indicate the stupidity of those idiot scientists, and all of those periods intimidate me.

    But I will say that the downward longwave radiation can be and has been measured.

    00

  • #
    Paul

    Rob Ryan:
    May 31st, 2011 at 2:37 pm

    But I will say that the downward longwave radiation can be and has been measured.

    And how do those measurements compare with measurements of direct radiation from the sun at ground-level?

    How would they compare in their ability to heat the surface?

    Paul

    00

  • #
    Myrrh

    Re 417

    The AGWScience claim, KT97, is that Solar energies, UV&Visible&NearIR, convert to heat, which heats the Earth, land and oceans, which then radiates out Thermal IR.

    Downwelling energy from the Sun can be scattered, which is not the creation of heat but the re-distribution of light or deflection of light into another path; it can be used in a chemical process which is not the creation of heat but a chemical change such as in photosynthesis; it can absorbed raising the temperature, creating heat, by exciting the molecules to thermal motion.

    We see plants as green because they do not absorb green visible light, they reflect it, no heat is created, it’s scattered. The molecules of Oxygen and Nitrogen in our atmosphere scatter Visible Light, clouds and dust scatter, etc., scattering takes several forms, reflection, refraction, transmission, etc., none of these excite to thermal motion therefore no heat is created. The sky is blue because blue visible light is scattered by the molecules of the gas Air which is our atmosphere, no heat is being created.

    Blue Visible Light and the bath is absolutely pertinent, because the claim from AGWScience is that this wavelength converts to heat the land and oceans. The example given is that it penetrates deeper into the ocean and so creates heat deeper than the other wavelengths, but, it is merely being transmitted, scattered, through the water molecules, not absorbed and exciting thermal motion, it is Light reflecting off the water molecules.

    Light energies may well be ‘highly energetic’, but that does not mean they are somehow ‘powerful’ enough to create heat, to raise the temperature of matter. All highly energetic means is that they move more quickly in the same distance than longer waves and what matter does with them, the water and land of the Earth, depends on the energies and the matter. The Heat energies of Thermal IR are larger, they are more powerful in size than the smaller microscopic Light energies. All these things have to be taken into consideration, the properties of these energies do different things depending on the matter and life encountered.

    Until AGWScience can prove that Visible Light converts to heat the land and oceans of Earth, AS THEY CLAIM, then these have to be taken out of their downwelling Energy Budget. Try heating a cup of water with Blue Visible Light. Don’t expect it to heat the water for your coffee. If you cannot get it to heat the water for your coffee you cannot claim that it heats the oceans to raise the temperature of Earth which then radiates out this heat as Thermal IR.

    AGWScience does not include Thermal IR in its downwelling Energy Budget, that itself shows AGWScience is nonsensical.

    AGWScience is not Real physical world science, it is imaginary, science fiction.

    Arguing with it is like arguing with any science fiction worlds created in storytelling, it makes up its own descriptions of properties and reactions and processes and so it should not be taken seriously by anyone, let alone any ‘skeptics’.

    00

  • #
    Myrrh

    Jo Nova says: “The 2nd Law of Thermodynamics applies to net flows of heat</i., not to each individual photon, and it does not prevent some heat flowing from a cooler body to a warm one."

    So, if there are larger amounts of heat flowing from a cooler body to a warm one than there are amounts of heat flowing from a hotter to a cooler, the net amount will be heat flowing from the cooler to the hotter, so the cooler will heat the warm. The net flow then is from the cooler to the warmer.

    00

  • #
    Myrrh

    sorry, put full stop instead of closing bracket for the italic.

    00

  • #
    Paul

    For another perspective on the effect of doubling the atmospheric carbon dioxide, take a look at this paper by Dr. Heinz Hug and hosted by John Daly : —

    The Climate Catastrophe – A Spectroscopic Artifact?

    One comment that ensued after the publication of this paper is very relevant to this present discussion : —

    Subject: Hug report, continuing comment
    Date: Fri, 18 Sep 1998 13:35:14 +0300 (EET DST)
    From: Jarl Ahlbeck AT
    To: [email protected] (John Daly)

    This is to continue the comment I sent earlier today: P.S. A description of the etablished way of calculating the radiative mechanisms of the atmosphere and the temperature profile can be found in the book by

    Goody and Young “Atmospheric Radiation, Theoretical Basis” 2. ed., Oxford Univ. Press, 1989. Calculating the profile acc. to these equations gives a global surface temperature of 350 K (77 deg. C).

    I remember that Lindzen has mentioned that the modellers simply have put in the correct temperature profile (15 deg C at the surface) and claimed that they have performed a “convective adaption”, and then went on by using the same radiative equations to calculate radiative forcing……

    So much for that science.

    regards. Jalle

    So you see, it is the latent heat and convection that are cooling the surface, by about 60 deg. C, and the sensitivity of the climate to increased atmospheric carbon dioxide has been grossly exaggerated by the IPCC.

    Paul

    00

  • #
    Myrrh

    Jo Nova – the AGWScience is a distinct package in its own right. It messes with basic physics understanding of properties and processes by various tweaking, giving the property of one to another which does not have it, applying out of context laws to process and so on. The differences is worth a study, a systematic analysis of what and how these changes have been made from traditional science, and the consequent effect, which is the creation of an impossible physical world, at least, not of this world, but one Alice might have discovered through the looking glass, where believing impossible things encouraged.

    Heat travelling from colder to the hotter is physically impossible in our real world without work being done, it doesn’t happen spontaneously. Any more than spontaneously water sometimes travels uphill without some extraneous force acting on it.

    Nitrogen, Oxygen and Carbon Dioxide do not have the qualities of an ideal gas, i.e. none, in our real world as AGWScience claims. Hence from AGWScience they argue that these molecules travel at great speeds in our atmosphere bouncing off each other and so ‘thoroughly mixed’. What that is actually saying is that our atmosphere is empty space with molecules which have no volume, no weight and not subject to gravity. An impossible world for the real gas molecules but not of the imagination. But the arguments always stop there, if asked how then does sound travel, there is no answer from AGWScience because they’d have to admit that molecules have volume, don’t speed through empty space, but are squashed together (in the fluid volume of gas which is Air). This science fiction is limited to sound bites of misinformation, misdirection,it’s only from constant repetition that people think it is real science. And then they try to justify it because it is such a prevalent meme.

    A story I’ve told elsewhere. One scientist brought up with the meme that gases in our atmosphere are well mixed because they behave like the imaginary ideal gas, therefore would spontaneously move at great speed bouncing off each other, and hearing the argument from real science that real gases have volume and weight relative to each other in our real atmosphere of the fluid gas Air, would not stay mixed, but would separate out into layers as they do in mines, for example.

    Methane being lighter than the molecules of Oxygen and Nitrogen is often to be found in a layer at the ceiling of mines. Not very long ago it was standard practice for miners to cover themselves with wet towels and taking a lit candle on a long stick enter the mine to rid it of the danger of methane.

    He decided to test this out for himself. He and his team found a mine, introduced a load of methane and finding that it did indeed gather in a layer at the ceiling waited for it to spontaneously thoroughly mix as per the AGWScience meme. It didn’t, it stayed pooled in a layer at the ceiling, which is had done spontaneously. His team then searched every inch of the mine because they were convinced that there must be another source of methane which was entering and so pooling quicker than the pooled could diffuse into the rest of the Air. Though they couldn’t find any such source, they still believed that this is what was happening and they had missed it. They couldn’t bring themselves to working out the logic of what they had witnessed. That the methane spontaneously pooled at the ceiling because it was lighter than the air in the rest of the mine, and that it was not going to then spontaneously thoroughly mix up again..

    What heavier than Air Carbon Dioxide does in mass amounts, such as pooling on floors of breweries and travelling downhill from volcanic vents etc., it also does at individual molecule level. It’s not because they somehow magically stick together when they are found in large amounts that gases heavier than air sink and gases lighter than air rise, but because each individual molecule of these gases will sink if heavier and rise if lighter than the gases in the volume around them.

    Re the difference between Light and Heat energies, http://wattsupwiththat.com/2011/05/25/snowball-earth-ended-by-methane-now-an-impossible-theory/#comment-672215 for a look at AGWScience’s claim that Solar, UV and Visible and Nr IR, heat our land and oceans.

    I hope that someone with the resources to do this will pull all this kind of information together, a real comparison between the tweaked memes of properties and processes as promoted by AGWScience fiction and the real science facts of properties and processes as still taught by some and still used in various applications because known and understood.

    This misdirection is very clever because it takes in so many disparate disciplines, it is like the many headed Hydra, difficult to destroy because someone from one discipline can find something amiss in his own field but takes on trust AGWScience’s misinformation about something else as real fact if from another discipline, or simply, because ‘that is what was taught in school’…

    00

  • #
    Myrrh

    Jo Nova – I’ve linked to my post here from a discussion on WUWT re the point I’m trying to make that there are basic false science memes from AGWScience promotion which are so prevalent that they are believed to be real world science even by ‘skeptics’, who in turn argue against real science by trying to make these false memes make sense – please see the following and other posts I’ve mentioned the link: http://wattsupwiththat.com/2011/05/30/skeptic-strategy-for-talking-about-global-warming/#comment-673358

    00

  • #
    Myrrh

    Wayne says:
    June 4, 2011 at 12:50 pm

    See, Myrrh defines the bands differently but some of what he is saying is correct.

    I haven’t gone into the detail much here because the complications in it would become a distraction to the proof I’m looking for from IRA et al re these short energies converting to heat the Earth, because it so complicated and my explanation and some thoughts about the divisions would be rather long.., so I chose the Visible range to concentrate on that aspect. Consequently, I haven’t explored it in the depth that I keep wanting to get distracted into exploring it, but enough to choose instead to make the distinction by example as on the NASA page, that NrIR is not felt as heat, and another link I’ve now lost, which described IR in three sections, near, mid and far, and said that near is like visible light, (reflective), and mid and far are thermal, because of the different way they act on meeting matter, (for example a NrIR camera is taking a picture of this wavelength reflecting off the subject, just as a Visible light camera does, not to be confused with thermal imaging infrared cameras which measure the thermal IR given off.)

    In other words I’ve stuck with the “NrIR”/”short wave infrared” of the KT97 not being thermal as a ‘working model’, while wondering if their actual numbers includes the thermal of ‘my working model’, i.e., that they are over-estimating Visible, as you’ve been exploring.

    [Just to remind and for MattG]. The AGW energy budget claims these, the “Solar”, are all thermal, as IRA has been arguing here; that they convert to heat land and oceans.] From the blurb they say that the “Solar” which is heating the Earth is the Visible plus the two short waves immediately adjacent, stated as being UV and Near Infrared, and according to the wiki page on the Electromagnetic Spectrum in the section on Visible it says these are also called light when it doesn’t matter to human sight, “Other wavelengths, especially near infrared (longer than 760 nm) and ultraviolet (shorter than 380 nm) are also sometimes referred to as light, especially when the visibility to humans is not relevant.” So technically I suppose, the explanation for these together being commonly called “Solar”, as in the AGW energy budget.

    Different areas of interest define where NrIR stops and the next begins with a certain arbitrariness it seemed to me, but in more technical descriptions I’ve most often seen actual NrIR stop at 1.4 um.

    There is a difference in the way these act on meeting matter, molecules and such, and NrIR, from what I’ve read so far on it, is like Visible, reflective, in that it excites atoms to a higher energy state and as these relax back they give off light (not heat, and how cameras and oximeters work and how Blue light is scattered on meeting molecules of oxygen and nitrogen, and how we see things, etc.).

    This it appears to me is why the classic desciption of the difference between Light and Heat energies in trad science, the near is reflective and not thermal, the thermal working on a different principle on meeting matter, rotational changes and such, but I haven’t explored this in depth, except that many references in more technical say water having affinity for absorbing thermal because of it (as it is absorbed by our bodies as heat, converts to heat, because of our own high water content.)

    The problems of what divisions of IR are in wavelength and name varies a lot, several here on wiki/Infrared, one example:

    Under the heading:

    CIE division scheme

    The International Commission on Illumination (CIE) recommended the difisionof infrared radiation into the following three bands.

    IR-A: 700 nm-1400 nm (0.7 um-1.4 um).
    IR-B: 1400 nm-3000 nm (1.4 um-3 um)
    IR-C: 3000 nm-1 mm(3 um-1000 um)

    A commonly used sub-division scheme is:

    . Near-infrared (NIR, IR-A DIN 0.75-1.4 um in wavelength, defined by the water absorption, and commonly used in fiber optic telecommunication because of low attenuation losses in the Si)2 glass (silica) medium. Image intensifiers are sensitive to theis area of the spectrum. Examples include night vision devices such as night vision goggles.

    . Short-wavelength infrared (SWIR, IR-B DIN)1.4-3 um, water absorption increases significantly at 1,450 nm. The 1,530 to 1,560 nm range is the dominant spectral region for long-distance telecommunications.

    . Mid-wavelength infrared (MWIR, IR-C DIN) also called intermediate infrared (IIR): 3-8 um. In guided missile technology the 3-5 um portion of this band is the atmospheric window in which the homing heads of passive IR ‘heat seeking’ missiles are designed to work, homing on to the infrared signature of the target aircraft, typically the jet engine exhaust plume.

    . Long-wavelength infrared (LWIR, IR-C DIN):8-15 um. This is the ‘thermal imaging region, in which sensors can obtain a completely passive picture of the outside world based on thermal emissions only and requiring no external light or thermal source such as the sun, moon or infrared illuminator. Forward-looking infrared (FLIR) systems use this area of the spectrum. Sometimes also called the “far-infrared”. [we and the earth typically radiate out at around 10-12]

    . Far infrared (FIR): 15- 1,000 um (see also far-infrared laser).

    NIR and SWIR is sometimes called “reflected infrared” while MWIR and LWIR is sometimes referred to as “thermal infrared”. Due to the nature of the blackbody radiation curves, typical ‘hot’ objects, such as exhaust pipes, often appear brighter in the MW compared to the same object in the LW.

    So, this fits in with AGW’s use of NrIR, that it includes near and shortwave, but not mid onwards. (Like the ‘near,mid,far of my ‘working model’) and are therefore reflective and not thermal, however, I have also been using the NASA page description of Nr as not thermal, but, if you look at the diagram of it, it calls Shortwave IR, Mid, if shortwave is according to above division, and that adds confusion here, because reading it it can appear to be saying, if one has the information that mid is thermal but not the numbers, that AGW 3 and under includes thermal…

    Not much more. Because this is so complicated re divisions I’ve stuck with two things here, that the AGW NrIR in their Solar includes the Short wave which are reflective and not thermal, that they are deliberately not including actual thermal infrared in this description, but instead claiming that these reflective are thermal and act as real thermal does to convert to heat, and, that the heat we feel from the Sun is the real thermal infrared, exactly as tradition science teaches, and it is this which heats the land and oceans, and us.

    So my insistence that they produce the actual proof that these shortwave energies are capable of converting to heat for the Earth to radiate out that amount of real thermal infrared, and why I’ve stuck with Visible..

    That UV doesn’t convert to heat is a fact in real science, that it barely penetrates the skin, no further than the epidermis etc., can be stopped by putting on a shirt, and descriptions of the way energies can be utilised beside creating heat, and what it means to be ‘reflective’ as in blue light tranmitting through air and water is not creating heat, that near infrared is not hot, and so on, are really only an aid for me to show that these energies do other things; I think I have the right, since this is impacting my life to its degradation, to insist they prove these energies are actually heating the land and oceans to produce the said amount of thermal infrared.

    I know this energy budget comes from their science fiction producing department, that these energies, even with the overlap confusion of ‘near ir’, cannot heat the land and oceans to the extent they claim. They must either put up or shut up.

    This means those who keep promoting it ad nauseum as does Ira here.

    Those who don’t understand the difference between light and heat energies might even learn something if they concentrate on looking for proof of what these energies are capable or not of doing, those others who promote this knowing full well that what they are saying is nonsense and so party to the fraud perpetrated against us by this AGWScience will no doubt continue to weasel their way out of having to show proof, we can recognise them..

    Now I don’t know what to do, having realised that I’ve posted this not just in the wrong discussion, but on a different web-site, JoNova’s!

    JoNova, my computer is reduced to basics and I have no easy way of transferring this, I can’t cut and paste, so as I’ve linked these two pages anyway and I’m a bit concerned about losing this post, I’ll submit here and hope my abject apologies accepted. I really do feel it very cheeky and have been sitting here worrying about doing it, I wish I hadn’t noticed until after I’d posted. If it’s not deleted I’ll get my answer, as of course I will if I find it deleted.. Many thanks if I find it still here.

    00

  • #
    Ted O'Brien

    May 20th, 2011 at 5:10 pmbananabender:

    “The temperature of the Earth is entirely due to EMR output of the Sun”.

    As an occasional viewer and not a scientist, I challenge this as surely wrong.

    The “temperature” that all the fuss is about is atmospheric temperature.

    For the atmosphere there is surely also a lot of heat which is rising through the surface from Earth’s hot innards.

    So much so that, for example, this could be and probably is in some areas causing Greenland’s ice to melt from the bottom, as the thickness of the ice insulates that zone from surface temperatures.

    00

  • #
    Bobc

    Kevin (@343):
    May 25th, 2011 at 11:43 am

    To Bobc, just once more for the h—l of it;

    Please, by all means go ahead and re-consider your opinion of my knowledge, I considered your knowledge from your first post about “demonstrates energy amplification” and my opinion could not go much lower.

    Enjoy your little unicorn ride there Bob.

    All you’re doing Kevin, is demonstrating that you haven’t been able to understand my post at #268 beyond the first sentence — perhaps you could object to something substantial, like what I was talking about, instead of your objection to the way I choose to label it?

    Nah, would require reading more than the 1st sentence.

    Your completely incoherent (and wrong) description of the fields inside an etalon (@276), and the fact that you (apparently) missed my reference to how this was impossible as shown by the Fourier decomposition of the supposed field (@304) seems to indicate that wherever you got your “graduate degrees in optics and electrical engineering” neglected to cover the Fourier Transform.

    (I can only guess what your “mastery” of Maxwell’s equations consists of — perhaps you can copy them in a nice hand?)

    You should get your money back, Kevin (if your “university” is still in business), because you still don’t know jack about optics.

    Cheers,
    BobC

    00

  • #
    BobC

    Bryan (@376):
    May 26th, 2011 at 9:38 am

    BobC

    It appears to me that your powers of comprehension are rather limited.

    Perhaps you’re right — I’m having a deal of trouble understanding the nonsense you are spewing.

    Maybe you could help by adding a few standard references. (Not “Slayer”)

    00

  • #
    BobC

    Bryan:
    May 26th, 2011 at 9:13 am

    BobC

    It appears that you and KR would rather talk about anything other than the atmosphere and the second law.

    Here is a more technical description of the operation of the microwave oven

    http://www.lsbu.ac.uk/water/microwave.html

    Whew! Hold on a moment while I recover from momentary dizziness due to sudden cognitive dissonance.

    It is clearly not about simple absorption of microwaves.

    It is, in fact, a detailed description from the wave point of view (the only POV that has a real theory behind it — Maxwell’s Equations) of the interaction of microwaves and water and the transfer of energy from the waves to the water.

    According to Planck’s Quantum Assumption (and 100+ years of experiment), this interaction is also quantized. However, because the microwave quanta are so small (~1/20,000 of light quanta), the analysis is not changed by ignoring them.

    If, however, you want to describe the operation of an MRI medical imaging machine (using 20 meter radio waves — quanta another ~1/1000 of microwave quanta) you must consider the quantized nature of the interaction (“photons”), due to the tiny energy level of the resonances being excited (reversing the magnetic orientation of a proton in the machine’s magnetic field).

    It adds nothing to our understanding of the atmosphere.

    I believe that KR’s point was that microwave radiation is emitted by rather cold objects, but can be absorbed by hot objects — a fact you seem to be disputing.

    BTY, any absorption calculation can be done from the wave point of view — even ones involving very large quanta — as long as you maintain the Quantum Assumption that nothing happens until a full quanta has been absorbed. This is often a way used to estimate effective cross-sections for radiation absorption (not as good as measuring them, however).

    00

  • #
    L.J. Ryan

    Bobc: 427

    Perhaps you missed my response/questions (378 & 379) to your post 327. For you to answer, I suspect, will forces reconsideration of your most deeply held beliefs. So take some time Bobc, realize the box your in and decide whether you should remain wedded to GHG physics.

    00

  • #
    BobC

    430L.J. Ryan:
    June 7th, 2011 at 4:18 am
    Bobc: 427

    Perhaps you missed my response/questions (378 & 379) to your post 327.

    I didn’t miss them, Ryan — they simply demonstrated that you still haven’t read the Wiki page on Fabre-Perot etalons that I linked. If you had read it, you would find that most of your questions are not relevant. The assumptions you made are easily corrected by skimming the Wiki page — don’t bother about the equations (unless you want to check my figures); just look at the description and drawings. Get back when you have read the page.

    So take some time Bobc, realize the box your in and decide whether you should remain wedded to GHG physics.

    The “box” I’m in, Ryan, is that I can’t have a meaningful conversation with you, since you won’t take even a few minutes to find out what I’m talking about. I don’t intend to publish a discourse on etalons when very simple explanations are available to anyone who knows about Google and has a couple of minutes.

    The point of my post was to show that passive physical systems existed that had much higher radiation fluxes inside them than the outside flux driving them. This was to answer the claim that it was impossible for the radiation arriving at the surface of the Earth to exceed the input from the Sun.

    I will, however, “remain wedded” to experimentally established facts, despite your unsupported opinions to the contrary.

    00

  • #
    L.J. Ryan

    Bobc 431

    Bobc I read the Fabry–Pérot interferometer link…AGAIN. I see nothing regarding power amplification. The equations provided within the your liked page, do not support your supposition. To all whom don’t subscribe GHG physics, this is obvious…that is output power can not exceed input power. Further, internal power within a system can not better input power. Self actualizing power amplification in not physical.

    So back to your various statements regarding REOs 99.9% reflective etalon. You said 1,000,000 W/m^2 are store within the cavities. I assumed the 0.1% not reflected was absorbed and therefore dissipated as heat. You said 0.1% was transmitted as the output…but then hedged your answer by saying If it’s not dissipated as heat, then it just passes through the etalon. (Duh!). I then asked if the tenth percent inefficiency was intentional. You never answered.

    1) So is the .001 of 1,000,000 W/m^2…1000 W/m^2 the output (transmitted) or is it loss (absorbed)?

    If the loss is not intentional and is dissipated as heat, a need to know variable is surface area. If area is unknown and loss is heat, then you can make no prediction of surface temperature. Since you can not provide surface area you said, I gave the intensity, which is power/area. Since heat dissipation also goes as the area, the area cancels out of any calculation about heating. Bob this last statement, is nonsensical.

    2)How long does take for the etalon to accumulate a 1,000,000 W/m^2?

    3)Can the output be closed as to store the 1,000,000 W/m^2 flux within the etalon?

    00

  • #
    L.J. Ryan

    Bobc 431 Since it seem you are again avoiding my other questions; I’ve re-posted my 379

    Bob 327

    I asked:A floor radiating 240 W/m^2 with a well insulated, highly reflective ceiling should, according to GHG physics, begin radiating 390 W/m^2. Or maybe the ceiling need be matte black…emulating blackbody. Does 390 W/m^2 happen instantaneously or should I get one of those GHG physic jackets? If they still legal?

    You replied:

    Do I really need to tell you why this is stupid?

    I agree, my question is stupid, idiotic shows a complete lack understanding of radiant physics…and Bob C it is a scenario you conceived (240) to describe GHG physics. It is much like the endorsement(184) of KRs wrapped radiator test. For those following along, KR and Bob C believe a radiator (say cast iron-hydronic) filled with 65 C water, will warm itself do to own reflection…assuming it’s wrapped in a IR reflective insulation.

    So yes Bob C, please tell me why my bolded question/scenario is stupid.

    00

  • #
    BobC

    L.J. Ryan:
    June 7th, 2011 at 7:41 am

    Bobc 431 Since it seem you are again avoiding my other questions; I’ve re-posted my 379

    Bob 327

    I asked:A floor radiating 240 W/m^2 with a well insulated, highly reflective ceiling should, according to GHG physics, begin radiating 390 W/m^2.

    So, tell me what “GHG physics” is, and why it says this. Imagine all those HVAC engineers designing radiant heating systems for 80+ years and not knowing what they are doing! (I’ll bet you haven’t even tried to look up any data on radiant heating.)

    Yes — that is a stupid statement.

    Bob C it is a scenario you conceived (240) to describe GHG physics

    It is yet another scenario (the Fabre-Perot etalon is another) I conceived to show that the idea that the radiant flux between two objects cannot in principle be larger than the outside driving flux is wrong. To be explicit: Something cannot be in principle impossible if there are examples of its existence. Got that?

    KR and Bob C believe a radiator (say cast iron-hydronic) filled with 65 C water, will warm itself do to own reflection…assuming it’s wrapped in a IR reflective insulation.

    No — We say something with a fixed source of heat energy will, in general, be warmer if you slow down the rate at which it loses heat. If I took the same liberties with strawman arguments that you do, I might say that you don’t believe that a coat will keep you warmer in the winter. (Have you tried the simple experiment with a heating pad yet? #385)

    So yes Bob C, please tell me why my bolded question/scenario is stupid.

    Well for one thing, you still haven’t corrected the typos in it:

    If they still legal?

    That’s kind of stupid.

    01

  • #
    BobC

    L.J. Ryan:
    June 7th, 2011 at 7:33 am

    Bobc 431

    Bobc I read the Fabry–Pérot interferometer link…AGAIN. I see nothing regarding power amplification.

    Apparently you didn’t notice that the radiant flux is always greater inside the cavity than outside. Since radiant flux is power per unit area, I see nothing wrong with calling that “power amplification”. Others, like Kevin, obviously do, so I will stick with “radiant flux” in the future to avoid triggering peoples’ phobias.

    that is output power can not exceed input power

    Well, it can actually, for pulsed systems (see below). What I actually said, however, is that the power (excuse me: “radiant flux”) between the two mirrors can be many times greater than the radiant flux outside the mirrors. This is obvious from the Wiki explanation — if you understand it, that is.

    Further, internal power within a system can not better input power.

    Hell, output power can better input power. How do you think that they build terawatt lasers (link)? With terawatt power supplies? Where do you find those? (Note that “power amplification” is a commonly used term when describing pulsed lasers.)

    I have another experiment for you to try. I’ve strung a number of electric fences. They typically run on 6 volt batteries that put out little power and can’t hurt you. So, by your reasoning (and ignoring of counter examples), the output of the fence can’t hurt you either. Grab the fence wire sometime and see.

    00

  • #
    Bryan

    Myrrh

    I think you bring up some very interesting points.

    More power to you since I believe you do not claim a formal training in orthodox science.
    After reflection on your posts this example you could use is exactly the one used by IPCC proponents.

    A 15um photon from Earth surface is absorbed by an atmospheric CO2 molecule.
    Does this imply an immediate rise in temperature?

    No – because the photon is absorbed by modes other than translational.
    Only the translational mode is involved in determination of temperature or thermal energy.

    Subsequent changes involving the second law however lead to a thermal effect of this interaction although the time lag may involve milleniums.
    IPCC advocates tie themselves in knots over this since they want a heating effect but also a full backradiation effect to “heat” the Earth surface.

    Having the cake and eating it springs to mind!

    Any subsequent atmospheric radiation emitted will be of a “lower quality” longer wavelength.
    Once these lower quality wavelengths become sufficiently long their thermal effects will again become insignificant.

    At the other end shorter wavelengths, < 2um implies thermal effects definately should be ruled out unless the radiation can be linked to proven thermalisation route.

    Photosynthesis and other routes are more likely as your posts point out.

    BobC (above) thinks that microwaves ovens can heat a warmer body by means of a radiative excange.

    I consider that machines such as a cold microwave oven can heat a warmer chicken since they expend "work" in the form of high qulity electical energy to this end.

    What do you think about the microwaveoven example?

    Dont be too polemical with BobC since he too is a sceptic.
    I made the mistakeof identifying Bob C with KR who is an AGW propagandist

    00

  • #
    BobC

    Bryan:
    June 7th, 2011 at 9:18 am

    BobC (above) thinks that microwaves ovens can heat a warmer body by means of a radiative excange.

    I consider that machines such as a cold microwave oven can heat a warmer chicken since they expend “work” in the form of high qulity electical energy to this end.

    What do you think about the microwaveoven example?

    I think we’re both right. The microwave oven example was brought up as a counter example to the claim that radiation from a cold object cannot in principle be absorbed by a hotter object.

    In the hopes of damping down the anger, here is what I believe:

    1) There is nothing physically impossible about the atmospheric greenhouse effect. Its relevance to the Earth’s climate is debatable. I think it is a mistake to argue that the greenhouse effect is impossible by using arguments that can be shown to be wrong by simple counter-examples.

    2) The contribution of CO2 to the Earth’s temperature is trivial, no matter who calculates it or how. The IPCC must postulate never seen positive feedbacks to get any significant effect at all. (And, they exaggerate the effect of temperature increases as well — why they don’t want you to know about the Medieval Warm Period, Roman Warm Period, Holocene Optimum, etc.)

    3) Human activity doesn’t seem to be affecting the atmospheric CO2 concentration at any detectable level — it has been growing at ~0.5%/year for 50 years, despite multiple times increases in human emissions. At this rate, it will double in ~140 years, producing a trivial effect on climate. Eventually, one would assume that this would become an embarrassment to the Warmists. I think they were hoping that they could claim the non-acceleration of CO2 concentration was their doing by getting limits in, but time is (has) run out on that play.

    There is already enough bad science in AGW — attack that, not some esoteric theory that doesn’t matter anyway.

    00

  • #
    BobC

    L.J. Ryan:
    June 7th, 2011 at 7:33 am
    Bobc 431

    2)How long does take for the etalon to accumulate a 1,000,000 W/m^2?

    I gave an explicit answer to this in post #268. Here it is again:

    “During the short transient period I referred to, the light intensity was building up inside the etalon due to multiple reflections back and forth between the mirrors. (Conceptually, this is the same as the time delay it takes to fill a reservoir after you close the spill gates.) If you measure the length of this transient period you will find that it is roughly the time required for light to bounce back and forth between the two mirrors 1000 times.”

    00

  • #
    BobC

    332Kevin:
    May 25th, 2011 at 9:47 am
    to BobC;

    So per your posts an eatlon both;

    “Demonstrates Energy Amplfication”

    and;

    “Stores Energy”

    Thusly, I should be able place my eatlon in front of a 1 watt laser and “fill” it with 1000 watts of “energy” because it amplifies energy. Then I can walk across town with my eatlon in my pocket with my 1000 watts of stored “energy” and give to you,

    Is that about how an eatlon works Bob ?

    Reasonably accurate except for:
    1) Watts are not energy, as you have pointed out ad nauseum. The etalon would contain a flux that could be measured in watts/square meter, an energy flux.

    2) You would have to walk across town in a few dozen nanoseconds, the “group delay” of the etalon.

    00

  • #
    wayne

    Bryan: 436
    June 7th, 2011 at 9:18 am

    A 15um photon from Earth surface is absorbed by an atmospheric CO2 molecule.
    Does this imply an immediate rise in temperature?

    No – because the photon is absorbed by modes other than translational.
    Only the translational mode is involved in determination of temperature or thermal energy.

    Brian, you are so very close, really. Just play into your statements above that there is always an equilibrium between how many ghg molecules are in an excited mode compared to the amount at ground state, water vapor or co2 included. (search wuwt: tom vonk) Thermalization and re-excitation both happen in equal amounts at all times, there is an equal sign in the equations governing these processes of equipartition of energies via thermalization and an equilibrium is always maintained on the average. So there is always the same average number of molecules in any given state without concentration changing. That is exactly why I keep claiming, contrary to AGW/IPCC claims, that energy cannot be “trapped” in a gravitationally held, open ended, atmosphere with 3K at one end of the system.

    Increased concentrations of water vapor or co2 doesn’t cause higher thermalization of energy, increased radiation from increased surface temperature from increased solar radiation does, as when total solar irradiance increases over the decades or comparing cloudy conditions to clear conditions.

    So, that implies that neither can water vapor for the same reasons and that makes perfect sense. No one has ever proven that temperature rides up and down with just humidity changes when no condensation is also occurring (clear sky). If this was not true you should literally cook every time humidity reaches 80% with a clear sky, doesn’t happen, and when it is rather hot you will find other effects are causing the rise.

    For water has a few more properties than co2 besides thermalization. High humidity prevents a portion of evaporation preventing a portion of surface cooling. High humidity causes more IR portion of incoming solar radiation to be absorbed by the atmosphere that cannot, on a net basis, ever move downward due to the decrease in temperature with altitude (see: SB). Water vapor can clump as dimers, trimers, polymers and droplets that start to radiate closer and closer to true gray bodies. This can cause warmer cloudy nights. Water vapor can evaporate and condense transferring huge quantities of energy across great distances, vertically and horizontally. CO2 lacks all of these effects, in fact it does nothing except absorb radiation as water vapor does and both are always in a thermalization equilibrium between their excited and ground level states and this causes no change in temperatures.

    There, that will give some here fodder for others to attack but I still claim it is very close to what is actually happening in atmospheres similar to Earth’s and why the global temperatures do not track co2 concentrations. Sounds like you are thinking the same. History will prove what is actual, the proof is happening right now as temperatures ease downward.

    00

  • #
    Bryan

    wayne

    I have read the Tom Vonk post but I do not agree with it.

    1.If we were dealing with only an isolated sample of CO2 at a constant temperature then collisional excitation and de-excitation would be matched as per Boltzmann and Kirchoff.
    2.However we are dealing with a gas sample which has 0.000380% CO2

    3.Further in the Earths atmosphere at night (for simplicity) there is an Earth upward stream of 15um
    So 15um absorption is relatively easy

    4.The vibrationally active CO2 has a longer relaxation time than the probability of collisional de-excitation, hence collisional loss to mainly N2 molecules.
    5. The energy of the 15um photon is dispersed among mainly N2 and O2 molecules translational modes causing local heating.
    6. For a vibrationally deactivated CO2 molecule to get enough energy to to become activated is statistically unlikely.
    However even if it does then back to 4.
    7. Any radiation emissions will probably be of a longer wavelength as there are around 30 H2O molecules for every CO2.
    The H2O molecules have a large number of possible transitions > 15um.
    8. Satellite obtained spectra show a “bite” around 15um.

    What this analysis tells us is that any “backradiation” is a considerably reduced fraction of the upward Earth surface radiation.
    It is also of (on average) a longer wavelength.
    This is in conflict with the K&T energy budget diagrams

    00

  • #
    Mark D.

    Bryan, Check your figure (item 2) above. .000380% ?

    10

  • #
    Bryan

    Thanks Mark D.

    …”Bryan ….. Check your figure (item 2) above. .000380% ?”…..

    The % sign should not be there!

    00

  • #
    wayne

    Bryan:
    June 7th, 2011 at 9:37 pm

    to wayne:
    I have read the Tom Vonk post but I do not agree with it.

    1.If we were dealing with only an isolated sample of CO2 at a constant temperature then collisional excitation and de-excitation would be matched as per Boltzmann and Kirchoff.

    Right, at every layer is at ~equal temperature as you say.

    2.However we are dealing with a gas sample which has 0.000380% CO2

    Of course, very thin.

    3.Further in the Earths atmosphere at night (for simplicity) there is an Earth upward stream of 15um
    So 15um absorption is relatively easy

    Right, there is a paper out of scientists in the Alps who took spectrums at near surface, 10 meters, 100 meters and 100,000 meters measuring upwelling radiation superimposed on top of each other. It is not until you get above the TOA that the huge ‘bite’ of CO2 appears to any great degree and that is showing decreased emission at co2’s wavelengths, not greater. Most of that co2 energy has moves toward lower energies and to greater that 17-1000 µm in wavelengths.

    4.The vibrationally active CO2 has a longer relaxation time than the probability of collisional de-excitation, hence collisional loss to mainly N2 molecules.
    5. The energy of the 15um photon is dispersed among mainly N2 and O2 molecules translational modes causing local heating.
    6. For a vibrationally deactivated CO2 molecule to get enough energy to to become activated is statistically unlikely.
    However even if it does then back to 4.

    But it is statistically likely. Otherwise, as you pointed out, what occurs is 4. to 6. over and over when in the lower troposphere. Is it not until you get above water vapor and at low pressures therefore long mean free paths that spectrum of upwelling shows co2’s final absorption and transfer out of co2’s range. There are more that one processes causing this. One is multiple photons each with lower frequencies (lower rotational shifts). Another is re-excitation of water vapor molecules at same or lower frequencies. That is what make the low emission at 15µm ‘bite’ in the spectrums looking downward.

    7. Any radiation emissions will probably be of a longer wavelength as there are around 30 H2O molecules for every CO2.
    The H2O molecules have a large number of possible transitions > 15um.

    8. Satellite obtained spectra show a “bite” around 15um.
    What this analysis tells us is that any “backradiation” is a considerably reduced fraction of the upward Earth surface radiation.

    It is also of (on average) a longer wavelength.
    This is in conflict with the K&T energy budget diagrams

    Right, most re-emissions are mostly of multiple emitted ‘photons’ at always lower frequencies. I agree.

    The one exception in all of this process is the “window” frequencies that mostly pass to space with none or one absorption and re-emission.

    But if thermalization/re-excitation and emission is not an equation with equal bi-directional transitions, show me the paper or book. I can’t find any. You ought to re-consider. It took me quite a while to see that this is what appears to be happening exactly as other physicists have described.

    No need to comment back, my wording is usually wrong somewhere, it just deserves consideration for all of the physics in this area of atmospheric radiation that I can find says this is what is happening.

    00

  • #
    Paul

    For some detailed investigation on the subject of heat transport through the atmosphere : —

    Solar radiation transport in the cloudy atmosphere – Davis_Marshak_2010.pdf [5.39 MB]

    Solar radiation transport in the cloudy atmosphere:
    a 3D perspective on observations and climate impacts.
    Anthony B Davis1,3 and Alexander Marshak2
    1 Los Alamos National Laboratory, Space and Remote Sensing Group, Los Alamos, NM 87545, USA
    2 NASA—Goddard Space Flight Center, Climate and Radiation Branch, Greenbelt, MD 20771, USA
    E-mail: [email protected] and [email protected]
    Received 8 July 2009, in final form 13 July 2009
    Published 19 January 2010
    Online at stacks.iop.org/RoPP/73/026801

    Abstract
    The interplay of sunlight with clouds is a ubiquitous and often pleasant visual experience, but it
    conjures up major challenges for weather, climate, environmental science and beyond. Those
    engaged in the characterization of clouds (and the clear air nearby) by remote sensing methods
    are even more confronted. The problem comes, on the one hand, from the spatial complexity of
    real clouds and, on the other hand, from the dominance of multiple scattering in the radiation
    transport. The former ingredient contrasts sharply with the still popular representation of
    clouds as homogeneous plane-parallel slabs for the purposes of radiative transfer
    computations. In typical cloud scenes the opposite asymptotic transport regimes of diffusion
    and ballistic propagation coexist. We survey the three-dimensional (3D) atmospheric radiative
    transfer literature over the past 50 years and identify three concurrent and intertwining thrusts:
    first, how to assess the damage (bias) caused by 3D effects in the operational 1D radiative
    transfer models? Second, how to mitigate this damage? Finally, can we exploit 3D radiative
    transfer phenomena to innovate observation methods and technologies? We quickly realize
    that the smallest scale resolved computationally or observationally may be artificial but is
    nonetheless a key quantity that separates the 3D radiative transfer solutions into two broad and
    complementary classes: stochastic and deterministic. Both approaches draw on classic and
    contemporary statistical, mathematical and computational physics.

    There are 70 pages to study in this paper and as I begin my impression is that it will be well worth the read.

    Paul

    00

  • #

    wayne,

    But it is statistically likely. Otherwise, as you pointed out, what occurs is 4. to 6. over and over when in the lower troposphere.

    This is, of course, what happens with a system in equilibrium. That is, the gas (atmosphere) achieves a certain temperature, so that everything balances.

    But in the case where you add more CO2, you’ve altered the equilibrium. The temperature of the gas (atmosphere) goes up until the balance is restored. This in turn happens in each layer of the atmosphere.

    The net result is that lower down, more IR is being trapped, and temperatures rise until as much is emitted as absorbed. Higher up, the CO2 is actually allowing the heat to escape, because with the air being less dense the CO2 relaxation time is shorter than the mean time between meaningful collisions. Hense, CO2 is more likely to “accept” energy from collisions, and emit it away through radiation. This is why CO2 cools the stratosphere, while warming the troposphere.

    But the important takeaway points are that (1) your logic applies to a system in equilibrium, and (2) the equilibrium attained does represent a net zero flow of energy, but that net zero flow is necessarily at temperature which maintains the equilibrium, and that temperature is predictably higher for a higher concentration of CO2.

    00

  • #
    L.J. Ryan

    Sphaerica (Bob):

    If every additional molecule of CO2 absorbs supplementary IR, then doesn’t each one of those same CO2 molecule radiate IR isotropically?

    And if IR is radiated isotropically, then isn’t equilibrium is forever realized?

    00

  • #
    L.J. Ryan

    BobC 435

    Apparently you didn’t notice that the radiant flux is always greater inside the cavity than outside. Well, it can actually, for pulsed systems (see below). What I actually said, however, is that the power (excuse me: “radiant flux”) between the two mirrors can be many times greater than the radiant flux outside the mirrors. This is obvious from the Wiki explanation — if you understand it, that is. Hell, output power can better input power. How do you think that they build terawatt lasers (link)? With terawatt power supplies? Where do you find those? (Note that “power amplification” is a commonly used term when describing pulsed lasers.)

    BobC if Chip amplification and/or etalon work as you describe, terawatts of output power can be fabricated by pushing kilowatts in, forget renewables an unrealized energy solution has been at hand for years. Never mind, this 2nd law thread, you should guest post on acceptable 1st law violations.

    Here is a hint on misunderstanding;
    Given 1W=1J/s

    What is the average energy released over one second wherein all 1J is released in a 1 ps burst….it’s not a terawatt.

    As I said previously self actualizing power amplification in not physical…you can not get more out then put in.

    00

  • #
    Paul

    Sphaerica (Bob):
    June 9th, 2011 at 3:22 am

    This is why CO2 cools the stratosphere, while warming the troposphere.

    But the important takeaway points are that (1) your logic applies to a system in equilibrium, and (2) the equilibrium attained does represent a net zero flow of energy, but that net zero flow is necessarily at temperature which maintains the equilibrium, and that temperature is predictably higher for a higher concentration of CO2.

    And the question is, by how much?

    To which I might add, that since the temperature at the surface is mainly controlled by convection and latent heat transport, any such warming of the troposphere will be lost in the noise and will remain virtually immeasurable.

    Paul

    00

  • #
    Myrrh

    Bryan – thank you for your post, which I’ve only just seen as I couldn’t get back to this earlier.

    If there’s too much maths language involved I find it too hard a slog to follow (because it would take too long to look up all the terms used), but I’ll go through the argument you’ve been having and see if I can understand it.

    00

  • #
    Bryan

    Myrrh

    It had occurred to me that the CO2s absorption of the 15um photon (which is well within the thermal IR spectrum) can be absorbed without causing a direct rise in temperature.
    OK, later it does, but only because of collisions.
    This obviously supports your arguments that shorter wavelengths do not directly cause an immediate heating effect.
    The photosynthesis example is very strong, where shorter wavelength energy is directly converted into chemical energy.
    This is sometimes locked away for millions of years as in the case of the fossil fuels.

    Your observations further confirm that the K&T energy budget diagrams are so far out as to be useless.
    Wayne above agrees with most of my post except below (3) where he says there is evidence that the 15um active population is stable till near TOA
    However keep up the good work and dont let the IPCC advocates intimidate you with PHD and BSc supposed authority.

    00

  • #
    Paul

    I have just downloaded and started to read Collected Scientific Papers by John Henry Poynting, Cambridge University Press, 1920 in ePub format.

    From the preface : —

    He was a remarkably clear thinker and had that characteristic insight into fundamental ideas which intuitively distinguishes between hypothesis and fact; and it was probably for this reason that he viewed with suspicion some of the more recent developments of mathematical physics. He withheld his judgment when the experimental foundations were either wanting or else inadequate to bear the superstructure erected upon them.

    I wonder how he would regard the present ‘discussion’ on ‘Global Warming’ and the state of ‘Climate Science’ today!

    Any way, his work is foundational to our understanding of radiation and I find it both interesting and helpful to read some of these early papers which are blessedly free from the bias of so much so-called science today.

    Paul

    00

  • #
    Paul

    The more things change, the more they stay the same … excerpt from ‘Collected Scientific Papers’ of John Henry Poynting, P30

    OBITUARY NOTICES XVll

    And so in the efficient discharge of his duties as a Professor, in successful original research, in the fulfilment of municipal duties, the time passed placidly on, the only cloud on an almost idyllic domestic life being his somewhat indifierent health, the first threatenings of the disease from which he ultimately died. To see if a country life would suit his health better than a town one…

    Life in the country too gave free scope to his taste for Natural History, in which he always took great interest; he was a keen and excellent observer, and a favourite contention of his was that physicists were somewhat too much inclined to confine their observations to experiments made in the laboratory and did not sufficiently avail themselves of the opportunities of studying the physical phenomena going on in the sky, the sea, and the earth.

    Bolding added.

    Paul

    00

  • #
    Paul

    Another excerpt from ‘Collected Scientific Papers’ of John Henry Poynting, P53

    Pressure of Light.

    For some years before his death Poynting devoted much attention to the question of radiation and the pressure of light. On the theory of this subject he published {Phil. Trans., A, vol. ecu) a very valuable paper, in the first part of which he discusses the application of the fourth-power law of radiation to determine the temperature of planets (in this he found afterwards he had been anticipated by Christiansen). Among other interesting results he arrived at the conclusion that the temperature of Mars must be so low that life, as we know it, would be impossible on its surface, this result was criticised by Lowell, but Poynting maintained his ground in a paper published in the Philosophical Magazine, December, 1907. The second part of the paper in the Philosophical Transactions contains investigations of the repulsive force between two hot spheres which arises from the radiation from the one tending to repel the other. He showed that if the bodies are in radiation equilibrium with the Sun at the distance of the Earth from it, the repulsive effect will be greater than the gravitational attraction between them if their radii are less than 19-6 cm., if their density were that of water; if they were made of lead the corresponding radius would be 1-78 cm. Thus if Saturn’s rings consisted of very small particles it is possible that the effect of radiation might make them repel instead of attract each other. He considers at the end of the paper the effect produced by radiation on the orbits of small bodies round the Sun and shows that this would ultimately cause them to fall into that body. To quote his own words: ” The Sun cannot tolerate dust. With the pressure of his light he drives the finest particles altogether away from his system. With his heat he warms the larger particles. They give out this heat again and with it some of that energy which enables them to withstand his attraction. Slowly he draws them to himself and at last they unite with him and end their separate existence.” {Pressure of Light, “Romance of Science” Series.)

    This reminded me of the video clip of a recent massive ejection of matter from the sun which was followed quickly by its absorption along the lines of magnetic force.

    Also interesting in connection with discussion here as to the varying effects of different wave lengths within the spectrum of solar radiation.

    There seems to be a lot more to be understood about solar radiation than that being disseminated by the IPCC.

    Paul

    00

  • #
    L.J. Ryan

    Paul: 452

    Thanks very much for this link. Paul you have a very well considered explanatory progression.

    00

  • #
    BobC

    @Ryan (#448): Please stop and think for a moment — you seem to have the driving need to disagree with everything you think I’m saying, even while you acknowledge what I am saying is correct.

    BobC if Chip amplification and/or etalon work as you describe, terawatts of output power can be fabricated by pushing kilowatts in

    I have no idea what you mean by “Chip amplification” — if you meant to say “power amplification”, then you are exactly right. This is how terawatt lasers work — they get peak output powers in the terawatt (10^12 watts) range, even though the peak input power is only in the kilowatt or less range. “Power amplification” is even a commonly used term in the laser community.

    …forget renewables an unrealized energy solution has been at hand for years. Never mind, this 2nd law thread, you should guest post on acceptable 1st law violations.

    You should try understanding the difference between power and energy — you seem to be confusing them.

    Here is a hint on misunderstanding;
    Given 1W=1J/s

    Right: Power is energy per time. If a certain amount of energy comes into a system slowly, it has a low peak input power — if it goes out more rapidly, it has a higher peak output power.

    If this is hard to understand, think of a stream: It flows through a valley with little effect. Then, you dam the stream and fill a large reservoir. Then the dam breaks and lets the accumulated water out all at once. The amount of water out is no more than the amount that came in, but the effect on the valley below is significantly different.

    What is the average energy released over one second wherein all 1J is released in a 1 ps burst….it’s not a terawatt.

    The averate is one watt. If averaged over 1000 seconds, it’s a milliwatt. If averaged over 300,000 years, it’s a picowatt. Who gives a damn, Ryan? A terawatt laser is a laser that has a peak output power of over a terawatt — not averaged over some arbitrary time defined for the entire industry by one L.J. Ryan.

    As I said previously self actualizing power amplification in not physical…you can not get more out then put in.

    You’re obviously wrong, as the entire pulsed laser industry demonstrates (not to mention the electric fence industry — tried my experiment about grabbing the output wire yet?)

    Read your own hint, Ryan: Try to understand the difference between power (energy/time) and energy.

    Perhaps you are misreading the equation you put down: In general, it should read W=J/s. If you want to specify How many Watts, Joules and seconds, you can by writing, say: 1W = 1J/1s. Only specifying 2 of the three values, as you have done, leaves the equation unsolved. The solution is obviously 1s, in your example.

    However, these are also valid equations:

    1J = 1W * 1s

    1J = (10^12 W) * (10^-12 s) = 1 terawatt * 1 picosecond

    Notice: Same amount of energy — vastly different peak powers.

    00

  • #
    L.J. Ryan

    BobC: 456

    I have no idea what you mean by “Chip amplification”

    Let me retype Chirp amplification or chirp pulse amplification

    Right: Power is energy per time. If a certain amount of energy comes into a system slowly, it has a low peak input power — if it goes out more rapidly, it has a higher peak output power.

    We agree BobC…I think. Until you make the stream analogy; which is completely dissimilar to lasers…completely dissimilar to radiative energy transfer. Water accumulates behind a dam, building potential energy as forced by gravity on water’s mass. Photons, however, do not accumulate behind a wall of mirrors…bouncing around waiting to be released to produce a burst of power.

    Laser’s continuously maximize input energy based on wavelength…there is no vessel amassing photons. Rather a continuation, which at any instant, looses more energy as heat and non-targeted wavelength, then it “focuses” at intended frequency.

    The averate is one watt. If averaged over 1000 seconds, it’s a milliwatt. If averaged over 300,000 years, it’s a picowatt. Who gives a damn, Ryan? A terawatt laser is a laser that has a peak output power of over a terawatt — not averaged over some arbitrary time defined for the entire industry by one L.J. Ryan.

    Strictly, the peak power as defined above (the maximum occurring optical power) is ambiguous; it depends on the temporal resolution (or bandwidth) of the power measurement.

    You make a similar error when referencing electric fences. Increasing voltage does not increase power…alternating current does not increase power. Shortening the the period of measurements does not increase power it only serves to detail transient spikes.

    Now, how reflective insulation acts to increase IR energy of the overall construct of radiant heating system, is just as misunderstood by your explanation. So once again, I asked: A floor radiating 240 W/m^2 with a well insulated, highly reflective ceiling should, according to GHG physics, begin radiating 390 W/m^2. Or maybe the ceiling need be matte black…emulating blackbody. Does 390 W/m^2 happen instantaneously?

    00

  • #
    BobC

    L.J. Ryan (@457):

    Photons, however, do not accumulate behind a wall of mirrors…bouncing around waiting to be released to produce a burst of power.

    You’re mistaken: This is exactly how a “cavity dumped” Q-switched laser works (link).

    “The cavity end mirrors are 100% reflective, so that no output beam is produced when the Q is high. Instead, the Q-switch is used to “dump” the beam out of the cavity after a time delay. The cavity Q goes from low to high to start the laser buildup, and then goes from high to low to “dump” the beam from the cavity all at once.”
    (Most Q-switched lasers store potential energy in the excited atomic states (“population inversion”), but cavity-dumped Q-switching stores the energy as light within a cavity bounded by 100% reflective mirrors, until a fast switch dumps it to the outside.)

    So once again, I asked: A floor radiating 240 W/m^2 with a well insulated, highly reflective ceiling should, according to GHG physics, begin radiating 390 W/m^2. Or maybe the ceiling need be matte black…emulating blackbody. Does 390 W/m^2 happen instantaneously?

    You appear to believe that this is a real question, Ryan — However, it appears to be complete nonsense to me. If you want an answer, you need to clarify a few things:

    1) What is “GHG physics”?
    2) How do these numbers follow from GHG physics? (Please demonstrate the calculation.)

    Radiant heating calculations are done with ordinary physics — Here is a description.

    00

  • #
    BobC

    Just a note Ryan:

    You have a habit of making authoritative pronouncements about what is and is not possible — about areas of science and technology that you are apparently unfamiliar with. This one, for example:

    Photons, however, do not accumulate behind a wall of mirrors…bouncing around waiting to be released to produce a burst of power.

    would, if true, make the science and art of “cavity-dumped Q-Switching” lasers impossible. However, this technology has been proposed for over 50 years and existant for over 40. Here, for example, is an authoritative explanatory link. A selected quote:

    A notable difference from [standard] Q switching is that before generation of the output pulse the energy is stored in the intracavity light field, rather than in the gain medium.

    obviously completely contradicts your pseudo-authoritative pronouncement.

    If you wish to be taken seriously, you should either broaden your knowledge base (and/)or tone down your claims in areas outside your field of expertise.

    (And just as a matter of common sense — what did you think would prevent light from being temporarily stored by bouncing between two mirrors? Surely, light can bounce off of any number of mirrors; If the mirrors are arranged so that the light retraces its path, then why is it not stored?)

    00

  • #
    Myrrh

    Bryan – done and dusted re PhD’s and their claims to be oh so far ahead of me that I couldn’t possibly understand and/or that I am just too thick to learn from them and therefore unteachable…

    http://wattsupwiththat.com/2011/05/30skeptic-strategy-for-talking-about-global-warming/#comment-678885

    Ira has posted several discussions so my run in with him on the basic premise of the KT97 begins before this last one.

    So, my gripe with the AGWScience fiction Energy Budget, as I have come to call it, is that it is nonsense from the very beginning, because Light energies simply do not convert to heat the land and oceans as it claimed. They have excluded all down-welling thermal infrared and given the properties of Heat to Light.

    This, I’ve found is of a pattern in AGWSci-fi, they take descriptions out of context in applying them to something in basic premises, like calling oxygen, nitrogen and carbon dioxide ideal gas and ‘statistical net in heat’ by adding in a concept not intrinsic to the 2nd Law and more, and so create an impossible, imaginary, world where Light is Heat and heat flows from cold to hot and carbon dioxide gets thoroughly mixed in the atmosphere which has no weight or volume and is just empty ideal space with dimensionless molecules zipping around bouncing off each other without interacting.

    It is really quite extraordinary that this is being taught in schools and consequently believed to be real science.

    Carbon dioxide being one and half times heavier than Air isn’t going to hang around TOA once the wind stops blowing or when the water in the rain grabs it, but it’s cold up there. What kind of heat could possibly be radiated back? Even in the tropics the rain forms in colder regions and when it falls is still cooler than the surface.

    The microwave oven is of the same problem I had arguing about the differences between light and heat energies, there’s this disjunct I’ve found because of trying to fit the real world into an imaginary meme, the actual properties of the constituent parts being looked at are simply excluded. The microwave oven is simply a container for the process of safely radiating a particular kind of energy into matter to cook it.

    Someone earlier gave the example re heat being “net exchange” that this meant if a body was surrounded in ice it would be saying that the ice would then raise the temperature of the body, but I was actually seriously given this as an example in one discussion by someone who had merely been fed this meme and hadn’t thought it through – that a chunk of raw meat could be left in an igloo for a few hours while off hunting for more and one would come back to a cooked dinner..

    They’ve simply added “heat can flow from cold to hot” into the statistical net of the 2nd Law and can’t then explain how they come to their claim that this means the net is always from hotter to colder. They don’t have a mechanism in place to stop a larger volume of cold from being the predominant heat source.

    A though I had while thinking about the posts here and still working through the ideas on the other board, is that they have actually created a one dimensional world because they think that all electromagnetic energy is the same and all of it creates heat, there’s simply no appreciation of the differences between things so other interactions are a closed world.

    We each think the other’s world is ludicrous!
    What is missing is that all these energies have their own characteristics, properties, things they can and cannot do.

    00

  • #
  • #
    Myrrh

    Bryan – ..trying to understand your question..

    Are you saying that at 15um the only effect on CO2 is to move it?

    00

  • #
    BobC

    Myrrh (@460):

    …Light energies simply do not convert to heat the land and oceans as it claimed.

    So, what happens to the energy of light, when it is absorbed, if it doesn’t manifest as heat?

    There is a very simple device used to estimate the output power of large lasers. You let the laser light be (nearly) totally absorbed by a black body and measure the temperature increase of the blackbody. From the difference between the blackbody temperature and ambient temperature, you can calculate the heat energy being lost by IR radiation — this is a pretty good measure of the laser output energy.

    The point here is that the laser doesn’t have to be an IR laser — it can radiate in the visible, or even in the ultraviolet: The energy, when absorbed, becomes heat.

    There is a simple way to build your own blackbody to try this. Take a stack of double-edged razor blades — about 20 or 30 is fine — and bolt them into a tight stack using screws and nuts in the blade holes. The center hole makes a good place to mount the thermocouple, using thermal cement. When you look at the stack edge on, it will be the blackest thing you have ever looked at. I guarentee this device will absorb light energy at any wavelength and convert it to heat.

    00

  • #
    Bryan

    BobC

    I think Myrrhs point is that light or higher energy photons do not automatically get thermalised, which seems to be the IPCC position.
    Photosynthesis is a ready example.

    Thermal infra red on the other hand does get automatically thermalised.

    Some then say well eventually all energy will end up thermalised -heat death of the universe.
    But that point in time is hopefully some way off.

    But I’m sure that Myrrh will have much more to say.

    00

  • #
    Myrrh

    BobC – ditto what Bryan said.

    Please do the following experiment for me:

    Go outside in the daytime on a warm sunny day with lots of blue sky, or remember such.
    Can you/did you see any lasers of blue visible light from the Sun?
    Even one?

    What you see is Blue Visible Light being reflected off all the oxygen and nitrogen molecules in our atmosphere, the voluminous and weighty mass of the gas Air pressing down on us a ton/sq ft.

    Lie down on the ground/remember doing so, and stare into the blue.

    Tell me how long it takes for your eyes to burn.

    00

  • #
    BobC

    What does this experiment prove, Myrrh? Are you maintaining that it proves that blue light cannot be converted into heat?

    If so, 1) You are wrong, and 2) Your proposed experiment needs some more control. (Or, maybe you’re just trying to prove that there are no blue lasers on the Sun? If so, whom are you arguing with who says there are?)

    Now, my proposed experiment with the argon (blue-green) laser directly shows that blue light (488 nm) can be converted into heat. (Or, you can just look up medical uses of argon lasers to see that they are used to cauterize blood vessels in the eye by burning them.)

    But heck! Don’t let common, everyday, well-known facts interfer with your grand theories! I mean, what an imposition on a Grand Theory to have to conform to pedestrian, everyday fact.

    00

  • #
    Paul

    BobC:
    June 17th, 2011 at 2:04 pm

    But heck! Don’t let common, everyday, well-known facts interfer with your grand theories! I mean, what an imposition on a Grand Theory to have to conform to pedestrian, everyday fact.

    Yesterday I watched the presentation of Willis Eschenbach, at the 4th International Conference on Climate Change, on how thunder storms act as a thermostat in the tropics, bypassing any effect of radiative slow-down, and largely explaining the current surface temperatures in the tropics. I have made my own similar observations in a temperate region and come to a similar conclusion. All the emphasis on atmospheric CO2 is really pointless and futile and so are esoteric experiments with lasers and other man-made instruments that build on what is known of the behaviour of radiation in tightly-controlled environments.

    In order to control the climate mankind will first have to control thunder storms. lol

    Paul

    00

  • #
    Graeme Bird

    This all proves that the Venus surface is being heated from the inside-out. Since the satellite showed that the lowest level of the Venus atmosphere was basically still. The first layer of CO2 is “lazy”. So there is absolutely no way that the dark side of Venus can be the cold surface heated from above. It must be a hot surface heated from within. With the other energy sources merely slowing the cooling process.

    This also implies (though on its own does not prove) that Vellikovsky was right and Carl Sagan was wrong. and that Venus is a new planet.

    The super-rotation of the clouds on Venus can help explain the basically identical heat profiles on both sides of the planet. But it cannot deny the fact that the heat direction is from the inside out, because the lowest layer has been shown to be stagnant.

    Ergo the explanation that greenhouse is the cause of Venus’ hot temperature is a theory lacking any basic motivating reason for anyone to believe it. We have a hot planet and overturning, greenhouse and air-pressure can only help explain why it has not yet cooled down.

    Also this proves that with the sun the Corona heats the photosphere of the sun and not the other way around. So that the primary, if not the only fusion, is going on in the Corona or at least on the surface. Or otherwise the energy is sourced from the outside …….. or both.

    01

  • #
    Graeme Bird

    “So, what happens to the energy of light, when it is absorbed, if it doesn’t manifest as heat?”

    Its hard to be absolutist about the effect of this backradiation when the mainstream don’t know what light is and don’t care for anyone to find out. But consider an analogy to the energy in two compressed air tanks:

    We join them together and the pressure equalizes. The flow of air molecules is from the high pressure tank to the lesser pressure tank.

    Now we cannot let our understanding of Brownian motion fool us on this matter. If we hone in on the individual molecules flowing from the high pressure tank to the low pressure tank we can see some “back-radiation” going on there. We know that via brownian motion a lot of the air molecules will be going the wrong way.

    We know what compressed air is and how the molecules move about. We don’t know what light is and so until it can be shown otherwise we would be safe to assume that the effect of this back-radiation is akin to the random wrong-way action of many of the molecules in this example.

    So if greenhouse does anything at all it ought to be assumed that it can only slow cooling a bit. And thats being a little bit charitable since we don’t have an anomaly that greenhouse needs to explain.

    01

  • #
    Graeme Bird

    “The Greenhouse effect” is a terrible misnomer since the real greenhouse effect, on earth, on Venus, and most particularly in greenhouses is OVERTURNING.

    The irony is that if the radiative properties of gases were so overpoweringly important in this context, then a greenhouse could not work. Nor would there be any problem leaving the twins in the car on a hot day, so as to do some gambling to pay the bills.

    If the radiative properties of gases were more important than overturning in the car, then the hot air in the car would quickly radiate all this infrared radiation out through the windows again. The inside of the car would never have much of a heat advantage to the air outside.

    The thing to focus on then is stratification. If there is stratification between the troposphere and the stratosphere then there will be overturning in the troposphere and more heat retention. If a cloud effectively forms another mobile strata for a short time that may help. If there is stratification between the water in your open kettle and the air outside, then the kettle can build a heat budget. But the heat budget is limited by the breakdown of the strata when the water boils.

    This is before we bring up the electrical energy flowing down from the ionosphere but thats an whole other matter. But its manifest that without stratification the radiative properties of the gases are going to do very little for your heat budget. Otherwise no greenhouse could work effectively.

    If only people would label things properly.

    http://www.youtube.com/watch?v=RZ2ow5lo4VQ

    01

  • #
    Graeme Bird

    “What does this experiment prove, Myrrh? Are you maintaining that it proves that blue light cannot be converted into heat?”

    But Bob in your proposed experiment the direction of where the energy ought to go is clear. You have a high-powered laser. There is no problem that the flow of energy will be from that laser to the thing the laser is heating.

    The problem comes when you assert a flow of energy from the heated gas or other entity BACK TO THE LASER, and if you assert also that this flow of energy is significant and not negligible.

    00

  • #
    Graeme Bird

    “in the first part of which he discusses the application of the fourth-power law of radiation to determine the temperature of planets”

    This fourth power law, along with everything else, makes a mockery of the global warming fraud. But the fourth power law does point to some very important things we ought to keep in mind. An implication is that TO SPREAD THE JOULES OUT IS TO RETAIN MORE OF THEM. Hence the importance of stopping any obstacles to the uninhibited flow of the great ocean conveyor, and more specifically the gulf stream.

    It may be and seems to be that the most severe phases of the glacial periods get locked in via ice obstruction. Also the globe ought to heat and cool step-fashion because of the extra viscosity of colder water. Such tentative conclusions flow pretty much directly from this fourth power law.

    00

  • #
    Graeme Bird

    “Clasius and Kelvin predate Bohr Theory relating to the structure of atoms.”

    Thats a good thing. Their work was solid. Bohr’s is not. The Quantum crowd don’t even buy into his model anymore. But then it doesn’t really matter what those dogmatic clowns say.

    We have to get away from science-by cult-of-personality.

    00

  • #
    Graeme Bird

    “Still, you’re doing everybody a favour by trying to educate the “It violates the 2nd law of thermodynamics!!!11!1!” cranks.”

    Just amazing how many perverts there are out there. So much for the “cranks” label. Obviously if the 2nd Law can be beaten, we have to be shown how. It won’t be something that happens on a casual basis. If it does happen it will be an extraordinary thing, and as a result of special circumstances.

    00

  • #
    Graeme Bird

    “This is preposterous. God doesn’t make rules like “photons can travel in any direction except the way it came”. Atoms radiate photons randomly and some return in the same direction.”

    We don’t know what light is, but we ought to know that photons are a fantasy. You cannot use what you imagine is the behavior of imaginary entities to contradict the 2nd law. Not without a good reason.

    00

  • #
    L.J. Ryan

    BobC 458 459

    Cavity dumping is not what you seem to think it is…that is, analogues to a dam holding back water. I suppose you do have a point, photoninc light energy, is “stored” within the reflective cavity. Similar to the water which adheres to the pipe walls as a volume flows or a mass of water flows through. Or more precisely, that water which accumulates within a pipe prior to, specified input…and that water which remains after discharge (dump). Perhaps the analogy is not completely accurate, however, I stand by my authoritative pronouncements about what is and is not possible:
    lasers continuously maximize input energy based on wavelength…there is no vessel amassing photons. Rather a continuation, which at any instant, looses more energy as heat and non-targeted wavelength, then it “focuses” at intended frequency.

    For instance BobC, a “40 W of optical power were assumed from the pump diodes, of which 80 percent might be absorbed in a useful lasing mode. Applying the Nd quantum efficiency of 0.75 leaves 24 W of possible cavity power at 1064 nm. Thus, within a 10-ns round-trip cavity time (corresponding to a laser cavity length of roughly 1.5 m), as much as 0.24 μJ might be added to the photon field”.

    When you walk through the calculation, you will realize as the start up time approaches time max energy input, photon accumulation within the cavity parasitically drops.

    Though this quick loss, quick “gain” is exactly why cavity dump is utilized…high frequency, high data transmitting communications.

    BobC without going back a reviewing the path with got us to cavity dumping, I’m sure it has very little relation to his thread topic…or maybe not. The fact lower temperature/lower energy can not make warmer higher temperature or make more aroused higher energy, is completely supported by both our links. Why else would constructed power need be supplied to lasers…otherwise, shouldn’t such effects be spontaneous?

    Now, regarding GHG physics (radiative forcing) and how this non-physical conjecture is demonstrably false. For instance, radiative floor heating can not be increased beyond input simply by adding radiative barriers. Also, proponents atmospheric models betray their stated objective. When a focused review of applied variables is processed… ie.is entropy decrease when radiative forcing is assumed.

    I’m having trouble with posting the images…I will re-work my GHG physics post tomorrow.

    00

  • #
    Paul

    CO2ISLIFE: @114
    May 21st, 2011 at 4:48 pm

    This is a topic visited on Roy Spencers Blog site.

    http://www.drroyspencer.com/2010/07/yes-virginia-cooler-objects-can-make-warmer-objects-even-warmer-still/

    I believe the following is the killer argument on this issue, But would someone like to muddy the water, by telling us that vectors have both magnitude and direction and both the emitting gas and the surface would be directionally noisy and suggest a calculus that resolves the potential impulse leakage problem if there is one.

    I’ve gone back and read Roy Spencer’s Blog and my take on what he contends is this : —

    1) In some circumstances the presence of a cooler body near a warmer body can slow the cooling of the warmer body. But when Spencer uses the words “can warm a warmer body” he is going beyond what can be experimentally verified and laying the ground for verbal twisting. Definitely “slowing the cooling of the warmer body” cannot be made into a catastrophic problem whereas “warming the warmer body” can.

    I don’t think that anybody disagrees with the fact that our atmosphere moderates our surface temperatures making the surface cooler by day and warmer by night. The point in contention is whether or not a doubling of atmospheric CO2 will make the surface warmer by a significant amount. Conflating the effect of the whole atmosphere with the effect of doubling atmospheric CO2 is a logical fallacy and it is probably because of this kind of fallacious argument that this discussion on the ‘Second Law’ is taking place at all.

    2) When Spencer talks about clouds warming the surface at night he is making the same mistake. Cloud cover formed locally by day that persists over night truly will reduced the loss of heat at the surface. But when a clear sky by day is followed by clouds coming over during the night, with warmer near-surface air temperatures, that warming is not by means of radiated energy at all. What is then happening is that a body of warmer air from elsewhere, probably from over the warmer ocean surface, has blown over the land. These are the familiar conditions referred to as a ‘maritime’ climate. In settled conditions, when no wind is blowing air from elsewhere, the temperature never rises at night due to the presence of clouds. The rate at which the near-earth air cools may slow in their presence, but that is not warming.

    3) Spencer admits that the flow of energy is always from the warmer to the cooler object. He simply contends that a body of matter, such as the atmosphere, covering the surface of the earth will slow the rate at which the surface looses energy. This may be of some significance during the hours of darkness when the direct rays of the sun are not warming the surface and the air in the Troposphere is no longer overturning. But then the effect is to slow the rate of cooling, not to warm the surface. During the hours of light, when the rays of the sun are actively warming the surface, the Troposphere, by one means or another, circumvents the insulating effect of a still atmosphere, conveying the heat from the surface to the top of the Troposphere. At that height, with 80% of the atmosphere’s mass and virtually all of the atmospheric water vapour below, there are no significant barriers to prevent radiant heat from dissipating into the cold of outer space. Hence, with no influence of CO2 on daytime warming of near-earth air temperatures and merely slowed cooling at night, there is left no possibility of catastrophic warming due to a doubling of the trace atmospheric gas CO2.

    In conclusion, I suggest that the issue of ‘back radiation’ has been completely overblown and exaggerated out of all proportion to its real significance. It has been applied to a flat, static earth rather than to a rotating globe. Calculating the effect of a doubling of atmospheric CO2 as a continuous ‘forcing’ operating while the Sun is warming the earth’s surface at an ‘average’ rate and ignoring both the rotation of the earth and the overturning of the Troposphere makes this a non-physical supposition.

    Paul

    00

  • #
    Myrrh

    BobC – Visible, light, waves are puny, they get easily scattered by the molecules of oxygen and nitrogen of our atmosphere; however you want to look at that, size to size or electronic to vibrational/rotational.

    Reflection/scattering of light on the electronic level is by the electron of the atom briefly becoming energised and emitting the light back out. This is what happens in Air. Water is different, like glass it is a transparent medium for light. Transparent on this level means that light is not absorbed at all, but passes through.

    See UV-Vis: Electronic transitions http://en.wikipedia.org/wiki/Transparency_and_translucency for the four possible outcomes when Visible, light, meets matter. The two middle descriptions are of reflection/scattering and transmission as I’ve just described re air and water.

    Photosynthesis an example of matter selectively absorbing light and reflecting that not absorbed, the green we see plants is the Green Visible not absorbed but reflected back out. The absorbed in this case is not the red and blue light’s energy used in heat creation, but for a chemical change which is not producing heat, of converting water and carbon dioxide to sugars.

    See http://en.wikipedia.org/wiki/Electromagnetic_absorption_by_water for clear graphic that Visible, light,is not absorbed by water and that it is the invisible Infrared which is absorbed, and therefore heating the oceans.

    Try heating a cup of water with Visible Blue, let me know when you’ve got it hot enough for a cup of coffee (let alone a bath* of water or the oceans which are 70% of the Earth’s surface).

    Visible electromagnetic spectrum are zilch capable of heating land and oceans of Earth as claimed in the AGWScience fiction energy budget, KT97, which says it is “Solar”, Visible and UV and Near Infrared either side, which converts to heat the Earth’s surface.

    * http://www.azonano.com/news.aspx?newsID=18696

    Most of those arguing that Light heats the land and oceans as per KT97 have been ‘educated’ into thinking this, but this has gone even further in misdirection as the example of Ira’s teaching in the linked WUWT discussion – the claim that it is these Solar energies which we feel as the heat from the Sun. All basic science at junior level is being destroyed in our education system by such manipulations of real physics. The heat we feel from the Sun, from a fire, from the stove even not producing visible light, is the heat carried by invisible thermal infrared.

    Thermal infrared is what actually heats organic matter of Earth, including ourselves.

    Visible light from the Sun is an effect produced by the Sun’s great heat, it is not hot itself. When you see great temperatures associated with the different colours you are actually seeing the temperature required to produce it, not the temperature of the light. An ordinary light bulb emits 95% of its energy in thermal infrared, 5% in light.

    NASA did have a very good children’s introduction to all this, http://science.hq.nasa.gov/kids/imagers/ems/ionfrared.html It has now been taken down but the page is saved for the record here:

    http://www.webcitation.org/5y68yeeRD

    Note the traditional teaching here, it is far infrared that we feel as heat from the Sun, near infrared is not hot. We cannot feel it anymore than we can feel blue light hot. (And note the difference in size between far and near infrared.)

    Traditional, real world science, the outcome of generations of hard work and thinking about our natural world, is being deliberately destroyed by AGWScience fiction memes. For the masses.

    This is found in every description of the properties and processes in this sciencefiction. Here, thermal infrared which we all feel as heat from the Sun has been deliberately excluded from the AGW energy budget, and in its place the properties of thermal infrared have been given to visible and shortwave which are incapable of being that.

    It is insidious. Please think about this. The manipulation of the 2nd Law is another example of the technique.

    You, generic, can argue til the cows end up in your stew pots, but the fact remains that the claim heat flows from colder to hotter is nonsense and those arguing ‘a physics’ supporting it tie themselves into logical knots because of it.

    How has this misdirection been achieved?

    In the 2nd Law simply by adding the meme ‘heat can flow from colder to hotter’ and associating it with the idea of ‘net’. Net in the real 2nd Law is in the hotter also losing heat when its energy converts to heat something colder as these interact to equilibrium.

    The AGWScience fiction meme ‘heat is the net of energy exchange from hotter to colder and colder to hotter’ can’t be logically supported when coupled with the meme ‘that net flow is always from hotter to colder’ – because there is no ‘mechanism’ which logically joins the two memes. That this is missing is the trick, the sleight of hand from AGWScience, deliberately misleading. Without that missing mechanism there is nothing to stop a greater volume of cold heating something hotter than itself. Therefore, that ‘heat is the net flow from hotter to colder’ isn’t possible to conclude.

    These, that Visible, light, is thermal, that the colder can heat the hotter, that the molecules nitrogen, oxygen and carbon dioxide have the properties of ideal gas and obey ideal gas laws, that carbon dioxide can stay up in the atmosphere for hundreds and even thousands of years accumulating even though it is heavier than Air – all these are built on giving the real world properties and processes a twist out of shape by a meme specially thought of to do this, calculated to do this.

    All sense of volume, size, weight, and interactions between such, are gone. The real world is destroyed by AGWScience fiction. It is full of impossible scenarios bearing no relation whatsoever to the real physical world around us in which we have our being and out of which we are created. We are Carbon Life Forms, carbon dioxide is the building block of our existence and our continuing food source. Only someone who has no appreciation of the real physical world could ever believe that carbon dioxide is a toxic.

    AGWScience deliberately and maliciously engineers these teachings.

    In other words, you, generic supporters believing AGWScience is real, need to look at every basic claim and compare them with what traditional and still taught science teaches about them, before you begin arguing for these memes in these convoluted illogical justifications for them. While the information is still available.

    00

  • #
    Paul

    One thing that helps put all this into perspective is viewing the earth and its atmosphere from outer space. For example, this photo of the earth clearly shows its thin atmosphere, its oceans and land-masses. Looking at that and imagining the cold of outer-space in which it is suspended by gravity and you can form some intelligent idea of how a slight increase of a trace gas, from 0.03% to 0.04% of the atmosphere, cannot be the sole cause of Global Warming, ignoring all other potential causes.

    Photo of the Earth and its Atmosphere.

    Paul

    00

  • #
    BobC

    Paul: (@467)

    Yesterday I watched the presentation of Willis Eschenbach, at the 4th International Conference on Climate Change, on how thunder storms act as a thermostat in the tropics, bypassing any effect of radiative slow-down, and largely explaining the current surface temperatures in the tropics.

    I had a friend who modeled thunderstorms at NCAR for ~ 10 years — this is pretty much what he said also (but not officially).

    All the emphasis on atmospheric CO2 is really pointless and futile

    You’ll get no argument from me that the CO2-based greenhouse effect is trivial and unimportant in the Earth’s climate system. Even the IPCC thinks so — that is why they have to postulate so many unseen positive feedbacks.

    … and so are esoteric experiments with lasers and other man-made instruments that build on what is known of the behaviour of radiation in tightly-controlled environments.

    It is precise the tightly-controlled experiments that tell us most about the basic behavior of electromagnetic waves. There is too much going on in real world environments to seperate the effects.

    If you want to argue that the greenhouse effect has been miscalculated, or is trivial w.r.t. global warming, fine — I will support you. But when you argue (like some here) that the greenhouse effect is impossible; and postulate NON-PHYSICAL behavior of radiation as “proof”, then anyone who knows anything about science is just going to tune you out. You won’t be doing the skeptic cause any good, and may well be doing it harm.

    My point in bringing up these “tightly-controlled” environments is to demonstrate that ill-considered statements like “blue light can’t become heat when absorbed”, and “The energy density inside a system can’t be larger than the input or outputs” are B.S. There are many reasons that CO2 is a trivial player in the Earth’s climate system, but fantasy physics is not one of them.

    00

  • #
    L.J. Ryan

    BobC:

    But when you argue (like some here) that the greenhouse effect is impossible; and postulate NON-PHYSICAL behavior of radiation as “proof”, then anyone who knows anything about science is just going to tune you out.

    Said otherwise, conform to the falsehood or be ignored.

    And what hubris BobC, you presume to speak for the masses whom have at least a nominal understanding of science. In actuality, the body of scientists whom recognize the NON-PHYSICAL nature of atmospheric forcing, is growing.

    If the image post correctly below, lets talk specifics problems with proponents radiative models. If it does not I will follow-up with linked images, and you can explain why it is valid.

    00

  • #
    Myrrh

    What is fantasy physics is the claim by AGWScience that Visible light is thermal. This is about the energy budget of the whole Earth! Experiments using artificially intensified lasers doensn’t prove anything, all you’re creating is a high speed drill.

    Prove that Blue visible light from the Sun is able to heat the organic land and oceans. Show the mechanism.

    I have given you a page which shows the four possibles of visible light in its meeting with matter. None of these are capable of raising the temperature of land and oceans.

    Let’s get a sense of perspective here.

    Bog standard traditional physics divides the electromagnetic waves into various catergories, the division into visible, called light, and thermal, called heat, is because that is their nature. Visible is Reflective, thermal infrared is Absorptive. The above graph of water absorption of electromagnetic waves shows very, very clearly, that visible, light, is not absorbed by water, and, that all the absorption by water is in the infrared.

    Unless you can show that classic, very well understood, and still taught traditional physics is wrong, and the AGWScience claim is right, then you’re the one fantasising.

    It doesn’t pass the Smokey test – there are countless applications using infrared for heating, where are the physical appliances which use blue light to heat water and buildings or cook food?

    The heat we feel from the Sun is thermal infrared, invisible. It warms us up as it does the oceans and land. AGWScience claims that it doesn’t even reach Earth’s surface! Can you feel it or not? That alone shows that AGWScience is fiction.

    00

  • #
    Graeme Bird

    “The point of my post was to show that passive physical systems existed that had much higher radiation fluxes inside them than the outside flux driving them. This was to answer the claim that it was impossible for the radiation arriving at the surface of the Earth to exceed the input from the Sun.”

    What a load of nonsense. You are not looking for amplification here. What you are after is a 2nd source of energy.

    00

  • #
    BobC

    L.J. Ryan (@481):
    June 18th, 2011 at 4:34 am

    Said otherwise, conform to the falsehood or be ignored.

    This is what you want to believe I said: I am not a gatekeeper of “religious truth”. What I actually said was that, if you persist in promoting ideas that can easily be shown to be false, you will eventually become ignored. That’s not me, or some cabal of evil scientists — that’s just the way people prune crackpot ideas.

    To be specific, let’s take just one of your claims:

    …near infrared is not hot. We cannot feel it anymore than we can feel blue light hot.

    Now, there are any number of ways that this claim can be tested. Not all are easily available, but the first one is easy for anyone:

    1) Google the medical uses of argon (blue-green) lasers, and note that the are used to heat cauterize blood vessels in the eye. It seems obvious that, if they can burn blood vessels in your eye, they can also burn other parts of the body and hence be felt as heat.

    2) I have personally received a painful, deep burn by holding my hand in the 1mm dia beam of a 3 milliwatt argon laser for ~5 seconds (not a wise thing to do, obviously). The laser was set up to produce a pure spectral beam (@ 488 nm) and did not contain more than 0.001% light outside of the ~ 0.1nm bandwidth. It certainly didn’t contain any measurable amount of far infrared light.

    3) Edmund scientific sells “heat mirrors” for a reasonable price online. These are coated glass plates that reflect near and far infrared light, but pass visible. (Usually, not UV, however.) One can put one of these mirrors over a magnifying glass to reject the IR and focus the remaining visible light from the Sun. It is still possible to burn and char objects with the focused visible light, although it takes longer.

    It is easily seen by your responses to criticism that you have not the slightest interest in testing your claims. That is not science. (This is also a characteristic of those government-funded researchers who flack AGW — why I don’t like calling them “scientists”.)

    If you want to do science, you must accept the primacy of experience. If you insist on promoting claims contrary to experience, you will be pigeonholed as a crackpot. Sorry L.J., that’s the way the world works.

    00

  • #
    BobC

    Correction (@484): It was a 3 watt argon laser.

    00

  • #
    Bryan

    BobC

    I have an open mind on this topic i.e. light being readily thermalised.
    Your illustration of the laser perhaps could perhaps cloud the real physics going on as the intensity is so enormous.
    Likewise lenses burning holes in things after the IR has been filtered out could be misinterpreted.
    It seems to me that burning indicates chemical change rather than a thermal one.
    I know Myrrh has a keen interest in this topic.
    What would settle the matter would be to filter out all the IR from a solar beam and direct the remaining fraction at an insulated beaker of water.
    An identical set up without the solar beam could be used as a control.
    Sensitive thermometers could be used to find if there is a heating effect.
    Has anyone tried this?

    00

  • #
    Myrrh

    BobC – you either don’t read or don’t take in what I’ve been trying to tell you here, stuff I’ve found out in exploring these aspects for myself – the AGWScience basic claims are junk.

    http://webcitation.org/5y68yeeRD

    Far infrared waves are thermal. In other words, we experience this type of infrared radiation every day in the form of heat! The heat that we feel from sunlight, a fire, a radiator or a warm sidewalk is infrared.

    Shorter, near infrared waves are not hot at all – in fact you cannot even feel them. These shorter wavelengths are the ones used by your TV’s remote control.

    So, the heat we feel from the Sun is thermal infrared, the kind we can feel as heat and not the near infrared which we can’t.

    This has been taken out of the AGWScience’s KT97 Energy Budget, and the qualities of thermal infrared given to the short wave visible which we cannot feel as heat.

    That, is simply a fact.

    That you are spending considerable time defending it is of course up to you, but you are defending a clear and deliberate manipulation of basic well known physical properties between these two categories of electromagnetic waves.

    How did you come to believe AGWScience fiction on this?

    00

  • #
    BobC

    Bryan @ 486: Your beaker experiment is a good one, if the difference between water absorption of visible and IR is controlled for. I suggest putting a black painted material in both beakers so that essentially all the light is absorbed in each beaker equally.

    This takes some care: My experience with IR is that many things that look dark in visible light (including many black paints) are, in fact, shiny bright in IR. Either buy an IR-certified black paint, or check them out with an IR camera.

    Given these caveats, I think this would be a good experiment to try if you have any questions about these things.

    Myrrh:
    June 19th, 2011 at 1:59 am
    BobC – you either don’t read or don’t take in what I’ve been trying to tell you here,

    Precisely my observation about you.

    stuff I’ve found out in exploring these aspects for myself – the AGWScience basic claims are junk.

    If you would expand your definition of “exploring” to include some of the simple experiments I have suggested, you might draw a different conclusion. I have pointed out many examples of actual, everyday uses of light that violate your assumptions. Perhaps you can explain why blue-green light can burn blood vessels?

    Shorter, near infrared waves are not hot at all – in fact you cannot even feel them.

    Really? I invite you to try my foolish experiment with the argon laser.

    This has been taken out of the AGWScience’s KT97 Energy Budget, and the qualities of thermal infrared given to the short wave visible which we cannot feel as heat.

    That, is simply a fact.

    You can feel anything as heat, that deposits sufficient energy on your skin.

    That you are spending considerable time defending it is of course up to you, but you are defending a clear and deliberate manipulation of basic well known physical properties between these two categories of electromagnetic waves.

    How did you come to believe AGWScience fiction on this?

    Because I know of numerious examples (I have given some) of experimental fact that contradict your claims.

    00

  • #
    BobC

    Bryan: A simpler way to do your experiment (to determine if visible light can produce heat) is to use the razor blade black body I described in post #463. I know, from experience, (and also from calculation) that this device is a very good black body, for all wavelengths up to nearly 1 mm, over at least a +/- 20 degree acceptance angle. This is sufficient to measure the heat content of focused sunlight with and without the IR component.

    Simply stick a temperature probe (such as are available for most multi-meters you can buy) into the main hole of the razor blade stack to get the temperature reading.

    This might be sensitive enough to use a cheap diffraction grating (also available from Edmund Scientific as plastic film) to split the sunlight into various colors and measure the heat content of each. It was just such an historical experiment that showed that IR existed — heat was produced by invisible radiation beyond the wavelength of visible red.

    00

  • #
    BobC

    Bryan, Myrrh, and anyone else who is still interested in this discussion:

    Here is a description of Herschel’s original experiments in 1800 that resulted in the discovery of infrared. If you’ll read at least the first three paragraphs, you’ll find that Herschel found that all colors of light produced heat when intercepted with a thermometer.

    Here’s a quote:

    He directed sunlight through a glass prism to create a spectrum (the rainbow created when light is divided into its colors) and then measured the temperature of each color. Herschel used three thermometers with blackened bulbs (to better absorb heat) and, for each color of the spectrum, placed one bulb in a visible color while the other two were placed beyond the spectrum as control samples. As he measured the individual temperatures of the violet, blue, green, yellow, orange, and red light, he noticed that all of the colors had temperatures higher than the controls. Moreover, he found that the temperatures of the colors increased from the violet to the red part of the spectrum. After noticing this pattern Herschel decided to measure the temperature just beyond the red portion of the spectrum in a region where no sunlight was visible. To his surprise, he found that this region had the highest temperature of all.

    This is a pretty simple experiment. It is probably still done in schools today (at least ones who care about early science education). Why should I believe anyone who tells me that this is impossible, who hasn’t even tried to repeat it?

    00

  • #
    BobC

    Addendum on Hershel’s experiment:

    Since solar radiation peaks in the green, why didn’t Hershel find that green light had the highest temperature? The reason is that a simple prism doesn’t spread the spectrum out evenly, but spreads the blue out the most and the red (and IR) the least, thus concentrating the energy pre unit area in the IR region. (A compound prism could be designed to provide a more uniform spreading.)

    You would also get a different result using a diffraction grating (depending on the pitch of the grating).

    Here is an analysis which includes the non-linear spreading.

    00

  • #
    L.J. Ryan

    BobC: 484

    if you persist in promoting ideas that can easily be shown to be false, you will eventually become ignored. That’s not me, or some cabal of evil scientists — that’s just the way people prune crackpot ideas.

    To what crackpot idea are you referring? Do you mean, hot flows to cold, high to low, energetic to passive? Not my idea BobC, and certainly not crackpot and certainly not wrong.

    Suggesting radiative insulation will increase source temperature, is crackpot. Suggesting cold surface radiation can increase the temperature of a warmer surface is crackpot. And these ideas are not mine BobC, they belong to those scientists whom promote atmospheric forcing. They belong to all whom agree with those scientists, that’s you BobC.

    00

  • #
    Myrrh

    BobC – artificially intensifying light will get you a drill, which is all you are doing in using the highly energetic, small and rapidly moving, electromagnetic waves in concentration. That says zilch about blue light’s ability to raise the temperature of land and oceans. Your experiments with lasers are irrelevant here unless you can prove they are actually raising temperature and not producing a chemical change (as Bryan mentioned).

    The closest wavelength to artificially concentrated lasers is UV. You can put a shirt on and block it. Just how intense is that? UV cannot even penetrate deeper than the first layer of three of skin,the epidermis.

    Just how capable of raising the actual temperature of the Earth’s lands and seas is UV?

    UV is shortwave which does not convert to heat, but acts on the DNA and melanin traps it. Our bodies use its energy to create vitamin D.

    There is a wealth of real life experiments going on all the time with light which means there is a wealth of information about it, you can look at these for yourself.

    Healing is one area where both intensified light to cauterise with pinpoint accuracy and deeply penetrating into tissue and muscle thermal infrared are used to directly affect changes, but also in such non-invasive applications as oximeters, which use near infrared and red to measure cholesterol levels.

    But your best place to look is where the differences are most obvious, in the field of plant growth where lamps which produce light but not much heat are in common use, as is adding extra CO2, even using different bands of light to stimulate the different stages of growth. They don’t use lasers.., unless you want your greenhouse full of fried tomatoes before they’re grown. Try LED’s.

    OK, here’s one example from people who know the difference:

    http://www.ncbi.nlm.nih.gov/pmc/articles/PMC1198233/

    ..With this configuration the aluminum block temperature did not exceed 45°C. A low operating temperature not only assures consistent LED output, but also is important as radiant heat may perturb plant growth and/or development.

    In real life, the difference between these two categories of electromagnetic waves are so well known it’s almost beyond belief that there is confusion about it from those either supporting AGW or from skeptics who’ve been subject to the spread of this fictional meme that short wave is thermal. People just don’t bother to check. That’s how they get away with it.

    00

  • #
    Myrrh

    I think the Herschel experiment interesting, but how did he exclude thermal when measuring the other colours since he used black taped bulbs to better trap heat? And what exactly were the temperatures that he got? I can’t find it.

    00

  • #
    Graeme Bird

    This has gone off track since the alarmist has steered the conversation towards whether light can be thermalised or not. Well of course it can. But the real question is whether the 2nd law of thermodynamics is being subverted by nature to heat up the world in a bizzare science fantasy.

    The photon model of light would seem to make this possible. But there is no such thing as a photon and never could be. The comparison is to sound. Because light is waves through an unknown medium. Could we have a runaway echo effect? I don’t THINKso.

    If a furnace of (x) volume was driving a turbine. The turbine caused an electrical current. The electrical current was connected to a light. The light was in a well-insulated room that was covered on the inside with mirrors of excellent quality. The room was of a volume of 2x. Could then the light eventually heat up the room to being hotter than the furnace?

    If the photon model of light were correct than this would be easy. Since the photons would come pouring in and accumulate and when they went out of existence they would do so in favor of thermal energy.

    If the 2nd law is the better guide than this would not be easy. But the thing is the 2nd Law has form. The photon model is ridiculous, and the planet is cooling. So there really is no contest here.

    Less is more. People are being fooled by a provisional model that they have gotten stuck in their head. It may even be a useful model for some purposes. Here this model, which we know for a fact is untrue, is leading people astray.

    00

  • #
    Roy Hogue

    Hey guys, I did some work about 16 years ago for company that uses UV lasers. I’ve seen a UV laser actually set a sheet of paper on fire in a few seconds. I’m an unbeliever about UV not being able to heat. Where have I got it worng?

    00

  • #
    Graeme Bird

    You haven’t got it wrong. Of course UV of sufficient strength is going to be able to set paper on fire. I don’t know how you public servants managed to steer things to such an argument cul-de-sac.

    In your experience do you believe that you’ve managed to beat the 2nd law thus opening the way up for a full-blown greenhouse effect?

    00

  • #
    Graeme Bird

    I can set paper on fire with a match or a magnifying glass. Its really no great feat. If you can’t do it with a laser you’d better get the laser fixed. Global warming arguments go on forever and they never get to the point since the warmers have no evidence to work with.

    00

  • #
    Graeme Bird

    “I was watching a Feynman video recently and he was saying(1960′s) that they could detect a single photon.”

    “Even your eyes can do this, when they are dark-adapted.”

    Riiiiiiggggggggggghhhhhhhhhhttttt. You’ve seen this mythical unicorn have you?

    00

  • #
    Graeme Bird

    Can two believers see the same photon at the same time? Thus confirming a new religious belief that a single photon is also a series of waves going off in many directions?

    00

  • #
    BobC

    Graeme Bird:
    June 19th, 2011 at 7:36 am
    This has gone off track since the alarmist has steered the conversation towards whether light can be thermalised or not. Well of course it can.

    First: I’m not an alarmist, which you would know if you read anything here instead of just popping in and popping off.

    Second: You’ve just aligned yourself with the evil AGW crowd, in Myrrh’s view — how dare you suggest non-IR light can create heat? Why don’t you try to educate them?

    The photon model of light would seem to make this possible. But there is no such thing as a photon and never could be.

    You’ve seen this mythical unicorn have you?

    You’re preaching to the choir here (me, anyway), as you would also know if you had read my post #239.

    So, do you have anything to add other than snark?

    00

  • #
    Graeme Bird

    Well I think I’ve added a lot. If you cannot give us an anomaly that greenhouse needs to explain, and you cannot show that you can break the 2nd law, then you have to concede. As usual the warming fraud is an evidence-free-zone.

    But light can be converted to heat. There is no argument there. The warmers would have to be tripping if they think they have a monopoly on this knowledge.

    00

  • #
    BobC

    If a furnace of (x) volume was driving a turbine. The turbine caused an electrical current. The electrical current was connected to a light. The light was in a well-insulated room that was covered on the inside with mirrors of excellent quality. The room was of a volume of 2x. Could then the light eventually heat up the room to being hotter than the furnace?

    You seem to be confused here, Bird. If the mirrors were of “excellent quality” they wouldn’t be converting the light to heat, but would be simply redirecting it.

    But, yes, the light intensity (but not the heat) will build to the quantity where the losses (whatever they are caused by) equal the input light flux.
    Elementary

    00

  • #
    Graeme Bird

    “You’re preaching to the choir here (me, anyway), as you would also know if you had read my post #239.”

    You believe in photons some-of-the-time?

    00

  • #
    BobC

    But light can be converted to heat. There is no argument there. The warmers would have to be tripping if they think they have a monopoly on this knowledge.

    Fine, then. You take over the argument with our local fantasists, who deny what you just proclaimed.

    00

  • #
    BobC

    Graeme Bird:
    June 19th, 2011 at 9:08 am
    “You’re preaching to the choir here (me, anyway), as you would also know if you had read my post #239.”

    You believe in photons some-of-the-time?

    What’s the matter, Bird? Couldn’t read through the whole post? (Or couldn’t understand what I said?)

    00

  • #
    Graeme Bird

    “You seem to be confused here, Bird. If the mirrors were of “excellent quality” they wouldn’t be converting the light to heat, but would be simply redirecting it.”

    No you are confused. Absolutist thinking. I’m not the least bit confused. If light were photons, then they could go on accumulating in the room. They would accumulate and bounce backward and forward, when they weren’t being turned into heat. And then you could easily beat the volume and heat of the furnace. But since light isn’t photons and light is waves through an unknown medium, and since light like all other forms of energy, conforms to the 2nd law, then the experimenters will find that they are not likely to be able to pull this off. No matter how they try to keep streaming heat and light into the room they will struggle to beat the heat of the furnace itself.

    Now of course, in your absolutist thinking the light will keep on accumulating AS LIGHT and I suppose we’ll eventually have a great starburst of some sort and everyone will go blind.

    But the 2nd Law rules this out as it does the greenhouse effect, for the most part.

    00

  • #
    Graeme Bird

    No no. Whats the matter with YOU. Clearly you are delusional. So you believe in photons SOME OF THE TIME?

    Thats just like a Catholic who says the Pope is infallible. You point out that one Pope accidentally poisoned himself while trying to poison some others. After consulting her parents the Catholic tells you that “The Pope is infallible” SOME OF THE TIME.

    Now whats the matter with you?

    00

  • #
    Graeme Bird

    “Fine, then. You take over the argument with our local fantasists, who deny what you just proclaimed.”

    Well its a form of energy. Forms of energy can be converted one to the other. And as a form of energy it conforms to the 2nd Law. Now so far you haven’t shown any special circumstances where the 2nd law is broken. A cold object mid-troposphere cannot heat up warmer air at the bottom of the troposphere. You cannot warm your hands with the contents of the refrigerator. Cold clouds made of ice cannot heat up a warm sea.

    00

  • #
    Graeme Bird

    Lets go over it again. You believe that you’ve seen an individual photon.

    Now. Can two believers see that same photon at the same time?

    You see this is a fantasy. You go. And this time don’t change the subject.

    Can two believers see that same unicorn at the same time?

    Answer the question. Its a serious question.

    00

  • #
    Bryan

    BobC

    I think that Myrrh has raised a very interesting point here.
    He also provides evidence to support his view.
    Ive just printed of his link to the LED bulbs use in agriculture for a closer read.
    Everyone knows that light can be through stages turned into thermal energy.
    The question is how much light is immediately thermalised and how long can the unthermalised energy remain unthermalised.
    The % of the total light energy with a time tag for each process.
    The broad brush approach of the IPCC who lump practically all radiation should be scrutinised.

    00

  • #
    Graeme Bird

    Recognition and application of these simple laws has big implications. Venus cannot be an old planet, it must have been molten recently. It is being heated from the inside out. The current theory of how the Suns energy is produced is also wrong. It must have its main energy production in the Corona, or from the outside in. And of course the greenhouse effect is all nonsense.

    These are important matters. We ought to get them right. They could lead to real breakthroughs. But right now we are stuck in a stagnant scientific environment.

    00

  • #
    Myrrh

    BobC re Addendum in 491 and link http://home.znet.com/schester/calculations/herschel/index.html

    “On first consideration, this result is surprising. The energy peak of the solar spectrum is at 0.60 micron (orange light), and definitely not in the infrared. So why did Herschel observe the highest reading in the infrared?”

    What we have here is an example of one of the points of confusion created by the AGWSF meme ‘peak energy in the visible’.

    All this shows is that the object emitting the heat is now hot enough to produce even visible light, all the time it is still producing thermal infrared, the hotter it gets the more thermal infrared it produces. This chart shows only the brightness of the light emitted, it says nothing, nothing, about the peak energy.

    All the heat you feel from the moment you first put on your toaster is thermal infrared, the hotter it gets the more thermal infrared you feel. The peak energy is in the infrared. Just as in an ordinary lightbulb which gives off 5% light to 95% heat, the 95% heat is the peak energy!

    The AGWSF meme is to confuse highly energetic with high energy, power.

    YOU CANNOT SEE THERMAL INFRARED, YOU CANNOT FEEL VISIBLE.

    Highly energetic means only that it moves more quickly in the same distance. To move that much more quickly than longer wavelengths of thermal in the same time, visible will be shorter and smaller.

    The highly energetic waves of visible are tiny. Tiny. So tiny they can be deflected and blocked not only by your shirt, but by molecules, and not only by molecules, but by electrons in those molecules; in air deflected by the electrons in the molecules of oxygen and nitrogen, in water blocked from joining in, not absorbed, but transmitted. Thermal infrared having greater energy actually moves the molecules of water into vibration, vibrational resonance; that’s what creates heat. The heat you feel from your toaster is all thermal infrared.

    There is no sense of scale in AGWScience Fiction. That some can’t see any difference between artificially greatly concentrating light to produce a laser which can burn a tiny little hole in a piece of paper and not see that this isn’t what we’re getting from the Sun, or we’d all be burned to a crisp, is because AGWSF deliberately messes with sense of scale. Build a laser of blue light that can ‘heat’ every inch of Earth’s surface and deep into land and the oceans. That’s the AGWSF claim.

    00

  • #
    Roy Hogue

    In your experience do you believe that you’ve managed to beat the 2nd law thus opening the way up for a full-blown greenhouse effect?

    That should answer itself. 🙂

    00

  • #
    Graeme Bird

    It does answer itself Roy. You haven’t done so, so that greenhouse is a silly concept. Worst of all it has no known anomaly to rest its tired butt on;

    This 33 degrees silliness. For starters the 33 degrees propaganda implies that the light-energy is the only energy available. Secondly it ignores overturning and lateral distribution of energy. Thirdly it fails to distinguish between temperature and heat content.

    The last is the worst and most brazen omission. Because it is the heat content of water vapor that is what the average person experiences and has been taught to associate with greenhouse.

    00

  • #
    Myrrh

    Graeme Bird – this isn’t actually ot here because the claim is that ‘heat is the net exchange of energy’ and all photons of energy are considered equally able to convert to heat which is how they get their ‘heat can flow from colder to hotter’.

    [What they don’t have is any mechanism to justify their conclusion that this obeys the 2nd Law by saying this net is always from hotter to colder.]

    If there is such a thing as a photon of light, and I’m not disputing it here, it has to relate to the wavelength it is in, doesn’t it? Isn’t this where the photon idea is adjusted to make it ‘quanta’? A package of photons. I don’t know, I haven’t thought much about that.

    As the NASA link I gave says, the size of the infrared goes from microscopic in the near infrared to pin head size in the far infrared, and it’s the far infrared we feel as heat from the Sun.

    The more highly energetic means smaller in every way, less powerful in meeting matter; UV is stopped by the first layer of skin, thermal infrared isn’t. Gamma rays highly concentrated will vapourise you instantly. http://exampleessays.com/viewpaper/38226.html Though, while looking for a description I also found this: http://avstop.com/news/ab2.html which shows how much confusion there is. It’s the gamma rays which vapourise, those further away terribly burned as UV can burn to a much lesser degree when intense as up a mountain, or skin unused to it.

    Anyway, my thinking so far including photons has been along the lines, if there are such things as photons then can they exist without the wave carrying them? When we turn on the light what happens to the light when we turn it back off again? The light is gone. Where has it gone?

    You’ve touched on a problem I was discussing on WUWT, that AGWSF thinks that created light is eternal.. It must be, because they say it stops when it is absorbed and turned to heat. For them, water must therefore absorb them because they only travel so far down in the ocean, and being thus absorbed they are heating the water to the depth they go.

    They have no concept of light going out because it runs out of puff; that the body, the volume, of the ocean itself stops them. They have no concept of volume.

    00

  • #
    Myrrh

    p.s. in other words, all mentions of ‘absorbed’ mean only one thing, as ‘all electromagnetic waves are the same’. They don’t have any concept of context.

    00

  • #
    Graeme Bird

    Right. Well I think the problem is that models are adopted for provisional reasons, and then people get taken in by them. In reality the idea of a volley of photons contradicts wave motion. So there is no photons. We have to struggle on without a proper, well-worked-out view of what light is. But so far we know it conforms to the 2nd law. And in absence of a proper model of light we have to assume then that greenhouse is negligible or total bunkum.

    You couldn’t keep squeezing photons and thermal energy into that room I was talking about. Any more than you could store up sound waves in a similar manner. And I’m sure your experience in the field will conform to this general understanding.

    00

  • #
    Graeme Bird

    I mean you couldn’t keep squeezing light waves. Or light energy. There I go slipping and talking about photons when there isn’t such a thing.

    00

  • #
    L.J. Ryan

    BobC

    Rereading your posts, I’m not sure I understand your overall stance. So, to clarify your position and to focus the trajectory of this thread please clarify the following:

    1) Do you agree solar input to the Earth’s surface is ~240 W/m^2?

    2) Do you agree with the GHG affect of atmospheric radiative forcing?

    3) Do you agree AGW is occurring?

    4) Do you agree CAGW is not occurring?

    00

  • #
    Paul

    Graeme Bird:
    June 19th, 2011 at 12:05 pm

    There I go slipping and talking about photons when there isn’t such a thing.

    The only valid way to think about light is in terms of an electro-magnetic flux. That this flux also behaves in a quantum manor, while true, does not mean that photons are real, simply that the energy is emitted, at the atomic level, by quanta of energy but still in the form of electro-magnetic waves.

    The way that these electro-magnetic waves operate is described mathematically by vectors and hence, when there are opposing vectors the weaker is offset by the greater and there is a net flow of energy from the more to the less energetic body, as described verbally by the second law of thermodynamics.

    Hence, in real life, we see a slowing of radiation from the earth’s surface at night, preventing the sudden plummeting of temperatures as would be the case without an atmosphere, with cloud-cover a major contributor, but no warming can be occurring as postulated by the ‘Greenhouse Effect’. Heat is never transmitted from a cooler to a warmer body, either by conduction or by radiation. And no build-up of energy in the atmosphere can occur in the absence of a physical barrier to convection.

    By day, when the surface is warmed by the Sun’s radiation, the amount of warming, by whatever frequencies, is limited by the cooling processes that result. The evaporation of water, convective air currents, formation of cloud cover, production of thunder-storms, etc, all observable on a daily basis, effectively limit the day-time temperatures well below what would eventuate in their absence. These cooling processes are entirely dependent on absolute temperature and do not allow for a secular trend of temperature in the atmosphere.

    The only significant warming agent is the sun’s radiation with large daily variations due to the rotation of the earth. Longer term variations in near-earth atmospheric temperatures occur due to the natural variability of the climate, with the oceans providing the major heat-sink, not the atmosphere. The daily regulatory mechanisms operate to keep the temperature within a fixed range of temperature limited by the under-shooting and over-shooting of the control mechanism, as is observed with a thermostat-controlled system. Within the limitations of a slowed-cooling over-night and the thermostat-controlled temperature by day, it is clear that the earth’s surface temperature is moderated by the atmosphere and that no run-away warming is possible.

    Those cyclical oscillations may be imposed on a secular change in temperature that is mediated by the amount of heat stored in the oceans. Such movements in stored heat will result from changes in external forcing such as from changes in the orbital distance of the Earth from the Sun or from the well-known Solar cycle over about 22 years with its changes in insolation energy levels as well as its impact on cloud cover and not from internal changes in the atmosphere.

    By ignoring these cyclical changes in external forcing, or assuming them to be zero in the long term, by creating simplistic models that relate near-earth air temperatures solely to changes in atmospheric CO2, the effect of an increase in atmospheric CO2 has been grossly exaggerated. Then, to make a bad matter worse, by assuming positive feed-backs when the feedbacks are all measured as being negative, they have compounded the exaggeration by an order of magnitude.

    Now, to defend such exaggeration by appealing to a two-way exchange of photons and arguing that this does not contradict the second law of thermodynamics, is really an admission that their claims are indefensible!

    Paul

    00

  • #
    Graeme Bird

    Right. Except for this:

    “The only significant warming agent is the sun’s radiation with large daily variations due to the rotation of the earth.”

    There is also electrical energy coming down between the ionosphere and troposphere. The electrical energy also coming from the sun.

    00

  • #
    Kevin Moore

    Paul @ 521

    I read that Venus has an atmospheric pressure 92 times that of Earth and has an average surface temperature of 464 plus degrees celcius while the atmosphere is contained of 97% carbon dioxide.

    Mars has an average surface temperature of minus 63 degrees celcius. Its atmosphere is 95% carbon dioxide.The atmospheric pressure there is just one thousandth that of Earths.

    So it seems as you suggest that the Earths atmosphere – particularly its pressure, is a regulator of temperature.

    10

  • #
    Kevin Moore

    Graeme Bird @ 522

    This may be of some interest to you.

    Recent measurements have determined that the voltage difference between the Earth and the ionosphere is 400,000 volts. Source, Scribd Tesla and the Pyramid.

    00

  • #
    Graeme Bird

    Right. That means when we point some sort of light detector upwards we aren’t necessarily seeing referred light from the ground. Part of what we see is of course the water vapor working its way out of the air and releasing its latent heat. But there is also this electrical energy making its way down from the ionosphere. Thanks for that Kevin. People must be getting to feeling I’m talking nonsense the whole time right about now. Good to get some confirmation that I’m not just making it all up.

    00

  • #
    Graeme Bird

    “Source, Scribd Tesla and the Pyramid.”

    That looks pretty cryptic.

    00

  • #
    Roy Hogue

    Kevin Moore @523,

    So it seems as you suggest that the Earths atmosphere – particularly its pressure, is a regulator of temperature.

    …that and the fortuitous circumstance that we are neither as close to the sun as Venus nor as far away as Mars.

    As with being in business, three things count: location, location and location. 😉

    00

  • #
    BobC

    Graeme Bird (@510):
    June 19th, 2011 at 9:36 am
    Lets go over it again. You believe that you’ve seen an individual photon.

    Well actually, you haven’t yet gotten through it the first time. If you had read my post (and understood it) you would realize that I said that when a detector system is sensitive enough to detect a single quanta of electromagnetic energy, it is commonly referred to as “detecting a single photon”. If you had also read my post #239 (about photons) you would know that I believe that “photons” are a figment of our imagination — the only thing we have any actual evidence of are waves that interact discretely with matter.

    Now. Can two believers see that same photon at the same time?

    Since photons have the same physical existance as “rays of light” (meaning the mathematical constructs used in lens design), you have the same ability to “detect” them as any other abstract mathematical construct.

    And, changing the question to “can two detectors detect the same quanta” is as meaningless as asking if two people can “eat the same bean”.

    You see this is a fantasy. You go. And this time don’t change the subject.

    Can two believers see that same unicorn at the same time?

    Answer the question. Its a serious question.

    Well, no it’s not. It’s a dumb question that you thought was snarky. You seem to be having trouble following arguments more complicated than a bumper sticker slogan.

    You appear to be too lazy (or incapable) of reading my previous posts to see what I think and believe. I have no interest in repeating them for you, since I have the same concern about what you think and believe.

    00

  • #
    BobC

    L.J. Ryan:
    June 19th, 2011 at 1:40 pm
    BobC

    Rereading your posts, I’m not sure I understand your overall stance. So, to clarify your position and to focus the trajectory of this thread please clarify the following:

    Well hey Ryan! Thanks for taking the time to read (and think about) my posts. Here are my serious answers:

    1) Do you agree solar input to the Earth’s surface is ~240 W/m^2?

    I haven’t looked it up — I’m assuming you are correct in this.

    2) Do you agree with the GHG affect of atmospheric radiative forcing?

    I find the mechanism plausible, as it is similar to several other methods of temporarily storing energy that I have outlined in my several posts. I haven’t gone through any of the calculations, and I am aware that the magnitude of this hypothesized effect is debated by serious people. I am also (as you know) in serious opposition to any attempt to falsify the proposed GHG mechanism that violates experimental fact — not only does this give the CAGW alarmists fodder against us, but the magnitude (or existance) of any proposed GHG mechanism is not relevant in the face of historical temperature records and paleo data that show it cannot be a major factor in climate.

    3) Do you agree AGW is occurring?

    I think it likely that some AGW is occurring, but I also think it most probable that the effect is below the noise level of our ability to measure it. I am sympathetic to the argument that the GHG effect of CO2 is too small to matter, but most of my belief is based on the data that convince me that mankind is contributing a minor amount (<4%) to the overall growth of atmospheric CO2 during the last century. (I have written and done calculations about that aspect a number of times on this blog, and can find the references if you want to read them.)

    4) Do you agree CAGW is not occurring?

    Emphatically YES. I think that, if nothing else, the historical and paleo record demonstrate that the Earth’s climate system is dominated by negative feedbacks, which would absolutely prevent anything like CAGW from occuring. Additionally, I don’t think a significant (or even measurable) amount of AGW is occuring, so the point is both false and moot.

    00

  • #
    Graeme Bird

    Bob we are going to go over it until you give a straight answer. In your religion, can two believers see the same photon at one time.

    Simple question. It requires a simple answer.

    You see the reality is that there are no photons. And being a specialist ought not allow one to dismiss logic and reason.

    You have said that you can see a single photon. I’ve not seen you retract from that position.

    00

  • #
    Graeme Bird

    You are just a liar Bob. I’ve read your post with a bit of history of Max Plank and thats just you shirking the question.

    00

  • #
    BobC

    I said:

    You appear to be too lazy (or incapable) of reading my previous posts to see what I think and believe.

    And you reply:

    Graeme Bird:
    June 20th, 2011 at 3:44 am

    You are just a liar Bob. I’ve read your post with a bit of history of Max Plank and thats just you shirking the question.

    You have said that you can see a single photon. I’ve not seen you retract from that position.

    So, the answer is “incapable”.

    Post away — I’ve no intention of arguing with a half-wit who can’t read.

    For anyone not inclined to scroll back to post 251 to see what it is that Graeme is having trouble understanding, here it is:

    Even your eyes can do this [detect a single “photon”], when they are dark-adapted. Since EM waves interact with matter in discrete units of energy, called “quanta”, when we can detect a single quanta of interaction it is common to say that we have “detected a single photon”.

    What we have never done is observe a photon in flight. It is not even certain that that is a meaningful statement, or that photons represent anything real other than the final interaction of a wave with matter.

    They certainly are a convenient mental concept, however, as long as you don’t get too carried away and hypothesize properties contrary to the known facts of EM theory.

    00

  • #
    BobC

    Graeme Bird:
    June 20th, 2011 at 3:42 am

    Bob we are going to go over it until you give a straight answer. In your religion, can two believers see the same photon at one time.

    Simple question. It requires a simple answer.

    Here was my answer (@528):

    Since photons have the same physical existance as “rays of light” (meaning the mathematical constructs used in lens design), you have the same ability to “detect” them as any other abstract mathematical construct.

    And, changing the question to “can two detectors detect the same quanta” is as meaningless as asking if two people can “eat the same bean”.

    Most people have enough intelligence to extract the fact that my short answer is “no”. Perhaps you can file that for future reference.

    00

  • #
    Graeme Bird

    If photons exist at all, they are not merely mathematical constructs dopey. A mathematical construct is something some fellow mucks about with in his room with a pencil and a piece of paper.

    Now you believe that you can see these mathematical constructs. An individual one of them. Of course to achieve this amazing superhuman feat, you need to get your eyes acclimatised to the dark first. I guess that makes it alright and plausible in your eyes.

    Clearly you can make lenses without having a belief in photons.

    00

  • #
    BobC

    Graeme Bird:
    June 20th, 2011 at 4:54 am

    If photons exist at all, they are not merely mathematical constructs dopey. A mathematical construct is something some fellow mucks about with in his room with a pencil and a piece of paper.

    Who says they exist? It takes a distinct lack of reading comprehension to assign such an idea to me. My opinion (which you would know if you could read for content) is that photons are figments of our imagination. This makes them abstract concepts which you cannot see or detect. (I go into some detail about this in post 239, which you were unable to understand, apparently.)

    If you assign rules to photons so that the concept is useful for calculations — like in Feynman diagrams — then that makes them mathmetical (or logical) constructs. You still can’t see or detect them, because they aren’t real objects, just as the “rays” used in lens design aren’t real objects, even though we define rules for their “propagation”. (Useful for approximating wave propagation through systems sufficiently larger than the wavelength.)

    (I see some faint signs that your brain may be catching up to your mouth. We can hope.)

    00

  • #
    BobC

    Graeme: I can see that I’m being too subtle for your brain.
    RE:
    When I say that “you have the same ability to “detect” them [“photons”] as any other abstract mathematical construct”, I mean, of course, that you can’t detect them physically since they are not real physical objects. To maintain otherwise would be to commit a severe logical “category error”.

    This seemed obvious to me at the time, but I see now that it went right over your head. So, let me give you the “bumper sticker slogan” version that you might be able to follow — these are my positions on the subject of photons (which were clear to most people from my previous posts):

    1) Photons don’t exist — they are figments of our imagination.

    2) All we really know is that EM waves are absorbed and created in discrete quanta of energy. Since this seems “particle-like” behavior, it has prompted the idea of photons.

    3) There is no “theory of photons”: We can’t describe them or their behavior. Any basic calculations about absorption or emission must be done from the wave point of view using Maxwell’s equations.

    4) When a system is sensitive enough to detect a single quantum of EM interaction is is commonly referred to as “detecting a single photon” — even by Richard Feynman. All this really means is that a single quantum event has been detected — it doesn’t mean that photons exist. It’s just sloppy description.

    5) Many physicists fall into the habit of using “photons” as a description. All of the “particle-wave duality” paradoxes disappear when you realize that there is no evidence that propagating “light particles” exist: All we really have is that EM waves interact with matter in discrete chunks of energy. That is strange enough — you don’t have to make it worse by postulating things that have no evidence to support them.

    Perhaps this will head off any more dumb questions from you.

    00

  • #
    Graeme Bird

    Right. I can see I misconstrued what you were about. I see myself in almost violent agreement with pretty-much everything you are saying. I don’t have a great deal more to add. I will add something a bit later. Whether it will be particularly relevant is another matter.

    00

  • #
    L.J. Ryan

    BobC 529

    Given your answers in 529, do agree with the radiative equilibrium portion of GHG theory? Specifically, because a portion of terrestrial radiation is absorbed by atmospheric GHGs, Earths system radiation output does not equal input. Because of this imbalance, surface temperature must rise to achieve equilibrium.

    00

  • #
    BobC

    Graeme Bird (@537):

    Thanks. Sorry I got a bit testy.

    00

  • #
    BobC

    L.J. Ryan (@538):
    June 21st, 2011 at 2:47 am

    BobC 529

    Given your answers in 529, do agree with the radiative equilibrium portion of GHG theory? Specifically, because a portion of terrestrial radiation is absorbed by atmospheric GHGs, Earths system radiation output does not equal input. Because of this imbalance, surface temperature must rise to achieve equilibrium.

    I don’t agree that “system radiation output does not equal input”, as that would require either an energy source or sink in the Earth system, which has neither (to first order — obviously a small amount of energy comes from the heated interior of the Earth). So, if you observe the Earth from afar (a satellite, say), the energy emitted (by reflection and re-emission) must equal the incident energy (almost all from the Sun). This is what satellites see.

    What I see the radiative greenhouse effect hypothesis claiming is that some energy is temporarily trapped between the atmosphere and the Earth, raising the temperature of both, and resulting in a local radiation flux that is greater than both the system input (Sun) and system output (Earth’s radiation signature from far out in space).

    I don’t see this as implausible or impossible because I can think of a number of other passive systems for which the internal energy density is greater than the input and output. One example is an optical etalon, which I really, really don’t want to go into again 🙂

    A more intuitive example, perhaps, is a child’s swing. The input energy density is quite small (gentle pushes), and at equilibrium must equal the energy lost due to swivel friction and wind resistance. However, the energy stored in the swing at equilibrium equals the energy of many pushes, as you would become painfully aware of if you stepped in front of it at the bottom of the swing’s arc.

    Given that I find the mechanism plausible, however, doesn’t mean I think it’s significant. Only a very small amount of energy can be stored due to CO2 absorption and re-emissions given the best assumptions. The problem is difficult to calculate and some of the historical assumptions are under intellectual attack. Even the IPCC has to postulate other, totally unobserved interactions (positive feedbacks) in order to even make a weak claim that CO2 matters.

    00

  • #
    BobC

    Graeme: Reading through your posts (the informative ones, not the combative ones 😉 ) especially re Venus and electrial energy in the solar system, I’m betting you would really enjoy James P. Hogan’s tour of “heretical” science, “Kicking the Sacred Cow”. (In fact, it’d be a good read for all skeptics.)

    I’d bet even money you already have a copy.

    00

  • #
    Paul

    BobC:
    June 21st, 2011 at 4:57 am

    Only a very small amount of energy can be stored due to CO2 absorption and re-emissions given the best assumptions. The problem is difficult to calculate and some of the historical assumptions are under intellectual attack. Even the IPCC has to postulate other, totally unobserved interactions (positive feedbacks) in order to even make a weak claim that CO2 matters.

    BobC, I’m with you all the way on this.

    I’d like to add that the heat-sink that needs to be taken into account most is that of the oceans which can store three orders of magnitude more energy than the atmosphere. The land-surface of the Earth also stores energy by day and releases it by night though on a lower order of magnitude and with a shorter cycle. Underground temperatures 3 M below the surface have almost no variation throughout Earth’s annual solar cycle. But the 30 -40 year cycles of ocean temperatures contribute significantly to observed cyclical climate change.

    The analogy of the swing I find helpful and it is the daily push from the Sun, stored as heat in the oceans, that is the Earth’s fly-wheel storing the energy from many daily ‘pushes’. It would be interesting to compute both the daily and the cumulative quantities of energy so stored and relate those to the quantity of daily energy received at the surface from the Sun. It is worth noting that while we experience it, on the surface, as a daily recurrent cycle, from the perspective of the Sun it is a continuous process as the Earth rotates beneath the Sun.

    The ocean-stored energy is redistributed around the globe, from the exposed to the unexposed hemisphere, by rotation initially, from the ocean to the atmosphere by conduction and convection [and radiation when the air is cooler], from the equatorial regions to the temperate and finally to the polar regions by means of ocean circulations. Some is also returned to the atmosphere during the hours of darkness, along with energy radiated from the land surface, contributing significantly to the moderating day-time to night-time temperature swings as well as moderating the summer to winter temperature swings away from the tropics.

    It should be easily understood that it is not the warm air that is imparting heat to the oceans in any significant quantity. By day the sun’s radiation warms the surface causing it to evaporate, warms the air causing it to convect, thunder-storms accelerate this process moving vast amounts of energy from the oceans to the atmosphere, transporting that energy to the top of the troposphere. A small part of the solar energy is retained by the ocean, mainly in the tropics, and it would be interesting to quantify that amount.

    All this was once generally understood but has been eclipsed by the new doctrine of man-made warming via the mechanism of increased atmospheric carbon dioxide. Under this doctrine the main cause of retained energy at the surface of the earth is the presence of molecules in the atmosphere that absorb certain wavelengths of radiation, of which carbon dioxide is considered to be the most ‘significant’ due to the fact that it does not condense out of the atmosphere as does water vapour. This is shown in such diagrams as the Earth’s Energy Budget where ‘back-radiation’ is quantified as of greater significance than the ‘average’ solar radiation as measured on a flat surface equal in area to the surface of the Earth. See for example Earth’s Annual Global Mean Energy Budget where solar radiation absorbed at the surface is given as 168 Watts per square meter while back-radiation absorbed at the surface is given as 324 Watts per square meter. Where are the scientific experiments, in the real atmosphere, to check this central and crucial claim? Without experimental data to support this speculation there is no scientific validity to it.

    To me, this is the crux of the issue both in terms of this thread and in terms of the overall political issue. While the physics of the process is of the essence of the matter, the real battle is with the perceptions of the general public, most of whom have neither the means nor the motivation to enquire directly themselves into the difficulties and complexities of the physics and who therefore accept what is being told them by their political, environmental, scientific and mass-media leaders.

    The problem here is that there has been a deliberate and subtle attempt to sway the minds of the multitudes into believing that there is a dire and immanent threat to their and their children’s safety. They are being panicked into sudden, extreme and ill-considered actions through the selective and exaggerated use of supposedly scientific data. Meanwhile, instead of proper debate between scientists with sufficient knowledge and capability to get to the bottom of the matter, there has been a long-standing attempt to by-pass such debate and to proceed immediately to convincing the masses with incessant propaganda.

    It can be seen right here, on this thread, that when the protagonists shout at and talk past each other, little progress in understanding the complexities of the climate system is made. But once people start to listen to each other there is progress in elucidating the matters where opinions are divided. That is as it should be. Then a true assessment of the risks can be undertaken and appropriate action taken to mitigate those risks.

    Instead, what we have at the present is the attempt to convince the mass of mankind that the world’s climate system has been transformed by the burning of fossil fuels from a refrigerator, cooling the earth’s surface, into a micro-wave oven that is set to cook both us and our children unless we turn it off! And all that, forgetting that it is indeed our life-support system that is really what we would end up turning off if we can be panicked into flicking the switch!

    Paul

    00

  • #
    BobC

    Paul: When you consider the oceans, the atmospheric CO2 seems puny indeed, even if you allow the improbable effectiveness the alarmists argue for. I find it particularly absurd when climate scientists respond to the measured loss of upper ocean heat by claiming that the “heat is going into the deep oceans”. One wonders how it bypasses the upper oceans on the way?

    OT, but one interesting thing I got from James Hogan’s book (referenced in post # 541) is that the corruption we see in climate science is, unfortunately, not a new thing, but has been going on in almost all sciences practically since science was invented. (Huygen, for example, couldn’t publish his wave theory of light while Newton lived, since Newton championed a particle theory and Huygen was afraid to face the backlash from Newton’s supporters. In the end, it was published posthumously.)

    Hogan’s book is full of examples of suppression and corruption of far more consequence than a theory of light.

    Scientists, alas, are no more (or less) human than politicians.

    00

  • #
    Graeme Bird

    “I’d bet even money you already have a copy.”

    No I don’t have a copy. I’ve been getting more and more scandalised by the state of things since about 2005. I was aware of the ghastly primitive thinking in mainstream economics. But I thought it was just my subject and not all the others as well.

    I tend to find people only after I’ve been scratching at the surface of the topic myself. Then I wind up wondering “why didn’t people tell me about this fellow earlier.” Usually its someone who is known as a “crank.” But being a crank often just entails applying simple realities from all different areas to a problem thats been rorted by some specialist priesthood or other.

    00

  • #
    L.J. Ryan

    BobC 540

    What I see the radiative greenhouse effect hypothesis claiming is that some energy is temporarily trapped between the atmosphere and the Earth, raising the temperature of both, and resulting in a local radiation flux that is greater than both the system input (Sun) and system output (Earth’s radiation signature from far out in space).

    One problems I have with radiative greenhouse effect, the magnitude of energy claiming temporarily trapped. With a 240 W/m^2 solar input and 390 W/m^2 terrestrial output, the delta 150 W/m^2 is back radiation. Said otherwise, energy temporarily trapped between the atmosphere and the Earth, is 62.5% of surface radiation. Adding that re-radiated trapped radiation to solar input, the required 390 W/m^2 is achieved…162.5% of input!

    And herein lies the origin of my “stupid” radiative heating question. I can not realize such performance though radiative insulation is incorporated within my abode.

    So what am I missing…is the Earth system flux difference simply a sampling difference (seconds vs ms) or does forcing play a significant role in global mean temperature, or what?

    00

  • #
    BobC

    L.J. Ryan:
    June 22nd, 2011 at 1:07 am

    One problems I have with radiative greenhouse effect, the magnitude of energy claiming temporarily trapped. With a 240 W/m^2 solar input and 390 W/m^2 terrestrial output, the delta 150 W/m^2 is back radiation. Said otherwise, energy temporarily trapped between the atmosphere and the Earth, is 62.5% of surface radiation. Adding that re-radiated trapped radiation to solar input, the required 390 W/m^2 is achieved…162.5% of input!

    It does seem suspiciously large, doesn’t it? Also, it is a “bait and switch” as this is the claimed “total GH effect” and no one claims that CO2 can possibly account for more than a very minor component of the total GH effect.

    I am not at all convinced that the “standard” calculations of the GH effect are correct. I think that a certain amount of effect is needed to account for the Earth’s temperature increase over a bare blackbody, and this is just arbitrarily assigned to the GH effect. As others have noted, there are many other things going on (including many we aren’t aware of, most probably) so this conclusion is logically unsuportable “argument from ignorance”: (“We can’t ‘explain’ it unless we assume the GHE is this.”).

    00

  • #
    L.J. Ryan

    BobC 546

    So GHG radiative forcing, anthropogenic and natural, is only a minor player (delta minor) in the mechanism which explain the energy delta. And of the this minor role, human induced CO2 accounts for <4% of the delta minor.

    Do I have this right?

    What, approximately, is the magnitude of this delta minor?

    00

  • #
    Graeme Bird

    “One problems I have with radiative greenhouse effect, the magnitude of energy claiming temporarily trapped. With a 240 W/m^2 solar input and 390 W/m^2 terrestrial output, the delta 150 W/m^2 is back radiation.”

    That doesn’t follow. If the terrestrial output is more than the solar input, there is a second energy source. If the earth puts out more energy than it receives that would negate the greenhouse effect. Not support it.

    00

  • #
    Graeme Bird

    Well just to correct myself a bit; We have to be sure we are comparing like with like. We don’t want to compare downwards energy at the tropo-pause with upward energy on the surface. We don’t want to assume that all energy trapped is trapped by greenhouse. I assume almost none of it is. And we want to know that we are comparing actual measurements with estimates or with other actual measurements.

    All planets give off more light-energy then they receive. As all of them receive electrical energy as well. Venus is still cooling down. And Jupiter and Saturn appear to have fusion going on in their atmosphere akin to the sun.(But not akin to current mainstream versions of how the sun works.) None of these excesses of outward to inward light-energy can be put down to greenhouse. But its no fair contest if we are measuring inward against outward at two different altitudes.

    00

  • #
    BobC

    L.J. Ryan:
    June 22nd, 2011 at 1:53 am
    BobC 546

    So GHG radiative forcing, anthropogenic and natural, is only a minor player (delta minor) in the mechanism which explain the energy delta. And of the this minor role, human induced CO2 accounts for <4% of the delta minor.

    Do I have this right?

    So I believe.

    What, approximately, is the magnitude of this delta minor?

    I don’t know with any certainty what the magnitude of the GH effect is, other than what is published. I do know that there have been several challanges to both the details of the “standard” calculation (not surprising, as it’s nearly a century old) and the simplifying assumptions that allowed it.

    I base my 4% anthropogenic estimate on the 36 (peer-reviewed) papers that actually tried to measure the atmospheric lifetime of CO2. They mostly found lifetimes between 4 and 12 years. Summary Chart here You read a lot of hand-waving at places like RealClimate about how this doesn’t matter — interestingly, however, in peer-reviewed papers no one tries this; They just ignore these measurements and reference only other CO2 cycle model papers that claim hundreds of years lifetime.

    The importance of the lifetime is that anthropogenic CO2 emissions can’t accumulate high enough to matter unless the lifetime is very long. (Kind of like trying to fill a bucket with lots of holes — it takes a greater inflow to do it.)

    Norwegian geologist Tom Segalstad has written and published a number of papers on the errors in CO2 estimates from ice cores, and how the usual evidence cited for human CO2 contributions doesn’t actually show that at all. Here is a paper (HTML) on that subject (read section 5). Segalstad also estimates (using, apparently, the standard calculations) that the human contribution to the total greenhouse effect is “less than half a Watt/m^2 of the 146 W/m^2 “Greenhouse Effect” of a cloudless atmosphere”.

    If we use Segalstad’s estimates, and the usual AGW claim that the total greenhouse effect produces 33 deg C of warming, then you can easily calculate that the Human contribution to GW is ~ 0.1 degrees — a value too small to be detectable in the century or two that we have been taking climate records (even if those records weren’t tampered with).

    00

  • #
    BobC

    Graeme,

    I’m not claiming that this is a direct analogy to the hypothesized GH effect, but an optical etalon is an example of a passive device that has a higher (sometimes much higher) radiative flux internally than the input or output fluxes. It doesn’t do this by using a separate energy source, but by temporarily storing the radiant energy between partially transparent mirrors. (Here’s the Wiki article.)

    You can decide for yourself if this has any relevance to the hypothesized GH effect — I don’t have anything more to say on the subject, than I’ve already said earlier in this thread.

    Interesting point about electrical energy input to the planets. The whole subject of the “Plasma Universe” (meaning the effect on the visible universe of the gigantic plasma currents in space) is nearly completely ignored by astrophysicists — the more so since it explains many observational features that require lots of imagination otherwise. Swedish physicist Hannes Alfven, who did much of the seminal work on this theory and got a Nobel prize for his more terrestrial work on plasmas, used his Nobel speech to a captive audience (1970) to tweak the astrophysicist community:

    But it is only the plasma that does not understand how beautiful their theories are, and absolutely refuses to obey them.

    00

  • #
    Myrrh

    Er – the obvious missing input is from thermal infrared..

    Though again, we’ve left with the strange idea that light energies convert to heat organic matter of Earth to raise it temperature enough to produce the thermal infrared claimed.

    But also, what does this 22°C ‘greenhouse’ warming actually refer to?

    Another sleight of hand from AGWScienceInc. The minus 18°C is Earth with the greenhouse – the real greenhouse – all our gaseous atmosphere, not, as AGWSF has it. Which it does by changing the meaning of greenhouse gas, by taking out the water cycle as Paul eloquently described it. Without the water cycle, the Earth would be 67°C. The difference then, is between what the temp would be without the water cycle and what it is, the 15°C. What we have is the real greenhouse gas water acts to cool the atmosphere.

    Earth without the real greenhouse, our gaseous atmosphere including water, -18°C

    Earth with our gaseous atmosphere, but without water cycle, 67°C

    Current temp, 15°C

    Just as AGWS avoids including downwelling thermal infrared which we can all feel from the Sun as heat at the Earth’s surface, so it takes the greenhouse gas water vapour out of its main role in cooling the Earth. Think deserts.

    And since CO2 accumulating mentioned, it takes out the properties of CO2, that it is heavier than Air, gives it and oxygen and nitrogen the propeties of ideal gas, i.e. no properties at all, and so claims our real gaseous atmosphere, the fluid heavy weight, is also non-existant.

    I think, imho, it’s pretty much useless to keep arguing about these things if one hasn’t taken on board first exactly what AGWSF has done to real properties and processes.

    People just take these figures and descriptions from AGWS as a given, just as here the 33&degC ‘representing’ the ‘addition of greenhouse gas’.

    No one bothers checking.

    In post 460 here I gave a link to a WUWT discussion, I’m not going back to it to find the actual post, but not a lot of posts from the end I have an attempt at deconstructing the AGWS presentation of this information which is ‘based on’ the real figures as I’ve just given, but in giving them the goal posts are moved as I’ve described, there is a disjunct. The effect is to slide over the fact that water has been excluded in its main real greenhouse role which is to cool the planet.

    00

  • #
    Myrrh

    Ah, should be the 33°C referred to, not 22.

    00

  • #
    Myrrh

    /#comment-673855 (I found a note on it).

    00

  • #
    Paul

    BobC:
    June 22nd, 2011 at 1:24 am

    L.J. Ryan:
    June 22nd, 2011 at 1:07 am

    One problems I have with radiative greenhouse effect, the magnitude of energy claiming temporarily trapped. With a 240 W/m^2 solar input and 390 W/m^2 terrestrial output, the delta 150 W/m^2 is back radiation. Said otherwise, energy temporarily trapped between the atmosphere and the Earth, is 62.5% of surface radiation. Adding that re-radiated trapped radiation to solar input, the required 390 W/m^2 is achieved…162.5% of input!

    The first thing that alerted me to the need for scepticism was that the CAGW idea was first floated, to my knowledge here in New Zealand, by our Prime Minister at the time, a person whom I would trust no further than I can throw an elephant.

    Then the astonishing magnitude of the ‘effect’ of atmospheric H2O and CO2 was one of the first things that I noticed as soon as I began to look into the preposterous claims of CAGW. Standing outside in the sun and feeling its warmth, then standing in the shade of my house and comparing the ‘warmth’ coming down from the sky, with the sun hidden from view, or comparing the difference between with and without the shade of a cloud on a warm summer’s day, those figures did not make the slightest sense to me. Nor did standing outside at night and observing the ‘down-dwelling heat’ corroborate the claim. Utter nonsense!

    If you postulate a downward electro-magnetic flux that partially offsets the upward electro-magnetic flux from the surface of the earth, with a net upward flux leading to heat flowing from the warmer to the cooler body, then that makes perfect sense though the absolute measures of those fluxes must be accurately quantified.

    One would expect that the atmosphere, being above the 3 K temperature of outer space, would slow the loss of body-heat even in the entire absence of molecules that ‘trap’ radiation. Even the official web sites admit that all matter emits radiation when its temperature is above absolute zero.
    [See this useful site as an example. Climate and Earth’s Energy Budget]

    Therefore I take exception to the assumption that the surface has to lose all its energy via radiation in the absence of atmospheric ‘Greenhouse gases’. I also take exception to the claim that, in the absence of atmospheric ‘Greenhouse gases’ there would be zero energy flux from the warmed atmosphere at the surface. Both claims seem based on thought experiments that exclude all other means of heat transfer than radiation from the surface to the atmosphere.

    It does seem suspiciously large, doesn’t it? Also, it is a “bait and switch” as this is the claimed “total GH effect” and no one claims that CO2 can possibly account for more than a very minor component of the total GH effect.

    One always has to keep one’s eye on the pea with these Warmists. One of their main tactics is to conflate unequal terms, stating something about one term that most would agree on then attributing that to another term that it does not apply to at all. Since the wider public do not know enough to perceive the unwarranted switch they are taken in by the wrongful assertion. The scientists who do know the difference either do not see the issue as being important enough or are on board with the deception so do not speak out against it. Thus public perceptions are swayed to agree with this great scam. That’s all that they need to keep the gravy train going.

    I am not at all convinced that the “standard” calculations of the GH effect are correct. I think that a certain amount of effect is needed to account for the Earth’s temperature increase over a bare blackbody, and this is just arbitrarily assigned to the GH effect. As others have noted, there are many other things going on (including many we aren’t aware of, most probably) so this conclusion is logically unsuportable “argument from ignorance”: (“We can’t ‘explain’ it unless we assume the GHE is this.”).

    For a start, the Earth is not a blackbody, it is a graybody, and the effective temperature of the earth, calculated on its albedo without any atmosphere, is on the order of +6 degrees Celsius, not the -18 degrees Celsius often quoted for the Earth as a blackbody. So, right at the start, there is a four-fold exaggeration of the total effect of the atmosphere on surface temperatures.

    On the aspect of checking their calculations, I downloaded one of the latest ‘peer reviewed’ ‘scientific’ papers on the Earth’s energy balance, to see for myself what these scientists are saying in the literature, and am [not really!] astonished to find that the error-bars are orders of magnitude greater than the supposedly ‘scientific’ results. The amount of ‘forcing’ by anthropogenic atmospheric carbon dioxide, which is the one thing under contention, is merely the residual, expressed to decimal places of a Watt, after a large number of estimates, all of which have greater uncertainties than the residual.

    Download the PDF here : —
    EARTH’S GLOBAL ENERGY BUDGET
    by Kevin E. Trenberth, John T. Fasullo, and Jeff rey Kiehl

    Here is a sample extracted from the paper : —

    There is a TOA imbalance of 6.4 W m−2 from
    CERES data and this is outside of the realm of current
    estimates of global imbalances (Willis et al. 2004;
    Hansen et al. 2005; Huang 2006) that are expected
    from observed increases in carbon dioxide and other
    greenhouse gases in the atmosphere. The TOA energy
    imbalance can probably be most accurately determined
    from climate models and is estimated to be 0.85 ±
    0.15 W m−2 by Hansen et al. (2005) and is supported by
    estimated recent changes in ocean heat content (Willis
    et al. 2004; Hansen et al. 2005). A comprehensive
    error analysis of the CERES mean budget (Wielicki
    et al. 2006) is used in Fasullo and Trenberth (2008a)
    to guide adjustments of the CERES TOA fluxes so as
    to match the estimated global imbalance. CERES data
    are from the SRBAVG (edition 2D rev 1) data product.
    An upper error bound on the longwave adjustment
    is 1.5 W m−2, and OLR was therefore increased uniformly
    by this amount in constructing a best estimate.
    We also apply a uniform scaling to albedo such that
    the global mean increase from 0.286 to 0.298 rather
    than scaling ASR directly, as per Trenberth (1997),
    to address the remaining error. Thus, the net TOA
    imbalance is reduced to an acceptable but imposed
    0.9 W m−2 (about 0.5 PW). Even with this increase, the
    global mean albedo is significantly smaller than for
    KT97 based on ERBE [0.298 versus 0.313; see Fasullo
    and Trenberth (2008a) for details].

    Bottom line, the key value in the CAGW argument, the amount of ‘forcing’ attributable to human additions to atmospheric CO2, is not even directly measurable!

    Now who’d a thought that?

    Paul

    00

  • #
    Myrrh

    Ah, another one, graybody… Ta. I’m so glad you put in the bottom line.

    So, is the 67°C without water, but with atmosphere from the -18°C, or was that originally worked out from greybody figure?

    00

  • #
    Paul

    Myrrh:
    June 22nd, 2011 at 10:30 am

    Ah, another one, graybody… Ta. I’m so glad you put in the bottom line.

    So, is the 67°C without water, but with atmosphere from the -18°C, or was that originally worked out from greybody figure?

    No, the -18°C is calculated on the Earth being a blackbody.

    If the calculation is done correctly, using the actual albedo of the Earth’s surface, I understand that the graybody calculation comes out at +6°C.

    So the overall difference made by the atmosphere is not +33°C, it is about +9°C. How much of that can be attributed to atmospheric CO2 is another calculation, and then the portion attributable to an anthropogenic increase in atmospheric CO2 may be 4% of that.

    Paul

    00

  • #
    BobC

    Paul @555:

    Truly amazing! The “new witch doctors” don’t look at chicken entralls anymore — they look at the measurement noise and “see” CAGW!

    00

  • #
    Paul

    Truly amazing! The “new witch doctors” don’t look at chicken entralls anymore — they look at the measurement noise and “see” CAGW!

    Yes, BobC, that’s about it in a nut shell!

    And I might add, there is a strong measure of circular reasoning going on, too.

    One of the main sources of their estimate of ‘CO2 forcing’ is the rise in ocean levels. That ‘estimation’ is on the basis simply that CO2 is the only factor that they can think of that might have caused the oceans to rise. They then base their threat of more ocean level increases on their certainty that CO2 levels will rise, due to the burning of fossil fuels.

    Paul

    00

  • #
    Graeme Bird

    The whole thing is done on a god-of-gaps basis. They see a gap and instead of saying that “God did it” they blame it on the greenhouse effect. They leave out the “REAL” greenhouse effect (of overturning,) when calculating the alleged anomaly that is the cause of their theory in the first place.

    Skeptics are constantly owning up to things they have no right to own up to. They say ….. “its standard radiative physics that a doubling of CO2 will increase the atmosphere by 1 degree C in the first instance.”

    Leaving aside that this is an ambit claim, rounded up from 1 degree F, there is absolutely no basis for it. If you find yourself saying that stop. Remember you have been put-upon, and you are only offering agreement so as not to seem churlish.

    For is it not the case that this calculation of 1 degrees is sitting on the back of the assumption that greenhouse is responsible for the 33 degrees anomaly???

    If the 1 degrees meme does not come from there it comes from nowhere. Or does someone have some other idea?

    Every word they say is a lie “including “and” and “the.””

    00

  • #
    Graeme Bird

    “I’m not claiming that this is a direct analogy to the hypothesized GH effect, but an optical etalon is an example of a passive device that has a higher (sometimes much higher) radiative flux internally than the input or output fluxes. It doesn’t do this by using a separate energy source, but by temporarily storing the radiant energy between partially transparent mirrors. (Here’s the Wiki article.)”

    Right but this is not comparing like with like. For example suppose I’ve got my light-meter (readout in watts) and I put it on the roof of the actual greenhouse, pointing upwards. Then I get another one and I point it at the ceiling, pointing downwards. I expect the downward pointing meter to do better, due to it whipping the other one in the evening through to almost mid-morning. And this is due to the “REAL greenhouse effect” that is to say overturning.

    But from what I can make out these guys are comparing the tropo-pause for the inward light, and the ground for the outward light. But of cause it is the troposphere that is known for its overturning out of all the strata. And thats only the beginning of non-greenhouse sources for the differential.

    Now suppose we go down to where the magma is, and we do the same with the light meter. Its going to work out the same whether its pointing up or down, supposing it could even make the reading and survive the experience.

    I’d have to put these etalon’s under the general heading the effects of strata. Thats my guess anyhow. Once I get around to figuring out exactly what they are.

    00

  • #
    L.J. Ryan

    BobC 550

    I agree GHG radiative forcing plays an insignificant role. You say delta minor = ~0 I say delta minor = 0. When considering warming above Earth’s black body, 255K seems a bit low. I understand how such a low temperature is calculated, but it seems…erroneous. More correctly, 240 W/m^2 seems low. Considering Graeme 2nd energy source point, Myrrh solar IR argument and assuming 255K is correct, to what do contribute delta major?

    00

  • #
    L.J. Ryan

    Paul 557

    No, the -18°C is calculated on the Earth being a blackbody.

    If the calculation is done correctly, using the actual albedo of the Earth’s surface, I understand that the graybody calculation comes out at +6°C.

    I agree -18 is Earth’s blackbody equivalent temperature. However, +6 C can not be a straight up greybody calculation. Since a theoretical blackbody will absorb all radiation it represents the theoretical maximum temperature a radiation spectrum can confer. A grey body will reflect or transmit a portion of that same radiation spectrum, therefore absorbing less…lower temperature.

    This backbody discussion is inline with an article written by Alan Siddon. Below is a excerpt:

    The idea of trapping light was intriguing, however, and Gustav Kirchhoff (1824-1887)
    conceived a solution: A hole in a cave. A beam of light could enter this hole but the walls
    inside would absorb any reflections and prevent the light from escaping. Thus, by confining
    incoming radiation, the thermal energy which light confers could be shown to its maximum
    advantage. Kirchhoff’s scheme was superior to selectively transmitting glass because a cave
    absorbs and traps all wavelengths of light, thus creating a complete radiative imbalance. At
    least theoretically.
    Well, so what was found by cavity experiments? That a perfectly absorptive (“black”) body
    rises to a temperature a bit higher than an actual black body that’s free to radiate to its
    surroundings. A theoretical blackbody thereby defines the upper limit of temperature vs
    radiant absorption.
    Try to grasp the implication, then. A blackbody cavity mimics the radiative restriction that
    ‘greenhouse gases’ are said to induce. Indeed, virtually none of the thermal radiation
    generated inside this cavity is allowed to escape. It “re-circulates” instead, and is sampled
    through a tiny hole. Does this confinement lead to a runaway greenhouse effect, though? No,
    it only sets an upper temperature limit — the SAME limit that’s applied to the earth in the
    first place, for its estimated temperature is based on a blackbody equation!

    00

  • #
    Myrrh

    Is the cave ‘experiment’ theoretical or actual? I don’t understand it.

    00

  • #
    L.J. Ryan

    Myrrh 564

    The cave/cavity experiment were, are actual. Blackbody spectrum ovens are in fact the reverse of Kirchhoff’s conception. That is, heating the cavity to generate a spectrum. A second source can be found at this University of Virgina.

    00

  • #
    L.J. Ryan

    Myrrh

    In support of you solar IR argument, I happened across an interesting observation. Ice cubes in direct sun light, contained within a transparent double wall vessel, take hours to melt. Move the tumbler to dark counter top and the ice melts quite quickly. Now one might conclude the higher temperature black surface conducted directly tho the interior wall, but I don’t think so. If this were the case the bottom side of the cup would have a condensate pool at the base…humidity nearing 90% at 88 F. I not sure exactly what conclusion can be drawn, but it is fare to say the visible light had little effect on the ice.

    00

  • #
    L.J. Ryan

    Wow L.J. Ryan! Perhaps you should read your post prior to hitting the SUBMIT COMMENT button. Take two:

    Myrrh

    In support of your solar IR argument, I happened across an interesting observation. Ice cubes in direct sun light, contained within a transparent double wall vessel, take hours to melt. Move the tumbler to dark counter top and the ice melts quite quickly. Now one might conclude the higher temperature black surface conducted directly tho to the interior wall, but I don’t think so. If this were the case, the bottom side of the cup would have a condensate pool at the base…humidity nearing 90% at 88 F. I ‘m not sure exactly exactly sure what conclusions can be drawn, but it is fare fair to say the visible light had little effect on the ice.

    00

  • #
    Paul

    Here is a discussion of the blackbody or graybody effective temperatue of the Earth, showing that the correct figure is 6 degrees Celsius regardless of whether the reflectivity is taken as less than zero or not. You will need to visit the site to see the equation properly formatted.

    http://en.wikipedia.org/wiki/Stefan-Boltzmann_law

    Temperature of the Earth

    Similarly we can calculate the effective temperature of the Earth TE by equating the energy received from the Sun and the energy transmitted by the Earth, under the black-body approximation:

    \begin{align} T_E & = T_S \sqrt{r_S\over 2 a_0 } \\ & = 5780 \; {\rm K} \times \sqrt{696 \times 10^{6} \; {\rm m} \over 2 \times 149.598 \times 10^{9} \; {\rm m} } \\ & \approx 279 \; {\rm K} \end{align}

    where TS is the temperature of the Sun, rS the radius of the Sun, and a0 is the distance between the Earth and the Sun. Thus resulting in an effective temperature of 6°C on the surface of the Earth.

    The above derivation is a rough approximation only, as it ignores any temperature differences and changes on the planet and also the greenhouse effect. However, the result does not change if the Earth is a grey body rather than a black body because any change in both the absorption and the emission will cancel each-other out.

    The IPCC[3] and other sources[4] present an effective temperature of 255 K (−18 °C), which can be found by considering that the Earth’s albedo is 0.3, meaning that 30 % of the solar radiation that hits the planet gets scattered back into space without absorption. The energy from the Sun is thus multiplied by 0.7 to account for this reduction in absorption. Even though the absorption is reduced, perfect blackbody emission is assumed, leading to this lower equilibrium temperature.

    However, compared to the 30% reflection of the Sun’s energy, a much larger fraction of long-wave radiation from the surface of the earth is absorbed or reflected in the atmosphere instead of being radiated away, by greenhouse gases, namely water vapor, carbon dioxide and methane.[5][6] Since the emissivity (weighted more in the longer wavelengths where the Earth radiates) is reduced more than the absorptivity (weighted more in the shorter wavelengths of the Sun’s radiation), the equilibrium temperature is higher than the simple black-body calculation estimates. As a result, the Earth’s actual average surface temperature is about 288 K (14 °C), rather than 279 K.

    Paul

    00

  • #
    Paul

    Just for the record, here is a discussion of what happens between two parallel surfaces of differing emissivity : —

    Emissivity between two walls

    Given two parallel walls whose facing surfaces have respective emissivities \varepsilon_1 and \varepsilon_2 at a given wavelength, a certain fraction of the radiation of that wavelength just inside one wall will leave that wall and enter the other. By Kirchhoff’s law of thermal radiation for a given wavelength, whatever portion of the radiation incident on a surface, from either side, that does not pass through the surface as emission to the other side, is reflected. When this reflected radiation is neglected, the proportion of radiation emitted from the first wall is \varepsilon_1, and the proportion of that entering the second wall is therefore \varepsilon_1\varepsilon_2.

    When reflection is taken into account, what does not enter the second wall is reflected back to the first wall, initially in an amount \varepsilon_1(1-\varepsilon_2). A fraction 1-\varepsilon_1 of this is then reflected back to the second wall, thereby augmenting the original emission from the first wall. These reflections bounce back and forth in diminishing quantity. Solving for the steady state then gives as the total proportion of radiation entering the second wall

    {\varepsilon}_{1,2}=\frac{1}{{\frac{1}{\varepsilon_1}}+{\frac{1}{\varepsilon_2}}-1}=\frac{\varepsilon_1\varepsilon_2}{\varepsilon_1+\varepsilon_2-\varepsilon_1\varepsilon_2}

    This formula is symmetric, and the proportion of radiation just inside the second wall that enters the first wall is the same. This is true regardless of what reflections and absorptions take place inside the two walls away from their facing surfaces, since the formula only concerns the radiation leaving one wall for the other.

    The quantities in these formulas are intensities rather than amplitudes, the appropriate choice when the walls are many wavelengths apart as the reflected and transmitted beams will then combine incoherently. When the walls are only a few wavelengths apart, as arises for example with the thin films used in the manufacture of optical coatings, the reflections tend to combine coherently, resulting in interference. In such a situation the above formula becomes invalid, and one must then add amplitudes instead of intensities, taking into account the phase shift as the gap is traversed and the phase reversal that occurs with reflection, concerns that did not arise in the incoherent large-gap or thick-film case.

    Once again you will need to go to the URL to read the formulae correctly formatted, and the subject matter is not strictly relevant to the situation that exists between the earth’s surface and its atmosphere, but I think that this was discussed earlier and here is a formula that describes the situation quantitatively.

    Bottom line is that I don’t think that this will help much to scare the public based on human emissions of carbon dioxide to the atmosphere.

    Paul

    00

  • #
    Graeme Bird

    You can see how light being waves and not photons might tend to put the kibosh on hopes of retaining and accumulating energy by this method. Its like hearing that sound waves can cause heat, and then hoping to get a runaway heating effect via the containment of echoes.

    00

  • #
    Graeme Bird

    “The difference then, is between what the temp would be without the water cycle and what it is, the 15°C. What we have is the real greenhouse gas water acts to cool the atmosphere.”

    The thrust of this was pretty clear to me very early on upon investigating this matter. Near Chiang Mai there is a roadside restaurant. At that elevated altitude you come off the road in the hot season, and the heat is appalling. Above the restaurant is this shade cloth and there is water running down on all sides. Only some of it making it to the ground before evaporating.

    The thing is that once you are under the substantial volume that the shade cloth covers you are blissfully cool. It really quite astonishing just how effective this refrigeration effect is.

    So here is the water vapor refrigerating the oceans when its being created. Water vapor is proof of refrigeration. How could they possibly have been so delusional as to count evaporation a positive feedback?

    The lunatics were running this myopic land-based model that did not care about actual weather. They did not take into account the location of where the latent heat was finally released. So they would say that the latent heat isn’t lost to the atmosphere, since of course its released mid-troposphere, or more likely they just didn’t think about it. Too busy adding up watts per square metre.

    Where water vapor greenhouse may be relevant is a situation of diminished buoyancy of water-vapor-saturated air. That is to say right in the middle of the tropical zone, newly evaporated water might find itself not as buoyant as under normal conditions. Not in a position to rise up and let loose its latent heat.

    If the clouds are having trouble forming due to diminished “cosmic ray” availability it might be that the tropical zone expands, pushing the other zones north and south and up the mountains.

    Supposing proper drops of water cannot form, but the air is hovering around 100% saturation. Then you will get water dithering either side of its phase-change, and since full-blown drops cannot form perhaps you have microscopic liquid water hovering about. It may be that the only serious greenhouse substance is airborne microscopic liquid water. Since you want a warmer-substance to be a lot heavier than air so that it does not simply carry the latent and thermal energy right up and away.

    00

  • #
    Graeme Bird

    In my efforts to put back the weather into climate science here is a neat little lecture that splits up the land into 14 different ecological niches. I’ve gotten the impression, that when the earth heats up or cools down, looking at it from one angle, its a bit like the tropical zone expanding and pushing the others north south and up.

    Note that unlike other greenhouse gases, more water vapor in the troposphere won’t mean more blocking of incoming IR far higher up. Whereas more methane and CO2 will lead to blocking of incoming since there will be increased levels above the level where the water vapor peters out.

    http://www.youtube.com/watch?v=HktMEQIx7FY

    00

  • #
    L.J. Ryan

    Paul 568

    I think we agree. Your point, Earth’s temperature (279K) as calculated based on proximity and magnitude is in fact equal(assuming no albedo)to Earth’s blackbody temp. Take the equality:

    flux density emitted via earth = flux density absorbed via earth

    σTe^4= S/4 * (1-A)

    Te= Earth’s temperature

    S=solar constant= 1370 W/m^2

    A=albedo

    So we get

    σTe^4= S/4

    Te={S/(4σ)}^.25

    Te= 278.8K

    Two sides of the same coin. Assuming albedo is zero may not be completely accurate, but I agree solar input to the Earth’s surface is greater then 240 W/m^2. Moreover, 255K should not be a starting temperature from which additional energy need be applied to reach 288K.

    The alarmist however, need to begin at 255 K in order to exaggerate, or fabricate the GHG effect. It is from starting point all of their idealized models are based. And inexorably linked, is the new equilibrium argument. Paraphrased as follows:

    Terrestrial radiation absorbed by atmospheric GHGs act to unbalance Earth’s system radiation budget…and because input energy = output energy, the Earth temperature must rise in order to counter that which is GHG absorbed. More GHG more absorption, more absorption higher temperature.

    It is this equilibrium argument which foils the whole GHG mechanism. If you click back to the An Idealize Earth + atmosphere diagram, notice To is purposely not assigned a value. Should the f (atmospheric absorption efficiency) increase, more CO2, To will increase. As in any formula, it must work backwards and forwards. Testing this formula forward, increasing f with To= 288K, works as alarmist plan. Testing backwards forces a reevaluation of the mechanism. With To= 255K, as required to draw warning of the potentially dangerous GHG apparatus, the model achieves equilibrium. Said otherwise, to realize 288K, To must first be 255K and T1= 255K…ie. when To = T1, input power equals output power,fσT1^4 + (1-f)σTo^4=σTo^4= 240W/m^2, equilibrium is achieved, no need for temperature increase.

    Of course, a model based on a false premise (radiative forcing) will undoubtedly prove faulty.

    00

  • #

    @ L.J. Ryan (573)

    What are the sources for the plots in your above links “idealized models” and “required to draw warning of the potentially dangerous GHG apparatus”? Where are those being referenced from?

    I have other sources for similar plots but I would like the sources for those as well, as I am writing a new paper debunking this idealized model and showing it to be pure fiction and sophistry, logically, mathematically, and physically. The more references to who uses these models, the better.

    Thank you.

    00

  • #
    Paul

    I downloaded a spreadsheet that calculates the emitted radiation of a blackbody for any given temperature and it confirms that the temperature of a planet at the same distance from the Sun as the Earth, considered as a blackbody, would be approximately 279 k [ or + 6 degrees Celsius]
    Spreadsheet here

    The figure of – 18 degrees Celsius is obtained by first reducing the amount of sunlight reaching the earth due to the presence of the atmosphere and then considering the earth as a blackbody, which it would be only in the absence of the atmosphere. So they are not comparing like with like.

    Looking at the energy balance of the Earth, it is the case that they introduce ‘back-radiation’ which it is claimed is absorbed by the surface, that is greater than the amount of radiation absorbed from the Sun’s incident radiation.

    Paul

    00

  • #
    Paul

    A resource for the ‘official’ version of the Greenhouse Effect is here : —

    CHAPTER 7. THE GREENHOUSE EFFECT

    This chapter seems to cover the entire field, including radiation theory, feed-backs, the GCM models, etc

    00

  • #

    Thanks Paul. I do already have that one referenced.

    Cheers.

    00

  • #
    L.J. Ryan

    Postma

    The idealized model I referenced is here I. The source link contained therein has not been updated and should link here II page 9.

    The potentially dangerous GHG apparatus source can be found here III and here IV page 13.

    Hope this aids your effort.

    I may have a few others references to pass along, I’ll post them this weekend.

    00

  • #
    BenAW

    @Paul #575

    The figure of – 18 degrees Celsius is obtained by first reducing the amount of sunlight reaching the earth due to the presence of the atmosphere and then considering the earth as a blackbody, which it would be only in the absence of the atmosphere.

    Afaik is the -18C calculated by taking the total amount of solar energy that is intercepted by the earth, 1364 W/m^2 on a disk with same radius as the earth. Since the earth is ~a sphere the radiation is divided by 4. > 341 W/m^2. Using albedo 30 only 70 % of the energy heats the earth.
    > ~240 W/m^2. Using the Stefan-Boltzmann blackbody formula you get the 255K = -18C often cited.
    > 341 W/m^2.

    00

  • #
    Myrrh

    L.J. Ryan 565-567

    Many thanks, first for the links to blackbody history from which I’ve explored some more, still don’t understand it..

    ..but it seems ‘no one’ understands quantum so I don’t feel too bad about it.

    The experiment you mention is interesting. Wayne in another discussion was trying to figure out how to do this and suggested running water over a glass box to take out the thermal ir and so testing if visible could actually heat as KT97 claims. The Chiang Mai restaurant seems a variation of this too, somehow, I’m trying to get my head around these.

    00

  • #
    BobC

    Graeme Bird:
    June 23rd, 2011 at 3:53 pm

    You can see how light being waves and not photons might tend to put the kibosh on hopes of retaining and accumulating energy by this method. Its like hearing that sound waves can cause heat, and then hoping to get a runaway heating effect via the containment of echoes.

    May I make a slight clarification? You can contain light “echos” between highly efficient mirrors and build up a very high energy flux — as long as you are very careful to not turn it into heat. Once you start absorbing the light (turning it into heat) you just get the input energy rate as heat output. In practice, an etalon’s internal intensity is limited by the value where the loss (due to leakage or absorption) equals the input.

    (It seems to me this is a significant difference between an etalon and the hypothesized GH effect, where total absorption occurs at each end. I’ll have to cogitate on it some more.)

    00

  • #
    L.J. Ryan

    Graeme Bird: 570

    Spot on Graeme! Don’t fear the tsunami, fear the wake of it’s undertow. It’s not the quake to fear, it the vibration of the falling structure which is really devastating. The lava is scorching, but mind the steam… I might have to barrow your insightful truism…in fact I will,

    Thanks Graeme.

    00

  • #
    L.J. Ryan

    Paul 575

    Looking at the energy balance of the Earth, it is the case that they introduce ‘back-radiation’ which it is claimed is absorbed by the surface, that is greater than the amount of radiation absorbed from the Sun’s incident radiation.

    Earth’s blackbody/graybody is insignificant to atmospheric forcing…duh! Paul, just fall inline and accept the consensus…please.

    How absurd, reflected or re-radiated energy is greater then the professed source! Quick cover the mirrors, fear your reflection! As Graeme poignantly mused,

    retaining and accumulating energy by this method…is like hearing that sound waves can cause heat, and then hoping to get a runaway heating effect via the containment of echoes.

    00

  • #
    Graeme Bird

    We are talking about the color of CO2. A scattering effect is akin to a muddying effect.

    You can see how on Venus, the CO2 can help the planet retain heat. Since the Venus atmosphere is heated from the inside-out. So scattering or muddying the light of course might be expected to help Venus stay hot. Just as the thick atmosphere and the dense clouds can help Venus retain its heat.

    But on earth, and especially since delta-extra-CO2 will eventually manifest above the troposphere …… But on earth it ought to be very clear that CO2 is a negative feedback. Since the light will be scattered prior to that light punching into the ocean.

    Penetration is the key here. Joules that penetrate more deeply do double and triple duty. Watts per square metre means nothing without penetration.

    Now we see that CO2 is a negative feedback and the data clearly bears this out. At least the ice core data does. This is not definitive because its a single proxy study. But clearly when CO2 peaks we don’t expect temperature to be rising.

    One cannot say too much because we ought not build upon single proxies. But we ought to be clear that on Venus CO2 helps retain heat because the direction of energy is from the inside out. Whereas on Earth the CO2 ought to be a net cooler since the net energy is from the sun inwards.

    00

  • #
    Graeme Bird

    “retaining and accumulating energy by this method…is like hearing that sound waves can cause heat, and then hoping to get a runaway heating effect via the containment of echoes.”

    Well I’m right about that. You can focus the light but you aren’t going to store it any more than you can store echoes.

    Thats just a fact. Get used to it. Any other scenario, where you are accumulating light has to be more or less instantaneous. Bob can speculate that this is not the case if we can avoid turning the light into heat. But you aren’t going to avoid this.

    My analogy stands. Light is waves and not photons. So light will defy accumulation. Thats something everyone ought to simply get used to.

    00

  • #
    Graeme Bird

    I want to emphasise this, because I suspect that Ryan is being a dope.

    You simply cannot accumulate waves. If you could accumulate waves you could accumulate sound waves as well. Or water waves. Any attempt has waves interfering with eachother and neutralising eachother.

    No-one has ever been able to accumulate waves. And Ryan being a smart alec isn’t about to change this reality.

    00

  • #
    L.J. Ryan

    Myrrh: 580

    If you’re interested in a gratis thermal radiation quick course, I can point you this informative series. It is a bit long but thoroughly covers the pertinent equations.

    Regarding the ice cube/tumbler observation, it occurred to me “re-test” when ambient temperature is low…say O C. Maybe, increase visible light by seating on foil backed insulation. I suspect ice will remain ice. Take a standard pint glass and re-run the “test”, I suspect you will find a cool water in the glass.

    00

  • #
    L.J. Ryan

    Graeme Bird: 586

    I suppose my sarcasm is a bit dry, or dopey, but my intention was/is to echo your point. Seriously Graeme, I completely agree with your sound wave analog…and…and the folly of photons.

    00

  • #
    Graeme Bird

    Holy crap. You are the second person on this one thread I’ve completely taken the wrong way.

    I guess its me thats being a bit dopey. In my defense I should note that I have a long history arguing with these incredible physics ideologues. So I always now suspect the worst.

    00

  • #
    L.J. Ryan

    Graeme Bird: 589

    As I tip my pint, cheers Graeme.

    00

  • #
    L.J. Ryan

    Postma: 574

    From:

    Environmental Science
    Toward a Sustainable Future 11e

    Richard T. Wright
    Dorothy F. Boorse

    Gorgon College

    published by Benjamin Cummings

    Page 458

    Page 459

    00

  • #
    L.J. Ryan

    Postma: 574

    From: Environment
    The Science Behind the Stories 4e

    Jay Withgott
    Scott Bernnan

    published by Benjamin Cummings

    page 496

    00

  • #
    Myrrh

    L.J.Ryan 587

    Er.. first lecture, first boo boo.

    He’s put Visible into “thermal radition”, and, gives light bulb as example saying that it’s hot, and gives off light..

    An ordinary incandescent light bulb gives 95% of its energy output as thermal infrared, heat. Only 5% or less is given off as visible, light. Light is not hot. Near infrared is not hot, as the now defunct NASA page used to teach. So why is he teaching this?

    It’s that AGWScience meme again, the well known still in traditional science distinction between Light and Heat electromagnetic waves is being scrubbed out of teaching.

    [In a nutshell, Light is not thermal, but the AGWScience energy budget claims it is. Visible light is tiny. Is reflected and bounced around by the nitrogen and oxygen molecules in the sky, hardly something then that can raise their vibrational/rotational mode to convert to heat as does thermal infrared. And in the oceans, water is transparent to visible, it is transmitted through the water unchanged and having no effect except being delayed in passing through. These simply have no ability to raise the temp of organic matter, they have other uses more suitable to the energy they carry, like photosynthesis, creation of vitamin D and so on.]

    This is why I have the problem with the ‘blackbody’ emissions chart/planck stuff and from the Sun. It doesn’t make any sense in terms of peak volume. If 95% of the energy emitted from the lightbulb is thermal infrared, the amount of visible is negligible. The amount of visible from the Sun is tiny, how can it, tiny and easily stopped even by a shirt, be given so much credibility as ‘highly energetic’ to mean powerful?

    All those black body stuff is really saying is that it is heat creating these energies and the hotter it is the shorter and smaller the wavelengths can be given off.

    Why does this page http://m.plantengineering.com/index.php?id=2831&tx_ttnews%5Btt_news%5D=33209&cHash=db4db9479b
    have to say the following?:

    “Not as widely recognized is the fact that incandescent objects emit a tremendous amount of invisible infrared radiation. For example, the radiance of a steel billet at 1500 F is 100,000 times greater in the infrared spectrum than in the visible spectrum.”

    100,000 times greater!

    Where is that acknowledged in the ‘blackbody cavity’ stuff? This is what I can’t get my head around. Nothing seems to explain this, and that’s before they go into creating quantum to explain figures they can’t get to work.

    And for the lightbulb – http://www.newton.dep.anl.gov/askasci/eng99/eng99505.htm

    00

  • #
    BobC

    You simply cannot accumulate waves. If you could accumulate waves you could accumulate sound waves as well. Or water waves. Any attempt has waves interfering with eachother and neutralising eachother.

    No-one has ever been able to accumulate waves.

    With some trepidation, I feel the need to clarify this. As a blanket statement, it simply isn’t true. Etalons do this, lasers do this (cavity-dumped Q-switched lasers do it in a particularly explicit way). My bathroom is near enough a cube that it has a distinct resonance in a note somewhat below C-below middle C: If I can find this note and hum it softly, soon the sound level in the bathroom greatly exceeds the sound level of my hum. Organ pipes do this (accumulate sound). My daughter’s flute does this — blow in it all you want, but the only strong sound coming out will be the one where the flute is resonant — that is, where it accumulates the sound. Rogue waves (on the ocean) are thought to be an accidental combination of many smaller waves. Any resonant cavity or device can be driven to a higher intensity than the input intensity. A number of my patented and tested inventions depend on this principle.

    HOWEVER: All these examples are single-frequency devices (with the exception of rogue waves, perhaps, which we don’t really understand). Any attempt to accumulate waves over a broad frequency band fails due to (as Graeme says) “waves interfering with each other and neutralising each other”.

    To put this in “Engineering-Speak”: You cannot have a high-Q cavity (or resonant circuit) with a broad bandwidth. This doesn’t mean that you can’t store broad-band sunlight — just that you can’t store very much.

    How is this related to the GH effect? Just that the GH effect isn’t impossible because it’s impossible to “store radiant energy”. There are lots of other un-confirmed assumptions and flaky calculations associated with the GH effect that we don’t need to claim it’s impossible for the wrong reasons.

    00

  • #
    Myrrh

    That’s not really storing it though, is it? It takes a continual note to create that amplification which will stop and run down when the imput stops. Which is the problem with the AGWScience thing about photons increasing the heat ‘by amplification’, making the photons ‘continual’ makes them ‘eternal’.

    00

  • #
    Myrrh

    If you’ve ever been to a production, or just visited and tried it out, but the acoustics in the ancient open air theatres are brilliant. Someone talking quite normally ‘on stage’ can be heard perfectly clearly at the top steps, with no loss of quality. A whisper on stage is heard as a whisper in the gods.

    00

  • #
    Paul

    The diagrams that purport to show the ‘Greenhouse Effect’ claim that the ‘back-radiation’ is absorbed by the surface. See Earth’s Global Energy Budget and page 4 of the PDF of this paper.

    Fig. 1. The global annual mean Earth’s energy budget for the Mar 2000 to
    May 2004 period (W m–2). The broad arrows indicate the schematic flow of
    energy in proportion to their importance.

    There are two things wrong with these claims.

    1) The first is a scientific matter. While it may be true that the warm air and clouds radiate energy that is detectable using a radiative temperature-sensing device, this does not equate with a transfer of energy. Since the surface is warmer than the air and clouds all the energy that is transferred will be from the surface, not to the surface. The energy flux from the surface being greater than the opposing energy flux from the air and clouds will transmit a positive but reduced flow of energy upwards.

    2) The second is a propaganda matter. By showing the back-radiation at an ostensibly higher value than that of down-dwelling radiation from the Sun, this lends credibility to the fiction that atmospheric CO2 is responsible for much of the surface temperature and that therefore a slight increase due to the use of fossil fuels is likely to have a catastrophic effect on the climate.

    The thing that dominates this diagram is the long-wave Surface Radiation which at 396 W/m^2 is shown as nearly two and a half times more ‘significant’ than the radiation received at the surface from the Sun, at 161 W/m^2 and is almost matched by the 333 W/m^2 of ‘Back Radiation’ which is asserted to be “Absorbed by Surface”.

    If, instead, the diagram had correctly shown Surface Radiation at a net figure of 63 W/m^2 of which 40 W/m^2 is directly emitted to space via the Atmospheric (frequency) Window, leaving 23 W/m^2 to be absorbed by the atmosphere and clouds, then the role of atmospheric CO2 as a trace component in the atmosphere would, quite obviously, be limited. In that case it would appear to be difficult to conjure a major international scare from the data.

    By removing the element of ‘Back Radiation’ and arriving at a net of 23 W/m^2 of radiation from the surface to be absorbed by the atmosphere rather than the 356 W/m^2 shown in the diagram, they can be seen to exaggerate the potential effect of an increase in atmospheric CO2 by a factor of over fifteen times.

    I can see absolutely no justification for this kind of ‘adjustment’ and presentation of the data. And isn’t this situation just what the ‘Second Law of Thermodynamics’ is all about?

    But the problem is that most people either do not have the capability or the interest in the science to evaluate the claims and so the propaganda sticks in their minds leaving them vulnerable to the manipulation intended.

    Paul

    00

  • #
    Graeme Bird

    “That’s not really storing it though, is it? It takes a continual note to create that amplification which will stop and run down when the imput stops.”

    Yes Bob surely? What you are talking about must be a more or less instantaneous effect right? At least in human terms, these sorts of things cannot last long once the lights go out?

    00

  • #
    Myrrh

    OK, here’s the thing.. shock, horror. I think the blackbody stuff is nonsense.. At least, I can’t find any explanation that makes sense to me. This the idea that it has anything to do with capacity to absorb light appears to me to come out of nowhere.

    I’ve got too many windows open and my computer is struggling, but I’ll try to give examples, may take a day or two.

    Paul – I’m also struggling because my pdf isn’t working properly and I can’t get the paper up to see the diagram. I see it’s a 2009 update from the KT97, and it says that downwelling ir has now been included to account for some misbalance in the earlier paper. I can’t follow where you get the 23W/m^2.

    00

  • #
    Paul

    Myrrh:
    June 26th, 2011 at 6:13 pm

    Paul – I’m also struggling because my pdf isn’t working properly and I can’t get the paper up to see the diagram. I see it’s a 2009 update from the KT97, and it says that downwelling ir has now been included to account for some misbalance in the earlier paper. I can’t follow where you get the 23W/m^2.

    Surface Radiation 396 W/m^2, less
    Back Radiation 333 W/m^2, equals
    Net [Actual] Radiation 63 W/m^2, less
    Atmospheric Window 40 W/m^2, equals
    Surface Radiation absorbed by the Atmosphere 23 W/m^2

    So if atmospheric water vapour (95%) and carbon dioxide (4%) ‘inhibit the loss of heat by radiation, the question is “by how much?”
    Global Warming: A closer look at the numbers

    And if the increase in atmospheric carbon dioxide contributed by burning fossil fuels is 4% of the total, the next question is, “How much extra heat will be retained due to the human contribution?”

    Paul

    00

  • #
    BobC

    Graeme Bird:
    June 26th, 2011 at 5:25 pm
    [Myrrh:“That’s not really storing it though, is it? It takes a continual note to create that amplification which will stop and run down when the imput stops.”]

    Yes Bob surely? What you are talking about must be a more or less instantaneous effect right? At least in human terms, these sorts of things cannot last long once the lights go out?

    Well yes. But I never thought of defining “storage” as “permanent storage”. Take a damed reservoir, for instance: If the input stream is stopped and some output remains (even if only evaporation), eventually there will be no water left in the reservoir — so even that isn’t “permanent”.

    For my bathroom, the storage time might be several hundred milliseconds (could be determined with a fast recorder — when I stop humming, the sound level should drop exponentially with a time constant of a few dozen or more milliseconds; related to the time it takes for a sound wave to traverse the room.)

    The most dramatic “storage” effect I have personally experienced was walking in the Grand Canyon between two large (100+m) alcoves eroded into opposite canyon walls. The echos were so strong that, if you walked at the right place, it sounded as if there were several people walking right behind you. It took nearly a second for the “phantom footsteps” to die out after you stopped. A yodel let off here had dramatic effects.

    But, yes, I’m talking about very short “storage times” that don’t have much relation to the GW effect (if it even exists):
    For an etalon, the “storage time” is only a few dozen nanoseconds, at the most. For all practical purposes, this sounds identical to “no storage”, but there are useful effects (in lasers and optical filters) that depend on this miniscule “storage time”. To a light wave, this is significant — an etalon might delay a light pulse for several feet, even though the physical thickness of the etalon is only a millimeter or so.

    So, yeah — I can see how these effects seem irrelevant to normal human time scales. (Unless you are reading this over a WDM fiber optic line — very likely, if you are far enough away — and then you are depending on just these effects in the wavelength selection filters used to route the signal 🙂 )

    So, to reiterate: I no of no way to “store” light waves that would be detectable at normal human time scales. The hypothesized GW effect doesn’t really store waves, but is more related to radiant heating methods.

    (Perhaps working with light for so long has warped my perception of what is a significant time scale 😉 )

    00

  • #
    Graeme Bird

    “OK, here’s the thing.. shock, horror. I think the blackbody stuff is nonsense.. ”

    You bet its nonsense. Its not the least bit analogous to what is going on. For starters we are talking about real AIR temperature. Whereas they are talking about hypothetical SURFACE temperature. Two entirely different things.Its so unbelievable because its not even an air temperature model.

    We are talking about real air molecules. They are talking about hypothetical phantom molecules that stay fixed in place, don’t bump into each-other, and simply radiate backwards.

    And in a way they aren’t even talking about that. Because the calculations they have made, of the temperature these phantom molecules contribute, is based on an anomaly where its simply assumed that the greenhouse gases are responsible for this anomaly (or most of it) which is clearly wrong.

    How they can just simply decide to substitute air temperature for surface temperature???? Intellectual paralysis. And it doesn’t comport with the reality. The reality is that its where the air is thin thats when the surface temperature heats up a bunch on those hot days. Up in the Atherton tablelands in summer, you walk on the road and you’ll burn your feet. But you step in the shade and its fine.

    Forget the tablelands. How about the Moon!!!!! Always its where the air is thin that you get this terrible heating at the surface.

    Nothing about this jive shows any willingness to compromise with the actual planet we are on.

    00

  • #
    Myrrh

    NASA in Shock New Controversy
    http://sppiblog.org/news/nasa-in-shock-new-controversy#more-1804

    NASA covered up for forty years proof that the greenhouse gas theory was bogus. But even worse, did the U.S. space agency fudge its numbers on Earth’s energy budget to cover up the facts?

    As Per my article this week, http://www.suite101.com/content/apollo-mission-a-giant-leap-to-discredit-greenhouse-gas-theory-a241363
    forty years ago the space agency, NASA, proved there was no such thing as a greenhouse gas effect because the ‘blackbody’ numbers supporting the theory didn’t add up in a 3-dimensional universe:

    “During lunar day, the lunar regolith absorbs the radiation from the sun and transports it inward and is stored in a layer approximately 50cm thick.. in contrast with a precipitous drop in temperature if it was a simple black body, the regolith then proceeds to transport the stored heat back onto the surface, thus warming it up significantly over the black body approximation..”

    A Greenhouse Effect on the Moon? by Alan Siddons, Martin Hertzberg & Hans Schreuder
    http://climaterealists.com/?id=5770

    A Greenhouse Effect on the Moon? May 2010 http://www.ilovemycarbondioxide.com/pdf/Greenhouse_Effect_on_the_Moon.pdf

    Apollo Mission: A Giant Leap Contradicting Greenhouse Gas Theory John O’Sullivan

    http://www.suite101.com/content/apollo-mission-a-giant-leap-to-discredit-greenhouse-gas-thoery-a241363

    00

  • #
    Paul

    I draw your attention to a new study that discredits the Greenhouse Gas theory, showing that atmospheric CO2 acts as a coolant.

    ‘Coolant’ Carbon Dioxide

    In his new paper, ‘Determination of the Total Emissivity of a Mixture of Gases Containing 5% of Water Vapor and 0.039% of Carbon Dioxide at Overlapping Absorption Bands’ the Mexican biologist turned climate researcher proves that in nature, CO2 and water vapor mix together to decrease infrared radiation emissions/absorptions in the air. This is the opposite of what conventional climatology has been saying for years.

    Read more at Suite101: Greenhouse Gas Theory Discredited by ‘Coolant’ Carbon Dioxide | Suite101.com http://www.suite101.com/content/greenhouse-gas-theory-discredited-by-coolant-carbon-dioxide-a365870#ixzz1QWUf5rG4

    The PDF of the study may be downloaded from http://www.biocab.org/Mean_Free_Path.pdf

    _____________________________________

    Then there is the older paper discrediting the ‘blackbody’ assumption as the starting point for estimating the Earth’s surface average temperature.

    Apollo Mission: A Giant Leap Contradicting Greenhouse Gas Theory

    http://www.suite101.com/content/apollo-mission-a-giant-leap-to-discredit-greenhouse-gas-theory-a241363#ixzz1QWVcRLxF

    Published online on May 24, 2010, the study argues that the flaw has always lain in Stefan-Boltzmann’s equations. The long-trusted formula has been used by climatologists without question – until now. The researchers report that the numbers used in those equations are the “first assumption that climate science makes when predicting the Earth’s temperature.”

    NASA Abandoned Flawed Climate Calculations in 1960’s

    Siddons, Hertzberg and Schreuder were astonished to find that “the principal method for predicting a planet’s temperature is surprisingly arbitrary and simplistic.” That was, they believe, why NASA needed to set aside the blackbody equations when doing their own calculations for the Moon landings.

    So now just about anything and everything underpinning the CAGW scam has been discredited. How much longer can this scam hold onto political power?

    Paul

    00

  • #
    L.J. Ryan

    One more for consideration: Mathematical Physics of
    BlackBody Radiation
    .

    The author, Professor of Applied Mathematics KTH,Claes Johnson is challenging the “consensus” of atmospheric forcing, the misconceptions radiative energy transfer and existence of the photon.

    00

  • #
    Graeme Bird

    The reason why NASA would cover this one up, more than just out of habit (N.EVER A. S.TRAIGHT A.NSWER)is because they had the dogma that all planets are around 4.5 billion years old.

    Any scattering will cool. Any scattering. Or the exceptions will be special and of note. The scattering on Venus won’t cool since the heat is from the inside out. NASA involved itself in a tendentious argument to prove that Venus wasn’t a new planet. But Venus must be a new planet and one way we know this is its atmosphere.

    How much of an atmosphere a planet can hold is worked out via gas laws, temperatures and escape velocities. And the people who work these things out find out that Venus cannot possibly sustain its atmosphere at its current temperature. Which means it must have gotten its atmosphere when it was further out from the sun. We also know indeed that Venus is casting off its atmosphere.

    If Venus isn’t cooling down then it would be the case that there is fission going on which is not going to last forever.

    I don’t think we can beat this lie unless we come out full steam against the lie that Venus is an old planet. The one lie has created the other. Its only the thought of a risk of some sort of upward heating spiral to Venus that can keep this idiocy going.

    You cannot be shy about this. The convergent evidence proving Venus as a new planet is in. The bad scientist who tried to say otherwise was Carl Sagan.

    00

  • #
    BobC

    Graeme, Ryan, Myrrh, and anyone else interested in whether there is any evidence of the “Greenhouse Effect”:

    Here is an interesting paper by the Friends of Science Society, published on the web (gasp! No “peer review” — guess people like KR will have to read it to find out what it says).

    They simply start with the GH claim that the Earth’s surface emission of IR is greater than the emission of the upper atmosphere into space, due to the GH effect. Then they set out to measure it.
    Here is a summary of what they are looking for:

    The Anthropogenic Global Warming (AGW) theory postulates that greenhouse gas emissions would cause a reduction of out-going longwave radiation (OLR) from the top of the atmosphere for a given global average surface temperature. This reduction of OLR would cause an energy imbalance with the incoming solar radiation resulting in a
    “forcing”, meaning more incoming energy than out-going energy, causing an increase in global temperatures. Increasing temperatures would cause OLR to increase until the radiation balance is restored.
    The greenhouse effect results in the upward surface radiation (Su) being greater than the OLR. The global surface temperatures are directly related to the surface radiation. As global warming may be caused by factors unrelated to greenhouse gases, the strength of the greenhouse effect is characterized by the fractional change in the radiative flux between the top and the bottom of the atmosphere, defined as the normalized greenhouse factor (NGF):

    They use satellite measurements and also analyze thousands of radiosonde measurements (using a radiation transfer model). It’s their results that I find interesting:

    Using radiosonde data from 1960 to date and a line-by-line radiation code, the normalized greenhouse factor was calculated to have increased by 0.19% over 49 years, which is not significant. Climate sensitivity at doubled CO2 concentration was calculated to be 0.26 deg C. This is less than 9% of the 3.0 deg C estimate by the International Panel on Climate Change (IPCC).

    Note that they measured the GH effect, so there is no room for modifications due to hypothesized feedbacks — all possible feedbacks are taken into account by measuring what actually occurs.

    00

  • #
    Bobc

    No idea why my avatar quit working in #607 — but it’s me

    [there was an “r” missing when you typed your e-mail address. WordPress likes perfect typing when you log in. I fixed it.]ED

    00

  • #
    Paul

    The link for that study is at Out-going Longwave Radiation and the Greenhouse Effect and is in PDF format.

    While I do not fully subscribe to the idea that one can calculate the radiative properties of the atmosphere by treating it as follows

    longwave absorption and radiation fluxes
    across the longwave spectrum utilizing 3490 spectral intervals, 150 atmosphere layers,
    9 directional streams and 11 greenhouse gases.

    yet that is indeed what all the IPCC models assume.

    Hence this analysis can be seen as demonstrating that on their own assumptions human influence on the climate would be minimal and by no means catastrophic.

    Paul

    00

  • #
    Myrrh

    Ryan – thank you for the Claes Johnson link, the history is fun…

    I did spend a bit of time arguing that Einstein’s gravity was silly, the bendy space creating gravity couldn’t work in 3D so another cartoon treated with awe instead of seen as a joke, and I’d come across Einstein’s uncertainty about his ideas while researching this.

    But anyway, what is it with all these arguments, the fixation on high energy converting to lower energy heat completely oblivious to the reality that high energy, the shortwaves, don’t actually do this doesn’t seem to matter at all.

    The blackbody obviously doesn’t work in a dark room..

    I was reading the wiki page on it and it said black velvet was close to a perfect blackbody, so what about white velvet? S’far as I know they both burn with visible light.

    And there was something about the Japanese and graphite which they said was even closer to blackbody perfection absorbing light, does graphite even burn?

    Looks like I’m going to be some time engaged in getting through the links and info provided by you all, many thanks.

    00

  • #
    BobC

    Paul @ 609: That is the beauty of this paper — they take as a starting point all of the IPCC’s theory, then devise a way to measure the predicted effects, and show that CO2 has a non-detectable effect on climate — all by the IPCC’s own theory!

    I don’t expect gov’t scientists to start rejecting their grants en mas, however. People will go along with anything for money and security (some people, anyway — and the gov’t has been selecting for just those scientists for some time now). In this morning’s paper, a number of local “scientists” from the government labs in Boulder were ramping up the hype: CO2 increasing faster than expected! Now we know extreme weather really IS caused by AGW! Sea levels accelerating? It was sickening. I can’t wait for these bozos to get defunded.

    Oh, and NOAA spokespeople were saying that, if they lose any funding, it will impact their ability to continue short term weather prediction! Gotta keep that useless climate research going, no matter what, even if they have to cut the only useful thing NOAA does.

    I say, disband them and let the private sector take over commercial weather prediction — let the best predictor win!

    00

  • #
    Paul

    BobC @611

    I agree, but unfortunately we’d have to disband most scientific organisations, most Universities, and even the school system and change most governments before this scam can be excised. Not to mention a host of other organisation peddling this misinformation. And there would need to be a clean-up of the internet where thousands of web sites proclaim as undisputed truth the nonsense of AGW.

    I was watching some interesting videos on http://www.teachersdomain.org and the indoctrination is easily seen. eg Global Warming: The Physics of the Greenhouse Effect

    Another interesting clip was at Extreme Temperatures on the Moon where it is said that without an atmosphere, in full sunlight, objects can reach 200 degrees Fahrenheit but something in the shade nearby can be as low as -120 degrees Fahrenheit. Somehow I don’t think that that equates to an average of -18 degrees Celsius, does it? Also, perhaps the value of conduction and convection here on earth might be a little more valued than the IPCC seems to!

    Paul

    00

  • #
    Graeme Bird

    This doctrine is so very silly. It never gets off the ground. Take the idea of a black body. Twice as far from the sun as earth is and always noon. Now it ought to be very clear from that starting point that any atmosphere is going to take the edge off the temperature of the black body. It will cool the black body temperature. There is really no doubt about this. So even the core of the model has no apriori basis.

    No part of this story isn’t a jumble of contradictory ideas. So why do skeptics always say that “a doubling of CO2 will lead to a 1 degree increase in temperature in the first instance. This is standard radiative physics”

    No it isn’t its just nonsense. Or take the idea of a laser at the back wall, shining on the front wall. Then have a glass tube of CO2 in the middle. And the laser light is one of the three narrow wave-length bands that CO2 scatters/absorbs.

    Well the test-tube will get hot. And so what? It will do so at the expense of the back wall. And that it will do so is as much to do with the insulative properties of glass as anything else. In this story the back wall is the earths surface. So the CO2 will cool the back wall. But the greenhouse story expects the back wall to heat up the CO2 in the tube which will then further heat the back wall. Its all so very very stupid .

    Apart from the globalist propagandizing what makes this plausible to some people is just the confusion that the failure to understand Venus as being hot from the inside out ….. the confusion that this has caused.

    If the heat on Venus comes from the outside in, then the Venus situation is simply incomprehensible. Its this incomprehensibility that the believer has tried to re-tune his mind too. He cannot really “get it” but he’s so confused he starts babbling about risk and insurance.

    Imagine Venus was heated from the outside in and its sunward side had something like the temperature profile it has now? What would its night-time side be like?

    The answer is that it would be very cold. Because the night lasts a very long time on Venus and CO2 doesn’t have great heat retention capacity. So the ground would cool down very quickly. But you’d get overhead temperature reversals like you sometimes get at the South Pole. Hot air blowing in from above but it would be very cold on the ground.

    Greenhouse cannot explain Venus. But Venus haunts this greenhouse story by making matters impossible to comprehend. And this is simply because NASA, Carl Sagan and a bunch of dopey astronomers took a tendentious, tribal, and anti-scientific view, towards what was going on on Venus.

    If we get back to reality, and to standard science that can be repeated in the lab. If we accept that Venus is hot from the inside out, then it all makes perfect sense. The science of it all becomes perfectly comprehensible again. And the second law of thermodynamics works just fine again like it always did.

    00

  • #

    The several of you who have been commenting in these last 30 posts or so…

    It is really rewarding for me to see you guys discussing the things you are and in the way that you are. I promise you that my next paper, which is going through internal peer-review and editing with the Slayers and should be released within 4 weeks, will expose the fraud completely once and for all. There are fundamental mathematical, physical, and logical contradictions in the model greenhouse atmosphere, which are exposed, and which can be experimentally proven, and I have explained in detail the experiments which can be performed and replicated by anyone. Keep an eye out for the paper…I will post a link here.

    00

  • #
    Baa Humbug

    Postma: #614
    July 8th, 2011 at 12:32 am

    I look forward to it, thnx Postma

    00

  • #
  • #
    BobC

    A (belated) comment on photons:

    After the comments on photons (which I think are simply products of our imagination) on this thread, I have re-looked at some previous literature. It is clear that a great many physicists actually believe in their reality. Even Richard Feynman (in his “Lectures on Physics” ) claims (with Bohr) that the “particle” and “wave” aspects of light exist, but only the one you look for “exists” at that time.

    This rather bizare metaphysical conclusion leads him to confidently claim that, in the famous “double-slit” experiment, anything you do to determine which slit a “photon” came through will destroy the existance of the interference pattern between the two slits.

    HOWEVER; A recently performed, simple experiment (which could have been done anytime in the last 70 years) shows that Feynman, Bohr, and all the other physicists who have repeated this claim are wrong — the interference pattern is not affected by having separate detectors record the output from each slit.

    This is, of course, exactly what you would expect from the known facts which are:
    1) Light propagates as waves (hence goes through both slits and forms an interference pattern).
    And,
    2) Light is absorbed in discrete quanta of energy, so with a weak wave field (no more than one quanta of energy exists in the volume of the apparatus), only one detector can trigger at a time..

    So, why do so many physicists make up photons? I believe the answer is “To avoid the appearance of a ‘non-local’ effect”. Consider the facts: A weak wave impinges on two detectors. The wave is so weak, that a considerable volume of space (larger than the volume of the two-slit apparatus) contains only one quanta of energy. Suddenly, one of the detectors records this quanta, and the wave instantaneously disappears.

    Physicists (because of the legacy of Einstein) are highly allergic to any description that contains the term “instantaneously”, so they assume that the energy is “really” carried by small particles which don’t require the instant disappearance of a large volume of wave when absorbed.

    As a “fix”, this doesn’t work, however: Now, you have to assume that the hypothesized photon “knows” whether the slit the photon doesn’t go through is open or not — simply a different “non-local” effect. (See the referenced Wiki article above.) The ‘fix’ doesn’t work. Even the “Many-Worlds” interpretation of Quantum Mechanics is postulated to avoid small non-local effects by postulating the greatest non-local effect possible — the creation of myriads of parallel universes every second.

    The fascinating thing is: Why don’t all these intelligent scientists recognize that these kind of dodgy “explanations” are the logical equivalent of the famous “Turtles all the way down” explanation that they all make fun of?

    Intelligence, apparently, is no guarentee that you won’t fool yourself. Better just to accept what the World tells you and quit trying to make it fit your preconceptions.

    00

  • #
    Gary

    CERN ‘gags’ physicists in cosmic ray climate experiment
    What do these results mean? Not allowed to tell you

    18th July 2011 12:01 GMT

    The chief of the world’s leading physics lab at CERN in Geneva has prohibited scientists from drawing conclusions from a major experiment. The CLOUD (“Cosmics Leaving Outdoor Droplets”) experiment examines the role that energetic particles from deep space play in cloud formation. CLOUD uses CERN’s proton synchrotron to examine nucleation.

    CERN Director General Rolf-Dieter Heuer told Welt Online that the scientists should refrain from drawing conclusions from the latest experiment.

    “I have asked the colleagues to present the results clearly, but not to interpret them,” reports veteran science editor Nigel Calder on his blog. Why?

    Because, Heuer says, “That would go immediately into the highly political arena of the climate change debate. One has to make clear that cosmic radiation is only one of many parameters.”

    00

  • #
    Graeme Bird

    This is the sort of thing that happens when an entire field of study is caught up in cult hero-worship, with each practitioner feeling that the idolatry has rubbed off on himself. They don’t like what they see. They don’t feel the need to inform mere mortals about the embarrassment of it.

    00

  • #
  • #
    Graeme Bird

    Science at last. Oh Lord how long has it been.

    00

  • #
    Paul

    Gary:
    July 20th, 2011 at 5:38 pm

    CERN ‘gags’ physicists in cosmic ray climate experiment
    What do these results mean? Not allowed to tell you

    This is a real ‘Inconvenient Truth‘.

    It might cause catastrophic damage to their carbon tax aspirations!

    Paul

    00

  • #
    TonyM

    As I can’t find any reference here am advising that Joe Postma’s latest paper is available (release July 22nd). Sorry if I’m repeating the news.
    Title is:
    The Model Atmospheric Greenhouse Effect
    can be sourced from ( and others):
    http://www.tech-know.eu/uploads/The_Model_Atmosphere.pdf

    00

  • #
    Joe Lalonde

    Jo,

    This is where science has me laughing on the floor.
    Simplistic LAW that does not look at all the events around it such as motion. How do the objects move together? What pressure is it under? Is gravity a factor? Friction can generate heat or inhibit movement. Molecular density. Cold molecules are heavier and slower than warm molecules vibrate. Cold compression can change gases to a liquid. Superheated compression has the ability to generate a much denser compression due to the vibrating molecules can compact tighter.

    The stretching of an atmosphere is by warm molecules vibrating. What happens when these molecules cool? The atmosphere is still stretch and takes a long time to come back into the normal space it occupied.

    00

  • #
    TonyM

    Hello Jo, great site you have.
    Not sure where to post this as it is an issue for Joe Postma and his new paper.
    Great paper from Joe Postma; am still digesting it all!

    But,using Jo Nova’s prudent suggestion that we get it right, I would question :
    ” increase in GHG’s will increase the temperature of the bottom of the atmosphere,
    while decreasing that at the top”
    based on
    “atmosphere essentially fixed in depth” (p16) which is a follow on from “this ensemble is bounded at the bottom and top by the Earth’s surface and the top of the atmosphere” (p7)

    The paper then goes on to conclude that a GHG warmer ground T would then need a cooler top of atmosphere T and hence result in faster (-ve) lapse rate. Faster will refer to the absolute value (e.g. -9 is faster than -5).
    I can’t see a reason to allow such a constraint of fixed depth when the atmosphere is free to expand.

    Hence one would question the conclusion that the lapse rate needs to be faster (-ve) to offset the larger gap between the warmer T closer to the ground and colder T at the top. What if the warmer T at the bottom simply causes expansion of the atmosphere with a lower lapse rate and just pushes the T’s further out in height?

    By relaxing that assumption of a fixed depth it can be shown that lower lapse rates can result from higher ground Temp readily – just expand the atmosphere.

    I wonder whether the water vapour is a good example of a lapse rate argument given that it would undergo some type of phase change somewhere on the T gradient and release heat – hence its much lower lapse rate.

    Postma argues that due to water vapour GHE “the lapse rate should already be faster than -10K/km because there is (ostensibly) already a GHE in operation in the atmosphere” This conclusion again derives from the assumed constraint of fixed height of atmosphere. Again, I don’t agree.

    In any case, one could argue that the observed lower lapse rate with water vapour shows that it functions as a GHG and caused the observed warming and also resulted in a lower lapse rate compared to an ideal gas. In other words it becomes a de-facto proof of the GHE hypothesis. And it achieved this by expanding the atmosphere.

    This can be shown to occur without altering the amount of energy emitted by earth i.e. the -18 deg C average emitting T height just shifts to some other height.

    BTW: Fig 8 needs to be corrected at the of the top of troposphere. I make it 11.8 km if T is -55 deg C (or -105 C at 18.5 km) and not -55C and 18km which is shown..

    00

  • #

    Yes I don’t want to bother Jo with this and turn her forum into a discussion of my paper. Using the climate-realists link

    http://climaterealists.com/index.php?id=8119

    would be a better place.

    As for your comments Tony…much to ponder over all that. There are lots of possible solutions to what you brought up and for now it seems quite convoluted (not saying YOU are convoluted…just that all the possibilities are convoluted).

    What I intended to point out, was that the “lapse rate” comes from a fundamental equation of conservation of energy…a very simple one in fact. The only terms in the equation are the strength of gravity, and the weighted-mean thermal capacity of the gas. This value matches what meteorology derives for adiabatic parcels of air. Then, if there was temperature amplification in the atmosphere from a GHE, that should increase the lapse rate. Would the atmosphere expand at exactly, more, or less than the lapse rate? Well, for small changes in T, not likely measurably in any case, because the expansion in volume is in 3 dimensions…the linear increase in atmospheric depth would therefore be quite small and still accommodate a VERY large expansion in volume. I.E. the temperature could change linearly but the depth would change only as the third-root. In that case you would still expect an increase in the gradient rate vs. depth.

    I am copying TonyM’s comment and my response over to the Climate Realists link so that discussion can continue there if desired. I don’t want to bother Jo and thank her for letting me reference my paper here at all in the first place.

    On the other hand perhaps Jo wants the traffic…ahhh, what is the proper etiquette here!!??

    Will copy to CR in any case, for now. If Jo does or doesn’t like that she can let us know.

    00

  • #
    TonyM

    Hello Jo: Not sure of your your thoughts so will post here at least to round off the issues.

    Joe Postma:
    When I wrote that you reach a certain conclusion on the lapse rate I may have, inadvertently, given readers the impression that your paper was confined to the area where I raised that issue above. This is not so and it is not the main part the paper. But it is definitely an issue.

    I encourage anyone to read the whole paper as it delves very neatly and clearly into a quite different way of analysis using astro-physics models in a very straight forward, insightful and far more robust way to analyse the issues of earth atmospherics than nebulous models we seem to be given in climatology.

    Whilst I accept your comments that the issues I raised can be convoluted my thoughts were to try for a first pass model to raise them. Let’s assume something about GHG – that they can retain some energy from inputs, say radiative, either manifested as extra kinetic energy (T) or internal (rotational etc) such that they do release it at some stage (say up to 10 mins). Some of this energy release will increase the KE (T) of other gasses and hence the atmosphere.

    Further I assume the GHG is spread throughout the atmosphere. If so then I suggest there is a continuous manifestation of some extra kinetic energy, and hence extra T, right through the atmosphere, because of this GHG – at any layer compared to the absence of such a gas. But I do suggest some asymptotic limits apply as under direct heating this effect would not be noticed very close to the ground at least – i.e. it won’t increase the max T of the day say under direct sun.

    Ultimately, I don’t think the physical mechanism of molecular heat retention/distribution is so important for the analysis as it becomes embodied in the lapse rate which is directly observable. Also we are not trying to model all aspects of the atmosphere with clouds etc but trying to make a first pass comparison against an ideal gas atmosphere.

    Given this, I would use your model in fig 8 – basically the adiabatic expansion model. In effect this extra heat would expand the top of the atmosphere with the result of having a higher T right up the column without compromising the 1st Law and avoids too close a need to consider the volume increase as this happens automatically.

    This should manifest itself in the lapse rate. But, I suggest, it lowers that lapse rate and would also show up as a higher thermal capacity (C) of the GHG/air mix in your equation 32 i.e. Lapse rate = – g/C ( where g is gravity acceleration) derivation from the 1st Law of Thermodynamics.

    00

  • #
    TonyM

    Joe Postma:

    One of the variables that was avoided earlier is that there is a continual input of heat from radiation so the adiabatic assumption is compromised.

    Judging by your illustration model where 77% of radiation from the ground could be absorbed then that would represent quite an input of heat and hence develop severe deviations from the theoretical adiabatic model.

    I guess that would contribute greatly to the convolutions.. 🙂

    00

  • #

    Hi TonyM,

    Okay let me summarize a few points here: First, the atmosphere could not increase ind depth faster than the temperature increase. Depth would only increase as the cubed-root of the temperature increase, and therefore, the lapse rate (delta T over delta depth) would increase in magnitude, if there were temperature enhancement beyond the basic equilibrium state.
    Secondly, the “adiabatic model” of Figure 8 is far too simple of a way to illustrate the physics, as I pointed out in the text below it. It is a way of looking at it, sure, but it convolutes too much of the unique physics occurring over day and night separately. As you know, when things become convoluted, you can not deconvolve them, generally. Especially in a case like this.
    Also, just to be sure, the adiabatic model assumes a constant input of energy such as to establish the equilibrium with, so that actually isn’t a problem.

    The internal physics of the system is not something I wanted or needed to focus on. It was much more important to discuss the issue of boundary conditions. My Figure 6 was a summary of the actual boundary conditions that exist, and these are quite different from the standard GHE model. With physically real boundary conditions established, it thus becomes much easier and much more physical to discuss the issues I presented on pp11-12 – that the day-time doesn’t actually achieve the temperature of the solar insolation because energy gets absorbed into other degrees of freedom.

    Maybe this is a good way to summarize my paper:

    The GHE models the Earth without a night-time. The night-time obviously is where most of the cooling occurs, since there is no input heating.

    In order to mathematically compensate for not including the cooling of the night-time, the model reduces the solar heating intensity by a factor of 4, thus reducing it to -18C.

    At that point, the model cannot explain why it is ever warmer than -18C anywhere, and so it invents a greenhouse effect to raise the temperature.

    However, the original error was in not including the night time in the model in the first place. If you actually include the night-time and day-time individually, then you can use the physically real actual solar heating intensity which is +121C at maximum, and +30C on average. These are the REAL, ACTUAL values of the solar heating, and then you don’t need to use the mathematically & artificially low value of -18C.

    And then you also never need to put yourself into the situation of scratching your head over why the Sun feels so warm!

    This relates to the issue of convolution we’ve touched on: if you convolute the day and night, you miss out on lots of physics because you can’t “see” it anymore. If you keep them separate, then you can see.

    Alan Siddons recently summed it up as this:
    “When a recipe says bake for an hour at 200°C, a climatologist assumes that four hours at 50 degrees will produce the same result.”

    In other words, will baking your Turkey from all-sides at 50C for 4 hours produce the same result as baking it on one-side-only at 200C for one hour? Surely it will not.

    In the same way, baking the Earth from all-sides at -18C will NOT produce the same physics as baking the Earth from one-side at +30C (with a maximum of 121C around the solar zenith).

    I thought Alan’s summary was just perfect.

    00

  • #

    What we have are human efforts to best explain the world around us.

    00

  • #
    TonyM

    Hi Joe Postma:

    You are perfectly right in that volume alone will not account for the effect. Perhaps the whole V+P+T relationships with some higher equilibrium T might but, it still will not give more fruitful insights IMHO. The modelling you present is a far better way to work with the issues – and yield a good roast turkey!

    I had, like you, hoped that the elegant lapse formula you presented in that section would yield more but will move on and bounce some thoughts off you.

    Let’s assume that a wet air has no complications with phase changes – so assume ideal gas behaviour. That lapse rate (LR) ought not change irrespective of the column equilibrium T’s in adiabatic conditions. The L.R. would be around – 6.5 deg C per 1 km rise as in your paper.

    By column I mean the whole column say 1 sq km cell on the ground going right up to the top of the troposphere – say 10km.
    Hence the LR will be a constant no matter where it is positioned on the night/day cycle of your model. For now let’s ignore any night/day humidity change.

    The dry LR of a similar column cell in some dry desert conditions will similarly be a constant as again the total air mix without water would be an ideal gas. This would be close to -10 deg C per km.

    From this the most useful observations are those already widely proposed (assuming on the same latitude) i.e. The wet versus dry air:
    1) will have a lower lapse rate i.e. will drop it’s T at a slower rate as we go up height.
    2) day max temp will be lower (most likely)
    3) night max temp will be higher (most likely)

    The last two should be observable from places on a similar latitude at comparable times of the day/night cycle.

    Certainly experience suggests the daily wet T max will be lower than a dry T max. It may happen that the “average” wet T might increase by virtue of the night T being higher for wet air. So am not sure where that leaves warmist arguments as certainly it would not suggest a desiccated earth – rather the opposite where we have a “little more tropical type climate” with “lush” growth. Perhaps there might be some issues for Antarctica ice.

    Now let’s go back to your original question in this section of your paper and couple it with the above. If the GHE effect is to hold then what has been the effect so far of the existing GHG’s? Basically this was your original question which started this discussion.

    From Dr Roy Spencer’s site:
    …..“ Now, you might be surprised to learn that the amount of warming directly caused by the extra CO2 is, by itself, relatively weak. It has been calculated theoretically that, if there are no other changes in the climate system, a doubling of the atmospheric CO2 concentration would cause less than 1 deg C of surface warming. This is NOT a controversial statement…it is well understood by climate scientists. “
    http://www.drroyspencer.com/global-warming-101/

    There is a topic on Jo Nova’s site devoted to the logarithmic CO2 effect. In the following:
    http://www.climate-skeptic.com/2010/01/catastrophe-denied-the-science-of-the-skeptics-position.html?gclid=CIfu-ZWIrqoCFYskpAodVWNhVQ

    there is a formula which comes from some past IPCC reports:

    F(C) = Ln(1 + 1.2C + 0.005C^2 + 0.0000014C^3)

    F(C) would mean Forcing degrees K due to Carbon Dioxide (C in ppm) conc in the absence of any other influences. It seems to be attributed to Michael Mann; must be satisfying to be that accurate!

    Notable results are:
    CO2 from 0 to 380 ppm shows 7.14 deg K increase ( i.e. situation now)

    CO2 from 380 to 760 ppm shows about 1.25 deg K increase in temp (future)

    (from 280 to 380 ppm shows 0.5K increase – interesting for last 60 years)

    The positive feedback (mainly water, methane) is plugged in at anywhere from 2 to more than 7 times the direct CO2 effect for a doubling of CO2 conc.

    It is reasonable to assume that this positive feedback had to hold in the past. So to date, the total GHE would have contributed upwards of 21 deg K . This is over and above the effect of water evaporating naturally before any CO2 is in the system.

    Bizarre conclusion of the formula! Most land mass would be frozen at night. The last 60 years would need to show an increase upwards of 1.5 deg C. – again fantasy land.

    Thought you might be interested in this formula/inputs to examine the implications using your modelling.

    As an aside, in Australia we are about to face the highest per capita tax in the world on a trace gas that supposedly stops us “freezing” over, likely allows higher crop yields in future and is essential for life just because “consensus science” using weird models suggests it is now a “pollutant.”

    10

  • #
    Graeme Bird

    “Simplistic LAW that does not look at all the events around it such as motion. How do the objects move together?”

    [SNIP!]. Okay lets have it then. Tell us where you or anyone else has beaten the 2nd law in the lab, or where you have found in nature that the 2nd law can be beaten. Surely it can happen. So where have you found it. Hey dummy? Where?

    10

  • #
    Graeme Bird

    Look the deal is this:

    The greenhouse theory doesn’t get off the ground since we have absolutely no anomaly to attribute to it, and we can find no place where it actually works …. least of all in greenhouses, and even less on Venus.

    So what is this all about? Find a live anomaly people. Carl Sagan ought never have revived this rotting menace from out of the history of dead ideas. There was no rigor to it at any stage. Only rigor mortis.

    10

  • #
    Bryan

    Joseph Postma.

    Hope this reaches you.
    Thanks for a most worthwhile paper.
    You have examined and falsified many of the pillars underpinning the Greenhouse Theory.
    I would have replied at Climate Realist as you suggest but for some reason posting there turns out to be very complicated.
    There is nothing like the 634 posts here.
    I hope your paper gets a wider circulation it certainly deserves it.
    Perhaps one of the more interactive sceptic blogs will feature it.
    Its sure to get a massive response.
    One part you might like to look at again in on page 15.
    U = CpT + gh
    This I think should read

    U = CpT + mgh

    Even better for an instructive presentation would be to use Specific Heat Capacity instead of Heat Capacity and the equation now becomes.

    U = mCpT + mgh where Cp is the SHC.

    The masses cancel at the next step leaving equation 32 unchanged.

    00

  • #
    Bryan

    Joseph Postma

    I think I know where your equaion originates.
    Jebling and Gilberts papers.

    But note that they explicitly say for a unit mass i.e. M =1 and so does not need further explanation

    U = CpT + gh or upon differentiation dU = CpdT + gdh (1)

    where “CpT” is the enthalpy (or thermal energy) per mass unit, “g” is the gravitational acceleration, “h” is the vertical height and “gh” is the potential energy per mass unit.
    At static equilibrium dU = 0 and equation (1) becomes;

    CpdT + gdh = 0 (2)

    00

  • #

    UPDATE #1 From Michael Hammer

    Michael Hammer has suggested this as a line of reasoning that may help people discuss this. If you don’t agree with the end conclusion (that greenhouse gases can’t warm earth because they are not hotter than Earth), point out exactly which step in the sequence is the one you think is wrong and explain it so we can understand why.

    I’ll try my hand at this.

    1. Do you agree that if you stand surrounded by cold objects (say a ring of huge ice blocks) you feel cold?

    Yes

    2. Do you agree that the colder the object you are surrounded by the colder you feel? eg: if they were blocks of frozen CO2 (dry ice) instead of water ice would you feel colder?

    Yes

    3. OK now if you have been standing surrounded by extremely cold objects and then move so that you are now surrounded by merely cool objects does the move make you feel less cold than you were before?

    Yes but with all due respect so what? Your analogy is one of perceptions. Let me demonstrate..

    Take 3 buckets of water and stand them side by side.
    Fill the left bucket with ice cubes, heat the right bucket to 80DegC and have the middle bucket at body temperature 37DegC.
    Now plunge your left hand into the cold left bucket and right hand into the hot right bucket and leave them there for a while.
    Now take both hands and plunge them into the tepid middle bucket. Does the tepid middle bucket of water feel hot or cold? Which are you going to ask, your lying left hand or deceptive right hand?
    Perception is one thing, reality another.
    Your analogy is inappropriate to the discussion of thermodynamics.

    4. In the absence of green house gases the earth is surrounded by an extremely cold object – outer space at 4K (-269C). Green house gases make the atmosphere opaque at some wavelengths. With these in place the earths surface is in effect surrounded by a merely cool atmosphere instead of the truly frigid outer space. Because the surface is now surrounded by a less cold object than it was before it is less cold.

    Sounds reasonable, but as with the above false analogy, it doesn’t pass the test of reality.
    The Moon for all intents and purposes is the same distance from the Sun as is Earth. The Moon is “surrounded by an extremely cold object – outer space at 4K”, but the side of the Moon facing the Sun is a water boiling ~123DegC. But suppose we were to surround the Moon with a less cold object….say an atmosphere similar to Earths?

    5. Since warm and cold are opposites, less cold is the equivalent of warmer. Surrounding earth’s surface with an opaque cool atmosphere make it warmer than it would be if exposed directly to the ultra frigid outer space.

    See my response to 4 above regards an opaque cool atmosphere making the world warmer.
    Also, saying warm and cold are opposites is meaningless. Zero degrees celcius is a human concept. Negative temperatures Vs positive temperatures is a human concept. If I was to say 230 kelvin (cold) is the opposite of 330 kelvin (hot), I wouldn’t be making sense.

    In summary, steps 3, 4 and 5 I believe are either wrong or inadequate.
    My belief isn’t set in stone and am happy to learn by being corrected.

    00

  • #
    Paul

    I would make an observation about all the above discussion: since even an intelligent and informed person, such as Joanne Nova, has a hazy understanding of the laws of heat transfer, is it any wonder that the general public are easily deceived and led astray by verbal arguments about such matters?

    I am reminded of an occasion when studying in the University library and getting bored, I browsed through some nearby shelves and found a translation of an early Greek discussion attempting to elucidate the movements of a rolling hoop. Having never studied physics I was entranced by the laborious verbal arguments that those ancient logicians used to try and unravel the motions of the hoop. I am sure that even thousands of years later those discussions would remain unresolved, nor would any useful ‘rules’ have been produced that might be useful in the real world.

    I think that the present verbal discussion of ‘Climate Change’ is in exactly the same position. Logic alone will never elucidate the practical knowledge that will allow for verifiable calculations of future near-surface air temperatures of the Earth. The only thing that will do that is properly constructed physical experiments that can be replicated and the results studied leading to quantitative relationships that can be verified. The predictive value of such relationships should then be able to be applied in any situation, local, continental or global.

    That is the strength of the Scientific Method, bypassing and nullifying the almost infinite ability of the human mind to convince itself, by logical arguments, that it understands some physical process.

    It should be informative to the present situation, that the Greek Astronomy, which prevailed for many centuries until the time of Galileo, constructed an elaborate description of the movements within movements of the stars and planets, not based on any known physical processes, though of sufficient accuracy that most knowledgeable people back then simply could not conceive that it was in any degree in error. However, as we all know, it gave way under the onslaught of actual measurements and observations. And once the laws of gravity and motion had been derived from the latter there has been no going back. The complexity of the system of Greek Astronomy is completely analogous with the complexity of the General Circulation Models on which the IPCC relies, which, though they can be fine-tuned to simulate average global temperatures, fail to reflect quantifiable and verifiable physical processes.

    So, my counsel is to forego the interminable verbal, ‘logical’ arguments about how the laws of heat operate to bring about the earth’s surface temperatures and leave the solution of that to experiments, data, and analysis.

    The clincher for me is that I can observe every day changes in near-surface air temperatures that are entirely ignored by the prevailing paradigm of ‘Global Warming’ ‘due to’ the increase of a vanishingly small component of the atmosphere. I have never yet known anyone to observe any part of the globe that is bathed permanently in the sun’s rays. The diurnal process is universal, yet ignored by the paradigm. When my part of the world is bathed in sunlight the near-surface air temperature stays stubbornly below what it would be without the cooling effect of the atmosphere and at night time the bulk of the stratosphere cools slowly, overnight, with a small surface layer that stays warm from the outgoing radiation from the surface. Where, in that scenario, does the ‘second law’ come into effect and how does a small increase – a few parts per million – of atmospheric carbon dioxide change that process so as to bring about catastrophic change?

    Paul

    10

  • #
    BenAW

    4. In the absence of green house gases the earth is surrounded by an extremely cold object – outer space at 4K (-269C). Green house gases make the atmosphere opaque at some wavelengths. With these in place the earths surface is in effect surrounded by a merely cool atmosphere instead of the truly frigid outer space. Because the surface is now surrounded by a less cold object than it was before it is less cold.

    I don’t see how you can call outer space a cold object. It’s a near perfect vacuum, with hardly any matter in the voids of space. (some hydrogen atoms/cm^3)
    The low temperature is due to the measured background radiation from the Big Bang.
    Imo it acts as an almost perfect insulator, effectively blocking conduction and convection as means of heat transfer. This leaves only radiation as a way to shed heat to outer space.

    00

  • #
    BenAW

    at night time the bulk of the stratosphere cools slowly, overnight, with a small surface layer that stays warm from the outgoing radiation from the surface.

    I suppose you mean troposphere iso stratosphere.
    The bulk of the troposphere doesn’t change much in temperature during day or night.
    The lower part (< 1000m) cools pretty rapidly at night in dry areas (deserts), presumably because the earth radiates it's heat to outer space, and thus the air directly above it cools with it.

    00

  • #
    Ted O'Brien

    Surely a large part of the high day/night variation of temperatures in desert areas is due to the lack of dew. With no moisture to condense the temperature just keeps dropping at night.

    We see this during severe droughts in civilised areas.

    00

  • #
    • #
      Paul

      I am surprised that no one has replied to this post since it answers all the questions about Global Warming in simply, logically and with irrefutable demonstration.

      Basically, the trick to getting the entire world worked up over a slight increase in atmospheric carbon dioxide was to build the theory on a two-dimensional model. Like a magician who diverts the attention of his audience so they do not observe his entire action, this model has diverted people’s attention away from the real world into the uncertainties of science-speak, non-physical models and erudite discussions of irrelevant science, such as the Second Law of Thermodynamics.

      All the discussion above is based on the assumption that a two-dimensional model has an explanatory power for changes in climate in the real world. Once the model is made to conform to physical reality by becoming three dimensional, hence allowing for the sun’s rays to impinge diurnally over the earth’s rotating, near hemisphere, rather than continuously over the entire sphere, it all drops out.

      The atmosphere is seen to keep the surface temperatures cooler than would otherwise obtain during daylight hours, with a slow cooling-off during the nighttime hours, the rate of which depends mainly on atmospheric water vapour and cloud-cover. There is no need for a supposed ‘Green House Effect’ attributable to atmospheric carbon dioxide.

      There is left, for the increased atmospheric carbon dioxide, only a vanishingly small effect, confirmed by all real-world measured data. All the speculation on runaway global temperatures can be seen for what they are; hogwash.

      I think that this paper deserves to be highlighted with its own article on this site.

      Paul

      10

  • #
    BenAW

    Surely a large part of the high day/night variation of temperatures in desert areas is due to the lack of dew.

    Imo it’s due to a lack of water(vapour) in the atmosphere. This way earth can radiate freely to outer space. Backradiating CO2 doesn’t seem to be doing much here.
    Cloudcover prevents this radiating.

    00

  • #
    Graeme Bird

    “Surely a large part of the high day/night variation of temperatures in desert areas is due to the lack of dew. With no moisture to condense the temperature just keeps dropping at night.”

    Yeah there is just not enough talk about specific heat capacity, latent heat of phase changes, and all that sort of thing. What the laymen thinks he understands about the alleged greenhouse effect, is in reality, all, or nearly all, to do with the fact that water appears in abundance on our planet in all three of its phases.

    10

  • #
    Graeme Bird

    “The low temperature is due to the measured background radiation from the Big Bang.”

    Thats all bullshit. There never was any big bang, nor any evidence for a big bang, and the alleged cosmic background radiation constitutes no evidence for a big bang, since the advocates, here clutching at straws, cannot come up with any argument as to why space would ever be zero degrees Kelvin.

    10

  • #
    Paul

    J Postma @#641
    October 12, 2011 at 1:02 am
    Most of you will enjoy this new paper:

    http://www.tech-know.eu/uploads/Copernicus_Meets_the_Greenhouse_Effect.pdf

    Why isn’t this receiving any attention?

    Paul

    00

    • #
      Bryan

      Paul I agree and I hope Jo Nova can arrange that Joseph Postma is available to answer questions.
      Joseph gave a very good account of himself on Climate etc for his previous paper.

      00

      • #
        J. Postma

        Paul, Bryan;

        Thanks for the messages. You might like to know that, last week I engaged in an email debate with a very wide distribution list with a well-known and staunch supporter of everything AGW and the GHE. The outcome of that debate ended much like the one on Climate Etc., with my challenger bowing-out of the debate at the end, and with my central objections to the GHE remaining intact. I can not tell you who the debate was with as the other party has not agreed for it to be published, yet. We may publish it anonymously.
        And additionally, even more interesting, is that I have now formulated an equation which can model the input/output heat flow situation on the Earth in real time, with rotation, and with the real instantaneous inputs and outputs for & of the system. The equation shows that there is tremendous difference in outcome between modelling a 2-D flat Earth with a cold Sun vs. a 3-D spherical Earth with a hot Sun – the latter of which being obviously what actually exists. I am working to publish this soon, online. If we can publish the email debate sooner I will let you know. Please take care.

        —–

        Joseph, Sorry, I don’t know why your comments keep getting caught in the filter. I’ve looked through the filter specs and searched for clues, but found nothing. — Jo

        00

        • #
          Paul

          All good news, Joseph, all except that first the New Zealand and now the Australian ‘Government’ have already bolted and it is left now to finally shut the stable door.

          Paul

          00

  • #
  • #

    Craig Thomas wrote:
    May 20, 2011 at 4:18 pm · Reply
    “I’m curious – what’s with the americanism? Why is it a “darn” law?”

    Even though I’m an American, I agree with Craig. The title should have been:
    So What is the Second Bloody Law?

    01

  • #
    BobC

    JN: People keep warmer in Igloos, not because the heat comes from the ice, but because it comes from burning fat internally, and the ice cuts the heat loss. Why continually ignore the real heat inputs? No one is suggesting that ice or GHG’s raise the temp of a warmer body above what it would have been without another energy source…. I’m stuck in ground-hog day: the sun … the sun … the sun…. ?

    I have personal experience of being comfortable in an igloo, in pants and sweater(inside air temp: 0C), while the outside air temp is -40C. Sitting outside, similarly dressed, would have resulted in hypothermia and, eventually, death. My body temperature was always significantly higher than the walls of the igloo (0C), so why am I warmer? It is absolutely correct to say that it is “because of the igloo”. Would the temperature inside the igloo be greater than the outside if there were no heat source inside? No, it would not.

    Postma’s analysis says, in effect, that you could not be warmer inside the igloo than outside — hence Postma’s analysis is wrong, as demonstrated by experience.

    An experience that more people can share is an actual glass greenhouse on a cold, but sunny, day. The glass allows energy from the Sun to come in via visible radiation (glass is transparent to visible wavelengths), but impedes the loss of energy by both radiation (glass is opaque to infrared wavelengths) and air convection. As anyone can demonstrate for themselves, it can be quite warm in the greenhouse even if the outside air temperature is very low. Measurement of the glass (or just touching it) demonstrates that the glass is colder than the inside air temperature (glass conducts heat, so is colder than the inside air since it is losing some heat to the outside). It is easily demonstrated that the glass is responsible for the increase in inside temperature by simply opening the windows — the temperature inside will rapidly drop. So, here is another example of a colder object (the glass) causing a warmer object (the inside of the greenhouse) to heat up. Thus, the claim that this is impossible is falsified.

    The only reason this seems counter-intuitive is that we have left the Sun out of the description (or internal body heat production, in the case of the igloo) — just like the Slayers leave the visible radiation input from the Sun out of their analyses.

    There is nothing about the radiative greenhouse effect that violates any laws of physics — as demonstrated by examples like the above. The only valid questions are:
    1) How important is this effect for the Earth’s temperature balance?
    2) How much of this effect is due to CO2?
    3) How much of the atmosphere’s CO2 is due to Human activity?

    Empirical data falsifies the modeler’s claims that Human activities are causing a crisis — arguments about assumed physical laws (especially, falsified arguments) are completely irrelevant.

    10

  • #
    Richard111

    Weird and wonderful discussions. Molecules on a solid/liquid surface are losing heat energy to a cooler gas via radiation and conduction. Looking into the solid/liquid, a few molecular layers down, energy levels are higher. This is the source. Heat energy from the surface will not be passed DOWN unless the external source is at a higher energy level. We know the gas is cooler. So in the solid/liquid heat energy ONLY moves up and out unless the outside gets hotter.

    Now consider the molecules at the surface, interface with the gas, having lost energy via radiation or conduction, some of those molecules are able to absorb apropriate photons and recover their previous energy state. This energy state is extremely unlikely to be greater than the energy states of the molecules below so that energy will not pass into the solid/liquid. That newly absorbed energy will simply be radiated or conducted back to the gas.

    Conclusion. Photons from a cooler source can indeed be absorbed at the surface of a warmer body due to the statistical nature of the energy states of the surface molecules but that newly absorbed low intensity energy is promptly lost again. Effectively is had been REFLECTED, not absorbed.

    01

  • #
    Graeme Bird

    The problem is Richard, that there is no such thing as a photon. By their own admission even the quantam mechanics don’t take the electron-jumping shell model of Niehls Bohr seriously any more. As loveable and useful as that model is, its this random photon business, derived from this model, which makes people FEEL that the greenhouse-backradiation story must be true. Small loveable and sometimes useful models are one thing. But the empirical evidence must be there. The empirical evidence coming from unpolluted data.

    10

    • #
      Tom in Oregon City

      Oh, my: “no such thing as a photon”. Well, turn out the lights (pun intended) and everybody go home. There is electromagnetic energy, and that’s all we are talking about. The model of the atom can be rice pudding, for all I care: atoms still emit radiation, with varied energy levels, and thereby lose internal energy. That radiation travels at the speed of light, because it IS “light”. You don’t want to call it a photon? Or you truly believe there is no such thing as electromagnetic self-propagating emission? So, how are you reading this, again?

      One Stefan-Boltzmann derivation characterizes the net transfer of energy between two objects at differing temperatures, proportional to T1^4 – T2^4, matching observation. If you don’t want to call that “back radiation”, fine. I don’t like that particular name, anyway: just call it radiation.

      Does object 1 call object 2 to find out if it’s OK to send a few bits of energy, and object 2 says, “No, you can’t do that; you’re too cold!” Nope. Can’t happen.

      Does IR emitted from a hot liquid in a thermos flask know it’s being reflected back to the liquid by the mirrored liner? No. Does it care? No. Does the liquid somehow refuse to accept the return? No. And it is observable and measurable: the reason thermos liners are mirrored is because it WORKS, not because it lets the assembly line people more easily check their makeup.

      Net transfer is observable, and pretty obvious, unless you believe in fairy traffic cops forbidding energy from traveling uniformly.

      01

  • #
    Graeme Bird

    The second law of thermodynamics is empirically based. Laws of physics are human-derived and may possibly be broken, but not without very good reason. We cannot have leftists and public servants breaking them ad hoc. The Niels Bohr electron-jumping model was an example of creative excellence and it has its uses. But it cannot be used to overturn a law that has not been overturned in the laboratory.

    It doesn’t matter how vividly you “see” in your minds eye all these photons randomly heading in myriad diagonals earthward. Thats no confirmation of the theory that the dope-smoking Sagan laid on us when he could not believe that Venus was superhot for other reasons.

    10

  • #
    Graeme Bird

    Tom. CO2 doesn’t represent a thermos flask. Or a mirror. Or any of the things you mention. I think you are still using the Niels Bohr ad hoc model to over-ride the second law. What do you think?

    10

    • #
      Tom in Oregon City

      Graeme: Please don’t miss the point. You prefer the ultraviolet catastrophe, then? The concept of the photon describes a physically measured phenomenon. Energy release from a single atom does not affect all atoms, everywhere, as a wave front proceeds. It affects a limited target, with discrete amounts of energy, and that’s hard to describe otherwise.

      Light is most certainly a fascinating subject, but even if we are still dealing with bone knives and bear skins in science, it doesn’t help to throw out a useful duality description just because you despise a radiative transfer idea.

      The second law commands entropy not decrease without intervention (refrigerators work, after all, and even then by increasing energy at the power plant). S/B net radiative transfer does not violate that law: so-called “back radiation” is simply a reduction of the speed of cooling, and the warmer object is STILL cooling. The subject is covered in textbooks, described with the transfer of radiation between two blackbodies of the same temperature (ie, no net difference in emission).

      It would be helpful if you looked at static model charts and realize that the dynamic reality is that the surface is never getting more than insolation provides, it just gets some of its own radiative emission back again, which only has the effect of slowing the cooling process.

      Likewise, the return of IR to a hot liquid merely reduces the cooling rate: the mirroring is done because it works. That “back-radiation” is not a violation of the second law, either.

      Lastly, it would be most accurate to discuss all atmospheric absorption and emission, instead of pinning the discussion on a minority player, CO2. It is water vapor that provides most of the gas effect, and liquid water
      most of the reflective effect.

      It’s obvious you don’t think much of the IPCC, or CAGW alarmists. Neither do I. But that doesn’t mean you can toss useful Physics out to get rid of ridiculous mis-applications of it. Keep the baby, toss the bathwater.

      01

  • #
    Graeme Bird

    Its not a matter of me “not wanting to call it a photon”. There are no such things as photons capiche? The idea is irrational once we know that light moves in waves. I don’t think I’m going to fast for you here. Its important because we cannot use an ad hoc model, full of imaginary entities, as evidence running counter to actual empirical evidence or empirically based laws of physics.

    10

  • #
    Bryan

    Graeme Bird says

    “There are no such things as photons capiche?”

    Your new ‘theory’ will come as a big surprise to 99.999% of physics graduates since 1925.
    When will the rest of the world come to see that you are the new Einstein

    00

  • #
    Graeme Bird

    It may come as a big surprise. But you ought to be able to think about it for a few minutes and figure it out for yourself. By the way, Einstein was a cult figure and not a serious scientist. If you have waves moving at c, its clear that this is incompatible with a photon particle moving at c, as the building particles for this wave. Waves always involve a medium. They never don’t involve a medium. Always a wave is a sort of shock (shockwave) moving through a medium. No photons need apply to be part of this story. This is not something that requires high intelligence to figure out. Its just something that requires basic dishonesty to be in denial of.

    01

    • #
      KR

      “By the way, Einstein was a cult figure and not a serious scientist.”

      Oh my, I had missed this one. Graeme, you are delusional. Nothing more to be said.

      01

    • #
      Tom in Oregon City

      If you have waves moving at c, its clear that this is incompatible with a photon particle moving at c

      A basic misunderstanding of a photon, which has no mass, and is described by Maxwell’s pair of wonderful partial differential equations, describing the perpendicular forces alternately destroying and rebuilding the other — one magnetic and the other electric — thus requiring no medium for travel. Back to school, Graeme. You’re not sounding like someone who actually studies the uniqueness of light, who has never seen the rather demonstrative photo of the track of a gamma ray — yeah, an actual photon — shooting into an intense magnetic field, thus disrupting its unique field pair, being destroyed while becoming a positron and an electron (illustrated by their opposite circular tracks in the field).

      For your education: http://www.britannica.com/EBchecked/topic/225048/gamma-ray

      It would be helpful if you tried to sound a little less cocky when you are espousing things wrapped up in a rather century-old view of the world.

      00

  • #
    Graeme Bird

    “Please don’t miss the point. You prefer the ultraviolet catastrophe, then? The concept of the photon describes a physically measured phenomenon. Energy release from a single atom does not affect all atoms, everywhere, as a wave front proceeds. It affects a limited target, with discrete amounts of energy, and that’s hard to describe otherwise.”

    But the concept of a photon also includes the photon taking off at the speed of c. Whereas a tsunami in no way implies that the water molecule moves as far or as fast as the wave itself. You must acknowledge that the photon, as advertised, simply doesn’t exist. This is not something to blame me about. It doesn’t exist as described. When the wave comes in to crash on the rocks, the molecules may be moving, at the speed of the wave, at the point of impact. But the concept of the photon is something entirely different. And you should be honest enough to admit that this photon is a fantasy.

    01

  • #
    KR

    Graeme Bird“…the photon, as advertised, simply doesn’t exist.”

    Here is a perfect example of why people raising “2nd law” objections to the radiative greenhouse effect are not taken seriously. Photons were first described with Planck and Einstein’s 1900-1905 descriptions of the photoelectric effect, i.e. the quantization of electromagnetic energy. This has been repeatedly confirmed over the last 110 years, and is an underlying concept of physics. (I, in fact, work with photon counting devices on a daily basis, although I will _not_ put myself in the Einstein class). It’s a central concept of quantum mechanics, which has given us nuclear power and the H-Bomb, PET scans and two-photon confocal microscopes. Photons are a demonstrated and functional description of the world with well tested consequences.

    Yet Graeme claims that the last 110 years of physics are wrong, that photons don’t exist, are “fantasy”. Absurd.

    Einstein supported his theories with detailed physics, with testable predictions from the consequences of his work, with math. Graeme, by comparison, waves the woo. And hence will be taken as a crank by anyone with a scientific background.

    Up your game, Graeme. Or you will (deservedly) be dismissed as delusional.

    Note: If someone’s objections to the general thrust, consensus, of scientific opinion can be supported, I would love to see it. So would just about everyone else with a science background. But ridiculous BS contrary to all observations like “photons are a fantasy” will at best be ignored, and at worst lead to men in white coats and straight jackets…

    00

  • #
    Graeme Bird

    People who deny that the photons are a fantasy (as advertised) are simply dishonest or not that bright. There is also this delusional notion that the aether doesn’t exist. But if light moves in waves this directly implies a medium. Now the quantization of energy. So our testing equipment tells us that energy manifests in discrete amounts. Well thats okay. But these little bursts are not photons. This we have already proven. Since the photon concept is not consistent with wave motion.

    Its only public servants we are talking about. The iron law of the public service is to lock stupidity in. Often-times when they want to lock something really stupid in they shoot straight for Copenhagen.

    “Einstein supported his theories with detailed physics, with testable predictions from the consequences of his work, with math. Graeme, by comparison, waves the woo. And hence will be taken as a crank by anyone with a scientific background.”

    Now you are just being silly. Einsteins ideas are pure fantasy and fudge factors. He didn’t employ the scientific method. He injected mysticism into science. I just wonder where the source of all the hype came from. This fellow a media darling for some reason.

    20

  • #
    Graeme Bird

    Most people don’t want to get bogged down in the Einstein fantasy-physics. Thats one sacred cow too many for most people. The Einstein believers would never sit still for a full-blown test of special relativity. One where we isolated every last factor before testing if velocity was more than a purely relative concept. The test would fail of course. Which is why it is never carried out. The entirety of the special relativity corpus is pure irrationality. Its not necessary to any part of physics. Its a testimony to the mind control that is possible over a profession.

    11

  • #
    Tom in Oregon City

    One must never let Graeme wash a baby, as it will be tossed with the bathwater.

    In objecting to abuses by the CAGW crowd, Graeme, one should avoid being led into the weeds of worse pseudoscience found in the ditch on the OTHER side of the road.

    Photo-electric effect. Maxwell’s equations. Observable tracks showing energy conversion to matter, and vice-versa.

    Now you can have the last word, though that, too, will fail to convince any actual Physics students on this list.

    01

  • #
    Graeme Bird

    Well why are you holding onto pseudo-science then? You are holding onto crankery and quack-science as if it were a silver dollar held between your butt-cheeks. A volley of photons is not consistent with wave motion. Wave motion implies an aether. And why in the world are you invoking Maxwell in this matter? Maxwell would side with me, and would have none of your fantasy-physics. Its a character issue as much as anything. To claim that there is such a thing as a photon, after its just been explained to you very carefully why there isn’t …. this is basic dishonesty on your part.

    10

  • #
    Graeme Bird

    “Observable tracks showing energy conversion to matter…”

    Observable tracks of what? Is this where its alleged that a single electron can be responsible for an entire track of bubbles? Pseudo-science.

    10

  • #
    Graeme Bird

    “A basic misunderstanding of a photon, which has no mass, and is described by Maxwell’s pair of wonderful partial differential equations, describing the perpendicular forces alternately destroying and rebuilding the other — one magnetic and the other electric — thus requiring no medium for travel. ”

    A basic misunderstanding of Maxwell, who was never so idiotic as to imagine that waves were possible without a medium.

    10

  • #
    Graeme Bird

    “A basic misunderstanding of a photon, which has no mass, and is described by Maxwell’s pair of wonderful partial differential equations, describing the perpendicular forces alternately destroying and rebuilding the other — one magnetic and the other electric — thus requiring no medium for travel. ”

    Many features of quack science in the one statement. The deification of Maxwell, or his equations, whilst misrepresenting him. The concentration on bizarre notions of mathematical aesthetics at the expense of reality. And note the description of lights progress in direct contradiction to the notion of a volley of photons.

    10

  • #